Sie sind auf Seite 1von 523

Polling of the Managerial Economic subject:

Shir Mohammad “Omid”


Omid78611@gmail.com
The best definition of economics is
A) How choices are made under conditions of scarcity.
B) How money is used.
C) How goods and services are produced.
D) How businesses maximize profits.
Which of the following is the best example of "what goods and services should
be produced?"
A) The use of a capital intensive versus a labor intensive process of manufacturing
textiles
B) The production of SUVs versus the production of sub-compact cars
C) The manufacturing of computer workstations in China or in India
D) The leasing versus the purchasing of new capital equipment
Which of the following is the best example of "how goods and services should be
produced?"
A) Complying with the technical specifications in the production of an aircraft
B) The production of jet aircraft for the air force or for a commercial airline
C) The use of additional workers versus the use of machines in the production of goods
D) The production of a new manufacturing facility
A company will strive to minimize
A) Transaction costs.
B) Costs of internal operations.
C) Total costs of transactions and internal operations combined.
D) Variable costs.
The best example of an economic goal of a firm is

1
A) Providing good products/services to its customers.
B) Improving its public image.
C) Increasing employee morale.
D) Increasing shareholder wealth.
Which of the following is a microeconomics concern?
A) the reasons for a decline in average price level
B) the reasons why Naira buys less orange juice
C) the cause of why total employment may decrease
D) the effect of the government budget deficit on inflation
Which of the following is a macroeconomic issue?
A) why plumbers earn more than janitors
B) the reasons for the rise in average prices
C) whether the army should buy more tanks or more rockets
D) the reasons for a rise in the price of orange juice
Those things that must be forgone to acquire a good are called:
A) substitutes
B) Opportunity costs.
C) Explicit costs.
D) Competitors
Holding all other factors constant, consumers demand more of a good the:
(A) Higher its price.
(b) Lower its price.
(c) Steeper the downward slope of the demand curve.
(d) Steeper the upward slope of the demand curve.
If the cost of making bicycles falls, the price goes down, causing the demand
curve to shift to the right.
 True

2
 False
Which of the following will not cause a shift in the demand curve?
A. Price
B. Population
C. Income
D. Taste and Preference
The Supply Curve is upward-sloping because:
(a) As the price increases, consumers demand less.
(b) As the price increases, so do costs.
(c) As the price increases, suppliers can earn higher levels of profit or justify higher
marginal costs to produce more
(d) None of the Above
A specific tax on sellers will ………
(a) Shift the demand curve to the right.

(b) Shift the demand curve to the left.


(c) Shift the supply curve to the right.
(d) Shift the supply curve to the left
If the price elasticity of demand for a good is less than one in absolute terms, we
say consumers of this good
(a) Are not very sensitive to price.
(b) Are not very sensitive to the quantity they demand.
(c) Are very sensitive to price.
(d) Are elastic.
Which of the following is the non-price determinant of supply of wheat?
(a) A decrease in the price of corn.
(b) A decrease in the price of wheat
(c) Improvements in the technique of growing wheat.
(d) None of the above

3
An economist would estimate the ……….to determine the relationship between
two goods.
(A) Income elasticity of demand
(B) cross elasticity of demand
(C) price elasticity of demand
(D) price elasticity of supply
Which of the following is not a variable input?
A. Raw materials
B. Electricity
C. Land
D. Transportation
When MP=0, which of the following is true?
A. TP=0
B. TP=+ve
C. TP=Max
D. TP=Min
Causes of Applicability of law of variable proportions are:
(a) Imperfect substitutes
(b) Under-utilization of fixed factor
(c) Optimum production
(d) All of these
The slope of an isoquant is:
A. the marginal product
B. The average product.
C. The marginal cost.
D. The marginal rate of technical substitution.
Which of the following is not true about the Isoquants?
A. Isoquant are Convex
B. Isoquant are Concave

4
C. Slope of Isoquant depends on MRTS
D. Higher isoquant means more production
Iso-cost line is also known as :
a) Factor Price Line
b) Outlay Line
c) Firm’s Budget
Which of the following will cause a change in the iso-cost line?
A. Change in recourses
B. Change in factor input price.
C. Change in factor combination
D. All of above
The law of diminishing returns assumes that :

A: .there is at least one fixed input.

B:. all inputs are changed by the same percentage.

C. additional inputs are added in smaller and smaller increments.

D. all inputs are held constant.


Decreasing returns to scale in production means
a. more than twice as much of all inputs is required to double output
b. Less than twice as much of all inputs is required to double output.
c. More than twice as much of only one input is required to double output.
d. Isoquants must be linear.
Implicit costs are:
A) Equal to total fixed costs.
B) Comprised entirely of variable costs.
C) Payments for self-employed resources.
D) Always greater in the short run than in the long run.
Variable costs are:

5
A) sunk costs.

B) multiplied by fixed costs.


C) costs that change with the level of production.
D) defined as the change in total cost resulting from the production of an additional
unit of output.
If all resources used in the production of a product are increased by 20 percent
and output increases by 20 percent, then there must be:
A) Economies of scale.
B) Constant returns to scale.
C) Diseconomies of scale.
D) Increasing average total costs.
Which would be an implicit cost for a firm? The cost:
A) Of worker wages and salaries for the firm.
B) Paid for leasing a building for the firm.
C) Paid for production supplies for the firm.
D) Of wages foregone by the owner of the firm
When the total product curve is falling, the:
A) Marginal product of labor is zero.
B) Marginal product of labor is negative.
C) Average product of labor is increasing.
D) Average product of labor must be negative.
The cost recorded in the books of accounts are considered as
a. Total cost
b. Marginal cost
c. Average cost
d. Explicit cost
Under ______ price is determined by the interaction of total demand and total
supply in the market.

6
a. Perfect competition
b. Monopoly
c. Imperfect competition
d. All of the above
Going Rate Pricing is adopted in which of the following market?
a. Perfect competition
b. Monopoly
c. Monopolistic competition
d. None of these
What is the kind of elasticity of demand in perfect competition?
a. Elastic
b. Inelastic
In perfect competition, the product of a single firm
A) Is sold to different customers at different prices.
B) has many perfect complements produced by other firms.
C) has many perfect substitutes produced by other firms.
D) Is sold under many differing brand names.
Which of the following characterizes monopolistic competition?
A) Many interdependent firms sell a homogeneous product.
B) A few firms produce a particular type of product.
C) Many firms produce a particular type of product, but each maintains some
independent control over its own price.
D) A few firms produce all of the market supply of a good.
Entry into a market characterized by monopolistic competition is generally:
A) Entirely blocked by existing firms.
B) Very easy because few barriers exist.
C) As difficult as in oligopoly.

7
D) More difficult than entry into monopolized markets.
The following are key features of a monopoly EXCEPT
A) Diseconomies of scale. B) No close substitutes.
C) Influence over price. D) Barriers to entry.
Which of the following statements about a monopoly is FALSE?
A) A monopoly is the only supplier of the good.
B) Monopolies have no barriers to entry or exit.
C) The good produced by a monopoly has no close substitutes.
D) None of the above; that is, all of the above answers are true statements about a
monopoly.
4. A defining characteristic of a natural monopoly is that
A) It exists because of legal barriers to entry.
B) It has no close substitutes.
C) Its average total cost curve slopes downward as it intersects the demand curve.
D) Its demand curve slopes downward.
6. For a monopoly, the industry demand curve is the firm's
A) Profit function. B) Marginal revenue curve.
C) Supply curve. D). Demand curve
If the entire output of a market is produced by a single seller, the firm:
A) Is a monopoly.
B) Faces a perfectly inelastic demand.
C) Can charge any price it wants and not lose customers.
D) All of the above.
Monopolists
A) Face downward sloping demand curves. B) Are price takers.
C) Have no short-run fixed costs. D) Maximize revenue, not profits
. In long run a monopolist have

8
• a. Normal profit
• b. supernormal profit
• c. economic loss
• d. all of these
A monopoly is
• a. one of the few producers of homogenous product
• b. A single producer of a single product
• c. one of the many producer
A monopoly will not only charge a higher price, it will also produce _____ output
than a competitive market would produce.
A) More B) less C) better D) poorer
Price discrimination is:
A) Charging different prices to different customers because it costs the firm more to
serve some customers than others.
B) Changing the firm’s price frequently to respond to market conditions.
C) Charging different prices to different customers when the price differences are
not based on cost differences.
D) Charging the same price to all customers.
In monopolistic competition, a firm:
A) Has no market power.
B) Captures significant economies of scale
C) Has a downward-sloping demand curve.
D) Has a homogenous product that all firms produce.
If there are many firms in an industry producing goods that are similar but
slightly different, this is an example of:
A) Perfect competition.
B) Monopolistic competition.
C) Oligopoly.

9
D) Monopoly.
Which of the following market structures is composed of many firms producing
close substitutes that are differentiated according to consumers?
A) Perfect competition.
B) Monopolistic competition.
C) Oligopoly.
D) Monopoly
Which of the following is characteristic of monopolistic competition?
A) Many firms in an industry. C) Market power.
B) High concentration ratios. D) All of the above
A major difference between monopoly and monopolistic competition is:
A) One maximizes profits by setting MR equal to MC, and the other does not.

B) The number of firms in the market.


C) One type of firm has market power, and the other does not.
D) One has a downward-sloping demand curve, and the other does not.
One of the main differences between oligopoly and monopolistic competition is:
A) Monopolistic competition faces a horizontal demand curve; oligopoly does not.
B) The degree of interdependence among firms.
C) The amount of non-price competition that occurs.
D) All of the above.
Which of the following characterizes monopolistic competition?
A) Price leadership. B) Product differentiation. C) Price discrimination. D)
Economies of scale.
Product differentiation occurs when:
A) A completely new process is used to produce a familiar product.
B) One firm produces many varieties of a product.
C) Buyers, though not necessarily sellers, perceive differences in the products of
several companies.

10
D) Sellers, though not necessarily buyers, perceive differences in the products of
several companies.
Which of the following is an example of product differentiation?
A) Two bars of soap differ only in their label, but consumers pay $0.20 more for the
label they recognize.
B) Sugar can be made from sugar beets or sugar cane which consumers cannot
differentiate when looking at sugar.
C) Consumers substitute vans in place of cars because vans accommodate more
passengers.
D) Some sawmills specialize in producing softwood and others specialize in
producing hardwood, but the two types of wood are used for very different purposes
Perfect competition and monopolistic competition are best distinguished by:
A) The degree of product differentiation.
B) The long-run economic profits that are expected.
C) The number of firms in the market.
D) The ease of entry and exit.
Which of the following statements about a monopoly is FALSE?
A) A monopoly is the only supplier of the good.
B) Monopolies have no barriers to entry or exit.
C) The good produced by a monopoly has no close substitutes.
D) None of the above; that is, all of the above answers are true statements about a
monopoly.
Which of the following is LEAST likely to be a monopoly?
A) the sole owner of an occupational license
B) a pharmaceutical company with a patent on a drug
C) a store in a large shopping mall
D) the holder of a public franchise
Monopolists ……
A) Face downward sloping demand curves.

11
B) Are price takers.
C) Have no short-run fixed costs.
D) maximize revenue, not profits
If there are two markets A and B. In market A, E>1 and in market B, E<1. In which
market the seller can charge the more price.
A. Market Aim
B. Market B
C. Both Markets
D. None of the Market
Which of the following is not an argument in favor of the globalization of
business?
A) More efficient use of resources lowers operating costs and selling prices.
B) More products are made available and new markets are opened.
C) Economic and political security are enhanced.
D) Technology transfers improve living standards in poorer countries.
Risks faced by multinational corporations include:
A) Changes in exchange rates.
B) Restrictions on ownership.
C) Cultural and religious philosophies.
D) All of the above
Firms undertake multinational operations in order to
A) Hire low-wage workers.
B) Manufacture in nations they have difficulty exporting to.
C) Obtain necessary factor inputs.
D) All of the above
Scarcity is a condition that exists when?
A) There is a fixed supply of resources relative to the demand for the product.
B) There is a large demand for a product.

12
C) Resources are not able to meet the entire demand for a product.
D) All of the above
Which of the following is not considered as a factor of production?
A) Money
B) Machinery and equipment
C) Land
D) Unskilled labor
Each point on the demand curve reflects
A) The highest price consumers are willing and able to pay for that particular unit of a
good.
B) The highest price sellers will accept for all units they are producing.
C) The lowest-cost technology available to produce a good.
D) All the wants of a given household
Which of the following is consistent with the law of demand?
A) A decrease in the price of a gallon of milk causes a decrease in the quantity of
milk demanded.
B) An increase in the price of a soda causes a decrease in the quantity of soda
demanded.
C) An increase in the price of a tape causes an increase in the quantity of tapes
demanded.
D) A decrease in the price of juice causes no change in the quantity of juice
demanded
If income decreases or the price of a complement rises?
A) There is an upward movement along the demand curve for the good.
B) There is a downward movement along the demand curve for the good.
C) The demand curve for a normal good shifts leftward.
D) The demand curve for a normal good shifts rightward
Marginal rate of technical substitution determines the ............... of isoquant curve.
(a) meaning

13
(b) slope
(c) satisfaction
(d) aim
The term ‘marginal’ in economics means
(a) Incremental
(b) Unimportant
(c) Minimum Unit
(d) None of the above
Isoquants are equal to:
A. Product Lines
B. Total utility lines
C. Cost lines
D. Revenue lines
An isoquant is ______ to an iso cost line at equilibrium point:
A. Convex
B. Concave
C. Tangent
D. Perpendicular
It definitely pays a firm to shut down if the price of its product is:
A) Below its minimum average variable cost.
B) Above its maximum variable cost.
C) Above its minimum average variable cost.
D) Below its minimum total cost.
The costs incurred even when no output is produced are called:
A) Fixed costs.
B) External costs.
C) Variable costs.

14
D) Marginal costs.
By producing less, a firm can reduce:
A) Its variable costs but not its fixed costs.
B) Its fixed costs and its variable costs.
C) Its fixed costs but not its variable costs.
D) Neither its variable costs nor its fixed costs.
The long run is a time frame in which:
A) The quantities of all resources are fixed.
B) The quantities of all resources can be varied.
C) The quantities of some resources are fixed and the quantities of other resources
can be varied.
D) All costs are sunk costs.
Law of variable proportion is valid when:
A. Only one input is fixed and all other inputs are kept variable, if kept variable to
change in constant than answer is option A. because this says variable so the
answer is option “D”, none of these
B. All factors are kept constant
C. All inputs are varied in the same proportion
D. None of these
When Isoquants are straight line as shown in the diagram the MRTS…
A. >1
B. =1
C. <1
D. None of these
Managerial economics is best defined as the economic study of
A) How businesses can make the most profits.
B) How businesses can decide on the best use of scarce resources.
C) How businesses can operate at the lowest costs.

15
D) How businesses can sell the most products.
Scarcity is a condition that exists when

A) There is a fixed supply of resources relative to the demand for the product.

B) There is a large demand for a product.

C) Resources are not able to meet the entire demand for a product.

D) All of the above

Which of the following is not considered as a factor of production?


A) Money
B) Machinery and equipment
C) Land
D) Unskilled labor
Which of the following is the best example of how the question of "what goods
and services to produce?" is answered by the command process?
A) Government subsidies for windmill energy production
B) Laws regarding equal opportunity in employment
C) Government allowance for the deduction of interest payments on private
mortgages
D) Government regulations concerning the dumping of hazardous waste
From the standpoint of a soft drink company the question of "What goods and
services should be produced?" is best represented by which of the following
decisions?
A) Whether or not to hire additional workers
B) Whether or not to increase its advertising
C) Whether or not to shut down selected manufacturing facilities
D) None of the above are examples.

Shir Mohammad “Omid”


Omid78611@gmail.com
16
Economic Analysis for Business Decisions
Multiple Choice Questions
Unit-1: Basic Concepts of Economics

1. In a free-market economy the allocation of resources is determined by:

a. Votes taken by consumers b. A central planning authority


c. By consumer preferences d. The level of profits of firms

2. A rational person does not act unless:

a. The action is ethical


b. The action produces marginal costs that exceed marginal benefits
c. The action produces marginal benefits that exceed marginal costs
d. The action makes money for the person

3. Economics is a ----- science which deals with human wants and their satisfaction.

a. Social b. Political c. Natural d. Physical

4. ------ defined economics as a study of mankind in the ordinary business of life.

a. Adam smith b. Lionel Robbins c. Samuelson d. Alfred Marshall

5. The growth of an economy is indicated by an

a. Increase in savings b. Increase in investment


c. Increase in general prices d. Increase in national income

6.-----------equals revenue minus all explicit costs.

a. Accounting profit b. Economic profit c. Normal profit d. Loss

7. The father of New Economics is :

a. Marshall b. J.M.Keynes c. Adam Smith d. Karl Marx

8. The ------------------ problem refers to the possibility that owners and their managers may have
different objectives.

a. Company- Manager problem b. Principal-Agent Problem


c. Firm-Employee problem d. Problem of different objectives

9. Economic profit refers to ------------ minus all relevant costs , both explicit and implicit.

a. Profit b. Cost c. Expenses d. Revenues


10. The interaction of individuals and firms in a market can be described as a ----------------- of
money, goods and services and resources through product and factor markets.

a. Constant flow b. Stable flow c. Circular Flow d. Regular Flow

11. ----------------- focuses on the behavior of the individual actors on the economic stage , that is,
firms and individuals and their interaction in markets.

a. Macroeconomics b. Microeconomics c. Managerial Economics d. Economics

12. In free market economy, the organization and interaction of producers and consumers is
accomplished through the -------------- system.

a. Price b. Cost c. Profit d. Revenue

13. An economic system:

a. Requires a grouping of private markets linked to one another.


b. Is a particular set of institutional arrangements and a coordinating mechanism used to
respond to the economizing problem.
c. Requires some sort of centralized authority (such as government) to coordinate economic
activity.
d. Is a plan or scheme that allows a firm to make money at some other firm's expense

14. The regulatory mechanism of the market system is:

a. Self-interest. b. Private property. c. Competition. d. Specialization.

15. Which of the following is not an economic cost?

a. Wages. b. Rents. c. Economic profits.


d. Payments made to the entrepreneur for organizing production

16. From society's point of view the economic function of profits and losses is to:

a. Promote the equal distribution of real assets and wealth.


b. Achieve full employment and price level stability.
c. Contribute to a more equal distribution of income.
d. Reallocate resources from less desired to more desired uses.

17. The invisible hand refers to the:

a. Fact that the U.S. tax system redistributes income from rich to poor.
b. Notion that, under competition, decisions motivated by self-interest promote the social
interest. c. Tendency of monopolistic sellers to raise prices above competitive levels.
d. Fact that government controls the functioning of the market system.
18. The invisible-hand concept suggests that: .

a. Market failures imply the need for a national economic plan.


b. Big businesses are inherently more efficient than small businesses.
c. The competitiveness of a capitalistic market economy invariably diminishes over time.
d. Assuming competition, private and public interests will coincide

19. Two major virtues of the market system are that it:

a. Allocates resources efficiently and allows economic freedom.


b. Results in an equitable personal distribution of income and always maintains full employment.
c. Results in price level stability and a fair personal distribution of income.
d. Eliminates discrimination and minimizes environmental pollution.

20. The simple circular flow model shows that:

a. Households are on the buying side of both product and resource markets.
b. Businesses are on the selling side of both product and resource markets.
c. Households are on the selling side of the resource market and on the buying side of the
product market.
d. Businesses are on the buying side of the product market and on the selling side of the resource
market.

21. The two basic markets shown by the simple circular flow model are:

a. Capital goods and consumer goods. b. Free and controlled.


c. Product and resource. d. Household and business.

22. Which of the following is a limitation of the simple circular flow model?

a. Product markets are ignored. b. Resource markets are ignored.


c. The determination of product and resource prices is not explained.
d. Households are included, but not businesses.

23. In economics the central problem is:

a. Money. b. Scarcity. c. Allocation. d. Production.

24. Macroeconomics deals with:

a. The behavior of firms. b. Economic aggregates.


c. The behavior of the electronics industry. d. The activities of individual units.
25. Microeconomics is not concerned with the behavior of:

a. Consumers. b. Aggregate demand. c. Firms. d. Industries.


26. The total demand for goods and services in an economy is known as:

a. Aggregate demand. b. Gross national product.


c. Economy-wide demand. d. National demand.

27. Unemployment means that:

a. People are not willing to work at the going wage rate.


b. At the going wage rate, there are people who want to work but cannot find work.
c. There are some people who will not work at the going wage rate.
d. There is excess demand in the labour market.

28. Opportunity cost is

a. A cost that cannot be avoided, regardless of what is done in the future.


b. The cost incurred in the past before we make a decision about what to do in the future.
c. That which we forgo, or give up, when we make a choice or a decision.
d. The additional benefit of buying an additional unit of a product.

29. The circular flow of goods and incomes shows the relationship between:

a. Firms and households. b. Goods and services.


c. Income and money. d. Wages and salaries.

30. In a planned or command economy, all the economic decisions are taken by the:

a. Workers. b. Consumers. c. Voters. d. Government.

31. Which one of the following is a normative statement?

a. The richest 10 per cent of the population has had a bigger percentage increase in incomes over
the past 10 years than the poorest 10 per cent.
b. The proportion of people's income paid in taxes is higher under this government than under
the previous one.
c. Inflation is rising.
d. Inequality in the distribution of income is a more serious problem than unemployment.

32. Continues consumption of homogeneous product is a assumption for:

a. Law of Demand. b. Law of Supply. c. Law of Diminishing Marginal Utility.

33. Goods and Services bought and sold in:

a. Product Market. b. Factor Market. c. Capital Market. d. Money Market.


34. In Product market money flows from:

a. Individual to firm. b. Business to households. c. Government to household.

35. In factor market suppliers are:

a. Firms. b. Households. c. Government

36. In economic activities which are the outflows:

a. Government spending. b. Investment. c. Savings.

37. Indian economy is:

a. Capitalist Economy b. Socialist Economy c. Mixed Economy

38. Father of Economics:

a. Lionel Robbins. b. Adam Smith. c. Alfred Marshal.

39. Implicit Cost added in:

a. Accounting cost. b. Economic cost. c. Both.

40. Business Economics is also known as………….

a. Managerial Economics b. Economics for Executives


c. Economic analysis for business decisions d. All the above

41. State whether economics is

a. A positive science only b. Neither a positive nor normative science


c. A science but not art
d. A science or an art depending on who uses economics and for what purpose.

42. The branch of economics wherein mathematics and statistics are used to measure and analyze
economics activities is called……………..

a. Applied Economics b. Econometrics c. Statistics d. Macro Economics

43. The opportunity cost of a machine which can produce only one product is:

a. Low b. Infinite c. High d. Medium


44. It is defined as a state of knowledge in which one or more alternatives result in a set of
specific outcomes but where the probabilities of the outcomes are neither known nor meaningful.
a. Risk b. Uncertainty c. Peril d. All of the above
45. Opportunity cost is term which describes

a. A bargain for a factor of production b. Costs related to an optimum level of production


c. Average variable cost d. None of these

46. It is also known as prescriptive economics

a. Positive Economics b. Micro economics c. Normative economics d. Economics

47. ……cost are also known as Imputed Costs

a. Opportunity b. Marginal c. Total d. Historical

48. There are ……branches of economics.


a. 2 b. 3 c. 4 d. 6

49. It is the study of economics actions of individuals and small groups of individuals.
a. Micro-Economics b. Macro-Economics c. Managerial Economics d. Business Economics

50. An input should be so allocated that the value added by the last unit is the same in all cases.

a. Opportunity Cost Principle b. Equi-Marginal Principle


c. Incremental Principle d. Discounting Principle

51. The term `Economics‟ in English language has its origin in ------word.

a. Greek b. Italic c. Latin d. Indian

52. Economics includes the following economic activities:

a. Production b. Consumption c. Exchange d. All of the above

53. It refers to the determination of prices of all goods and services by the interaction of the
forces of demand and supply without any external interference.

a. Product-mechanism b. Price-mechanism c. Cost-mechanism d. None of these

54. The principle reasons behind economic problems

a. Unlimited wants b. Limited or Scarce of Means


c. Alternatives Uses of Means d. All of the above
55. Income flow is also known as------.

a. Product Flow b. Money flow c. Profit flow d. Cash flow

56. Flows of the factor of production and the goods and services between the different sectors is -
-----.
a. Real flow b. Money flow c. Cash flow d. Product flow

57. It is also called output flow or real flow

a. Profit flow b. Cash flow c. Product flow d. None of these

58. Accounting profit=------------Explicit Costs

a. Total Revenue b. Total Cost c. Implicit cost d. None of these

59. According to Professor S.E.Landsbury, “Firm is an organization that produces and sells
goods with the goal of------its profits”.

a. Minimizing b. Maximizing c. Optimizing d. All of the above

60. According to profit maximization theory of the firm, management.

a. Decides output level which maximizes revenue


b. Output level which minimizes cost.
c. Output level which maximizes difference between the two
d. None of these

61. According to Simon if a firm fails to achieve its target initially results in:

a. A sense of helplessness b. Search behavior


c. Sacking of its managerial team d. Appropriate revision of the aspiration level

62. Managerial utility function is expressed as:

a. U = S (S,M,I) b. U = S (S,M) c. U = f (S,M,I) d. U = F(S,M,I)

63. It is the difference between total revenue and total economic cost

a. Accounting Profit b. Economic Profit c. Gross Profit d. Net Profit

64. Invisible hand theory is given by----------


a. Lord Robbins b. Samuelson c. Marshall d. Adam Smith

65. Managers are Agent when:


a. Managers and workers. b. Shareholders and managers. c. Managers and contractors.
66. Accounting cost include:

a. Owner`s land for company establishment. b. Depreciation c. Management skill of a owner

67. If economic profit equal to zero then:

a. Owners receive a profit more than their opportunity cost.


b. Owners receive a profit less than their opportunity cost.
c. Owners receive a profit equal to their opportunity cost.

68. Trade-offs are required because wants are unlimited and resources are:

a. Economical. b. Unlimited c. Efficient d. Marginal e. Scarce

69. Economics is the study of:


a. how society manages its unlimited resources. b. how to reduce our wants until we are
satisfied. c. how society manages its scarce resources. d. how to fully satisfy our unlimited
wants. e. how to avoid having to make trade-offs

70. Which of the following statements regarding the circular-flow diagram is true?

a. If Vijay works for XYZ Solutions Ltd. and receives a salary payment, the transaction takes
place in the market for goods and services.
b. If XYZ Solutions Ltd. sells a military aircraft, the transaction takes place in the market for
factors of production.
c. None of these answers.
d. The factors of production are owned by households.
e. The factors of production are owned by firms.

71. Which of the following is not a Productive Resource?

a. Labour b. Land c. Money d. Capital e. All of these answers are factors of production.

72. Which of the following issues is related to microeconomics?

a. The impact of oil prices on car production b. The impact of money on inflation
c. The impact of technology on economic growth d. The impact of the deficit on saving

73. The word economy comes from the Greek word for

a. "Environment." b. "One who participates in a market." c. "One who manages a


household." d. "Conservation.“

74. Economics deals primarily with the concept of

a. Poverty. b. Scarcity. c. Change. d. Power.


75. The opportunity cost of an item is

a. The number of hours needed to earn money to buy it.


b. What you give up to get that item.
c. Always less than the dollar value of the item.
d. Always equal to the dollar value of the item.

76. Factors of production are :

a. Inputs into the production process.


b. Weather, social, and political conditions that affect production.
c. The physical relationships between economic inputs and outputs.
d. The mathematical calculations firms make to determine production.

77. In the circular-flow diagram,

a. Firms are sellers in the resource market and the product market.
b. Households are sellers in the resource market.
c. Firms are buyers in the product market.
d. Spending on goods and services flow from firms to households.

78. In the circular-flow diagram,

a. Spending on goods and services flow from firms to households.


b. Goods and services flow from households to firms.
c. Factors of production flow from firms to households.
d. Income from factors of production flows from firms to households.

79. Scarcity is a condition that exists when

a. There is a fixed supply of resources.


b. There is a large demand for a product.
c. Resources are not able to meet the entire demand for a product.
d. All of the above.

80. Company goals that are concerned with creating employee and customer satisfaction and
maintaining a high degree of social responsibility are called ___________ objectives.

a. Social b. Noneconomic c. Welfare d. Public Relations

81. The value of an entrepreneur‟s resources that she uses in production are known as:

a. Explicit costs. b. Sunk costs. c. Operating expenses. d. Technological expenses.


e. Implicit costs.
82. One of the most important differences between a firm‟s economic profit and its accounting
profit is the subtraction of:

a. Costs incurred when hiring labor, capital, and land.


b. Any explicit cost incurred by the entrepreneur for risk taking.
c. Any implicit charges for the use of capital owned by the entrepreneur.
d. Any taxes on the retained earnings of the firm.
e. The costs of distributing the firm‟s output.

83. That profit functions as an incentive for innovation was among the key contributions to
economic thought by:

a. Karl Marx. b. Frank Knight. c. Joseph Schumpeter. d. Adam Smith. e. Alfred Marshall.

84. Which two of the following are characteristics of the „principal-agent‟ problem?

a)Ownership and control in hands of same people


b)Ownership and control in hands of different people
c)Shareholders and managers pursue different objectives
d)Shareholders and managers pursue same objectives
e)Sole traders are the dominant form of business organization
a. b) and e)
b. b) and d)
c. b) and c)
d. a) and e)
e. a) and c)

85. Economic profit is…

a. Calculated by subtracting implicit costs of using owner-supplied resources from the firm's
total revenue.
b. a theoretical measure of a firm's performance and has little value in real world decision-
making.
c. Generally larger than accounting profit. d. Negative when costs exceed revenues.

86. Economic profit is…

a. The difference between total revenue and explicit costs.


b. The difference between total revenue and the opportunity cost of all the resources used
in production.
c. The difference between accounting profit and explicit costs.
d. The difference between accounting profit and the opportunity cost of the market-supplied
resources used by the firm.
87. The principal-agent problem arises when…

a. The principal and the agent have different objectives


b. The principal cannot decide whether the firm should seek to maximize the expected future
profits of the firm or maximize the price for which the firm can be sold.
c. The principal cannot enforce the contract with the agent or finds it too costly to monitor the
agent. d. Both a and c e. None of the above

88. Moral hazard…

a. Occurs when managers pursue profit maximization without regard to the interests of society in
general.
b. Is the cause of principal-agent problems.
c. Occurs only rarely in modern corporations.
d. Exists when either party to a contract has an incentive to cancel the contract.
E. Both a and b

89. Economic profit is the best measure of a firm's performance because…

a. Economic profit fully accounts for all sources of revenue.


b. Implicit costs are generally too difficult to measure accurately.
c. The opportunity cost of using ALL resources is subtracted from total revenue.
d. Only explicit costs influence managerial decisions since, in general, only explicit costs can be
subtracted from revenue for the purposes of computing taxable profit.

90. Business Economics is also known as _______

a. Managerial Economics b. Economics for Executive c. Economic Analysis for Business


Decision d. All the above

91. It is the study of economic actions of individuals and small groups of individuals:

a. Micro – Economics b. Macro Economics c. Managerial Economics d. Business


Economics

92. The principal reasons behind economic problems :

a. Unlimited Wants b. Limited or Scarce Resources


c. Alternative uses of Means d. All the above

93. Invisible hand theory is give by :

a. Lord Robbins b. Samuelson c. Marshal d. Adam Smith


94. The ultimate effect of the "invisible hand" of Adam Smith is that, in a competitive economy,
everyone:

a. Benefits if each acts in his/her own interest.


b. Will increase their profits in a free market. c. Should act to maximize economic growth.
d. Should act to promote the public interest.

95. The three fundamental questions of economic organization are:

a. When, for whom, and how. b. How, what, and for whom.
c. Who, how, and when. d. What, who, and why.

96. „„Economics is the science, which studies human behaviours as a relationship between ends
and scarce means which have alternative uses.‟‟ This definition of Economics is given by,

a. Lord Robbins b. Samuelson c. Alfred Marshal d. Adam Smith

97. Which of the following is true regarding the circular flow model?

a. Households provide the demand for the factor market and business provide the supply for the
goods and services market.
b. Households provide the demand for the goods and service market and business provides the
supply for the factor market.
c. Households provide the supply for the factor market and business provides the supply
for the goods and service market
d. Households provide both the supply and demand for the goods and services market

98. Income and revenues that are created within a country

a. always will remain within that country


b. Can leave that country only when goods are exported.
c. Can leave that country when goods are imported
d. Can leave that country when capital flows into that country's financial institutions.

99. In a circular flow model, the real variables are:

a. Money that flows from the factor market to the households.


b. Only the goods and services that are produced.
c. Only the resources that are used.
d. Both the goods and services produced and the resources that are used.

100. Money is

a. Backed by gold in Fort Knox b. The same as income.


c. The value of all coins and currency in circulation at any time.
d. Anything that is generally accepted as a medium of exchange.
101. The development of money as a medium of exchange has facilitated the expansion of trade
because

a. Holding money increases people's income b. No other mediums of exchange are available
c. Money eliminates the "double coincidence of wants" problem
d. Holding money increases people's wealth

102. As the interest rate falls, people hold ________ money instead of bonds because the
opportunity cost of holding money has ________.

a. More; fallen b. More; risen c. Less; fallen d. Less; risen

103. The concept of opportunity cost:

a. Suggests a major increase in public health-care spending means an expansion in other


areas will be harder to achieve.
b. Suggests all our wants can be achieved.
c. Would be irrelevant if we eliminated poverty.
d. Is relevant only for a capitalist economy like the United States.

104. Inflation is:

a. A decrease in the overall level of economic activity.


b. An increase in the overall level of economic activity.
c. An increase in the overall price level.
d. A decrease in the overall price level.

105. A recession is:

a. A period of declining unemployment. b. A period of declining prices


c. A period during which aggregate output declines d. A period of very rapidly declining
prices.

106. The statement "The unemployment rate for teens is higher than that for adults" is

a. A normative statement. b. A political statement.


c. An ethical statement. d. A positive statement

107. Which of the following is a positive statement?

a. Low rents will restrict the supply of housing.


b. Housing costs too much.
c. Low rents are good because they make apartments more affordable.
d. Owners of apartment buildings ought to be free to charge whatever rent they want.
108. A normative statement is

a. One that does not use the ceteris paribus clause. b. About what ought to be.
c. Always true. d. About what is.

109. The task of economic science is to discover ________ that are consistent with ________.

a. Positive statements; normative statements


b. Positive statements; what we observe
c. Normative statements; positive statements
d. Ways to make money; the law

110. Economic models

a. Are better if they include most of the detail of the real economy.
b. Rely on simplification.
c. Do not address questions about the economy.
d. Make no assumptions that have not been prove

111. An economic theory is


a. A generalization that summarizes what we understand about economic choices.
b. A positive statement that cannot use the ceteris paribus clause.
c. Usually more complex than the real world.
d. Always a mathematical, or nonverbal, model.

112. Ceteris paribus is the Latin expression for

a. A statement about the way the economic world ought to be.


b. An expression that means "other things being equal."
c. The (false) statement that what is true of the parts is true of the whole or what is true of the
whole is true of the parts.
d. The error of reasoning that a first event causes a second event because the first event
occurred before the second event.

113. Opportunity cost means

a. The accounting cost minus the marginal benefit. b. The highest-valued alternative forgone.
c. The monetary costs of an activity. d. The accounting cost minus the marginal cost

114. Economics is best defined as the study of how people, businesses, governments, and societies

a. Make choices to cope with scarcity. b. Attain wealth.


c. Choose abundance over scarcity. d. Use their infinite resources.

115. Scarcity requires that people must

a. Trade. b. Compete. c. Cooperate. d. Make choices


116. Microeconomics focuses on all of the following EXCEPT

a. The effect of increasing the money supply on inflation.


b. The purchasing decisions that an individual consumer makes.
c. The effect of an increase in the tax on cigarettes on cigarette sales.
d. The hiring decisions that a business makes.

117. Entrepreneurs do all of the following EXCEPT

a. Bear risk from business decisions.


b. Own all the other resources.
c. Come up with new ideas about what, how, when and where to produce.
d. Organize labor, land, and capital.

118. Principal Agent Problem is related to…….

a. Managers and Owner s b. Suppliers and Buyers


c. Producers and Sellers d. LIC Agents

119. Invisible Hand theory was described by …….

a. Robert Anthony b. Adam Smith c. Amartya Sen d. C.K. Pralhad

120. PV in Unit number one stands for ………

a. Postal Volume b. Past Value c. Programmable Value d. Present Value

121. Which of the following factor is important for demand along other factors?

a. Selling Power b. Ability to Buy c. Product Development d. New product Launch


Core course: EC02 B02 Micro economics II
B A Economics II semester

Multiple Choice Questions and Answers

1. Cost functions are derived from

a) demand functions, b) production functions, c) supply functions

2. The U shape of the average total cost curve reflects

a) LDMU, b) The Law Of Variable Proportions, c) Consumer’s


Surplus

3. The total fixed cost is a : a) horizontal straight line b) vertical c)


hyperbola

4. The shape of average fixed cost is

a) horizontal straight line b) vertical c) rectangular hyperbola

5. The shape of TVC and TC are:

a) rectangular hyperbola b) inverse ‘S’ shape c) horizontal


straight line

6. The inverse ‘S’ shape of the TVC and TC are due to

a) LDMU b) Law of Variable Proportionsc) MRS

7. The MC curve cuts the AC curve at

a) The maximum point b) The initial Point, c) The minimum Point

8. The minimum point of ATC is at ……………. position of the minimum


point of AVC: a) right b) left c) same

9. If the long run cost curve shifts down wards it is an indication of

a) technological progress b) lower factor prices c) both of


these

10. The U shape of the LAC reflects


a) Law of Variable proportions b) Laws of returns to scale c)
none of these

11. The long run cost curve in the traditional theory is

a) Envelops curve b) Planning curve c) Both of these

12. In the modern theory of cost, the short run average variable cost is

a) saucer shaped b) U shaped c) none of these

13. The short run average variable cost is saucer shaped because of the
presence of

a) excess capacity b) reserve capacity c) none of these)

14. Price discrimination is an essential feature of

a) Perfect competition b) Oligopoly c) Duopoly d)


monopoly

15. Under monopoly the slope of AR curve is:

a) Upward sloping b) downward sloping c) horizontal d)


None of these

16. Monopoly market is :

a) Single seller market b) single buyer market c) single buyer and


seller d) None

17. In a monopsony market there is:

a) Single seller b) single buyer c) Two sellers d) two buyers

18. Third degree price discrimination occurs when the monopolist charges
different prices for the same commodity in different

a) Markets b) places c) continents d) countries

19. Price discrimination is possible:

a) Under any market form b) only under


monopoly c) Only under monopolistic
completion d) Only in perfect competition

20. Monopolist maximizes profit at the point where


a) MC = AC b) MC = MR c) AC = AR d) MR = AR

21. At the point of equilibrium of a monopolist MC cuts MR curve

a) From below b) from above c) at point of equality of AC and


AR d) None

22. A multiplant monopolist maximizes his profit at the point where:

a) MR = MC1 b) MR = MC2 c) MR1 = MR2 d) MR = MC1


= MC2

23. Lerner Index is a measure of:

a) Elasticity of demand b) Monopoly power c) Inequality d)


None

24. For a firm with monopoly power

a) Price equals MC b) Price is less than MC c) Price exceeds MC


d) None

25. Railways is an example of :

a) Simple monopoly b) differentiated monopoly


c) Natural monopoly d) Monopsony

26. A market with only one buyer and one seller is called

a) Oligopsony b) monopsony c) Bilateral monopoly d) None

27. Bilateral monopoly is a market with

a) Single buyer b) Single seller c) Single buyer and single seller


d) Few buyers and sellers

28. The dual pricing system of charging high price during peak time and
low price during of peak time is called

a) Double pricing b) Dual pricing c) kinked pricing d) peak


load pricing

29. Selling more than one product at a single price

a) Dumping b) Bundling c) Discounting d) Off loading

30. An international price discrimination


a) Dumping b) Bundling c) Discounting d) Off loading

31.The marker structure which have very large number of sellers selling Identical
products is called
a) Perfect competition
b) Monopoly
c) Monopolistic competition
d) Oligopoly
32.The marker structure with Perfect mobility of factors and products is called
a) Perfect competition
b) Monopoly
c) Monopolistic competition
d) Oligopoly
33.The marker structure with Perfect knowledge is called
a) Perfect competition
b) Monopoly
c) Monopolistic competition
d) Oligopoly
34.The condition of perfect competition is fulfilled when
a) Sellers are large in number
b) Buyers are large in number
c) Commodity produced is homogenous
d) All the above
35.The following are conditions of perfect competition except
a) Sellers are large in number
b) Buyers are large in number
c) Commodity produced is homogenous
d) Commodity produced is differentiated
36.The following are conditions of perfect competition except
a) Strong barriers to entry
b) Sellers are large in number
c) Commodity produced is homogenous
d) Buyers are large in number
37.The following are conditions of perfect competition except
a) Sellers are large in number
b) Single buyer
c) Commodity produced is homogenous
d) Freedom to Entry and exit
38.The condition of short run equilibrium under perfect competition is
a) MC=MR
b) AC=MR
c) AC=AR
d) AR=Selling cost
39.The large number of firms producing the same commodity ensure that the
individual firm has no control over
a) Price of the commodity
b) The quantity of the commodity
c) Both of the above
d) None of the above
40.Individual firm has no control on the price of the commodity in the market is a
condition of
a) Perfect competition
b) Monopoly
c) Monopolistic competition
d) Bilateral monopoly
41.In a Perfect competitive market
a) Firm is the price giver and the industry is a price taker
b) Firm is the price taker and the industry is a price giver
c) Both are price makers
d) Both are price takers
42.One of the essential conditions of perfect competition is
a) Product Differentiation
b) Multiplicity of prices for identical product at any one time
c) Many sellers and few buyers
d) Only one price for identical goods at any one time
43.Under perfect market conditions the individual firm in the industry has
------------------- control over the price of the product.
a) Some
b) Full
c) No
d) None of the above
44.The condition of short run equilibrium under perfect competition is
a) MC=MR
b) MC cuts MR from below
c) MC is rising when it cuts AR
d) All the above
45.The marker structure which have large number of sellers selling differentiated
product is called
a) Perfect competition
b) Monopoly
c) Monopolistic competition
d) Oligopoly
46.The marker structure in which number of sellers is small with
interdependence is called
a) Perfect competition
b) Monopoly
c) Monopolistic competition
d) Oligopoly
47.The cost incurred to alter the position or slope of demand curve is known as
a) Marginal cost
b) Selling cost
c) Alternate cost
d) Additional cost

48.The condition of short run equilibrium under monopolistic competition is


a) MC=MR
b) AC=MR
c) AC=AR
d) AR=MR
49.Kinked demand curve explain which of the following features of oligopoly
a) Selling cost
b) Price rigidity
c) Non price competition
d) Product differentiation

50.Demand curve of a firm under monopolistic competition is


a) Parallel to X axis
b) Parallel to Y axis
c) Downward slopping
d) Upward slopping
51.Which of the following is not a feature of monopolistic competition?
a) Homogenous product
b) Large number of firms
c) Freedom to entry and exit
d) Differentiated product
52.In the short run, a monopolistically competitive firm can have
a) Abnormal profit
b) loss
c) Normal profit
d) Any of the above are possible
53.Selling cost is a feature of
a) Monopolistic competition
b) Perfect competition
c) Monopoly
d) Bilateral monopoly
54.Refrigerator company is an example of
a) Oligopoly
b) Perfect competition
c) Monopoly
d) Bilateral monopoly
55.Which of the following is a form collusive oligopoly
a) Bilateral monopoly
b) Monopoly
c) cartel
d) Kinked Oligopoly

56.OPEC is an example of
a) Bilateral monopoly
b) Monopoly
c) cartel
d) Kinked Oligopoly
57.In the long run, which of the following is applicable to a firm under
monopolistic competition
a) AR = AC
b) AR > AC
c) AR < AC
d) AR = MC
58. A discriminating monopolist will charge a higher price from which
group of customers?
a. Group with more elastic
b. Group with less elastic
c. Group with Unitary Elastic
d. Group with Infinitely Elastic

59. Perfect price discrimination means that every customer ____________


a. buys the same amount
b. pays the same price
c. contributes the same revenue
d. pays what she thinks the product is worth
60. Supernormal profit refers to
a) High proportion of net profit
b) Minimum necessary profit to induce an entrepreneur to remain in
business
c) Unexpectedly high Profit
d) Residual surplus

Answers:

1) a 2) b 3) a 4) c 5) b 6) b 7) c 8) a 9) c
10) b

11) c 12) a 13) c 14) d 15) b 16) a 17) b 18)


a 19) b 20) b

21) a 22) d 23) b 24) c 25) c 26) c 27) c 28)


d 29) b 30) a

31) a 32) a 33) a 34) d 35) d 36) a 37) b


38) a 39) c 40) a
41) d 42) d 43) c 44) d 45) c 46) d 47) b 48)
a 49) b 50) c

51) a 52) d 53) a 54) a 55) c 56) c 57) a


58) b 59) d 60) a
C04-Fundamentals of business economics

Sample Exam Paper

Question 1

The recession phase of the trade cycle

A. Is often caused by excessive consumer expenditure.


B. Is normally characterised by accelerating inflation.
C. Is most prolonged when the country has high levels of imports.
D. Is usually caused by falling aggregate monetary demand.

Question 2

Supply side policy is designed to

A. Raise the level of aggregate monetary demand in the economy.


B. Manage the money supply in the economy.
C. Improve the ability of the economy to produce goods and services.
D. Reduce unemployment by limiting the supply of labour.

Question 3

The effects of low real interest rates include all of the following EXCEPT which ONE?

A. Credit-based sales will tend to be high.


B. Nominal costs of borrowing will always be low.
C. Business activity will tend to increase.
D. Investment will be encouraged.

Question 4

In the circular flow model of the economy, the level of national income will always reach
equilibrium because:

A. Injections and withdrawals are always equal.


B. Withdrawals are a function of the level of income.
C. Governments will change taxes and expenditure to ensure equilibrium.
D. Expenditure equals income.

Question 5

All of the following government policies would tend to raise national income over time
EXCEPT which ONE?

A. Increased expenditure on the economic infrastructure.


B. Tax cuts to encourage higher demand from consumers.
C. Policies to encourage the training of labour.
D. Financial incentives to encourage personal and corporate saving.

Updated: Oct 2013 1


C04-Fundamentals of business economics

Question 6

Which ONE of the following would lead to a fall in the value of the multiplier?

A. A decrease in the marginal propensity to consume.


B. A fall in the level of public expenditure.
C. Consumers saving a lower proportion of their income.
D. A decrease in the marginal propensity to import.

Question 7

Which ONE of the following would lead a country’s balance of payments current account to
move towards a surplus?

A. A rise in commodity imports.


B. An inflow of foreign capital into the economy.
C. An increase in foreign tourism into the country.
D. An increase in government tax receipts.

Question 8

All of the following are benefits for a business from depreciation (reduction) in the rate of
exchange for the country’s currency EXCEPT which ONE?

A. The business could charge lower prices for its exports.


B. Imported raw materials used by the business would be cheaper.
C. The business could raise profit margins on exports without losing sales.
D. In its home market, the business would face reduced competition from imports.

Question 9

A slowdown in economic growth in the US would be transmitted to the rest of the world by all
EXCEPT which ONE of the following processes?

A. A fall in US interest rates.


B. A fall in the US demand for imports.
C. A decline in the level of US capital flows to the rest of the world.
D. Falling US stock market prices leading to similar falls in other countries.

Question 10

The balance of payments accounts are defined as:

A. The difference between the government’s receipts and its expenditure over the
period of a year.
B. The difference between the exports of goods and services and imports of goods and
services over the period of a year.
C. The surplus or deficit on a country’s international trade over a given period.
D. A statement of the economic transactions between residents of a country and the
rest of the world over a given period.

Updated: Oct 2013 2


C04-Fundamentals of business economics

Question 11

All of the following are disadvantages of inflation EXCEPT which ONE?

A. It redistributes wealth from debtors to creditors.


B. It reduces international competitiveness.
C. Market price signals are distorted.
D. Fixed income earners experience a fall in real income.

Question 12

An expansionary fiscal policy would be most likely to reduce unemployment if the country
had:

A. A high marginal propensity to import


B. A low marginal propensity to save.
C. A high marginal tax rate.
D. A low marginal propensity to consume.

Question 13

If a country were to join a free trade area, its business sector would gain because:

A. It could get tariff-free raw materials from countries inside the free trade area.
B. There would be exchange rate stability between the member countries.
C. Prices of competitors from countries not in the free trade area would rise.
D. There would be fewer barriers to establishing subsidiaries in other countries in the
free trade area.

Question 14

All of the following are reasons for trans-national companies locating production of a good in
more than one country except one. Which ONE is the EXCEPTION?

A. The existence of trade barriers.


B. Significant transport costs.
C. Economies of scale in production.
D. Differences in demand conditions between countries.

Question 15

All of the following are characteristics of a common market EXCEPT which ONE?

A. Free trade in goods and services among member states.


B. Common levels of direct taxation.
C. Free movement of factors of production between member states.
D. A common external tariff.

Updated: Oct 2013 3


C04-Fundamentals of business economics

Question 16

All of the following are features of globalisation EXCEPT which ONE?

A. Rising trade ratios for countries.


B. Increased international capital flows.
C. Improved terms of trade for all countries.
D. Reduced barriers to international factor movements.

Question 17

The process of globalisation leads to all of the following EXCEPT which ONE?

A. Increasing foreign competition in domestic markets.


B. Decreasing interdependence of national economies.
C. Greater international division of labour.
D. Rising levels of international trade relative to national income.

Question 18

Multinational companies locate production in more than one country for all of the following
reasons EXCEPT which ONE?

A. The existence of trade barriers.


B. High transport costs.
C. Capital is internationally immobile.
D. To increase market share.

Question 19

Which of the following is the most appropriate for indicating long term shareholder wealth?

A. Net present value


B. Rate of return on capital
C. Earnings per share
D. Profit

Question 20

Which of the following groups are stakeholders in a business organisation?

(i) Employees
(ii) Shareholders
(iii) Management
(iv) Customer’s
(v) Suppliers

A. (i), (ii) and (iii) only


B. (i), (ii), (iii) and (iv) only
C. (ii) only
D. All of them

Updated: Oct 2013 4


C04-Fundamentals of business economics

Question 21

Which of the following issues are true of objectives in non-for-profit organisations?

(i) Conflict between the goals of different stakeholders


(ii) A complex set of objectives
(iii) Difficulty in setting measures for their performance
(iv) Those that benefit from the good or service offered may not pay for it

A. (i), (ii) and (iii) only


B. (ii), (iii) and (iv) only
C. (i), (iii) and (iv) only
D. All of them

Question 22

Not-for-profit organisations have all the following features except one, which ONE is the
EXCEPTION?

A. They need to be efficiently run


B. They often have a range of organisation objectives
C. Their primary aim is to make financial surpluses
D. They have more than one group of stakeholders

Question 23

All but one of the following are characteristics of all organisations. Which ONE is the not a
characteristic of all organisations?

A. The pursuit of objectives


B. The need to measure performance
C. They need to compete with other organisations
D. They have a range of stakeholders

Question 24

X plc has 1 million shares issued. It has an EPS figure of $0.10. X is planning a new
investment project which is expected to raise profits after tax by $50,000. This project will be
financed by an issue of an additional 250,000 shares. After the project is completed, the new
EPS will be:

A. $0.2
B. $0.12
C. $0.08
D. $0.15

Question 25

Which of the following might lead to a rise in earnings per share for a company?

A. A rise in corporate tax

Updated: Oct 2013 5


C04-Fundamentals of business economics

B. An issue of new shares


C. A fall in its rate of return on capital employed
D. A fall interest rates

Question 26

Which of the following are reasons why future income is usually valued less than current
income?

(i) Inflation may reduce the value of future income


(ii) Future income flows are riskier than current flows
(iii) The lender must delay the pleasure from the consumption that the money could buy
now
(iv) Higher interest rates

A. (i) and (ii) only


B. (i) and (iii) only
C. (ii), (iii) and (iv) only
D. (i), (ii) and (iii) only

Question 27

If, in the long run, a business doubles all the inputs it uses, but total physical output less than
doubles, the business is experiencing:

A. Diminishing returns.
B. Decreasing returns to scale.
C. Price elasticity of demand of less than 1
D. Market saturation

Question 28

The minimum condition for a business to continue to operate in the short run is that:

A. Its revenue is sufficient to cover variable costs.


B. Its revenue is at least equal to its total costs.
C. Its marginal revenue is equal to its marginal cost.
D. It is making normal profits.

Question 29

All of the following are internal economies of scale for a firm EXCEPT which ONE?

A. The firm is able to reduce administration costs per unit of output when it opens a
second production plant.
B. The firm can buy raw materials from other firms at lower prices when it buys in bulk.
C. TV advertising costs can be spread over a larger output.
D. Training costs are reduced when the firm can draw on the development of pool of
skilled labour in the region due to colleges setting up training courses for the industry.

Updated: Oct 2013 6


C04-Fundamentals of business economics

Question 30

In order to remain in business in the short run, a firm’s revenue must be sufficient to cover:

A. Its fixed costs


B. Its variable costs
C. Its total costs
D. Its total costs minus normal profit

Question 31

The initial fall for a business in the short run average cost per unit of its output is the result
of:

A. Diminishing returns to a fixed factor


B. Diseconomies of scale
C. The spreading of fixed cost over a larger output
D. A fall in total variable costs

Question 32

The profit-maximising output will always be where:

A. Average cost = marginal revenue.


B. Marginal cost = marginal revenue.
C. Average cost = average revenue.
D. Marginal cost = average revenue.

Question 33

A business has the following costs:

Output Fixed Cost Total variable cost


10 $100 $150
11 $100 $164
12 $100 $176

The average cost of production for output 11 is $______

Question 34

The breakeven output for a firm will be where:

A. Average cost is equal to average revenue


B. Average cost is at a minimum
C. Average revenue starts to fall
D. Average cost starts to rise

Updated: Oct 2013 7


C04-Fundamentals of business economics

Question 35

The profit maximising level of output for a business is where:

A. Average costs are lowest


B. The difference between total cost and total revenue is greatest
C. Where average revenue is highest
D. Where average revenue is equal to average cost

Question 36

The optimum level of output for a business is where:

A. Profits are maximised


B. Revenue is maximised
C. Marginal cost is minimised
D. Average cost is minimised

Question 37

All of the following are example of the sources of internal economies of scale for a business
except one. Which ONE is the EXCEPTION?

A. Lower external financing costs for a larger firm


B. Greater specialisation of tasks in its labour force
C. Lower supply costs resulting from its bulk buying
D. Lower production costs resulting from long run technical change in the economy

Question 38

Which ONE of the following will tend to make the demand for a company’s product LESS
price elastic?

A. A fall in consumer incomes.


B. A rise in the price of complementary goods.
C. A fall in the number of substitute goods.
D. A lower price for the good.

Question 39

In a market economy, prices perform all of the following functions EXCEPT which ONE?

A. A means of allocating resources between competing uses.


B. A means of ensuring a fair distribution of incomes.
C. A signal to consumers.
D. A signal to producers.

Question 40

Which ONE of the following will produce the largest fluctuations in a market price?

A. Large shifts in supply with price elastic demand.

Updated: Oct 2013 8


C04-Fundamentals of business economics

B. Large shifts in supply with price inelastic demand.


C. Large shifts in supply with perfectly price elastic demand.
D. Small shifts in supply with price elastic demand.

Question 41

If the demand curves for Good A shifts to the left when the price of Good B rises, we may
conclude that:

A. The goods are substitutes.


B. Good A is an inferior good.
C. The goods are complements.
D. The demand for Good A is price elastic.

Question 42

Which ONE of the following would lead the demand curve for a good to shift to the right?

A. A rise in consumer income where the good is a normal good.


B. A decrease in the supply of a complementary good.
C. An increase in the supply of a substitute good.
D. A fall in the price of the good.

Question 43

If a business currently sells 10,000 units of its product each month at $10 each unit and the
demand for its product has a price elasticity of -2·5, a rise in the price of the product to $11
will?

A. Raise total revenue by $7,250.


B. Reduce total revenue by $17,500.
C. Reduce total revenue by $25,000.
D. Raise total revenue by $37,500.

Question 44

If the government imposed a price for a good that was above the equilibrium price, the
consequence would be:

A. A contraction of demand, an increase in supply and a market surplus


B. A decrease in demand, an extension of supply and a market surplus
C. A contraction in demand, an extension in supply and a market surplus
D. A rise in supply, a fall in demand and a market shortage

Question 45

If the government imposes a maximum price for a good that is below the equilibrium price,
the resulting market shortage will be greatest when:

A. The demand is price elastic and the supply is price inelastic


B. The demand is price elastic and the supply is price elastic
C. The demand is price inelastic and the supply is price elastic
D. The demand is price inelastic and the supply is price inelastic

Updated: Oct 2013 9


C04-Fundamentals of business economics

Question 46

All of the following would shift the supply curve for a product to the right except one. Which
ONE is the EXCEPTION?

A. A government subsidy
B. An improvement in production techniques
C. Lower input prices
D. An indirect tax on the product

Question 47

If the production of a good involves an external social cost, the appropriate policy for the
government is to:

A. Take the industry into state ownership


B. Impose an indirect tax on the good
C. Impose a higher rate tax on the profits of the producers
D. Provide a subsidy for the consumers of the product

Question 48

A government may prevent horizontal mergers in an industry because:

A. By controlling sources of supply, the merged firms can prevent the entry of new firms
into the industry
B. There will be a lack of synergy between the merging companies
C. The merged firms will be unable to reduce costs
D. Consumers may suffer if the merged firm achieve market dominance

Question 49

Which ONE of the following statements is true?

A. A legal maximum price for a good always results in a shortage of the good.
B. A price fall for a good will lead to an increase in demand for that good.
C. A price fall for a good will always lead to a contraction in supply of the good.
D. An excess supply of a good arises when a legal minimum price is set below
equilibrium price.

Question 50

Which ONE of the following does NOT restrict the number of firms in an industry?

A. Low levels of product differentiation.


B. Significant economies of scale.
C. Barriers to entry.
D. The use of capital-intensive technology in the industry.

Question 51

Updated: Oct 2013 10


C04-Fundamentals of business economics

Which ONE of the following is an example of an external social cost?

A. Bad weather reducing the output of the farm sector.


B. Possible illness caused to owners of mobile phones as a result of excessive use.
C. Smoke emissions by a factory causing health problems for nearby residents.
D. Cost increases caused by rising prices of imported raw materials.

Question 52

Which ONE of the following is NOT a potential source of market failure?

A. External costs.
B. External benefits.
C. An unequal income distribution.
D. The existence of monopolies.

Question 53

All of the following factors will lead to an imperfect allocation of resources EXCEPT which
ONE?

A. There are some production costs that are not borne by the producer.
B. Consumers only wish to buy a limited amount of the product.
C. There are spill over benefits in consumption.
D. Consumers have limited knowledge of market prices.

Question 54

Whenever government intervention prevents prices from reaching their equilibrium level, the
result will always include ALL of the following EXCEPT which ONE?

A. Shortages or surpluses.
B. Demand and supply not equal.
C. Reduced profits for producers.
D. Resources not allocated by price.

Question 55

The economic welfare case for governments increasing taxes on petrol to raise its real price
is that:

A. Oil is a scarce resource.


B. It would reduce the imports of oil.
C. There is a large demand for petrol.
D. Petrol consumption involves external social costs.

Question 56

Which ONE of the following would tend to increase the degree of monopoly power of a
company?

A. The ending of one of its patents.

Updated: Oct 2013 11


C04-Fundamentals of business economics

B. An increase in excess profits.


C. A fall in the cross price elasticity of demand for its product.
D. Diversification into a wider range of products.

Question 57

The main function of the money market is to:

A. Enable businesses and government to obtain liquidity.


B. Encourage saving.
C. Permit the efficient buying and selling of shares.
D. Deal in credit instruments of more than one year maturity.

Question 58

Which ONE of the following does NOT normally form part of the equity capital market?

A. Central bank.
B. Pension funds.
C. Retail banks.
D. Venture capitalists.

Question 59

All of the following statements about the monetary system are true EXCEPT which ONE?

A. Financial intermediation links net savers and net debtors.


B. In developed economies cash and notes form only a very small part of the money
supply.
C. The capital market deals in long term financial assets.
D. The more liquid a financial asset is, the higher is the expected rate of return.

Question 60

In order to finance an excess of expenditure over taxation receipts, a government could:

A. Reduce its current consumption expenditure


B. Issue government bonds
C. Raise direct taxes
D. Run an overdraft on its account at the World Bank

Question 61

All the following are sources of a short run lack of financial synchronisation for a business
except one. Which ONE is the EXCEPTION?

A. The need for working capital


B. Early receipt of payment of goods sold
C. The financing of the purchase of new capital equipment
D. A requirement to pay an element of end of year tax early

Updated: Oct 2013 12


C04-Fundamentals of business economics

Question 62

Which one of the following is not usually a source of long-term finance?

A. Bank overdrafts
B. Debentures
C. Preference shares
D. Mortgages

Question 63

Equity finance in high risk enterprises is known as:

A. Venture Capital
B. Working Capital
C. Debentures
D. Gearing

Question 64

In their role as financial intermediaries, banks fulfil all of the following functions except one.
Which ONE is the EXCEPTION?

A. Provide a transactions mechanism


B. Aggregation
C. Financial synchronisation
D. Maturity transformation

Question 65

A central bank has all of the following functions except one. Which ONE is the EXCEPTION?

A. Acting as banker to the banks


B. Providing very long term finance for investment
C. Acting as banker to the government
D. Lender of the last resort

Question 66

A bond has the following features:

Nominal Value $1000


Coupon rate 4%
Current Market Value $1250

The running yield on this bond is _____%

Question 67

The following data refer to X plc:

Issued shared 1m

Updated: Oct 2013 13


C04-Fundamentals of business economics

Current share price $1.20


Dividend payment $0.15 per share

The net dividend yield for X plc is _____%

Question 68

The main determinant of the cost of borrowing is:

A. Maturity
B. Base rate
C. Size of loan
D. Risk

Question 69

A commercial bank has the following data:

Required reserve assets ratio 12.5%


Additional cash deposit $1000

Total bank deposits will ultimately rise by $_____

Question 70

Which of the following is not provided by stock markets? A market for:

A. Large transactions in foreign currencies


B. Shares in publicly quoted companies
C. Long term government bonds
D. New share issues

Question 71

All the following contributed to the bank crisis of 2008/09 except one. Which ONE is the
EXCEPTION?

A. Problem in the US mortgage market


B. The over use of credit default swaps
C. Under regulation of the banking system by authorities
D. Banks holding too much capital relative to their liabilities

Question 72

The forward exchange market is an example of:

A. A perfect market
B. Reinsurance
C. A future market
D. Underwriting

Updated: Oct 2013 14


C04-Fundamentals of business economics

Question 73

All of the following would tend to raise the exchange rate (appreciate) for a country’s
currency except one. Which ONE is the EXCEPTION?

A. A fall in the volume of imports


B. A risen in foreign investment in the country
C. A fall in domestic interest rates
D. A rise in the country’s invisible earnings

Question 74

Which one of the following is an advantage for a country adopting a flexible exchange rate
system regime?

A. It provides certainty for organisations engaged in international trade


B. It eliminates transaction costs
C. Monetary policy can be used to manage to exchange rate
D. It reduces the need for central banks to keep reserves of foreign exchange

Question 75

Which one of the following is not a benefit to a country from joining a single currency area?

A. Reduced transaction costs


B. Lower interest rate
C. Reduced exchange rate uncertainty
D. Increased price transparency

Updated: Oct 2013 15


C04-Fundamentals of business economics

C04 – Answers

Question Answer Question Answer


1 D 44 C
2 C 45 B
3 B 46 D
4 B 47 B
5 D 48 D
6 A 49 C
7 C 50 A
8 B 51 C
9 A 52 C
10 D 53 B
11 A 54 D
12 A 55 C
13 A 56 C
14 C 57 A
15 B 58 D
16 C 59 D
17 B 60 B
18 D 61 C
19 C 62 A
20 D 63 A
21 D 64 A
22 C 65 B
23 C 66 3.2%
24 B 67 12.5%
25 D 68 D
26 D 69 $8,000
27 B 70 A
28 A 71 D
29 D 72 C
30 B 73 C
31 C 74 D
32 B 75 B
33 $24
34 A
35 B
36 A
37 D
38 C
39 B
40 B
41 C
42 A
43 B

Updated: Oct 2013 16


C04-Fundamentals of business economics

C04 – Explanations

1. (answer: D) A general slowdown in economic activity is termed an economic


recession. This is caused by fall in aggregate monetary demand [C+I+G+ (X-M)].
An economic recession indicated by:
- low or negative growth of GDP
- low business confidence, investment spending and reduced business profits
- low household income and widespread drop in consumer spending (A cannot be
correct)
- falling inflation rate (option B cannot be correct)
- increased bankruptcies and unemployment rates.

2. (answer: C) Supply side policy aimed at increasing the long term productive capacity
of an economy. This is based on the macroeconomic argument that economic
growth can be most effectively created by lowering barriers for people to produce
(supply) goods and services, such as lowering income tax and capital gains tax rates,
and by allowing greater flexibility by reducing regulation. As a result, consumers will
benefit from a greater supply of goods and services at lower prices.

3. (answer: B) Real interest rate = Nominal interest rate – Rate of Inflation


Therefore, real interest rate will be low when nominal interest rate is high with high
rate of inflation.
In times of low real interest rates, credit-based sales (option A) and the level
investment (option D) tend to be higher due to low cost of borrowing. This in turn
will enhance the general business activity in an economy (option C).

4. (answer: B) The circular flow of income and expenditure model describes the
reciprocal circulation of income between inter-dependant entities of firms
(producers) and households (consumers) in an economy. With government and
international trade (four-sector model), injections into an economy are government
spending (G), investments (I) and exports (X), whereas the withdrawals from
economy are taxation (T), savings (S) and imports (M. National income will reach an
equilibrium when G + I + X = T + S + M.

5. (answer: D) National income is the total value a country’s final output of all new
goods and services produced in one year. National income can be increased by
enhancing the productive capacity (increase in long run aggregate supply - LRAS) and
/ or by increasing aggregate monetary demand (AMD). Better infrastructure such as
transport, information technology etc (option A) and trained workforce (option C)
will enhance the productive capacity of an economy, shifting the long run aggregate

Updated: Oct 2013 17


C04-Fundamentals of business economics

supply curve outwards. Increased consumer expenditure (option B) will increase the
level of AMD. With more savings (option D) there will be less consumption, thus the
level of AMD will fall. [AMD = C+I+G+ (X-M)].

6. (answer: A) According to Keynesian economic theory, any injection into the economy
via investment capital (I), government spending (G) or exports (X) will result in a
proportional increase in overall national income. Decrease in MPC (marginal
propensity to consume) as in option A means, increase in the MPS (marginal
propensity to save). According to the below formula, decrease in MPS will lead to a
fall in the value of multiplier.

Multiplier = 1 / MPS + 1 / MRT + 1 / MPM

7. (answer: C) Balance of payments (BoP) current account records inflows and outflows
of funds into / out of a given economy due to international trade of goods and
services. If inflows exceed outflows, it will be in surplus. It will be in deficit if there is
a net outflow. Increase in foreign tourism into the country (option C) will result in
inflows of funds. Capital transactions between a given county and the rest of the
world (option B) are recorded in the BoP capital account, hence no impact on the
BoP current account. Government tax receipts (option D) is flow of funds within an
economy, and hence no impact on BOP.

8. (answer B) Depreciation of a country’s currency makes imports expensive in terms of


domestic currency (options B and D). On the other hand, depreciation makes exports
cheaper in foreign currency, thus businesses could raise price (increased profit
margins) in domestic currencies but the price of the exports in foreign currency will
remain unchanged (options A and C).

9. (answer A) slowdown in economic growth in US means that the US economy has


entered into an economic recession, caused by fall in aggregate demand. Reduction
in household income and consumer expenditure means demand for imports into the
US falls (option B). Low level of business confidence during recession will reduce
amount of US investments in other parts of the world (option C), and which in turn
will reflect in the stock market prices in other countries (option D). Interest rates
may be reduced (option A) in the US as a measure to help recovery.

10. (answer D) Refer explanation to Question 7.

11. (answer A) Inflation occurs when the general price level increase in an economy, and
as a result the buying power of money declines. This means, borrowers / debtors

Updated: Oct 2013 18


C04-Fundamentals of business economics

benefit at the expense of lenders / creditors which is technically interpreted as


‘redistribution of wealth from creditors to debtors (and not from debtors to
creditors, as in option A).

12. (answer A) an expansionary fiscal policy involves reducing tax and / or increasing
government expenses, in order to boost the level of aggregate demand. This will
result in higher consumer expenditure and increased demand for goods and services.
Demand for labour is derived from the demand for goods and services, but if the
consumers tend to spend their increased income on imported goods and services, it
will not create employment opportunities within the economy.

13. (answer A) A free-trade area is a trade bloc whose member countries have signed a
free-trade agreement, which eliminates tariffs, import quotas and preferences on
most (if not all) goods and services traded between them. Therefore, firms in a free-
trade area will be able to import raw materials without paying tariff from the
member countries (option A). Exchange rate stability occurs when two or more
countries share a common currency or decides to peg their exchange rates (option
B).

14. (answer C) Trans-national companies register in more than one country or have their
operations in more than one country (e.g. design in Italy, produce in Sri Lanka,
finance / IT in Philippines). The operations are spread in different countries for
variety of reasons including options A, B and D. To obtain economies of scale in
production (in order to reduce the long run average cost), firms need to expand the
output, and need not necessarily operate from different countries.

15. (answer B) Free-trade area (refer Question 13) is also referred to as common market.
The individual countries can decide on taxation independently.

16. (answer C) Terms of trade ToT) is the ratio of quantities of domestic goods that a
country must give up to obtain a unit of imported goods. A rise in the prices of
exports with the prices of imports unchanged indicates a rise in the terms of trade
because it will now take fewer exports to purchase the same quantity of imports.
However, globalisation will not lead to improvement in ToT for all trading partners.

17. (answer B) is the process of international integration arising from the interchange of
products, ideas, world views and other aspects of national cultures. Therefore,
globalisation will increase interdependence of national economies.

Updated: Oct 2013 19


C04-Fundamentals of business economics

18. (answer D) a company may continue to produce in the same country, but sell in
more than country to increase its market share.

19. (answer C) An important aspect of EPS is that it ignores the capital required to
generate the earnings (net income). Two companies could generate the same EPS,
but one could do so with less equity (investment) - that company would be more
efficient at using its capital to generate income and, all other things being equal
would be a "better" company. Therefore, EPS is the most appropriate for assessing
the long term wealth of shareholders.

20. (answer D) a stakeholder is a person or an organisation that has interest or concern


in an organisation. Stakeholders can affect or be affected by the organization's
actions, objectives and policies. Therefore, all of the listed parties are stakeholders
of a typical business organisation.

21. (answer D) An organisation that exists for any reason (e.g. educational or charitable
reasons) other than for profit where its owners or trustees do not benefit financially. Surplus
revenues (if any) will be retained to achieve its goals.

22. (answer C) Refer Question 21.

23. (answer C) An organisation is a social entity that has a collective goal and is linked to the
external environment. There are a variety of legal types of organizations, including public
corporations, governments, non-government organisations, armed forces, charities,
partnerships, limited and unlimited liability companies, cooperatives, universities etc, some
of which need not / do not compete with other organisations (e.g. armed forces).

24. (answer B) EPS = Profit after tax and preference dividends/ No. of equity share in
issue.
0.01 x 1,000,000 = Original profit after tax = 100,000
New EPS = (100,000 + 50,000) /(1,000,000+ 250,000) = 0.12

25. (answer D) EPS = Profit / Number of shares (very basic formula). Therefore, reduced
profit due to increase in corporation tax (option A) and issue of new shares (option
B) will reduce the EPS. A fall in interest rate (option D) will increase the profit,
increasing EPS.

26. (answer D) Future income isn’t affected by the change of interest rate, although
lower interest rate may lead to inflation as more people are able to borrow money
and spend more. Higher interest rate, in contrast, is likely to reduce inflation and
will not cause future income to worthless.

Updated: Oct 2013 20


C04-Fundamentals of business economics

27. (answer B) concept of returns to scale identifies a relationship between change in


output as a response of change in input. Decreasing returns to scale occurs when
output changes by a less proportion compared with change of input.

28. (answer A) in the short term, the price needs to cover at least average variable cost.

29. (answer D) Internal economies of scale occur when a firm expands its output and as
a result the long run average cost falls, due to various reasons such as:
- fixed costs being shared amongst more output (option A and C);
- bulk purchasing enables trade discounts (Option B).

External economies of scale occur when an industry expands so that all firms in the
industry are befitted with reduced long run average cost (option D).

30. (answer B) Refer Question 28.

31. (answer C) Short run is a time period that is long enough to change only variable
inputs, and not fixed inputs. Therefore, in the short run, fixed costs remain
unchanged. A larger output in the short term means that the fixed cost can be
spread amongst larger output, hence short run average cost falls (despite, average
variable cost is assumed remain intact)

32. (answer B) Profit maximising output (where MR = MC) of a firm is called the
equilibrium of the firm. Up to equilibrium, MR > MC and therefore with each
additional output produced / sold, total profit increases. Beyond equilibrium, MC >
MR and as a result total profit will start falling with each additional output produced
/ sold. Therefore, for a firm to record the highest profit, it needs to produce an out
where MR = MC.

33. (answer $24)


Total cost of producing 11 units ($100 + $164) = $264
Average cost of producing 11 units ($264 ÷ 11) = $24

34. (answer A) For a firm to break even its total cost (TC) must equal to total revenue
(TCR). Assuming that a firm sells all output it produces, its TR = TC, when average
cost is same as average revenue (price).

35. (answer B) Profit = TR – TC and therefore, highest profit will be recorded when the
gap between TR and TC is the greatest.

Updated: Oct 2013 21


C04-Fundamentals of business economics

36. (answer A) All typical business organisations aim for highest possible profit. The
optimal level of output is therefore the output that generates maximum profit.

37. (answer D) Refer Question 29.

38. (answer C) Price elasticity of demand refers to the level of responsiveness of


quantity demanded to change in price of the same commodity. A product is said to
be less price elastic (inelastic demand) when the demand change by a small
proportion to a given change in price. When a product has less substitutes,
consumers tend to buy almost the same quantity despite high prices (e.g. petrol)

39. (answer B) In a market economy, resources are allocated amongst competing uses
based via market mechanism (interaction of demand and supply). The economic
behaviour of both consumers and producers are influenced by price in a market
economy (e.g. at high prices, consumers tend to demand less). One main weakness
of market mechanism is that it leads to unfair allocation of income. For instance,
only those who have marketable resources may produce and earn an income.

40. (answer B) Obviously, large shift in supply or demand create the largest price
change, hence option D can be opted out. When demand is perfectly price elastic,
any quantity is demanded at a given price, so option C can be opted out. For a given
change in quantity demanded the largest price change can be expected when the
demand is inelastic, hence option A can be opted out.

41. (answer C) leftward shit of demand curve for Good A corresponds to reduction in
demand due any factor other than price of Good A (in this case, due to rise in price
of Good B). An inverse relationship between price and quantity demanded can be
seen for complementary goods (which are usually bought / used together)

42. (answer A) Rightward shift of the demand curve corresponds to increase in demand,
due to any factor other than price of the same commodity. When consumer income
rises, demand for normal goods increase, shown by rightward shift of demand curve.

43. (answer B) PED = % change in quantity demanded


% change in price
- 2.5 = % change in quantity demanded
+10%

Therefore, % change in quantity demanded is (-2.5 X 10%) -25%

New quantity demanded at $11 is (10,000 – 25% of 10,000) is 7,500 units

Updated: Oct 2013 22


C04-Fundamentals of business economics

Total revenue at the price of $11 = $82,500 (7,500 units @ $11)

Total revenue at the price of $10 = $100,000 (10,000 units @ $10)

Change in total revenue = reduction of $17,500 (from $100,000 to $87,500)

44. (answer C) At the equilibrium price quantity demanded = quantity supplied. Increase
in price will contract demand and expand supply, leading to market surplus.

45. (answer B) Market shortage occurs when demand exceeds supply. For a given
reduction in price, a greater proportionate expansion in demand and contraction of
supply can be expected when the demand and supply is price elastic.

46. (answer D) Rightward shift of the supply curve corresponds to increase in supply,
due to any factor other than price of the same commodity. An indirect tax on the
product is shown by leftward shift of the supply curve due to increase in costs.

47. (answer B) An externality is a cost or benefit which results from an activity or transaction
and which affects an otherwise uninvolved party who did not choose to incur that cost or
benefit. An example for an external cost is air pollution caused by manufacturing activities,
that impose health and clean-up costs on the whole society. An indirect tax imposed by the
government will add to the costs of the producer, thereby reduction in activity is
theoretically expected.

48. (answer D) Horizontal merger occurs when two (or more) firms in the same industry
and at the same stage of production integrate (e.g. two car manufacturers). The
merged firm will be able to exploit consumers due the size, so governments tend to
discourage horizontal mergers.

49. (answer C) When the demand is perfectly elastic (horizontal demand curve), at a
legal maximum price (lower than equilibrium price), demand = supply (option A,
watch the word ‘always’!). A fall in price will expand (not increase) the quantity
demanded (option B). At a price below equilibrium price, there will be excess
demand due to expansion of demand and contraction of supply (option D).

50. (answer A) Options B, C, D high level of product differentiation are examples of entry
barriers which makes firms difficult to enter into a given market. #

51. (answer C) Refer Question 47.

Updated: Oct 2013 23


C04-Fundamentals of business economics

52. (answer C) Market failure is a concept within economic theory describing when the
allocation of resources by market mechanism (demand and supply) is inefficient or
imperfect. An unequal income distribution is a consequence (not a source) of market failure.

53. (answer B) Refer Question 52.

54. (answer D) Government interference sometimes enables producers to earn more


profit. E.g. when the demand is inelastic, producers will earn a higher revenue /
profit when a legal minimum price is introduced.

55. (answer C) Refer Question 47.

56. (answer C) Fall in cross price elasticity of demand means the low level of
substitutability of the products, which gives a firm monopoly power. Ending of
patents (option A) and diversification into wider product range (option D) will reduce
monopoly power. Increase in excess profits (option B) is a consequence of monopoly
power, and not a cause.

57. (answer A) Money market assists businesses and governments to raise short term
capital in order to maintain the required level of liquidity.

58. (answer D) Central Bank is the banker to commercial banks, and banker to the
government, and hence is not part of the equity capital market.

59. (answer D) Liquidity is the ability to convert assets into means of exchange without
delay and loss of face value. Illiquid assets earn higher rate of return as
compensation for the loss of liquidity. Therefore, there is an inverse relationship
between level of liquidity and the rate of return.

60. (answer B) The key word to note in the question is ‘finance’ – an immediate measure
to bridge the gap between tax receipts and government expenditure. Options A, C
and D suggest medium or long term measures to address the problem.

61. (answer C) Financing capital equipment belongs to long term (not short term)
financial management.

62. (answer A) Bank overdrafts are usually a tool to support short term lack of financial
synchronisation.

Updated: Oct 2013 24


C04-Fundamentals of business economics

63. (answer A) Venture capitalist specialise in funding high risk business projects /
enterprises, usually for a higher rate of return (e.g. high-tech business start ups)

64. (answer A) Financial intermediaries link surplus units (lenders / creditors) and deficit
units (borrowers / debtors). Banks offer cheque books for current account holders as
a safer / reliable means to engage in financial transactions. Aggregation (option B) is
where banks accept deposits of varied amounts and lend in lump-sum. Financial
synchronisation (option B) takes place when banks lend short term using depositors’
money. Maturity transformation (option D) refers to banks accepting deposits for
varied time periods and lends money for varied time periods.

65. (answer B) Central bank is the banker to the government (not the banker to
commercial organisation).

66. (answer 3.2%) Running yield = annual interest/ current market value
Annual interest = norminal value x coupon rate = 1000 x 4% = 40
Running yield = 40/ 1250 = 3.2%

67. (answer 12.5%) Dividend yield = dividend per share / price per share = 0.15/ 1.20 =
12.5%

68. (answer D) Creditors lend money to those who have a likelihood of returning it at the
end of agreed maturity period. However, debtors may face financial difficulties and
may not repay. Further, inflation reduces the value of money (what money could
buy) – all adds up to risk. Interest is charged to compensate this risk.

69. (answer $8,000) The credit multiplier of the bank is (1/ cash retention ratio = 1/
0.125) 8 times. Therefore, the additional cash deposit of $$1,000 will finally rise by 8
times. This is called ‘credit-creation’ by banks that practice ‘fractional reserve
banking system where a fraction of the deposit (in this case 12.5%) is retained and
the rest is advanced to borrowers with each new deposit it receives.

70. (answer A) Foreign Exchange markets deal with foreign currencies.

71. (answer D) If banks hold more capital than its liabilities, banks will be more
financially stable. The 2008/09 banks crisis was in fact caused by banks not having
enough capital relative to their liabilities.

Updated: Oct 2013 25


C04-Fundamentals of business economics

72. (answer C) In forward exchange markets, lenders (banks) and borrowers (businesses)
contract to buy / sell foreign currency at a future date, at a pre-determined
exchange rate.

73. (answer C) Exchange rate is determined by the market forces – demand for (by
foreigners) and supply of (by residents) a given currency. Fall in domestic interest
rate is expected to reduce the demand for the domestic currency due to low returns.
Reduction in demand for the currency will reduce the exchange rate (depreciation).

74. (answer D) In a flexible exchange rate system, the exchange rate is determined by
the free play of demand for and supply of a given currency, without government
intervention.

75. (answer B) In a single currency area, the member countries collectives decide the
interest rate for all member countries, but there is no assurance that the interest
rate will always be low.

Updated: Oct 2013 26


Chapter 14
Monopoly

14.1 Monopoly and How It Arises

1) A major characteristic of monopoly is


A) a single seller of a product.
B) multiple sellers of a product.
C) two sellers of a product.
D) a few sellers of a product.
E) each firm makes an identical product.
Answer: A
Topic: Monopoly
Skill: Level 1: Definition
Objective: Checkpoint 14.1
Author: JC

2) The good produced by a monopoly


A) has perfect substitutes.
B) has no substitutes at all.
C) has no close substitutes.
D) can be easily duplicated.
E) must be unable to be resold.
Answer: C
Topic: Monopoly
Skill: Level 1: Definition
Objective: Checkpoint 14.1
Author: SA
598 Bade/Parkin œ Foundations of Economics, Third Edition

3) We define a monopoly as a market with


A) one supplier and no barriers to entry.
B) one supplier with barriers to entry.
C) many suppliers with no barriers to entry.
D) many suppliers with barriers to entry.
E) a few suppliers and barriers to entry.
Answer: B
Topic: Monopoly
Skill: Level 1: Definition
Objective: Checkpoint 14.1
Author: WM

4) A monopoly produces a product ____ and there ____ barriers to entry into the market.
A) identical to its many competitors; are
B) with no close substitutes; are
C) identical to its many competitors; are not
D) with no close substitutes; are not
E) slightly different from those of its many competitors; are
Answer: B
Topic: Monopoly
Skill: Level 1: Definition
Objective: Checkpoint 14.1
Author: PH

5) A monopoly
A) is not protected by barriers to entry.
B) produces a good with no close substitutes.
C) faces a downward-sloping demand curve.
D) Both answers A and B are correct.
E) Both answers B and C are correct.
Answer: E
Topic: Monopoly
Skill: Level 2: Using definitions
Objective: Checkpoint 14.1
Author: SA
Chapter 14 Monopoly 599

6) A monopoly
A) must determine the price it will charge.
B) faces extensive competition from firms making close substitutes.
C) cannot price discriminate because such a pricing strategy is illegal in the United States.
D) Both answers A and B are correct.
E) Both answers B and C are correct.
Answer: A
Topic: Monopoly
Skill: Level 2: Using definitions
Objective: Checkpoint 14.1
Author: SA

7) An important characteristic of monopoly is that there are


A) no close substitutes for the good or service sold if the firm price discriminates and
there are many close substitutes for the good or service sold if the firm sets a single
price.
B) a few close substitutes for the good or service sold.
C) no close substitutes for the good or service sold.
D) thousands of close substitutes for the good or service sold.
E) no close substitutes for the good or service sold if the firm sets a single price and there
are many close substitutes for the good or service sold if the firm price discriminates.
Answer: C
Topic: Monopoly
Skill: Level 1: Definition
Objective: Checkpoint 14.1
Author: JC

8) Monopolies arise when there are


A) many substitutes but no barriers to entry.
B) no close substitutes and no barriers to entry.
C) no close substitutes and barriers to entry.
D) many substitutes and barriers to entry.
E) None of the above answers are correct because the existence of a monopoly has
nothing to do with the presence or absence of barriers to entry.
Answer: C
Topic: Monopoly
Skill: Level 2: Using definitions
Objective: Checkpoint 14.1
Author: WM
600 Bade/Parkin œ Foundations of Economics, Third Edition

9) A major characteristic of monopoly is that


A) no barriers to entry exist.
B) the product is identical to that produced by other companies.
C) a barrier to entry keeps out competitors.
D) competition is intense.
E) a few firms compete with each other.
Answer: C
Topic: Monopoly
Skill: Level 1: Definition
Objective: Checkpoint 14.1
Author: JC

10) A monopoly will arise if


A) two out of three of a town's pizzerias go out of business and only one new pizzeria
opens.
B) the town council passes a law granting Nick's Pizza the exclusive right to operate in
that town.
C) Papa Joe's Pizza becomes the largest pizza producer in town and Nick's Pizza stays
small in size.
D) several big pizza chains force several small pizzerias out of business.
E) people decide they like pizza more than before so some pizzeria's gain new customers.
Answer: B
Topic: Monopoly
Skill: Level 2: Using definitions
Objective: Checkpoint 14.1
Author: SA

11) A natural barrier to entry is defined as a barrier that arises because of


A) technology that allows economies of scale over the entire relevant range of output.
B) patents or licenses that exclude others from producing a good or service.
C) many firms producing the good and thereby allowing choice for all consumers.
D) anticompetitive practices by a firm that keep other firms from producing.
E) one firm owning a key natural resource.
Answer: A
Topic: Monopoly, barriers to entry
Skill: Level 1: Definition
Objective: Checkpoint 14.1
Author: WM
Chapter 14 Monopoly 601

12) Natural barriers arise when, over the relevant range of output, there
A) are diseconomies of scale.
B) are constant returns to scale.
C) are several firms who produce at the lowest average cost.
D) are economies of scale.
E) is one firm that owns a key natural resource.
Answer: D
Topic: Monopoly, barriers to entry
Skill: Level 2: Using definitions
Objective: Checkpoint 14.1
Author: WM

13) A natural monopoly


A) arises as a result of legal barriers to entry.
B) occurs when one firm controls a natural resource.
C) arises when one firm can meet the entire market demand at a lower average total cost
than two or more firms.
D) Both answers A and B are correct.
E) Both answers A and C are correct.
Answer: C
Topic: Natural monopoly
Skill: Level 1: Definition
Objective: Checkpoint 14.1
Author: SA

14) For a natural monopoly, economies of scale


A) exist over the long-run average cost curve at least until it crosses the market demand
curve.
B) and diseconomies of scale exist over the long-run average cost curve at least until it
crosses the market demand curve.
C) lead to a legal barrier to entry.
D) as well as constant returns to scale and diseconomies of scale exist over the long-run
average cost curve at least until it crosses the market demand curve..
E) lead to an ownership barrier to entry.
Answer: A
Topic: Natural monopoly
Skill: Level 2: Using definitions
Objective: Checkpoint 14.1
Author: SA
602 Bade/Parkin œ Foundations of Economics, Third Edition

15) A natural monopoly arises when


A) one firm controls the supply of a unique resource.
B) a firm has many small firms that it can control.
C) there are firms which act together as a monopoly.
D) the long-run average cost curve slopes downward until it crosses the demand curve.
E) one firm naturally convinces the government to limit competition in the market.
Answer: D
Topic: Natural monopoly
Skill: Level 2: Using definitions
Objective: Checkpoint 14.1
Author: WM

16) The long-run average cost curve of a natural monopoly


A) is positively sloped until it crosses the demand curve.
B) intersects the demand curve while it is still negative sloped.
C) intersects the demand curve while it is positively sloped.
D) is the natural monopoly's supply curve.
E) is the same as the natural monopoly's demand curve.
Answer: B
Topic: Natural monopoly
Skill: Level 2: Using definitions
Objective: Checkpoint 14.1
Author: SA

17) Which of the following is an example of a natural monopoly?


A) the trademark protecting Gatorade
B) the talents of Tom Hanks
C) the local water company
D) the patent on an Intel processor
E) Debeers' ownership of a large fraction of the world's diamonds.
Answer: C
Topic: Natural monopoly
Skill: Level 2: Using definitions
Objective: Checkpoint 14.1
Author: JC
Chapter 14 Monopoly 603

18) Which of the following is an example of the ownership barrier to entry?


A) a patent
B) ownership of the entire supply of a resource
C) a copyright
D) a government license
E) a public franchise
Answer: B
Topic: Legal barriers to entry
Skill: Level 1: Definition
Objective: Checkpoint 14.1
Author: JC

19) Which of the following is a legal barrier to entry?


i) public franchise
ii) government license
iii) patent
A) iii only
B) i and iii
C) ii and iii
D) i, ii, and iii
E) i and ii
Answer: D
Topic: Legal barriers to entry
Skill: Level 1: Definition
Objective: Checkpoint 14.1
Author: CD

20) DeBeers has a monopoly in the diamond market because it ____.


A) buys up a significant portion of diamond resources.
B) can supply the entire market at a lower price than two or more firms.
C) has been granted a public franchise.
D) has been granted a government license.
E) has been granted a patent.
Answer: A
Topic: Legal barriers to entry
Skill: Level 2: Using definitions
Objective: Checkpoint 14.1
Author: CD
604 Bade/Parkin œ Foundations of Economics, Third Edition

21) The U.S. Postal Service has a monopoly over first-class mail service because
A) the government has granted this agency a public franchise.
B) stamps are copyrighted.
C) stamps are trademarked.
D) stamps are patented.
E) it owns a vital resource, namely all mailboxes.
Answer: A
Topic: Legal barriers to entry
Skill: Level 2: Using definitions
Objective: Checkpoint 14.1
Author: JC

22) The U.S. Postal Service's monopoly on first-class mail service is the result of
A) a natural monopoly.
B) a patent.
C) a public franchise.
D) a government license.
E) an ownership barrier to entry.
Answer: C
Topic: Legal barriers to entry
Skill: Level 2: Using definitions
Objective: Checkpoint 14.1
Author: CD

23) The makers of the movie Finding Nemo have some monopoly power over this film because
the
A) movie is patented.
B) name Nemo is trademarked.
C) movie is protected by copyright law.
D) government has issued the maker of this movie a public franchise.
E) owner has never price discriminated in marketing the movie.
Answer: C
Topic: Legal barriers to entry
Skill: Level 2: Using definitions
Objective: Checkpoint 14.1
Author: JC
Chapter 14 Monopoly 605

24) If Jack invents a machine that can record people's dreams and gets a patent on this machine,
how long will his patent last in the United States?
A) forever
B) 20 years
C) 5 years
D) 100 years
E) 35 years
Answer: B
Topic: Legal barriers to entry
Skill: Level 1: Definition
Objective: Checkpoint 14.1
Author: JC

25) To encourage invention and innovation, the government provides


A) patents.
B) public franchises.
C) government licenses.
D) natural monopolies.
E) easily obtained ownership barriers to entry.
Answer: A
Topic: Legal barriers to entry
Skill: Level 1: Definition
Objective: Checkpoint 14.1
Author: PH

26) Which of the following is an example of a person or firm that is likely to have been granted
a public franchise?
A) medical doctor
B) taxi cab driver
C) the local pizza parlor
D) the local telephone company
E) the local Honda dealership
Answer: D
Topic: Legal barriers to entry
Skill: Level 2: Using definitions
Objective: Checkpoint 14.1
Author: SA
606 Bade/Parkin œ Foundations of Economics, Third Edition

27) A gas station in the mountains of Oregon has a monopoly over the retail gas market within
a 50-mile radius. The station decides not to price discriminate. As a result, all consumers
will pay
A) the highest price each consumer is willing to pay.
B) the lowest price possible.
C) a single price.
D) multiple prices.
E) a price that depends on their willingness to pay.
Answer: C
Topic: Price strategies, single price
Skill: Level 1: Definition
Objective: Checkpoint 14.1
Author: JC

28) A single-price monopoly


A) sets a single, different price for each consumer.
B) sets a single price for all consumers.
C) asks each consumer what single price they would be willing to pay.
D) sets a single, different price for each of two different groups.
E) sells each unit of its output for the single, highest price that the buyer of that unit is
willing to pay.
Answer: B
Topic: Price strategies, single price
Skill: Level 1: Definition
Objective: Checkpoint 14.1
Author: WM

29) A single-price monopoly


A) charges all of its customers a different price for the good or service.
B) sells all of its output at different prices.
C) sells each unit of output for the same price to all of its customers.
D) sells its output to wholesale customers at a different price than it sells to retail
customers.
E) does not need to lower its price to sell more output.
Answer: C
Topic: Price strategies, single price
Skill: Level 1: Definition
Objective: Checkpoint 14.1
Author: PH
Chapter 14 Monopoly 607

30) A price-discriminating monopoly is a firm that


A) sells its output at a single price to all of its customers.
B) sells different units of a good or service at different prices.
C) has control over the resources used to produce the product.
D) has a license to sell the product.
E) illegally charges different customers different prices for the good it produces.
Answer: B
Topic: Price strategies, price discrimination
Skill: Level 1: Definition
Objective: Checkpoint 14.1
Author: PH

31) Price discrimination is


A) always illegal in the United States.
B) defined as charging the same price to all consumers.
C) defined as charging different prices for different units.
D) setting the price to minimize the quantity sold.
E) Both answers A and C are correct.
Answer: C
Topic: Price strategies, price discrimination
Skill: Level 1: Definition
Objective: Checkpoint 14.1
Author: WM

32) Movie theaters often practice price discrimination. Price discrimination means that movies
charge
A) men higher prices than women.
B) the same low price to everyone.
C) the same high price to everyone.
D) different customers different prices.
E) different customers the same price.
Answer: D
Topic: Price strategies, price discrimination
Skill: Level 1: Definition
Objective: Checkpoint 14.1
Author: JC
608 Bade/Parkin œ Foundations of Economics, Third Edition

33) A ____ monopoly sells different units of its good or service for ____.
A) price-discriminating; different prices
B) price-discriminating; the same price
C) single-price; the same price
D) single-price; different prices
E) Both Answers A and C are correct.
Answer: E
Topic: Price strategies, price discrimination
Skill: Level 1: Definition
Objective: Checkpoint 14.1
Author: CD

34) To be able to price discriminate, a firm must


A) have a public franchise.
B) be a natural monopoly.
C) be able to prevent resales of its good.
D) have a patent.
E) have an ownership barrier to entry.
Answer: C
Topic: Price strategies, price discrimination
Skill: Level 1: Definition
Objective: Checkpoint 14.1
Author: CD

35) In order for a hotel to successfully price discriminate so that senior citizens are given a
discount, the hotel must be able to
A) offset the economic loss from charging senior citizens a lower price by lowering the
marginal cost of renting rooms to senior citizens.
B) lower its prices to younger customers too.
C) prevent senior citizens from reselling their rooms to younger customers.
D) shift its demand curve rightward.
E) determine if a senior citizen can pay a higher price.
Answer: C
Topic: Price strategies, price discrimination
Skill: Level 2: Using definitions
Objective: Checkpoint 14.1
Author: JC
Chapter 14 Monopoly 609

36) Which of the following is an example of price discrimination?


A) UPS charges more if a package is sent from New York to Hawaii and less if it is sent
from New York to New Jersey.
B) Frank's Furniture shop charges no delivery fee for furniture delivered within Dutchess
County but charges $40 delivery fee outside of the county.
C) Albert pays 25 percent less on prescription drugs because he is a senior citizen.
D) Only answers A and B are correct.
E) Answer A, answer B, and answer C are all correct.
Answer: C
Topic: Price strategies, price discrimination
Skill: Level 2: Using definitions
Objective: Checkpoint 14.1
Author: SA

37) Firms that can effectively price discriminate


A) can be either perfectly competitive firms or monopolies.
B) can prevent the resale of their products.
C) have only one class of buyers, buyers willing to pay a high price.
D) Both answers A and B are correct.
E) Both answers A and C are correct.
Answer: B
Topic: Price strategies, price discrimination
Skill: Level 2: Using definitions
Objective: Checkpoint 14.1
Author: SA

14.2 Single-Price Monopoly

1) The demand curve for a monopoly is


A) horizontal because the demand is perfectly elastic.
B) downward sloping.
C) vertical because the demand is perfectly inelastic.
D) upward sloping.
E) undefined because it is the only supplier in the market.
Answer: B
Topic: Demand
Skill: Level 2: Using definitions
Objective: Checkpoint 14.2
Author: SA
610 Bade/Parkin œ Foundations of Economics, Third Edition

2) If a monopoly wants to sell a greater quantity of output, it must


A) lower its price.
B) raise its price.
C) tell consumers to buy more because it's a monopolist.
D) raise its marginal cost.
E) change its fixed costs.
Answer: A
Topic: Demand
Skill: Level 2: Using definitions
Objective: Checkpoint 14.2
Author: WM

3) A single-price monopoly
A) must practice price discrimination.
B) can lower its price for only a few select consumers if it wants to increase its output.
C) will set its price equal to a consumer's willingness to pay.
D) must lower the price for all customers if it wants to increase its output.
E) is able to raise its price as high as it wants and consumers must still buy from it
because it is a monopoly.
Answer: D
Topic: Demand
Skill: Level 2: Using definitions
Objective: Checkpoint 14.2
Author: SA

4) Total revenue is defined as


A) the change in price resulting from a one-unit increase in quantity sold.
B) the amount people will buy at a given price.
C) the change in the quantity sold when you change the price by one unit.
D) price multiplied by the quantity sold.
E) the price at which the good or service is sold.
Answer: D
Topic: Total revenue
Skill: Level 1: Definition
Objective: Checkpoint 14.2
Author: WM
Chapter 14 Monopoly 611

5) Marginal revenue is defined as


A) the amount people buy at a given price.
B) the amount people buy between two prices.
C) the change in total revenue brought about by a one-unit increase in quantity sold.
D) price multiplied by the quantity sold.
E) equal to the price.
Answer: C
Topic: Marginal revenue
Skill: Level 1: Definition
Objective: Checkpoint 14.2
Author: WM

6) For a single-price monopoly, price is


A) equal to marginal revenue.
B) greater than marginal revenue.
C) less than marginal revenue because the firm must lower its price in order to sell
another unit of output.
D) less than marginal revenue because the firm cannot increase its total revenue when the
demand curve is downward sloping.
E) equal to zero because the firm is not a price taker.
Answer: B
Topic: Marginal revenue
Skill: Level 2: Using definitions
Objective: Checkpoint 14.2
Author: PH

7) The marginal revenue for a single-price monopoly with a downward-sloping demand


curve
A) is less than the price.
B) is greater than the price.
C) is equal to the price.
D) might be more than, less than, or equal to the price, depending on whether the slope of
the demand curve exceeds 1.0 in magnitude.
E) might be more than, less than, or equal to the price, depending on whether the price
elasticity of demand exceeds 1.0 in magnitude.
Answer: A
Topic: Marginal revenue
Skill: Level 2: Using definitions
Objective: Checkpoint 14.2
Author: SA
612 Bade/Parkin œ Foundations of Economics, Third Edition

8) A single-price monopoly faces a linear demand curve. If the marginal revenue for the
second unit is $20, then the marginal revenue for the
A) first unit is less than $20.
B) third unit is less than $20.
C) third unit is more than $20.
D) third unit is also $20.
E) more information is needed to determine if the marginal revenue for the third unit is
more than, less than, or equal to $20.
Answer: B
Topic: Marginal revenue
Skill: Level 3: Using models
Objective: Checkpoint 14.2
Author: SA

9) A single-price monopoly can sell 2 units for $8.50 per unit. In order to sell 3 units, the price
must be $8.00 per unit. The marginal revenue from selling the third unit is
A) $24.00.
B) $8.50.
C) $7.00.
D) $6.50.
E) $17.00.
Answer: C
Topic: Marginal revenue
Skill: Level 4: Applying models
Objective: Checkpoint 14.2
Author: PH

10) Suppose a single-price monopoly sells 3 units of a good at $20 per unit. If the monopoly
sells 4 units, the total revenue increases to $72. What is the marginal revenue of the fourth
unit?
A) $52
B) $18
C) $60
D) $12
E) $20
Answer: D
Topic: Marginal revenue
Skill: Level 4: Applying models
Objective: Checkpoint 14.2
Author: SA
Chapter 14 Monopoly 613

11) Suppose a single-price monopoly sells 3 units of a good at $20 per unit. If the monopoly
sells 4 units, the total revenue increases to $72. What price is being charged for 4 units?
A) $52 each
B) $18 each
C) $60 each
D) $12 each
E) $20 each
Answer: B
Topic: Total revenue
Skill: Level 4: Applying models
Objective: Checkpoint 14.2
Author: SA

12) The demand curve facing a single-price monopoly is


A) below the marginal revenue curve.
B) above the marginal revenue curve.
C) the same as only the marginal revenue curve.
D) the same as only the marginal cost curve.
E) the same as both the marginal revenue curve and the marginal cost curve.
Answer: B
Topic: Marginal revenue curve
Skill: Level 2: Using definitions
Objective: Checkpoint 14.2
Author: PH

13) A single-price monopoly has a marginal revenue curve that is


A) horizontal and equal to price.
B) downward sloping and below the demand curve.
C) upward sloping and equal to the supply curve.
D) downward sloping and above the demand curve.
E) vertical at the profit-maximizing quantity.
Answer: B
Topic: Marginal revenue curve
Skill: Level 2: Using definitions
Objective: Checkpoint 14.2
Author: WM
614 Bade/Parkin œ Foundations of Economics, Third Edition

14) If Microsoft currently charges prices where its demand is elastic, then Microsoft's marginal
revenue is
A) negative.
B) positive.
C) zero.
D) minimized.
E) undefined.
Answer: B
Topic: Marginal revenue and elasticity
Skill: Level 1: Definition
Objective: Checkpoint 14.2
Author: JC

15) The relationship between marginal revenue and elasticity is


A) when demand is elastic marginal revenue is positive and when demand is inelastic
marginal revenue is negative.
B) whenever the elasticity is positive, marginal revenue is positive.
C) whenever the elasticity is negative, marginal revenue is positive.
D) when demand is elastic marginal revenue is negative and when demand is inelastic
marginal revenue is positive.
E) that total revenue equals zero at the quantity for which the demand is unit elastic.
Answer: A
Topic: Marginal revenue and elasticity
Skill: Level 2: Using definitions
Objective: Checkpoint 14.2
Author: WM

16) If the Boston Red Sox is currently charging ticket prices where its demand is inelastic, then
the Red Sox's marginal revenue is
A) negative.
B) positive.
C) zero.
D) maximized.
E) undefined.
Answer: A
Topic: Marginal revenue and elasticity
Skill: Level 1: Definition
Objective: Checkpoint 14.2
Author: JC
Chapter 14 Monopoly 615

17) If the single restaurant in an Eastern Kentucky town is currently charging prices on its ham
and eggs where the demand is unit elastic, the marginal revenue for ham and eggs is
A) negative.
B) positive.
C) zero.
D) maximized.
E) undefined.
Answer: C
Topic: Marginal revenue and elasticity
Skill: Level 1: Definition
Objective: Checkpoint 14.2
Author: JC

18) Suppose that along a linear demand curve, the elasticity of demand is equal to 1 when the
price is $4 and the quantity is 100 units. Then the
A) total revenue is at its maximum when 100 units are produced.
B) marginal revenue is positive at 100 units.
C) marginal revenue is negative at 100 units.
D) Both answers A and B are correct.
E) Both answers A and C are correct.
Answer: A
Topic: Marginal revenue and elasticity
Skill: Level 3: Using models
Objective: Checkpoint 14.2
Author: SA

19) Suppose that along a linear demand curve, the elasticity of demand is equal to 1 when the
price is $4 and the quantity is 100 units. Then the
A) marginal revenue is negative when output exceeds 100 units.
B) elasticity of demand is less than 1 when output exceeds 100 units.
C) marginal revenue is 0 when output equals 100 units.
D) Only answers A and B are correct.
E) Answers A, B, and C are correct.
Answer: E
Topic: Marginal revenue and elasticity
Skill: Level 4: Applying models
Objective: Checkpoint 14.2
Author: SA
616 Bade/Parkin œ Foundations of Economics, Third Edition

Price(dollars) Quantity(units)
6 1
5 2
4 3
3 4
2 5
1 6

20) The above table gives the demand schedule for a monopoly. The demand is elastic at all
points between
A) $6 and $1.
B) $5 and $1.
C) $3 and $1.
D) $6 and $4.
E) $4 and $3.
Answer: D
Topic: Marginal revenue and elasticity
Skill: Level 4: Applying models
Objective: Checkpoint 14.2
Author: PH

21) The above table gives the demand schedule for a monopoly. The demand is inelastic over
the entire range between
A) $6 and $1.
B) $5 and $1.
C) $3 and $1.
D) $6 and $4.
E) $4 and $3.
Answer: C
Topic: Marginal revenue and elasticity
Skill: Level 4: Applying models
Objective: Checkpoint 14.2
Author: PH
Chapter 14 Monopoly 617

22) To maximize its profit, a single-price monopoly the amount of output so that its marginal
revenue
A) equals zero.
B) equals its marginal cost.
C) exceeds its marginal cost but not necessarily by as much as possible.
D) is less than its marginal cost.
E) exceeds its marginal cost by as much as possible.
Answer: B
Topic: Single-price monopoly, profit maximization
Skill: Level 1: Definition
Objective: Checkpoint 14.2
Author: JC

23) To maximize its profit, a perfectly competitive firm produces so that ____ and a single-price
monopoly produces so that ____.
A) MR = MC; MR > MC
B) MR > MC; MR = MC
C) MR = MC; MR = MC
D) MR > MC; MR > MC
E) P = ATC; P = ATC
Answer: C
Topic: Single-price monopoly, profit maximization
Skill: Level 2: Using definitions
Objective: Checkpoint 14.2
Author: SA

24) In order to maximize its profit, a single-price monopoly equates


A) P = MC.
B) MR = MC.
C) P = MC - MR.
D) P = MR.
E) P = ATC.
Answer: B
Topic: Single-price monopoly, profit maximization
Skill: Level 2: Using definitions
Objective: Checkpoint 14.2
Author: PH
618 Bade/Parkin œ Foundations of Economics, Third Edition

Price(dollars) Quantity(units)
8 1
6 2
5 3
4 4
3 5

25) The above table gives the demand schedule for a single-price monopoly. The marginal
revenue first becomes negative when going from
A) 1 unit to 2 units.
B) 2 units to 3 units.
C) 3 units to 4 units.
D) 4 units to 5 units.
E) None of the above; the total revenue is always positive so the marginal revenue must
always be positive.
Answer: D
Topic: Marginal revenue
Skill: Level 3: Using models
Objective: Checkpoint 14.2
Author: SA

26) The above table gives the demand schedule for a single-price monopoly. If the marginal
cost is $3, the profit maximizing output for the monopoly will be between
A) 1 to 2 units.
B) 2 to 3 units.
C) 3 to 4 units.
D) 4 to 5 units.
E) Exactly 5 units.
Answer: B
Topic: Single-price monopoly, profit maximization
Skill: Level 3: Using models
Objective: Checkpoint 14.2
Author: SA
Chapter 14 Monopoly 619

27) Suppose the Busy Bee Café is the monopoly producer of hamburgers in Hugo, Oklahoma.
The above figure represents the demand, marginal revenue, and marginal cost curves for
this establishment. What quantity should the Busy Bee produce to maximize its profit?
A) 20 hamburgers per hour
B) 30 hamburgers per hour
C) 50 hamburgers per hour
D) 0 hamburgers per hour.
E) 10 hamburgers per hour
Answer: A
Topic: Single-price monopoly, profit maximization
Skill: Level 3: Using models
Objective: Checkpoint 14.2
Author: JC
620 Bade/Parkin œ Foundations of Economics, Third Edition

28) Suppose the Busy Bee Café is the monopoly producer of hamburgers in Hugo, Oklahoma.
The above figure represents the demand, marginal revenue, and marginal cost curves for
this establishment. What price should the Busy Bee charge to maximize its profit?
A) $5.00 for a hamburger
B) $3.00 for a hamburger
C) $2.00 for a hamburger
D) $1.00 for a hamburger
E) $4.00 for a hamburger
Answer: B
Topic: Single-price monopoly, profit maximization
Skill: Level 3: Using models
Objective: Checkpoint 14.2
Author: JC

29) Suppose the Busy Bee Café is the monopoly producer of hamburgers in Hugo, Oklahoma.
The above figure represents the demand, marginal revenue, and marginal cost curves for
this establishment. In order to maximize profit, the Busy Bee should produce ____
hamburgers per hour and set a price of ____ per hamburger.
A) 20; $3.00
B) 20; $1.00
C) 30; $2.00
D) 30; $4.00
E) 50; $5.00
Answer: A
Topic: Single-price monopoly, profit maximization
Skill: Level 3: Using models
Objective: Checkpoint 14.2
Author: JC
Chapter 14 Monopoly 621

30) Suppose the Busy Bee Café is the monopoly producer of hamburgers in Hugo, Oklahoma.
The above figure represents the demand, marginal revenue, and marginal cost curves for
this establishment. If the Busy Bee produces 40 hamburgers per hour, then
A) marginal revenue will exceed marginal cost.
B) profit will be maximized.
C) marginal revenue will be negative.
D) marginal revenue will be maximized.
E) both the marginal revenue and the price will be negative.
Answer: C
Topic: Single-price monopoly, profit maximization
Skill: Level 3: Using models
Objective: Checkpoint 14.2
Author: JC
622 Bade/Parkin œ Foundations of Economics, Third Edition

31) In the above figure, the profit-maximizing output for this single-price monopoly is ____
units and the price is ____.
A) 200; $10
B) 300; $20
C) 500; $50
D) 200; $30
E) 300; $30
Answer: D
Topic: Single-price monopoly, profit maximization
Skill: Level 3: Using models
Objective: Checkpoint 14.2
Author: SA
Chapter 14 Monopoly 623

14.3 Monopoly and Competition Compared

1) Compared to a perfectly competitive industry, a single-price monopoly will produce


A) more output.
B) less output.
C) the same output.
D) some amount that might be more, less, or the same depending on whether the
monopoly's marginal revenue curve lies above, below, or on its demand curve.
E) some amount that might be more, less, or the same depending on whether the
monopoly's marginal cost curve lies above, below, or on its marginal revenue curve.
Answer: B
Topic: Monopoly and competition compared, output
Skill: Level 3: Using models
Objective: Checkpoint 14.3
Author: JC

2) Compared to a perfectly competitive market, a single-price monopoly will charge


A) a lower price.
B) the same price.
C) a higher price.
D) a price that might be higher, lower, or the same depending on whether the monopoly's
marginal revenue curve lies above, below, or on its demand curve.
E) a price that might be higher, lower, or the same depending on whether the monopoly's
marginal cost curve lies above, below, or on its marginal revenue curve.
Answer: C
Topic: Monopoly and competition compared, price
Skill: Level 3: Using models
Objective: Checkpoint 14.3
Author: JC
624 Bade/Parkin œ Foundations of Economics, Third Edition

3) If we compare a perfectly competitive market to a single-price monopoly with the same


costs, we see that the monopoly sells
A) the same quantity at a higher price.
B) a smaller quantity at a higher price.
C) a larger quantity at a lower price.
D) a larger quantity at a higher price.
E) a smaller quantity at the same price.
Answer: B
Topic: Monopoly and competition compared, output and price
Skill: Level 2: Using definitions
Objective: Checkpoint 14.3
Author: WM

4) When compared to a perfectly competitive market, a single-price monopoly with the same
costs produces ____ output and charges ____ price.
A) a larger; a lower
B) a smaller; a lower
C) the same; a higher
D) a smaller; a higher
E) a smaller; the same
Answer: D
Topic: Monopoly and competition compared, output and price
Skill: Level 2: Using definitions
Objective: Checkpoint 14.3
Author: PH

5) A single-price monopoly transfers


A) consumer surplus to producers.
B) producer surplus to consumers.
C) economic profit to consumers.
D) economic profit to the government.
E) economic profit to deadweight loss.
Answer: A
Topic: Monopoly and competition compared, surpluses
Skill: Level 2: Using definitions
Objective: Checkpoint 14.3
Author: SA
Chapter 14 Monopoly 625

6) Which of the following statements is FALSE?


A) A perfectly competitive market produces more output and charges a lower price than a
monopoly.
B) A perfectly competitive firm produces where MR = MC but a monopoly produces
where MR > MC.
C) In a perfectly competitive market, the price is equal to the marginal cost, but in a
market with a single-price monopoly, price exceeds marginal cost.
D) The consumer surplus is smaller for a market with a monopoly than for a perfectly
competitive market.
E) In the long run, a monopoly can earn a larger economic profit than can a perfectly
competitive firm.
Answer: B
Topic: Monopoly and competition compared
Skill: Level 2: Using definitions
Objective: Checkpoint 14.3
Author: SA
626 Bade/Parkin œ Foundations of Economics, Third Edition

7) Suppose the grocery store market in Kansas City is perfectly competitive. Then one store
buys all the others and becomes a single-price monopoly. The figure above shows the
relevant demand and cost curves. When the market is perfectly competitive, the price of a
pound of steak is
A) $4.
B) $8.
C) $12.
D) $20.
E) $2.
Answer: B
Topic: Monopoly and competition compared, price
Skill: Level 3: Using models
Objective: Checkpoint 14.3
Author: JC
Chapter 14 Monopoly 627

8) Suppose the grocery store market in Kansas City is perfectly competitive. Then one store
buys all the others and becomes a single-price monopoly. The figure above shows the
relevant demand and cost curves. When the market is a monopoly, the price of a pound of
steak is
A) $4.
B) $8.
C) $12.
D) $20.
E) $2.
Answer: C
Topic: Monopoly and competition compared, price
Skill: Level 3: Using models
Objective: Checkpoint 14.3
Author: JC

9) Suppose the grocery store market in Kansas City is perfectly competitive. Then one store
buys all the others and becomes a single-price monopoly. The figure above shows the
relevant demand and cost curves. When the market is perfectly competitive, the price of a
pound of steak is ____ and when it is a monopoly, the price of a pound of steak is ____.
A) $4; $20
B) $8; $4
C) $8; $12
D) $4; $8
E) $4; $12
Answer: C
Topic: Monopoly and competition compared, price
Skill: Level 3: Using models
Objective: Checkpoint 14.3
Author: JC
628 Bade/Parkin œ Foundations of Economics, Third Edition

10) Suppose the grocery store market in Kansas City is perfectly competitive. Then one store
buys all the others and becomes a single-price monopoly. The figure above shows the
relevant demand and cost curves. When the market is perfectly competitive, the quantity of
steak is
A) 2,000 pounds.
B) 3,000 pounds.
C) 4,000 pounds.
D) 5,000 pounds.
E) less than 2,000 pounds.
Answer: B
Topic: Monopoly and competition compared, output
Skill: Level 3: Using models
Objective: Checkpoint 14.3
Author: JC

11) Suppose the grocery store market in Kansas City is perfectly competitive. Then one store
buys all the others and becomes a single-price monopoly. The figure above shows the
relevant demand and cost curves. When the market is a monopoly, the quantity of steak is
A) 2,000 pounds.
B) 3,000 pounds.
C) 4,000 pounds.
D) 5,000 pounds.
E) less than 2,000 pounds.
Answer: A
Topic: Monopoly and competition compared, output
Skill: Level 3: Using models
Objective: Checkpoint 14.3
Author: JC
Chapter 14 Monopoly 629

12) Suppose the grocery store market in Kansas City is perfectly competitive. Then one store
buys all the others and becomes a single-price monopoly. The figure above shows the
relevant demand and cost curves. When the market is perfectly competitive, the quantity of
steak is ____ pounds and when the market is a monopoly the quantity of steak is ____
pounds.
A) 2,000; 4,000
B) 3,000; 2,000
C) 4,000; 4,000
D) 5,000; 3,000
E) 4,000; less than 2,000 pounds.
Answer: B
Topic: Monopoly and competition compared, output
Skill: Level 3: Using models
Objective: Checkpoint 14.3
Author: JC
630 Bade/Parkin œ Foundations of Economics, Third Edition

13) In the above figure, a perfectly competitive market will have a price of ____ and a
single-price monopoly will have a price of ____.
A) P1 and quantity of Q1; P2 and quantity of Q2
B) P2 and quantity of Q2; P1 and quantity of Q1
C) P3 and quantity of Q3; P1 and quantity of Q1
D) P2 and quantity of Q2; P3 and quantity of Q1
E) P2 and quantity of Q1; P1 and quantity of Q1

Answer: B
Topic: Monopoly and competition compared, output and price
Skill: Level 3: Using models
Objective: Checkpoint 14.3
Author: SA
Chapter 14 Monopoly 631

14) In the above figure, for a single-price monopoly the consumer surplus is equal to the area
A) abP1.
B) acP2.
C) bce.
D) bed.
E) cQ20P2

Answer: A
Topic: Monopoly and competition compared, surpluses
Skill: Level 3: Using models
Objective: Checkpoint 14.3
Author: SA

15) In the above figure, for a single-price monopoly the deadweight loss is equal to the area
A) abP1.
B) acP2.
C) bce.
D) bed.
E) P1beP3.

Answer: C
Topic: Is monopoly efficient?
Skill: Level 3: Using models
Objective: Checkpoint 14.3
Author: SA

16) Comparing a perfectly competitive market to a single-price monopoly with the same costs,
we see that
A) both markets are equally efficient in their use of resources.
B) the monopoly market always is more efficient in the use of resources.
C) the perfectly competitive market achieves efficiency in resource use while the
monopoly market does not.
D) the monopoly market achieves efficiency in resource use while perfectly competitive
market does not.
E) None of the above answers is correct because comparing a perfectly competitive
market to a monopoly is impossible.
Answer: C
Topic: Is monopoly efficient?
Skill: Level 2: Using definitions
Objective: Checkpoint 14.3
Author: WM
632 Bade/Parkin œ Foundations of Economics, Third Edition

17) The Seattle Mariners baseball team has a monopoly on major league baseball in the
Northwest. If the Mariners could be purchased by anyone with enough money, we could
argue that this purchase is fair according to the
A) fair rules test.
B) fair results test.
C) fair price test.
D) fair output test.
E) allocative fairness test.
Answer: A
Topic: Is monopoly fair?
Skill: Level 2: Using definitions
Objective: Checkpoint 14.3
Author: JC

18) Rent seeking is the act of obtaining special treatment by ____ to create ____.
A) a monopoly; consumer surplus
B) the government; economic profit
C) consumers; a monopoly
D) the government; consumer surplus
E) competitive producers; a monopoly
Answer: B
Topic: Rent seeking
Skill: Level 1: Definition
Objective: Checkpoint 14.3
Author: JC

19) Rent seeking is defined as


A) charging higher prices for an apartment.
B) the act of obtaining special treatment by the government to create an economic profit.
C) charging a price below marginal cost.
D) selling a greater quantity than is profitable.
E) charging different prices for different units of the good or service.
Answer: B
Topic: Rent seeking
Skill: Level 1: Definition
Objective: Checkpoint 14.3
Author: WM
Chapter 14 Monopoly 633

20) Rent seeking


A) is the act of obtaining special treatment by the government to create economic profit.
B) is the attempt to get rent from tardy renters.
C) occurs when landlords advertise for apartments and other property for rent.
D) is an attempt to sell a property and capture economic profit.
E) occurs when a firm charges different prices for different units of its good or service.
Answer: A
Topic: Rent seeking
Skill: Level 1: Definition
Objective: Checkpoint 14.3
Author: SA

21) When a rent-seeking equilibrium is reached, the


A) economic profit is maximized.
B) economic profit is eliminated by legislation.
C) economic profit is eliminated.
D) consumer surplus is greater than without rent seeking.
E) consumer surplus is eliminated.
Answer: C
Topic: Rent-seeking equilibrium
Skill: Level 2: Using definitions
Objective: Checkpoint 14.3
Author: WM

14.4 Price Discrimination

1) To be able to price discriminate, a firm must


A) lower prices for all customers.
B) raise prices for all customers.
C) be able to identify and separate different types of buyers.
D) sell a product that can be resold.
E) Both answers B and C are correct.
Answer: C
Topic: Price discrimination
Skill: Level 1: Definition
Objective: Checkpoint 14.4
Author: JC
634 Bade/Parkin œ Foundations of Economics, Third Edition

2) Which of the following must exist for a firm to engage in price discrimination?
A) The firm must be able to identify and separate its buyers into different classes, and the
low-price buyers cannot resell the product to the high-price buyers.
B) The firm must face an inelastic demand.
C) The firm must be able to realize economies of scale.
D) The firm must have no more than one class of buyer.
E) The firm must be a natural monopoly.
Answer: A
Topic: Price discrimination
Skill: Level 1: Definition
Objective: Checkpoint 14.4
Author: PH

3) A price-discriminating monopoly charges


A) the same price to every buyer for the same product.
B) a different price to different classes of buyers for the same product, even though there
are no differences in costs.
C) a different price to different buyers, because the costs are different.
D) different prices to buyers for different products.
E) each customer a price that equals the marginal cost of serving that customer.
Answer: B
Topic: Price discrimination
Skill: Level 1: Definition
Objective: Checkpoint 14.4
Author: PH

4) With price discrimination, a monopoly


A) converts consumer surplus into economic profit.
B) converts producer surplus into economic profit.
C) can charge a single price to all customers.
D) produces less output than if it does not price discriminate.
E) converts consumer surplus into deadweight loss.
Answer: A
Topic: Price discrimination
Skill: Level 2: Using definitions
Objective: Checkpoint 14.4
Author: SA
Chapter 14 Monopoly 635

5) One way a monopoly can convert additional consumer surplus into economic profit is to
A) lower prices.
B) raise prices.
C) price discriminate.
D) become more competitive.
E) produce where price equals average total cost.
Answer: C
Topic: Price discrimination
Skill: Level 2: Using definitions
Objective: Checkpoint 14.4
Author: JC

6) The airline industry


A) cannot price discriminate because it is against the law.
B) price discriminates by charging higher prices to leisure travelers.
C) price discriminates by charging lower prices to leisure travelers.
D) identifies business travelers in order to charge them lower prices.
E) has fewer customers because it price discriminates than it would have if it did not price
discriminate.
Answer: C
Topic: Price discrimination
Skill: Level 2: Using definitions
Objective: Checkpoint 14.4
Author: WM

7) A price-discriminating monopoly
A) sells a larger quantity than it would if it were a single-price monopoly.
B) is illegal.
C) cannot offer discounts.
D) cannot control the price of its product.
E) makes a smaller economic profit than it would if it were a single-price monopoly.
Answer: A
Topic: Price discrimination
Skill: Level 2: Using definitions
Objective: Checkpoint 14.4
Author: SA
636 Bade/Parkin œ Foundations of Economics, Third Edition

8) With price discrimination, a monopoly ____ its economic profit and ____ its output.
A) increases; increases
B) increases; decreases
C) decreases; increases
D) decreases; decreases
E) increases; does not change
Answer: A
Topic: Price discrimination
Skill: Level 2: Using definitions
Objective: Checkpoint 14.4
Author: PH

9) Why do publishers print the first edition of a book by a popular author in hard cover and
not in paperback?
A) Hard cover books are long lasting and paperbacks can rip easily.
B) Readers who want to read the book as soon as it comes out will be willing to pay a
higher price as compared to those who can wait for the paperback edition.
C) A hardcover is the publisher's way of rewarding the avid readers.
D) Publishers are not sure of the demand.
E) Publishers cannot price discriminate.
Answer: B
Topic: Price discrimination
Skill: Level 5: Critical thinking
Objective: Checkpoint 14.4
Author: SA

10) With perfect price discrimination, a monopoly can extract the ____ price each customer is
willing to pay and thereby obtain the entire ____ surplus.
A) maximum; consumer
B) minimum; producer
C) maximum; producer
D) minimum; consumer
E) None of the above answers are correct.
Answer: A
Topic: Perfect price discrimination
Skill: Level 2: Using definitions
Objective: Checkpoint 14.4
Author: PH
Chapter 14 Monopoly 637

11) When a firm is able to engage in perfect price discrimination, its marginal revenue curve
A) lies below its demand curve.
B) is the same as its demand curve.
C) lies above its demand curve.
D) is the same as its supply curve.
E) is undefined because it does not exist.
Answer: B
Topic: Perfect price discrimination
Skill: Level 2: Using definitions
Objective: Checkpoint 14.4
Author: PH

12) If a monopoly can perfectly price discriminate, then its marginal revenue curve will be
A) the same as its demand curve.
B) the same as its supply curve.
C) the same as its marginal cost curve.
D) a vertical line at the profit-maximizing quantity of output.
E) undefined because it does not exist.
Answer: A
Topic: Perfect price discrimination
Skill: Level 2: Using definitions
Objective: Checkpoint 14.4
Author: JC

13) Compared to a single-price monopoly, when a monopoly can perfectly price discriminate,
the deadweight loss
A) increases.
B) decreases.
C) remains the same.
D) becomes infinite.
E) probably changes, but more information is needed to determine if it increases,
decreases, or remains constant.
Answer: B
Topic: Perfect price discrimination
Skill: Level 2: Using definitions
Objective: Checkpoint 14.4
Author: JC
638 Bade/Parkin œ Foundations of Economics, Third Edition

14) Which of the following industries is most likely closest to achieving perfect price
discrimination?
A) the airline industry
B) the wheat industry
C) the textbook industry
D) the toilet paper industry
E) the soft drink industry
Answer: A
Topic: Eye on the U.S. economy, airline price discrimination
Skill: Level 2: Using definitions
Objective: Checkpoint 14.4
Author: JC

14.5 Monopoly Policy Issues

1) When economies of scale exist so that one firm can meet the entire market demand at a
lower average total cost than two or more firms,
A) a natural monopoly develops.
B) the monopoly encounters competition.
C) economic profit is reduced to zero.
D) the monopoly converts all of the consumer surplus into economic profit.
E) there is always the opportunity to price discriminate.
Answer: A
Topic: Natural monopoly
Skill: Level 2: Using definitions
Objective: Checkpoint 14.5
Author: WM

2) Incentives to innovate
A) are a potential advantage monopoly has over a competitive alternative.
B) are why a monopoly price discriminates if it is possible.
C) create a legal monopoly.
D) do not exist when a firm is a monopoly.
E) are larger for monopolies that price discriminate than for monopolies that set a single
price.
Answer: A
Topic: Gains from monopoly
Skill: Level 2: Using definitions
Objective: Checkpoint 14.5
Author: MR
Chapter 14 Monopoly 639

14.6 Integrative Questions

1) A monopoly can arise when


A) there are diseconomies of scale.
B) there are barriers to entry and no close substitutes for the good being produced.
C) a firm cannot price discriminate.
D) firms engage in rent seeking.
E) a firm must set MR equal to MC in order to maximize its profit.
Answer: B
Topic: Integrative
Skill: Level 2: Using definitions
Objective: Integrative
Author: CD

2) The total revenue test using the price elasticity of demand


A) explains why monopolies will only operate on the elastic portion of their demand
curve.
B) explains why monopolies will only operate on the inelastic portion of their demand
curves.
C) demonstrates why a monopoly can earn an economic profit in the long run.
D) determines whether a monopoly can perfectly price discriminate or not.
E) cannot be used for a price discriminating monopoly.
Answer: B
Topic: Integrative
Skill: Level 4: Applying models
Objective: Integrative
Author: CD

3) A monopoly definitely incurs an economic loss if


A) it produces where its marginal revenue equals its marginal cost.
B) its average total cost is greater than price.
C) it cannot perfectly price discriminate.
D) it price discriminates.
E) The statement errs because a monopoly cannot incur an economic loss.
Answer: B
Topic: Integrative
Skill: Level 3: Using models
Objective: Integrative
Author: CD
640 Bade/Parkin œ Foundations of Economics, Third Edition

4) A difference between a perfectly competitive industry and a monopoly is that


A) in the long run, firms in a perfectly competitive industry earn a normal profit and a
monopoly can earn an economic profit.
B) a firm in a perfectly competitive industry can perfectly price discriminate but a
monopoly cannot.
C) only monopolies have an incentive to maximize profit.
D) perfectly competitive firms can have a public franchise.
E) a barrier to entry protects perfectly competitive firms in the short run and protects a
monopoly in the long run.
Answer: A
Topic: Integrative
Skill: Level 3: Using models
Objective: Integrative
Author: CD

5) If a monopoly engages in rent seeking,


i. its average total cost curve is lower than otherwise.
ii. it might or might not earn an economic profit depending on how many other
competitors also are rent seeking.
iii. it necessarily incurs an economic loss.
A) i only.
B) ii only.
C) iii only.
D) i and ii.
E) i and iii.
Answer: B
Topic: Integrative
Skill: Level 2: Using definitions
Objective: Integrative
Author: CD
Chapter 14 Monopoly 641

6) If a monopoly can perfectly price discriminate,


A) all the demanders pay one price.
B) it minimizes its profit.
C) it produces the same amount of output as would be produced if the market was a
perfectly competitive industry.
D) it produces less output than would be produced if the market was a perfectly
competitive industry.
E) it creates the same amount of consumer surplus as would be created if the market was
a perfectly competitive industry.
Answer: C
Topic: Integrative
Skill: Level 3: Using models
Objective: Integrative
Author: CD

7) Monopolies arise when there are


A) many substitutes but there are no barriers to entry.
B) no close substitutes and there are no barriers to entry.
C) no close substitutes and there are barriers to entry.
D) many substitutes and there barriers to entry.
E) None of the above answers are correct because the existence of a monopoly has
nothing to do with the presence or absence of barriers to entry.
Answer: C
Topic: Integrative
Skill: Level 2: Using definitions
Objective: Integrative
Author: WM
In a competitive market, the demand and supply curve represent the
behaviour of buyers and sellers. The demand curve shows how buyers
respond to price changes whereas the supply curve shows how sellers
response to the price changes. And we know from our earlier discussion that
demand is negatively related to price whereas supply is positively related.
However from our earlier discussion, we do not know how much the quantity
changes as a result of a price change. We only know the direction of the
change from our discussion but not the magnitude. In order to get more
precision into our analysis we need to discuss the concept of elasticity, which
is what we will do in this session.

Elasticity is one of the most readily available measures and its uses include:
Pricing.
Managing cash flows.
Impact of changes in competitors’ prices.
Impact of economic booms and recessions.
Impact of advertising campaigns.
And lots more!

1
The law of supply states that higher prices raise the quantity supplied. The
price elasticity of supply measures how much the quantity supplied responds
to changes in the price. Supply of a good is said to be elastic (or price
sensitive) if the quantity supplied responds substantially to changes in price.
Supply is said to be inelastic (or price insensitive) if the quantity supplied
responds only slightly to changes in the price. The elasticity of supply can be
zero or greater. The measure of elasticity of supply is closer to zero then it is
considered to be inelastic and if it is closer to infinity then it is more elastic.
Different goods have different elasticity of supply. For example, beachfront
house will be more inelastic than books. Firstly, the land on which beachfront
houses are built is limited and secondly it takes time to build the house even if
we find the land. However, more books can be relatively easily printed.

2
This brings us to our next point about the determinants of price elasticity of
supply. The main determinants of price elasticity can be classified as:

Time period: In a number of markets time period is a fundamental determinant


of the price elasticity of supply. In the short run supply is usually more inelastic
than in the long run. In most situations firms have limited capacity. They
cannot increase the goods produced as easily in the short run as in the long
run. So if price increases then the supply is likely to be more responsive in the
long run rather than immediately.

Productive capacity: The responsiveness of the firm to changes in prices is


also going to depend on the state of the economy. For example, when the
economy is experiencing recession most business will not be operating to their
full capacity and will be able to respond relatively quickly to changes in market
condition and hence supply will be more elastic. On the other hand in times of
boom most firms will be operating near full capacity and will find it much more
difficult to respond to any price changes and hence price will be less elastic.

The Size of the Firm/Industry: The price elasticity of supply may be more
inelastic for bigger firms than smaller firms. For example, if a small furniture

3
company gets an unexpected order then it might be able to buy extra raw
material at a short notice and produce more. A small increase in purchases of
raw material is unlikely to impact the unit costs of the firm. However, for a big
steel manufacturing firm buying iron ore in large quantities on the commodity
markets is likely to be a more difficult decision impacting the firms unit costs
and hence profitability. So the elasticity of supply is likely to be more elastic for
the furniture company than the steel firm.

Mobility of factors of production: If factors of production can be switched


relatively easily to produce a particular good then supply will be more elastic.
For example, wheat and rye can be produced using very similar resources. If
price of wheat goes up, farmers may switch from producing rye and supply
wheat instead. Thus, the supply of wheat will be elastic. However, if there is a
shortage of heart specialists then that does not mean that other doctors can
suddenly become heart surgeons. The supply of heart specialists is therefore
going to relatively inelastic.

Ease of storing stock/inventory: Ease of storing stock/inventory: Industries


such as light fittings or car parts are easier to store and therefore firms can
easily supply more if there is an increase in price. However, the same is not
true for fresh fruit. Mangoes are only available for instance in certain time
periods and a price hike in the off season period does not guarantee readily
available mangoes. Thus, price elasticity of supply of fresh fruit may be more
inelastic.

3
Elasticity is measured as percentage change in quantity supplied divided by
percentage change in price. It is very important to remember the ceteris
paribus assumption. We can calculate the price elasticity of supply only if other
things are held constant. The relationship between price and quantity is always
positive and so is the slope of the supply curve. This implies the price elasticity
of supply will always be a positive number.

The standard method of computing the percentage (%) change is: (end value
– start value /start value) x 100%. However, in the case of supply curve we are
interested in finding the elasticity between two points where the distinction
between start and end values is less meaningful.

For example take the following points:

Point A : Price = €4 Quantity Supplied = 80

Point B : Price = €6 Quantity Supplied = 125

If we use the standard method of calculating elasticity and use point A as the

4
starting point then price rise should comes to 50%. However, if we use the
point B as the starting point then the price fall should be 33%. Using these
numbers for elasticity calculations will provide very different estimates. To
avoid this problem we will use the midpoint method.

The midpoint is the number halfway between the start and end values, the
average of those values. It doesn’t matter which value you use as the start and
which as the end—you get the same answer either way!

4
Price elasticity of supply measures the seller’s responsiveness to price. If the
scales on the axes of the diagrams used are the same then supply curves can
be visually compared for elasticity. The diagram in panel a shows perfectly
inelastic supply: Elasticity=0. If we think about this then it is fairly
straightforward. If the price increases then the quantity supplied does not
change. Thus supply is completely insensitive to price changes and hence this
is represented as the vertical line. An example of this can be fresh fruit in the
short run. The capacity of the mango farms cannot be changed overnight and
once mangoes are harvested they are available in a fixed stock. Panel b
shows the supply curve where elasticity is less than one. The curve captures
the fact that in this case the change in price is proportionately greater than
change in quantity supplied.

5
If price elasticity of supply is equal to one then it is referred to as unit elastic.
On this slide we have a diagram of supply curve that is unit elastic, where
percentage change in quantity supplied is equal to the percentage change in
price.

6
Elasticity of supply can be greater than one as depicted on the slide in panel d
and e. As elasticity rises the supply curve gets flatter. In the case of panel d
the elasticity is greater than one. The supply curve becomes horizontal in the
case of perfectly elastic supply as illustrated on the slide.

7
In some markets the price elasticity of supply is not constant and varies along
the curve. In some cases points with low price and low quantity tend to elastic
due to unused capacity. Small increases in prices increase the quantity
substantially in order to capitalize on the potential for profit. As firms continue
increasing production, firms begin to reach full capacity. Thus points with high
price and high quantity may be inelastic.

8
Because firms often have a maximum capacity for production, the elasticity of
supply may be very high at low levels of quantity supplied and very low at high
levels of quantity supplied. Here an increase in price from $3 to $4 increases
the quantity supplied from 100 to 200. Because the 67% increase in quantity
supplied (computed using the midpoint method) is larger than the 29%
increase in price, the supply curve is elastic in this range. By contrast, when
the price rises from $12 to $15, the quantity supplied rises only from 500 to
525. Because the 5% increase in quantity supplied is smaller than the 22%
increase in price, the supply curve is inelastic in this range.

9
The total revenue (TR) received by sellers is P x Q, the
price of the good times the quantity of the good sold

The change in TR as one moves along the supply curve


depends on the elasticity of a supply

If supply is inelastic, then an increase in the price causes


an increase in TR that is proportionately less than the price
change

If the supply is elastic, then an increase in price leads to a


much greater increase in TR

10
The law of demand states that higher prices consumers reduce quantity
demanded. The price elasticity of demand measures how much the quantity
demanded responds to changes in the price. Demand of a good is said to be
elastic (or price sensitive) if the quantity demanded responds substantially to
changes in price. Demand is said to be inelastic (or price insensitive) if the
quantity demanded responds only slightly to changes in the price.

11
Price elasticity of demand measures the responsiveness of quantity demanded
to its own price. There is an inverse relationship between price and quantity
demanded, hence price elasticity of demand is always negative. For reasons
of convention, we drop the minus sign and speak of price elasticity in absolute
terms. The absolute value of own price elasticity can be greater than one in
which case the demand is said to be elastic. This means an increase in the
price of good will reduce the consumption considerably. Or a reduction in price
will increase the consumption substantially. It is very important to remember
the ceteris paribus assumption. We can calculate the own price elasticity of
demand only if other things are held constant.

12
Like supply a number of factors impact the elasticity of demand. We as
consumers do not respond to the change in prices of different goods in the
same fashion. For example, if the price of salt doubles- would you change your
consumption patterns? Alternatively is the price of the house you want to buy
doubles then you are unlikely to ignore it as you probably do for salt. Some of
the factors affecting the own price elasticity include:

Available Substitutes: The more substitutes available for the good, the more
elastic the demand. For example, butter and margarine can be easily
substituted for each other. If there is a small in the price of butter, assuming
the price of margarine is held fixed, causes a big drop in quantity of butter
demanded as people start to consume margarine instead.

Necessities versus Luxuries: Necessities will have a more inelastic demand as


compared to luxuries. For example, food is a necessity whereas a owning a
sailing boat is a luxury for most people. If the price of food increases, people
will still buy food. They may buy less of and avoid wastage but the demand of
food will largely stay intact. However, people are going to be more responsive
to the change in the price of a luxury such as a sailing boat. Thus, sailing boat
will have a more elastic demand curve.

13
13
Definition of the market: Narrowly defined markets have a greater price
elasticity of demand than a broadly defined market. For example, the demand
for beverages is more inelastic than the demand for coke.

Time: Demand tends to be more inelastic in the short term than in the long
term. Time allows consumers to seek out available substitutes.

Proportion of income devoted to the product: Goods that comprise a small


share of consumer’s budgets tend to be more inelastic (e.g salt) than goods for
which consumers spend a large portion of their incomes (e.g house).

14
Elasticity of demand is measured as percentage change in quantity demanded
divided by percentage change in price. It is very important to remember the
ceteris paribus assumption. We can calculate the price elasticity of demand
only if other things are held constant. There is an inverse relationship between
price and quantity demanded, hence own price elasticity of demand is always
negative. For reasons of convention, we drop the minus sign and speak of own
price elasticity in absolute terms.

As with supply, we use the midpoint method for calculating price elasticity of
demand- for the same reasons discussed earlier while calculating elasticity of
supply.

15
As with supply, demand curves can be classified according to their elasticity.
Demand is perfectly inelastic if price elasticity of demand = 0. In this case
demand curve is vertical as depicted in panel a. An example of a vertical
demand curve may include medication for diabetes where the consumer is
entirely dependent on the medication and therefore not responsive to price.
Demand is inelastic if price elasticity of demand < 1 as depicted in panel b.
Here an increase in price has a relatively small affect on demand. An example
of this would be a commodity like cigarettes that are highly addictive.

16
Demand has unit elasticity if price elasticity of demand = 1. This is an
intermediate case where demand is neither highly price-sensitive or completely
unresponsive.

17
Demand is elastic if price elasticity of demand > 1 as depicted in panel d. A
number of goods with a variety of substitutes will fit this case. Demand is
perfectly elastic if price elasticity of demand = infinite. In this case demand
curve is horizontal, shown in panel e.

As a rule of thumb the flatter the demand curve, the greater is the price
elasticity of demand. However, please note that this will only work if the
measurement scales across different demand curves are comparable
especially if demand curves are not perfectly elastic or inelastic.

18
The expenditure on a good or the total revenue received by the seller
ultimately depends on the purchasing habits of buyers. A producer therefore
has to be mindful of the consumer’s sensitivity to price changes in any
decisions as it has a direct impact on the total revenue received. The
consumer response to price changes in turn depends on elasticity. If demand
is inelastic, a price increase will cause TR to go up. If demand is elastic, a
price increase will cause TR to go down. In the next few slides we will analyse
the reason behind this step by step.

19
The total amount paid by buyers, and received as revenue by sellers, equals
the area of the box under the demand curve, P × Q. Here, at a price of $4, the
quantity demanded is 100, and total revenue is $400. This is illustrated by the
pink rectangle in the slide above.

20
The impact of a price change on total revenue (the product of price and
quantity) depends on the elasticity of demand. In panel (a), the demand curve
is inelastic. In this case, an increase in the price leads to a decrease in
quantity demanded that is proportionately smaller, so total revenue increases.
Here an increase in the price from $4 to $5 causes the quantity demanded to
fall from 100 to 90. Total revenue rises from $400 to $450.

21
The impact of a price change on total revenue (the product of price and
quantity) depends on the elasticity of demand. In panel (b), the demand curve
is elastic. In this case, an increase in the price leads to a decrease in quantity
demanded that is proportionately larger, so total revenue decreases. Here an
increase in the price from $4 to $5 causes the quantity demanded to fall from
100 to 70. Total revenue falls from $400 to $350.

22
To summarise our discussion on price elasticity of demand and TR, we have
two opposing forces when price changes. On one hand an increase in price
increases total revenue. However, a price increase also causes a fall in
quantity demanded. Elasticity helps us determine which of these opposing
effects dominate and whether the total revenue will increase or decrease.

When demand is inelastic (elasticity < 1)


-P and TR move in the same direction
If P ↑, TR also ↑

When demand is elastic (elasticity > 1)


-P and TR move in opposite directions
If P ↑, TR ↓

If demand is unit elastic (elasticity = 1)


-Total revenue remains constant when the price changes

23
The demand curves we have analysed so far have an elasticity that is same
along the whole curve. This, however, is not always the case. Most of the
times we will work with linear demand curve, which has a constant slope but
varying elasticity figures along the curve. Slope is defined as the ratio of
changes in two variables (rise over run), whereas the elasticity is the ratio of
percentage changes in the two variables.

The linear demand curve tends to be inelastic at points with low price and high
quantity. Whereas points with high price and low quantity tend to lie in the
elastic region of the linear demand curve.

24
The diagram above shows the elasticity at various points along a linear
demand curve. Unlike slope, elasticity is not constant along the linear demand
curve. As price increase demand becomes more elastic. The elastic area on
the demand curve shows the region where Elasticity>1, followed by region of
unit elasticity and then the area of where Elasticity<1. According to the formula
price elasticity, elastic area means that a percent fall in price increases the
quantity demanded by more than a percent. Connecting this to Total revenue
or revenue implies that if we are in the elastic region and we reduce the price
then total revenue will increase. On the other hand in the inelastic region a
percent reduction in price will lead to less than a percent increase in quantity,
causing the total revenue to fall. Total revenue is maximized at the point of unit
elasticity. The unit elastic point lies in the middle of the curve.

25
In addition to price elasticity, economists also rely on other elasticities to
describe consumer behaviour in the market. Income elasticity provides a
useful measure of how the quantity demanded changes as consumer income
changes. Income elasticity is equal to percentage change in quantity
demanded divided by the percentage change in income.

26
Most goods such as shoes, clothes, computers are normal good, which implies
that higher income leads to more demand. However, some goods such as bus
rides tend to be inferior goods. As income goes up we tend to use less of
public transport and therefore income elasticity tends to be negative.
Necessities such as food tend have small income elasticities because we all
rely on food regardless of income. On the other hand luxuries such as
diamonds have a high income elasticity because people can do without these
goods in times of income reductions.

27
As we know the demand for a good is also affected by change in prices of
related goods. For example the demand for bananas gets affected by the price
of apples/oranges etc. So we can calculate the effect of price changes of
related goods using cross price elasticity of demand. The same principles that
we discussed previously apply here. All you do is calculate the percentage
change of quantity demanded of good X and the percentage change of price of
good Y. Dividing the two gives us the cross price elasticity. Here the sign is
extremely important. If the sign is positive, it means that when the price of
good Y increases, the demand for good X goes up. Hence, the two goods are
substitutes (goods typically used in place of one another
). If the sign is negative, it means that when the price of good Y increases, the
demand for good X goes down. Implying that the two goods are complements
(Goods that are typically used together).

28
Imagine a situation where you are a wheat farmer and a new crop variety
increases the production per acre by 20%. This causes:
Supply curve shifts to the right
Higher quantity and lower price
Demand is inelastic: total revenue falls

29
When an advance in farm technology increases the supply of wheat from S 1 to
S2, the price of wheat falls. Because the demand for wheat is inelastic, the
increase in the quantity sold from 100 to 110 is proportionately smaller than the
decrease in the price from $3 to $2. As a result, farmers’ total revenue falls
from $300 ($3 × 100) to $220 ($2 × 110).

In this example we see the two opposing factors impacting the total revenue.
Since the quantity sold is now more than before TR increases. But at the same
time the price for each unit sold goes down. Since in this case demand is
inelastic, overall the revenue goes down.

30
The paradox of public policy arises from the fact that what may be good news
for the economy may not be good news for individual farmers. This may also
explain why despite the improvements in technology many farmers have left
the farm for jobs in the city.

31
Name Econ Unit 3 Test

Part 1: Multiple Choice: Choose the best answer for each question (2 points each)
1. Which of the following statements is true for both monopolistically 9. Which of the following would most likely be a monopoly?
competitive and oligopolistic industries?
A) An appliance store B) A supermarket
A) It is impossible for new firms to enter the industries. C) An electricity provider D) A dentist's office
B) Collusion and the creation of cartels is common. 10. Monopolistically competitive firms most frequently do which of the
C) Producers cannot benefit from knowing other firms' plans. following?
D) Firms have some degree of control over prices.
A) Compete in pricing wars with other firms in the industry
2. Which of the following best describes an oligopoly?
B) Advertise the traits that make their product identifiable
A) many monopolistically competitive firms C) Enjoy monopoly pricing power
B) a few firms sharing monopoly power D) Merge and consolidate into oligopolistic groupings
C) a former monopoly that has been broken up by the government 11. When the government grants an exclusive patent to one firm, that
D) a government-granted franchise or monopoly firm enjoys
3. Collusion most frequently occurs in industries that are A) productive efficiency B) antitrust regulation
A) oligopolistic C) monopoly powers D) collusive prices
B) monopolistically competitive 12. Which of these could be considered a government-created barrier to
C) monopolistic market entry?
D) perfectly competitive
A) Discretionary spending
4. If a few firms share most of an entire industry's revenues, the market B) Antitrust legislation
structure is most likely C) Patents and copyrights
A) monopolistically competitive D) Progressive income taxes
B) an oligopoly 13. The basic difference between copyrights and patents is that
C) perfectly competitive
A) copyrights never expire, while patents always have an
D) a monopoly
expiration date
5. All of the following can help break a monopoly EXCEPT B) copyrights are used to protect ideas, while patents protect
A) increased barriers to entry processes and products
B) changing consumer tastes C) copyrights are awarded by private entities, while patents are
awarded by the government
C) international competition
D) copyrights always last longer than patents
D) technology and innovation
14. A patent is given to a firm to protect that firm's
6. Which of the following best describes a successful monopolist?
A) monopoly power over a new process or product
A) The only buyer of a resource or type of labor
B) research and development investment
B) The only seller of a difficult-to-substitute product
C) ability to profit from its discoveries
C) The only buyer of a consumer product
D) all of the above
D) The only seller of a non-essential product
15. If a proprietorship fails, who is responsible for the firm's debts?
7. A monopoly is able to make greater profits than a perfectly
competitive firm because A) The workers B) The sole proprietor
A) the monopolist fixes prices in cooperation with its rivals C) The shareholders D) The public
B) barriers to entry in the monopolist's market prevent 16. Which of the following is a disadvantage faced by sole proprietors?
competition
A) Limited decision-making power
C) antitrust legislation protects the monopolist from regulation
B) Full responsibility for the business' debts
D) monopolistic firms are always larger than competitive firms
C) Demands placed on them by shareholders
8. Which of the following is most likely to be observed in a D) "Double taxation" of profits through corporate and dividend
monopolistically competitive market? taxes
A) Standardized, homogenous products 17. A proprietorship shared between multiple people is known as
B) Collusion and price-fixing between firms
A) a partnership B) a franchise
C) Government antitrust oversight
C) an oligopoly D) a conglomerate
D) Non-price competition, such as advertising

Page 1 Version 1
18. In general, a company whose shareholders collect dividends must 23. A vertical merger is
be
A) the combination of two firms that specialize in different
A) publicly traded B) a private bank stages of the same supply chain
C) taking a loss D) earning a profit B) the combination of two firms from completely unrelated
19. A corporation distributes profits to its many part-owners by industries
C) a combination of two firms that were previously in
A) raising the interest rate B) offering stock competition with each other
C) paying dividends D) issuing bonds D) a combination of two firms that are not corporations
20. Which of the following organizations is most likely to sell stock? 24. If a corporation that makes rubber combines with a corporation that
A) Sole proprietorship B) Partnership makes tires, this action would most likely be considered
C) Corporation D) Labor union A) a horizontal merger B) a conglomerate merger
21. A business that is related to an established corporation and shares C) a stock merger D) a vertical merger
the same trademarks, brands, and business model is known as a 25. The personal income earned by the sale of stock is known as
A) merger B) franchise A) preferred stock B) dividends
C) proprietorship D) entrepreneurship C) mutual fund income D) capital gains
22. Firm X and Firm Y were previously in direct competition, but now
they plan to merge. This combination would be considered a
A) horizontal merger B) vertical merger
C) complementary merger D) conglomerate merger

Page 2 Version 1
Answer Key
Unit 3 2015

1. D
2. B
3. A
4. B
5. A
6. B
7. B
8. D
9. C
10. B
11. C
12. C
13. B
14. D
15. B
16. B
17. A
18. D
19. C
20. C
21. B
22. A
23. A
24. D
25. D

Page 3 Version 1
Chapter 11 Perfect Competition - Sample Questions

MULTIPLE CHOICE. Choose the one alternative that best completes the statement or answers the question.

1) Perfect competition is an industry with 1)


A) a few firms producing identical goods.
B) many firms producing goods that differ somewhat.
C) a few firms producing goods that differ somewhat in quality.
D) many firms producing identical goods.

2) In a perfectly competitive industry, there are 2)


A) many buyers and many sellers.
B) many sellers, but there might be only one or two buyers.
C) many buyers, but there might be only one or two sellers.
D) one firm that sets the price for the others to follow.

3) In perfect competition, the product of a single firm 3)


A) is sold to different customers at different prices.
B) has many perfect complements produced by other firms.
C) has many perfect substitutes produced by other firms.
D) is sold under many differing brand names.

4) In perfect competition, restrictions on entry into an industry 4)


A) do not exist. B) apply to labor but not to capital.
C) apply to both capital and labor. D) apply to capital but not to labor.

5) In perfect competition, 5)
A) there are significant restrictions on entry.
B) each firm can influence the price of the good.
C) there are few buyers.
D) all firms in the market sell their product at the same price.

6) The price elasticity of demand for any particular perfectly competitive firm's output is 6)
A) less than 1. B) equal to zero. C) infinite. D) 1.

7) The demand for wheat from farm A is perfectly elastic because wheat from farm A is a(n) 7)
A) perfect complement to wheat from farm B. B) perfect substitute for wheat from farm B.
C) normal good. D) inferior good.

8) In perfect competition, the elasticity of demand for the product of a single firm is 8)
A) 0. B) infinite.
C) 1. D) between 0 and 1.

1
9) In perfect competition, the elasticity of demand for the product of a single firm is 9)
A) infinite, because many other firms produce identical products.
B) zero, because many other firms produce identical products.
C) zero, because the firm produces a unique product.
D) infinite, because the firm produces a unique product.

10) In perfect competition, an individual firm 10)


A) has a price elasticity of supply equal to one.
B) faces unitary elasticity of demand.
C) has a price elasticity of supply equal to infinity.
D) faces infinitely elastic demand.

11) If Steve's Apple Orchard, Inc. is a perfectly competitive firm, the demand for Steve's apples has 11)
A) elasticity equal to the price of apples. B) unitary elasticity.
C) infinite elasticity. D) zero elasticity.

12) In a perfectly competitive industry, the price elasticity of demand for the market demand is 12)
________ and the price elasticity of demand for an individual firm's demand is ________.
A) infinite; less than infinite B) infinite; infinite
C) less than infinite; less than infinite D) less than infinite; infinite

13) A perfectly competitive firm's demand curve is 13)


A) perfectly inelastic.
B) the same as the market demand curve.
C) downward sloping.
D) the same as the firm's marginal revenue curve.

14) The market for fish is perfectly competitive. So, the price elasticity of demand for fish from a single 14)
fishery
A) is sometimes greater than and sometimes less than the elasticity of demand for fish overall.
B) is greater than the elasticity of demand for fish overall.
C) is less than the elasticity of demand for fish overall.
D) equals the elasticity of demand for fish overall.

15) In perfect competition, the price of the product is determined where the industry 15)
A) elasticity of supply equals the industry elasticity of demand.
B) supply curve and industry demand curve intersect.
C) fixed cost is zero.
D) average variable cost equals the industry average total cost.

16) Economists assume that a perfectly competitive firm's objective is to maximize its 16)
A) revenue. B) economic profit. C) output price. D) quantity sold.

2
17) Total economic profit is 17)
A) total revenue minus total opportunity cost.
B) marginal revenue minus marginal cost.
C) total revenue divided by total cost.
D) marginal revenue divided by marginal cost.

18) The economic profit of a perfectly competitive firm 18)


A) is less than its total revenue.
B) is greater than its total revenue.
C) equals its total revenue.
D) is less than its total revenue if its supply curve is inelastic and is greater than its total revenue
if its supply curve is elastic.

19) In perfect competition, a firm that maximizes its economic profit will sell its good 19)
A) below the market price.
B) above the market price.
C) below the market price if its supply curve is inelastic and above the market price if its supply
curve is elastic.
D) at the market price.

20) The above figure shows a firm's total revenue line. The firm must be in a market with 20)
A) monopolistic competition. B) monopoly.
C) perfect competition. D) oligopoly.

21) For a perfectly competitive firm, curve A in the above figure is the firm's 21)
A) average fixed cost curve. B) average variable cost curve.
C) total revenue curve. D) total fixed cost curve.

3
22) The figure above portrays a total revenue curve for a perfectly competitive firm. Curve A is straight 22)
because the firm
A) has perfect information. B) wants to maximize its profits.
C) is a price taker. D) faces constant returns to scale.

23) The figure above portrays a total revenue curve for a perfectly competitive firm. The firm's 23)
marginal revenue from selling a unit of output
A) equals $1.00. B) equals $2.00.
C) equals $0.50. D) cannot be determined.

24) The figure above portrays a total revenue curve for a perfectly competitive firm. The price of the 24)
product in this industry
A) equals $1.00. B) equals $2.00.
C) equals $0.50. D) cannot be determined.

25) In the above figure showing a perfectly competitive firm's total revenue line, the firm's marginal 25)
revenue
A) does not change as output increases. B) falls as output increases.
C) rises as output increases. D) cannot be determined.

Quantity Price
5 $15
6 $15
7 $15

26) In the above table, if the firm sells 5 units of output, its total revenue is 26)
A) $30. B) $15. C) $75. D) $90.

27) In the above table, if the quantity sold by the firm rises from 5 to 6, its marginal revenue is 27)
A) $15. B) $75. C) $90. D) $30.

28) In the above table, if the quantity sold by the firm rises from 6 to 7, its marginal revenue is 28)
A) $90. B) $30. C) $105. D) $15.

29) In perfect competition, the marginal revenue of an individual firm 29)


A) equals the price of the product.
B) is positive but less than the price of the product.
C) exceeds the price of the product.
D) is zero.

30) In the case of a perfectly competitive firm, the 30)


A) firm's marginal revenue exceeds the price of the product.
B) change in the firm's total revenue equals the price of the product multiplied by the change in
quantity sold.
C) firm's marginal revenue is less than average revenue.
D) price of the product falls sharply when the quantity the firm sells doubles.

4
31) In perfect competition, the firm's marginal revenue curve 31)
A) cuts its demand curve from above, going from left to right.
B) always lies below its demand curve.
C) cuts its demand curve from below, going from left to right.
D) is the same as its demand curve.

32) At a firm's break-even point, definitely its 32)


A) marginal revenue equals its average fixed cost.
B) marginal revenue equals its average variable cost.
C) total revenue equals its total opportunity cost.
D) marginal revenue exceeds its marginal cost.

33) When Sidney's Sweaters, Inc. makes exactly zero economic profit, Sidney, the owner, 33)
A) makes an income equal to his best alternative forgone income.
B) will boost output.
C) will shut down in the short run.
D) is taking a loss.

34) The break-even point is defined as occurring at an output rate at which 34)
A) total cost is minimized.
B) total revenue equals total opportunity cost.
C) economic profit is maximized.
D) marginal revenue equals marginal cost.

Output Total Revenue Total Cost


0 $0 $25
1 $30 $49
2 $60 $69
3 $90 $91
4 $120 $117
5 $150 $147
6 $180 $180

35) In the above table, the price of the product is 35)


A) $30. B) $150. C) $147. D) $180.

36) In the above table, the firm 36)


A) must be in a perfectly competitive industry, because its marginal revenue is constant.
B) cannot be in a perfectly competitive industry, because its short-run economic profits are
greater than zero.
C) cannot be in a perfectly competitive industry, because its long-run economic profits are
greater than zero.
D) must be in a perfectly competitive industry, because its marginal cost curve eventually rises.

37) In the above table, the marginal revenue from the fourth unit of output is 37)
A) $180. B) $147. C) $150. D) $30.

5
38) In the above table, if the firm produces 2 units of output, it will make an economic 38)
A) loss of $60. B) profit of $60. C) loss of $9. D) profit of $9.

Output Total Cost


(balloons per hour) (dollars per hour)
0 $4.00
1 $7.00
2 $8.00
3 $12.50
4 $17.20
5 $22.00
6 $29.00
39) In the above table, the firm's total fixed cost of production is 39)
A) $29.00. B) $4.00. C) $3.00. D) $7.00.

40) In the above table, the average fixed cost at 4 units of output is 40)
A) $4.80. B) $4.70. C) $1.00. D) $4.50.

41) In the above table, the average variable cost at 2 units of output is 41)
A) $4.00. B) $2.00. C) $1.00. D) $4.80.

42) In the above figure, by increasing its output from Q1 toQ2, the firm 42)
A) increases its profit. B) increases its marginal revenue.
C) reduces its marginal revenue. D) decreases its profit.

43) In the above figure, by increasing its output from Q2 to Q3, the firm 43)
A) increases its marginal revenue. B) reduces its marginal revenue.
C) decreases its profit. D) increases its profit.

6
44) The above figure illustrates a firm's total revenue and total cost curves. Which one of the following 44)
statements is FALSE?
A) At output Q1 the firm makes zero economic profit.
B) At an output above Q3 the firm incurs an economic loss.
C) Economic profit is the vertical distance between the total revenue curve and the total cost
curve.
D) At output Q2 the firm incurs an economic loss.

45) The feature of the above figure that indicates that the firm is a perfectly competitive firm is the 45)
A) fact that the total cost and total revenue curves are farthest apart at output is Q2.
B) shape of the total revenue curve.
C) fact that the total cost and total revenue curves cross twice.
D) shape of the total cost curve.

46) In the above figure, the firm is making an economic loss at 46)
A) point a. B) points b and d.
C) points a, b, and d. D) point c.

47) In the above figure, the firm is breaking even at points 47)
A) a and d. B) b and d. C) c and d. D) a and c.

48) In the above figure, when the firm produces output corresponding to point c, the firm's marginal 48)
cost
A) is less than its marginal revenue. B) equals its average revenue.
C) exceeds its marginal revenue. D) equals its marginal revenue.

7
49) For a perfectly competitive firm, in a diagram with quantity on the horizontal axis and both total 49)
revenue and total cost on the vertical axis, the firm's ________ is a straight line ________.
A) total cost curve; through the origin B) total revenue curve; with zero slope
C) total cost curve; with zero slope D) total revenue curve; through the origin

50) A perfectly competitive firm maximizes its profit by producing the output at which its marginal 50)
cost equals its
A) average variable cost. B) marginal revenue.
C) average total cost. D) average fixed cost.

51) For a firm in perfect competition, a diagram shows quantity on the horizontal axis and both the 51)
firm's marginal cost (MC) and its marginal revenue (MR) on the vertical axis. The firm's
profit-maximizing quantity occurs at the point where the
A) MC curve intersects the MR curve from above, going from left to right.
B) slope of the MC curve is zero.
C) MC curve intersects the MR curve from below, going from left to right.
D) MC and MR curves are parallel.

52) A firm will expand the amount of output it produces as long as its 52)
A) average total revenue exceeds its average variable cost.
B) marginal revenue exceeds its marginal cost.
C) marginal cost exceeds its marginal revenue.
D) average total revenue exceeds its average total cost.

53) A perfectly competitive firm is producing at the point where its marginal cost equals its marginal 53)
revenue. If the firm boosts its output, its total revenue will ________ and its profit will ________.
A) fall; fall B) fall; rise C) rise; rise D) rise; fall

54) A perfectly competitive firm is producing at the point where its marginal cost equals its marginal 54)
revenue. If the firm boosts its output, its revenue will
A) rise and its total variable cost will rise, but not by as much.
B) fall but its total variable cost will rise.
C) fall and its total variable cost will fall, but not by as much.
D) rise and its total variable cost will rise even more.

55) A perfectly competitive firm's marginal revenue exceeds its marginal cost at its current output. To 55)
increase its profit, the firm will
A) increase its output. B) raise its price.
C) decrease its output. D) lower its price.

56) A perfectly competitive firm's marginal cost exceeds its marginal revenue at its current output. To 56)
increase its profit, the firm will
A) increase its output. B) raise its price.
C) lower its price. D) decrease its output.

8
57) A perfectly competitive firm is producing more than the profit-maximizing amount of its product. 57)
You can conclude that its
A) marginal revenue is less than the price of the product.
B) total cost exceeds its total revenue.
C) average total cost exceeds the price of the product.
D) marginal cost exceeds the price of the product.

58) The costs incurred even when no output is produced are called 58)
A) fixed costs. B) external costs. C) variable costs. D) marginal costs.

59) A firm's shutdown point is the quantity and price at which the firm's total revenue just equals its 59)
A) marginal cost. B) total variable cost.
C) total cost. D) total fixed cost.

60) It definitely pays a firm to shut down if the price of its product is 60)
A) below its minimum average variable cost. B) above its maximum variable cost.
C) above its minimum average variable cost. D) below its minimum total cost.

61) The owners definitely will shut down a perfectly competitive firm if the price of its good falls 61)
below its minimum
A) average marginal cost. B) wage rate.
C) average variable cost. D) average total cost.

62) A firm that shuts down and produces no output incurs a loss equal to its 62)
A) marginal costs. B) total fixed costs.
C) total variable costs. D) marginal revenue.

63) By producing less, a firm can reduce 63)


A) its variable costs but not its fixed costs.
B) its fixed costs and its variable costs.
C) its fixed costs but not its variable costs.
D) neither its variable costs nor its fixed costs.

64) The shutdown point occurs at the level of output for which the ________ is at its minimum. 64)
A) marginal cost B) total cost
C) average fixed cost D) average variable cost

65) A competitive firm is more likely to shut down during a recession, when the demand for its 65)
product declines, than during an economic expansion, because during the recession it might be
unable to cover its
A) external costs.
B) depreciation due to machinery becoming obsolete.
C) variable costs.
D) fixed costs.

9
66) If the price of its product falls below the minimum point on the AVC curve, the best a perfectly 66)
competitive firm can do is to
A) shut down and incur a loss equal to its total variable cost.
B) shut down and incur a loss equal to its total fixed cost.
C) keep producing and incur a loss equal to its total variable cost.
D) keep producing and incur a loss equal to its total fixed cost.

67) If the price of its product just equals the average variable cost of production for a competitive firm, 67)
A) total revenue equals total variable cost and the firm's loss equals total fixed cost.
B) total revenue equals total fixed cost and the firm's loss equals total variable cost.
C) total variable cost equals total fixed cost.
D) total fixed cost is zero.

Output Total cost


(tons of rice per year) (dollars per ton)
0 $1,000
1 $1,200
2 $1,600
3 $2,200
4 $3,000
5 $4,000

68) Based on the table above which shows Chip's costs, if rice sells for $600 a ton, Chip's 68)
profit-maximizing output is
A) less than one ton. B) between one and two tons.
C) between two and three tons. D) between three and four tons.

69) Based on the table above which shows Chip's costs, if rice sells for $600 a ton, Chip will 69)
A) stay open because he earns an economic profit.
B) stay open because the price is above his minimum average variable cost.
C) shut down because the price is below his minimum average variable cost.
D) shut down because he incurs an economic loss.

70) Based on the table above which shows Chip's costs, if rice sells for $600 a ton, Chip 70)
A) earns an economic profit, but should shut down in the short run.
B) incurs an economic loss, but should stay open in the short run.
C) incurs an economic loss and should shut down in the short run.
D) earns an economic profit and should stay open in the short run.

71) Based on the table above which shows Chip's costs, if Chip shuts down in the short run, his total 71)
cost will be
A) $1,200. B) $4,000. C) $1,000. D) $0.

72) Based on the table above which shows Chip's costs, if Chip shuts down in the short run, his 72)
economic loss will be
A) $1,000. B) $1,200. C) $0. D) $4,000.

10
73) In the above figure, if the price is P1, the firm will produce 73)
A) where ATC equals P1. B) where MC equals P1.
C) nothing. D) where MC equals ATC.

74) In the above figure, if the price is P1, the firm maximizes its profit by producing 74)
A) where ATC equals P1. B) nothing.
C) where MC equals P1. D) where MC equals ATC.

75) In the above figure, if the firm increases its output from Q1 to Q2, it will 75)
A) increase its profit. B) reduce its marginal revenue.
C) decrease its profit. D) increase its marginal revenue.

76) In the above figure, if the firm increases its output from Q2 to Q1, it will 76)
A) reduce its marginal revenue. B) increase its profit.
C) increase its marginal revenue. D) decrease its profit.

77) In the above figure, if the price is P1, the firm is 77)
A) incurring an economic loss. B) shut down.
C) breaking even. D) making an economic profit.

78) In the above figure, if the firm produced Q1, the firm's economic profit is ________ than if it 78)
produced Q2 and ________ than if it produced Q3.
A) more; less B) less; more C) more; more D) less; less

79) In the above figure, if the firm produced Q3, the firm's economic profit is ________ than if it 79)
produced Q1 and ________ than if it produced Q2.
A) more; less B) more; more C) less; more D) less; less

11
80) A perfectly competitive firm will have an economic profit of zero if, at its profit-maximizing 80)
output, its marginal revenue equals its
A) marginal cost. B) average variable cost.
C) average total cost. D) average fixed cost.

81) The figure above shows short-run cost curves for a perfectly competitive firm. If the price of the 81)
product is $8, in the short run the firm will
A) incur an economic loss.
B) earn an economic profit.
C) earn a normal profit.
D) None of the above answers is correct because more information is needed to determine the
firm's profit or loss.

82) The figure above shows short-run cost curves for a perfectly competitive firm. If the price of the 82)
product is $8 and the firm does not shut down, the firm's output in the short run
A) will be 0.
B) will be 10 or higher.
C) will be between 0 and 10.
D) cannot be determined without more information.

83) The short-run supply curve for a perfectly competitive firm is its 83)
A) marginal cost curve above the horizontal axis.
B) average cost curve above the horizontal axis.
C) average cost curve above its shutdown point.
D) marginal cost curve above its shutdown point.

12
84) The short-run supply curve for a perfectly competitive firm is its marginal cost curve 84)
A) below its shutdown point. B) above the horizontal axis.
C) everywhere. D) above its shutdown point.

85) The short-run supply curve for a perfectly competitive firm is its marginal cost curve above the 85)
minimum point on the
A) average variable cost curve. B) demand curve.
C) average total cost curve. D) average fixed cost curve.

86) A perfectly competitive firm's supply curve is made up of its marginal cost curve at all points 86)
above its minimum
A) average variable cost curve. B) average total cost curve.
C) average fixed cost curve. D) price.

87) The firm's supply curve is its 87)


A) marginal cost curve, at all points above the minimum average fixed cost curve.
B) marginal revenue curve, at all points above the minimum average total cost curve.
C) marginal cost curve, at all points above the minimum average variable cost curve.
D) marginal revenue curve, at all points above the minimum average revenue curve.

88) The figure represents a firm in a perfectly competitive market. The firm will shut down if price falls 88)
below
A) P2. B) P1. C) P3. D) P4.

89) The figure represents a firm in a perfectly competitive market. If the firm does not shut down, the 89)
least amount of output that it will produce is
A) 10 units. B) 8 units. C) 5 units. D) less than 5 units.

13
90) The figure represents a firm in a perfectly competitive market. If the price rises from P3 to P4 then 90)
output will increase by
A) 3 units. B) 0 units. C) 1 unit. D) 2 units.

91) The figure above represents a firm in a perfectly competitive market. The firm's supply curve is the 91)
curved line linking
A) point c to point e and continuing on past point e along the ATC curve.
B) point b to point f and stopping at point f.
C) point a to point c and stopping at point c.
D) point b to point d and continuing on past point d along the MC curve.

92) In a perfectly competitive industry, the industry supply curve is the sum of the 92)
A) average total cost curves of all the individual firms.
B) supply curves of all the individual firms.
C) average variable cost curves of all the individual firms.
D) average fixed cost curves of all the individual firms.

93) If there are 1,000 rutabaga farms, all perfectly competitive, an increase in the price of fertilizer used 93)
for growing rutabagas will
A) have no effect on the total quantity of rutabagas supplied, because each farm's supply curve is
a vertical line.
B) reduce the total quantity of rutabagas supplied, because each farm's supply curve is a
horizontal line and will shift upward.
C) have no effect on the total quantity of rutabagas supplied, because no farm has enough market
power to raise the price.
D) decrease the total quantity of rutabagas supplied, because each farm's supply curve shifts
leftward.

14
94) In the above figure, if the price is P1, the firm is 94)
A) earning a normal profit.
B) incurring an economic loss.
C) earning enough revenue to pay all of its opportunity costs.
D) making an economic profit.

95) Suppose the cost curves in the above figure apply to all firms in the industry. Then, if the initial 95)
price is P1, in the long run the market
A) supply will decrease. B) supply will increase.
C) demand will decrease. D) demand will increase.

96) Suppose the cost curves in the above figure apply to all firms in the industry. If the initial price is 96)
P1, firms are
A) making an economic profit and some firms will leave the industry.
B) incurring an economic loss and some firms will leave the industry.
C) making an economic profit and some firms will enter the industry.
D) incurring an economic loss and some firms will enter the industry.

97) New reports indicate that eating turnips helps people remain healthy. The news shifts the demand 97)
curve for turnips rightward. In response, new farms enter the turnip industry. During the period in
which the new farms are entering, the price of a turnip ________ and the profit of each existing
firm ________.
A) falls; rises B) rises; falls C) rises; rises D) falls; falls

98) If firms exit an industry, the 98)


A) profits of the remaining firms decrease. B) industry supply curve shifts leftward.
C) price of the product falls. D) output of the industry increases.

15
99) As firms leave an industry because they are incurring an economic loss, the economic loss of each 99)
remaining firm
A) increases and the price of the product rises.
B) decreases and the price of the product falls.
C) decreases and the price of the product rises.
D) increases and the price of the product falls.

100) In a perfectly competitive industry, a permanent decrease in demand initially brings a lower price, 100)
economic
A) profit, and entry into the industry. B) profit, and exit from the industry.
C) loss, and entry into the industry. D) loss, and exit from the industry.

101) In the above figure, the firm's initial average total cost curve is SRAC with an initial marginal cost 101)
curve of SRMC. The price of the product is P1. In the short run the firm will produce output equal
to the amount
A) Q2. B) Q1. C) Q4. D) Q3.

102) In the above figure, the firm's initial average total cost curve is SRAC. If the price is P1., in the long 102)
run the firm will
A) retain the same plant size. B) expand its plant size.
C) exit the industry. D) reduce its plant size.

103) In the above figure when the firm has reached its long-run equilibrium position, it will produce 103)
output equal to the amount
A) Q4. B) Q3. C) Q2. D) Q1.

16
104) If the cost curves shown in the above figure apply to all firms in the industry and the initial price is 104)
P1, in the long run the price will be
A) greater than P1. B) zero. C) equal to P1. D) less than P1.

105) In a perfectly competitive industry, a permanent increase in demand initially brings a higher price, 105)
economic
A) profit, and entry into the industry. B) profit, and exit from the industry.
C) loss, and entry into the industry. D) loss, and exit from the industry.

106) In the long run, fixed costs are 106)


A) zero and variable costs are zero. B) zero and variable costs are positive.
C) positive and variable costs are positive. D) positive and variable costs are zero.

107) In the long run, the economic profits of a firm in a perfectly competitive industry 107)
A) will equal zero. B) will be below zero.
C) will be above zero. D) can be above, below, or equal to zero.

108) Assuming long-run external diseconomies exist, when demand increases in a perfectly competitive 108)
market, in the long run, the price of the product
A) falls below the initial price (before the increase in demand) and the quantity decreases.
B) equals the initial price (before the increase in demand) and the quantity increases.
C) equals the initial price (before the increase in demand) and the quantity decreases.
D) rises above the initial price (before the increase in demand) and the quantity increases.

109) Assuming long-run external economies exist, when demand increases in a perfectly competitive 109)
market, in the long run, the price of the product
A) rises above the initial price (before the increase in demand) and the quantity increases.
B) equals the initial price (before the increase in demand) and the quantity increases.
C) falls below the initial price (before the increase in demand) and the quantity increases.
D) equals the initial price (before the increase in demand) and the quantity decreases.

110) In a perfectly competitive market, if there are no external economies or diseconomies, an increase 110)
in demand
A) raises average cost in the long run. B) lowers the price in the long run.
C) leaves the price the same in the long run. D) raises the price in the long run.

111) If there are external economies, as demand increases, 111)


A) output decreases in the long run.
B) the price falls in the long run.
C) the price rises in the long run.
D) firms exit from the industry in the long run.

17
112) External economies are factors beyond the control of an individual firm that ________ as the total 112)
industry output increases.
A) raise its marginal revenue B) raise its costs
C) lower its costs D) lower its profit

113) A long-run supply curve for a perfectly competitive industry can slope upward because of 113)
A) external economies. B) economic profit.
C) external diseconomies. D) diminishing marginal returns.

114) In the above figure, the industry short-run supply curve shifts from S0 to S2 as the 114)
A) wage rate falls. B) number of firms increases.
C) external economies rise. D) number of firms decreases.

115) The curve LS0 in the above figure is the long-run supply curve of a perfectly competitive industry. 115)
As the demand curve shifts rightward, the industry exhibits
A) external economies.
B) neither external economies nor external diseconomies.
C) external diseconomies.
D) technological advancement.

116) Congestion of airports and airspace causes the airline industry to experience external 116)
A) diseconomies and have a long-run supply curve with positive slope.
B) economies and have a long-run supply curve with positive slope.
C) diseconomies and have a long-run supply curve with negative slope.
D) economies and have a long-run supply curve with negative slope.

18
117) Assuming long-run external economies exist, when demand increases in a perfectly competitive 117)
market, in the long run the average total cost curve for a typical firm
A) shifts upward. B) shifts downward.
C) is no longer U-shaped. D) stays the same.

118) If the slope of the long-run supply curve for a perfectly competitive industry is positive, the 118)
industry experiences
A) internal economies. B) external economies.
C) external diseconomies. D) internal diseconomies.

119) If the slope of the long-run supply curve for a perfectly competitive industry is negative, the 119)
industry experiences
A) external economies. B) external diseconomies.
C) internal diseconomies. D) internal economies.

120) The gains from trade that go to households are called 120)
A) consumer surplus. B) income.
C) profits. D) producer surplus.

121) Among the obstacles to the efficient allocation of resources are all of the following EXCEPT 121)
A) competition. B) monopoly.
C) external benefits. D) external costs.

122) An example of an external cost is 122)


A) the damage created by a tornado.
B) pollution.
C) the price that a consumer pays for a new car.
D) the price that a firm pays for a consultant's advice.

123) Which of the following characterizes a perfectly competitive industry? 123)


A) Each firm produces a product slightly different from that of its competitors.
B) The industry demand curve is vertical.
C) The demand for each individual firm is perfectly elastic.
D) Each firm sets a different price.

124) Paul runs a shop that sells printers. Paul is a perfect competitor and can sell each printer for a price 124)
of $300. The marginal cost of selling one printer a day is $200; the marginal cost of selling a second
printer is $250; and the marginal cost of selling a third printer is $350. To maximize his profit, Paul
should sell
A) two printers a day. B) more than three printers a day.
C) three printers a day. D) one printer a day.

19
125) Because of a decrease in the wage rate it must pay, a perfectly competitive firm's marginal costs 125)
decrease but its demand curve stays the same. As a result, the firm
A) decreases the amount of output it produces and lowers its price.
B) increases the amount of output it produces and lowers it price.
C) increases the amount of output it produces and does not change its price.
D) decreases the amount of output it produces and raises its price.

126) For prices above the minimum average variable cost, a perfectly competitive firm's supply curve is 126)
A) the same as its average variable cost curve.
B) horizontal at the market price.
C) the same as its marginal cost curve.
D) vertical at zero output.

127) A perfectly competitive firm is definitely earning an economic profit when 127)
A) P > ATC. B) P > AVC. C) P < ATC. D) MR < MC.

128) In the short run, a perfectly competitive firm can 128)


A) earn a normal profit.
B) incur an economic loss.
C) earn an economic profit.
D) earn an economic profit, earn a normal profit, or incur an economic loss.

129) Suppose firms in a perfectly competitive industry are suffering an economic loss. Over time, 129)
A) some firms leave the industry, so the price falls and the economic loss decreases.
B) some firms leave the industry, so the price rises and the economic loss decreases.
C) other firms enter the industry, so the price falls and the economic loss decreases.
D) other firms enter the industry, so the price rises and the economic loss decreases.

130) As firms enter a perfectly competitive industry, 130)


A) the price falls and the existing firms' economic profits do not change.
B) the price falls and the existing firms' economic profits decrease.
C) the price falls and the existing firms' economic losses do not change.
D) the price rises and the existing firms' economic profits decrease.

131) In the long run, a perfectly competitive firm can 131)


A) earn an economic profit, earn a normal profit, or incur an economic loss.
B) earn an economic profit.
C) incur an economic loss.
D) earn a normal profit.

20
132) The demand for a product produced in a perfectly competitive market permanently increases. In 132)
the short run the price
A) rises and each firm produces less output.
B) does not change because each firm produces more output.
C) rises and each firm produces more output.
D) does not change as new firms enter the industry.

133) If there are external diseconomies in an industry, in the long run, after a permanent increase in 133)
demand, the price
A) will be the same as it was initially before the increase in demand.
B) will be lower than it was initially before the increase in demand.
C) may be higher or lower than it was initially before the increase in demand, depending on
whether or not the firms are earning an economic profit.
D) will be higher than it was initially before the increase in demand.

134) To which of the following situations does the term "external diseconomies" apply? 134)
A) Increases in an industry's output reduce the costs of the firms in an industry.
B) The firm's ATC curve slopes upward as the firm produces more output.
C) The firm's MC curve falls as more output is produced.
D) Increases in an industry's output raise the costs of the firms in an industry.

135) The above figure shows the total revenue curve for Dizzy Discs. The demand curve for CD's sold 135)
by Dizzy Discs
A) has positive slope. B) has negative slope.
C) is horizontal. D) is vertical.

21
136) In the figure above, a firm is operating at point A on the graph. At point A, the firm's average cost 136)
curve
A) is horizontal. B) has negative slope.
C) is vertical. D) has positive slope.

137) Carol's Candies is producing 150 boxes of candy a day. Carol's marginal revenue and marginal cost 137)
curves are shown in the figure above. To increase her profit, Carol should
A) decrease output to increase profit.
B) maintain the current level of output to maximize profit.
C) increase output to increase profit.
D) Not enough information is given to determine if Carol should increase, decrease, or not
change her level of output.

22
138) Joe's Shiny Shoes is a firm that operates in a perfectly competitive market. The figure above shows 138)
Joe's cost and revenue curves. If the number of firms in the shoe market decreases, Joe will
A) decrease his production. B) have an MR curve with positive slope.
C) have an MR curve with negative slope. D) increase his production.

23
Answer Key
Testname: UNTITLED3.TST

1) D
2) A
3) C
4) A
5) D
6) C
7) B
8) B
9) A
10) D
11) C
12) D
13) D
14) B
15) B
16) B
17) A
18) A
19) D
20) C
21) C
22) C
23) B
24) B
25) A
26) C
27) A
28) D
29) A
30) B
31) D
32) C
33) A
34) B
35) A
36) A
37) D
38) C
39) B
40) C
41) B
42) A
43) C
44) D
45) B
46) A
47) B
48) D
49) D
50) B
1
Answer Key
Testname: UNTITLED3.TST

51) C
52) B
53) D
54) D
55) A
56) D
57) D
58) A
59) B
60) A
61) C
62) B
63) A
64) D
65) C
66) B
67) A
68) C
69) B
70) B
71) C
72) A
73) B
74) C
75) A
76) D
77) D
78) B
79) D
80) C
81) A
82) C
83) D
84) D
85) A
86) A
87) C
88) A
89) B
90) C
91) D
92) B
93) D
94) B
95) A
96) B
97) D
98) B
99) C
100) D
2
Answer Key
Testname: UNTITLED3.TST

101) A
102) B
103) B
104) D
105) A
106) B
107) A
108) D
109) C
110) C
111) B
112) C
113) C
114) B
115) C
116) A
117) B
118) C
119) A
120) A
121) A
122) B
123) C
124) A
125) C
126) C
127) A
128) D
129) B
130) B
131) D
132) C
133) D
134) D
135) C
136) A
137) A
138) D

3
Presented by Group 2:
 Zuha Handoo
 Suhail Qadir
 Foziya Khanday
 Mariya Qurat-Ul-Ain
 Aqib Hussain
 Mujeeb Tariq
 Salman Farooq Dar
INTRODUCTION
Pure Monopolistic
(perfect) Oligopoly Monopoly
competition competition

In decreasing order of level of competition


Oligopoly :
 Derived from the Greek word, “oligo’(few) “polo”(to
sell).
 Market dominated by a few large firms, i.e.;
Competition amongst the few.
• Difficult • Competitors

Ease of
Competition
entry

Control over
Products
price

• Same- • Yes
Different
Characteristics of Oligopoly:
 Only “few” sellers.
 Homogeneous/Differentiated products.
 Imperfect knowledge.
 High barriers to entry.
 Mutual dependence.
 Non price competition.
Types of Oligopoly:
1. Pure or Perfect Oligopoly
2. Imperfect or Differentiated Oligopoly
3. Collusive Oligopoly
4. Non-Collusive Oligopoly
5. Open Oligopoly
6. Closed Oligopoly
1. Pure or Perfect Oligopoly:
If the firms produce homogeneous products,
then it is called pure or perfect oligopoly.
Though, it is rare to find pure oligopoly
situation, yet, cement, steel, aluminum and
chemicals producing industries approach pure
oligopoly.
2. Imperfect or Differentiated Oligopoly:
If the firms produce differentiated products, then it
is called differentiated or imperfect oligopoly.For
example, passenger cars, cigarettes or soft drinks.
The goods produced by different firms have their
own distinguishing characteristics, yet all of them
are close substitutes of each other.
3. Collusive Oligopoly:
If the firms cooperate with each other in
determining price or output or both, it is
called collusive oligopoly or cooperative
oligopoly.In other words, the firms in a
collusive oligopoly combines to avoid the
competition among themselves regarding the
price and output of the industry. For example,
OPEC(Organization for petroleum exporting
countries) serves the example for collusive
oligopolies.
4. Non-collusive Oligopoly:
If firms in an oligopoly market compete with
each other, it is called a non-collusive or non-
cooperative oligopoly.The firms in non-
collusive oligopoly tries to gain maximum
share of the market by developing policies and
strategies to outperform or beat their rivals.
5.Open oligopoly:
An open oligopoly provides full freedom to
new firms to enter into industry.In the
situation of open oligopoly there is no
restriction of any kind for the desiring firm to
enter into the market.
6.Closed oligopoly:
A closed oligopoly refers to that market
structure where only few firms control the
market and new firms are not allowed to enter
industry. Barriers are set to prevent the entry
of new firms into the industry. For example,
patents,licences,requirement of large
capital,control over crucial raw materials are
some of the reasons which prevent new firms
from entering into industry.
Barriers to entry:
1. Access to suppliers & distributors
2. Cost of entering a market
3. Legal requirements
4. Legal restriction
5. Fear of retaliation
Examples of oligopoly:
1. Smart Phone Operating Systems:
The smart phone market is similarly
dominated by a handful of companies, the
most powerful two being Google Android
and Apple IOS. Those companies have deep
relationships with the handset providers and
are able to have their system pre – installed
on each phone.
2.Computer Operating Systems:
New high tech markets can become
oligopolies when the companies provide
unique products that are supported by an
ecosystem of supporting technology.
Computer operating systems are dominated
by Microsoft’s Windows, Apple’s Mac OS and
the open source Linux operating systems.
These three systems capture close to 100 % of
the computer operating system market due
to their established positions. According to
the StatOwl website.
3.Music Industry:
The music entertainment industry is
dominated by four music companies that
control 80% of the market and these are
universal Music Group, Sony Music
entertainment, Warner Music Group.
21%

35.10% universal
sony
warner
21.10% others

22.80%
4.Auto industry:
Auto industry is another example of an oligopoly,
which is dominated by few firms and these firms
are Hero Motor Corp., TVS and Honda.
 HERO: 40% Market Share
 Honda: 25% Market Share
 TVS: 14% Market Share
21%

40% HERO
TVS
HONDA
OTHERS
25%

14%
5.Oligopoly in soft drink industry:
Two firms control 74 % of soft drink sales:
o 42.8% coca-cola’s 25 brands and 139 varieties.
o 31.1% Pepsi’s 18 brands and 163 varieties.
Coca-cola and Pepsi are in an oligopoly market.
They are mutually and strategically
interdependent, as a decision made by one firm
invariably affects the other. They are selling the
homogeneous product so they can control over
price.
Series 1

26.10%

42.80% coca-cola
Pepsi
others

31.10%
Some other common examples:
Airlines
Supermarkets
Steel Industry
Health Insurance, etc………
Models of oligopoly:
Although there are many models which
explain oligopolistic market structure, but we
will be discussing two over here.
These are:
1: Kinked model
2: Price Leadership model
Kinked Model:
A bend in a standard demand curve that is a
result of competitors decreasing their prices
to match each others, but not raising them
to achieve the same effect. The thought is
that once a business has reduced their price
to a certain level any fluctuation that raises
the price will cause the firm to lose
customers.
Price Leadership model:
 Firms follow the price leadership of a
particular firm.
 Tacit collusion i.e. unwritten and unspoken
agreement.
Types of Price Leadership Model:
 Price leadership by Low-Cost Firm.
 Price leadership by Dominant Firm.
 The Barometric price leadership.
Assumptions
 Small number of firms.
 Entry is restricted.
 Homogeneous product.
 Interdependence.
 Firms have similar cost curves.
Low-Cost Price Leadership
 Firm with the lowest cost of production sets
the price and others follow.
 Only the leader firm optimizes the profit.
Price Leadership by Dominant
Firm
 Dominant firm sets the price
 Influence of dominant firm on market is very high.
 Prices set by the dominant firm maybe pushed.
Barometric Price Leadership
 Price is generally set by a large and
experienced firm with good knowledge and
proven predictability.
 Leader consistently monitors the market.
 Leader considers common interest of all
firms.
 Forms due to the dissatisfaction of firms
with the level of competition.
Thank You!
Economics 101
Professor Wallace
Fall 2003

Homework 7 Answers
1. The average product of labor is
a. the inverse of the average product of capital.
b. the slope of the curve showing the marginal product of labor.
c. the slope of the curve showing the total product of labor.
d. total product divided by the total quantity of labor employed.

Total Product, Marginal Product, Average Product


Labor Total product Marginal Average
(workers per day) (units per day) product product
0 0 0 0
1 2 2 2
2 8
3 12
4 15
5 16 1

2. In the above table, the marginal product of the fourth worker is


a. 1.
b. 3.
c. 4.
d. 6.

3. In the above table, the average product is less than the marginal product
a. when the first worker is hired
b. when the second worker is hired.
c. when the third worker is hired.
d. for the entire range of output given.

4. In perfect competition, restrictions on entry into an industry


a. apply to both capital and labor.
b. apply to labor but not to capital.
c. apply to capital but not to labor.
d. do not exist.

5. A perfectly competitive firm’s demand curve is


a. perfectly inelastic.
b. the same as the market demand curve.
c. the same as the firm’s marginal revenue curve.
d. downward sloping.
Quantity sold Price
5 $15
6 $15
7 $15

6. In the above table, if the quantity sold by the firm rises from 6 to 7, its marginal
revenue is
a. $15.
b. $30.
c. $90.
d. $105.

7. In perfect competition, the marginal revenue of an individual firm


a. is zero.
b. is positive but less than the price of the product.
c. equals the price of the product.
d. exceeds the price of the product.

8. A perfectly competitive firm’s marginal revenue exceeds its marginal cost at its
current output. To increase its profit, the firm will
a. lower its price.
b. raise its price.
c. decrease its output.
d. increase its output.
Price and costs (dollars)
MC

ATC
P1

0 Q1 Q2 Q3
Quantity (per day)

9. In the above figure, if the firm increases its output from Q2 to Q3, it will
a. reduce its marginal revenue.
b. increase its marginal revenue.
c. decrease its profit.
d. increase its profit.

10. New reports indicate that eating turnips helps people remain healthy. The news
shifts the demand curve for turnips rightward. In response, new farms enter the
turnip industry. During the period in which the new farms are entering, the price
of a turnip ____ and the profit of each existing firm ____.
a. rises; rises
b. rises; falls
c. falls; rises
d. falls; falls
Price and costs (dollars)
MC

ATC

P1

0
Quantity (per day)

11. Suppose the cost curves in the above figure apply to all firms in the industry. If
the initial price is P1, firms are
a. making an economic profit and some firms will leave the industry.
b. making an economic profit and some firms will enter the industry.
c. incurring an economic loss and some firms will leave the industry.
d. incurring an economic loss and some firms will enter the industry.

12. If there are external economies, as demand increases,


a. the price falls in the long run.
b. the price rises in the long run.
c. firms exit from the industry in the long run.
d. output decreases in the long run.

S2
Price and costs (dollars)

LS0
S0
P2

D2
P0

D0

0 Q0 Q2
Quantity (per day)
13. The curve LS0 in the above figure is the long-run supply curve of a perfectly
competitive industry. As the demand curve shifts rightward, the industry exhibits
a. external economies.
b. external diseconomies.
c. neither external economies nor external diseconomies.
d. technological advancement.

Output Total cost


(tons of rice per (dollars per ton)
year)
0 $1,000
1 $1,200
2 $1,600
3 $2,200
4 $3,000
5 $4,000

14. Based on the table above which shows Chip’s costs, if rice sells for $600 a ton,
Chip will
a. shut down because he incurs an economic loss.
b. shut down because the price is below his minimum average variable cost.
c. stay open because he earns an economic profit.
d. stay open because the price is above his minimum average variable cost.

15. Based on the table above which shows Chip’s costs, if rice sells for $600 a ton,
Chip’s profit-maximizing output is
a. less than one ton.
b. between two and three tons.
c. between three and four tons.
d. between one and two tons.
Chapter 4 - Elasticity - Sample Questions

MULTIPLE CHOICE. Choose the one alternative that best completes the statement or answers the question.

1) The slope of a demand curve depends on 1)


A) the units used to measure quantity but not the units used to measure price.
B) the units used to measure price and the units used to measure quantity.
C) the units used to measure price but not the units used to measure quantity.
D) neither the units used to measure price nor the units used to measure quantity.

2) The price elasticity of demand depends on 2)


A) the units used to measure price but not the units used to measure quantity.
B) the units used to measure price and the units used to measure quantity.
C) the units used to measure quantity but not the units used to measure price.
D) neither the units used to measure price nor the units used to measure quantity.

3) The price elasticity of demand measures 3)


A) the slope of a budget curve.
B) how often the price of a good changes.
C) the responsiveness of the quantity demanded to changes in price.
D) how sensitive the quantity demanded is to changes in demand.

4) When the quantity of coal supplied is measured in kilograms instead of pounds, the demand for 4)
coal becomes
A) more elastic. B) neither more nor less elastic.
C) less elastic. D) undefined.

5) The price elasticity of demand equals 5)


A) the percentage change in the quantity demanded divided by the percentage change in the
price.
B) the change in the quantity demanded divided by the change in price.
C) the percentage change in the price divided by the percentage change in the quantity
demanded.
D) the change in the price divided by the change in quantity demanded.

6) If a rightward shift of the supply curve leads to a 6 percent decrease in the price and a 5 percent 6)
increase in the quantity demanded, the price elasticity of demand is
A) 0.83. B) 0.30. C) 0.60. D) 1.20.

7) A 10 percent increase in the quantity of spinach demanded results from a 20 percent decline in its 7)
price. The price elasticity of demand for spinach is
A) 0.5. B) 20.0. C) 2.0. D) 10.0.

8) A 20 percent increase in the quantity of pizza demanded results from a 10 percent decline in its 8)
price. The price elasticity of demand for pizza is
A) 2.0. B) 10.0. C) 0.5. D) 20.0.

1
9) Suppose a rise in the price of peaches from $5.50 to $6.50 per bushel decreases the quantity 9)
demanded from 12,500 to 11,500 bushels. The price elasticity of demand is
A) 0.5. B) 1000.0. C) 2.0. D) 1.0.

10) A fall in the price of lemons from $10.50 to $9.50 per bushel increases the quantity demanded from 10)
19,200 to 20,800 bushels. The price elasticity of demand is
A) 1.25. B) 1.20. C) 8.00. D) 0.80.

11) A fall in the price of cabbage from $10.50 to $9.50 per bushel increases the quantity demanded from 11)
18,800 to 21,200 bushels. The price elasticity of demand is
A) 1.20. B) 0.80. C) 8.00. D) 1.25.

12) Suppose that the quantity of root beer demanded declines from 103,000 gallons per week to 97,000 12)
gallons per week as a consequence of a 10 percent increase in the price of root beer. The price
elasticity of demand is
A) 1.66. B) 6.00. C) 0.60. D) 1.40.

13) The price elasticity of demand is 5.0 if a 10 percent increase in the price results in a ________ 13)
decrease in the quantity demanded.
A) 10 percent B) 50 percent C) 2 percent D) 5 percent

14) A shift of the supply curve of oil raises the price of oil from $9.50 a barrel to $10.50 a barrel and 14)
reduces the quantity demanded from 41 million to 39 million barrels a day. The price elasticity of
demand for oil is
A) 2 million barrels a day per dollar. B) 0.5.
C) $1 per 2 million barrels a day. D) 2.0.

Price Quantity demanded


(dollars per bushel) (bushels)
8 2,000
7 4,000
6 6,000
5 8,000
4 10,000
3 12,000

15) The table above gives the demand schedule for snow peas. The price elasticity of demand between 15)
$6.00 and $7.00 per bushel is
A) 1.0. B) 5.0. C) 2.0. D) 2.6.

16) The table above gives the demand schedule for snow peas. If the price of snow peas falls from 16)
$4.00 to $3.00 a bushel, total revenue will
A) increase because demand is elastic in this range.
B) increase because demand is inelastic in this range.
C) decrease because demand is inelastic in this range.
D) decrease because demand is elastic in this range.

2
17) The table above gives the demand schedule for snow peas. The demand curve for snow peas is a 17)
straight line and so the elasticity of demand is
A) lower at higher prices. B) higher at higher prices.
C) 1 at all prices. D) the same at all prices but not 1.

Price Quantity demanded


(dollars per bushel) (bushels)
A 10 0
B 8 4
C 6 8
D 4 12
E 2 16

18) The table above gives the demand schedule for peas. As you move from point A to point B, the 18)
price elasticity of demand equals
A) 0.50. B) 0.11. C) 9.09. D) 0.22.

19) The table above gives the demand schedule for peas. As you move from point C to point D, the 19)
price elasticity of demand is
A) 3.00. B) elastic. C) 0.75. D) unit elastic.

20) The table above gives the demand schedule for peas. Which of the following statements correctly 20)
describes the price elasticity of demand?
A) The price elasticity of demand is larger at point A than at point B.
B) The price elasticity of demand is constant because the slope is constant.
C) The price elasticity of demand increases moving from point A to point B to point C to point D
to point E.
D) The price elasticity of demand is larger at point D than at point A.

21) If demand is price elastic, 21)


A) a 1 percent decrease in the price leads to an increase in the quantity demanded that exceeds 1
percent.
B) a 1 percent increase in the price leads to an increase in the quantity demanded that exceeds 1
percent.
C) the price is very sensitive to any shift of the supply curve.
D) a 1 percent decrease in the price leads to a decrease in the quantity demanded that is less than
1 percent.

22) The price elasticity of demand can range between 22)


A) negative one and one. B) zero and infinity.
C) zero and one. D) negative infinity and infinity.

23) Demand is perfectly inelastic when 23)


A) the good in question has perfect substitutes.
B) shifts in the supply curve results in no change in price.
C) shifts of the supply curve results in no change in quantity demanded.
D) shifts of the supply curve results in no change in the total revenue from sales.

3
24) If the price elasticity is between 0 and 1, demand is 24)
A) inelastic. B) elastic. C) perfectly elastic. D) unit elastic.

25) Demand is inelastic if 25)


A) a large change in quantity demanded results in a small change in price.
B) the price elasticity of demand is greater than 1.
C) the quantity demanded is very responsive to changes in price.
D) the price elasticity of demand is less than 1.

26) A good with a vertical demand curve has a demand with 26)
A) infinite elasticity. B) unit elasticity.
C) zero elasticity. D) varying elasticity.

27) The demand curve in the figure above illustrates the demand for a product with 27)
A) unit price elasticity of demand at all prices.
B) a price elasticity of demand that is different at all prices.
C) infinite price elasticity of demand.
D) zero price elasticity of demand at all prices.

28) When the price elasticity of demand for a good equals 28)
A) 0, the demand curve is horizontal. B) 1, the demand curve is vertical.
C) 1, the demand curve is horizontal. D) 0, the demand curve is vertical.

29) A straight-line demand curve along which the price elasticity of demand equals 0 is one that 29)
A) forms a 45 degree angle with the vertical axis.
B) is horizontal.
C) is vertical.
D) forms a 60 degree angle with the horizontal axis.

4
30) The demand for movies is unit elastic if 30)
A) any increase in the price leads to a 1 percent decrease in the quantity demanded.
B) a 5 percent decrease in the price leads to an infinite increase in the quantity demanded.
C) a 5 percent increase in the price leads to a 5 percent decrease in the quantity demanded.
D) a 5 percent increase in the price leads to a 5 percent increase in total revenue.

31) Unit elastic demand 31)


A) means that the ratio of a change in the quantity demanded to a change in the price equals 1.
B) will be vertical.
C) means that the ratio of a percentage change in the quantity demanded to a percentage change
in the price equals 1.
D) will be horizontal.

32) A good with a horizontal demand curve has a demand 32)


A) with an income elasticity of demand of 0.
B) with a price elasticity of demand of infinity.
C) for which there are no substitute.
D) with a price elasticity of demand of 0.

33) The demand curve in the figure above illustrates a product whose demand has a price elasticity of 33)
demand equal to
A) infinity. B) zero at all prices.
C) a different amount at different prices. D) one at all prices.

5
34) The demand curve in the figure above illustrates the demand for a product with 34)
A) zero price elasticity of demand at all prices.
B) a price elasticity of demand that is different at all prices.
C) unit price elasticity of demand at all prices.
D) infinite price elasticity of demand.

35) On a linear demand curve that intersects both axes, 35)


A) the elasticity decreases as the price falls and quantity increases.
B) the elasticity is less than 1.00 at all prices.
C) the elasticity equals 1.00 at all prices.
D) the elasticity exceeds 1.00 at all prices.

36) On a straight-line downward-sloping demand curve, the maximum elasticity of demand occurs 36)
A) where it intersects the supply curve. B) at its vertical intercept.
C) at its horizontal intercept. D) at its midpoint.

37) A straight-line demand curve with negative slope intersects the horizontal axis at 100 tons per 37)
week. At the midpoint on the demand curve (corresponding to 50 tons per week) the price
elasticity of demand is
A) greater than 1.0. B) 0.5. C) 1.0. D) 0.

6
38) The figure above illustrates a linear demand curve. By comparing the price elasticity in the $2 to $4 38)
price range with the elasticity in the $8 to $10 range, you can conclude that the elasticity is
A) the same in both price ranges.
B) greater in the $8 to $10 range when the price rises but greater in the $2 to $4 range when the
price falls.
C) greater in the $8 to $10 range.
D) greater in the $2 to $4 range.

39) The figure above illustrates a linear demand curve. If the price falls from $8 to $6, 39)
A) the quantity demanded will increase by less than 20 percent.
B) total revenue will remain unchanged.
C) total revenue will increase.
D) total revenue will decrease.

40) The figure above illustrates a linear demand curve. In the range from $8 to $6, 40)
A) the demand is unit elastic.
B) the demand is price inelastic.
C) the demand is price elastic.
D) more information is needed to determine if the demand is price elastic, unit elastic, or
inelastic.

41) The figure above illustrates a linear demand curve. If the price falls from $6 to $4, 41)
A) total revenue will decrease.
B) total revenue will increase.
C) quantity demanded will increase by more than 100 percent.
D) total revenue will remain unchanged.

7
42) The figure above illustrates a linear demand curve. In the price range from $8 to $6, demand is 42)
________ and in the price range $4 to $2, demand is ________.
A) elastic; inelastic B) inelastic; inelastic
C) elastic; elastic D) inelastic; elastic

43) The figure above illustrates a linear demand curve. If the price rises from $6 to $8 demand is 43)
________ and if the price falls from $8 to $6 demand is ________.
A) inelastic; inelastic B) elastic; inelastic
C) elastic; elastic D) inelastic; elastic

44) The demand curve in the figure above illustrates the demand for a product with 44)
A) zero price elasticity of demand at all prices.
B) a price elasticity of demand that is different at all prices.
C) unit price elasticity of demand at all prices.
D) infinite price elasticity of demand.

45) A straight-line demand curve with negative slope intersects the horizontal axis at 200 tons per 45)
week. The point on the demand curve at which the price elasticity of demand is 1 corresponds to a
quantity demanded
A) that would be negative if a negative quantity demanded were possible.
B) of 100 tons.
C) of 0 tons.
D) of 200 tons.

8
46) Demand is inelastic if 46)
A) a leftward shift of the supply curve raises the total revenue.
B) the good in question has close substitutes.
C) the smaller angle between the vertical axis and the demand curve is less than 45 degrees.
D) large shifts of the supply curve lead to only small changes in price.

47) Demand is unit elastic when 47)


A) a shift of the supply curve leads to no change in price.
B) the slope of the demand curve is -1.
C) a change in the price of the product leads to no change in the total revenue.
D) a shift of the supply curve leads to an equal shift of the demand curve.

48) Producers' total revenue will decrease if 48)


A) the price rises and demand is inelastic.
B) income increases and the good is a normal good.
C) the price rises and demand is elastic.
D) income falls and the good is an inferior good.

49) Producers' total revenue will increase if 49)


A) income falls and the good is a normal good.
B) the price rises and demand is inelastic.
C) the price rises and demand is elastic.
D) income increases and the good is an inferior good.

50) If the demand for a good is unit elastic, 50)


A) a 5 percent increase in price results in a 5 percent increase in total revenue.
B) the demand curve is a straight line with slope of -1.
C) a 5 percent increase in price results in a 5 percent decrease in total revenue.
D) a 5 percent increase in price does not change total revenue.

51) A shift of the supply curve of oil raises the price from $10 a barrel to $30 a barrel and reduces the 51)
quantity demanded from 40 million to 23 million barrels a day. You can conclude that the
A) supply of oil is elastic. B) supply of oil is inelastic.
C) demand for oil is inelastic. D) demand for oil is elastic.

52) A shift of the supply curve of oil raises the price from $10 a barrel to $15 a barrel and reduces the 52)
quantity demanded from 40 million to 15 million barrels a day. You can conclude that the
A) demand for oil is elastic. B) supply of oil is elastic.
C) supply of oil is inelastic. D) demand for oil is inelastic.

53) A leftward shift of the supply curve of cookies raises the price of a cookie from 10 cents to 20 cents 53)
and decreases the quantity demanded from 700,000 to 500,000. You can conclude that
A) the supply of cookies is elastic. B) the supply of cookies is inelastic.
C) the demand for cookies is elastic. D) the demand for cookies is inelastic.

9
54) The demand for a good is elastic if 54)
A) a decrease in its price results in a decrease in total revenue.
B) the good is a necessity.
C) an increase in its price results in an increase in total revenue.
D) an increase in its price results in a decrease in total revenue.

55) If a price decrease results in your expenditure on a good decreasing, your demand must be 55)
A) unit. B) inelastic. C) linear. D) elastic.

56) An increase in subway fares in New York City will boost your expenditures on subway rides if 56)
A) the supply of subway rides is elastic. B) the supply of subway rides is inelastic.
C) your demand for subway rides is inelastic. D) your demand for subway rides is elastic.

57) The more substitutes available for a product, 57)


A) the larger is its income elasticity of demand.
B) the smaller is its income elasticity of demand.
C) the smaller is its price elasticity of demand.
D) the larger is its the price elasticity of demand.

58) Of the following, demand is likely to be the least elastic for 58)
A) Toyota automobiles. B) compact disc players.
C) Ford automobiles. D) toothpicks.

59) Of the following, demand is likely to be the least elastic for 59)
A) pink grapefruit. B) iceberg lettuce.
C) insulin for diabetics. D) diamonds.

60) The demand for food is most elastic in countries 60)


A) with low income levels. B) that are highly urbanized.
C) with intermediate income levels. D) with high income levels.

61) The demand for Honda Accords is 61)


A) probably inelastic and less elastic than the demand for automobiles.
B) probably elastic but less elastic than the demand for automobiles.
C) probably elastic and more elastic than the demand for automobiles.
D) probably inelastic but more elastic than the demand for automobiles.

62) The route from Dallas to Mexico City is served by more than one airline. The demand for tickets 62)
from American Airlines for that route is probably
A) elastic and more elastic than the demand for all tickets for that route.
B) inelastic and less elastic than the demand for all tickets for that route.
C) elastic but less elastic than the demand for all tickets for that route.
D) inelastic but more elastic than the demand for all tickets for that route.

10
63) The elasticity of demand for Gateway computers is probably 63)
A) elastic and smaller than the elasticity of demand for computers overall.
B) inelastic and smaller than the elasticity of demand for computers overall.
C) inelastic but larger than the elasticity of demand for computers overall.
D) elastic and larger than the elasticity of demand for computers overall.

64) Aglets are the metal or plastic tips on shoelaces that make it easier to lace your shoes. The demand 64)
for aglets is probably
A) perfectly elastic. B) inelastic.
C) elastic but not perfectly elastic. D) unit elastic.

65) The cross elasticity of demand measures the responsiveness of the quantity demanded of a 65)
particular good to changes in the prices of
A) its complements but not its substitutes.
B) its substitutes but not its complements.
C) its substitutes and its complements.
D) neither its substitutes nor its complements.

66) If goods are complements, definitely their 66)


A) income elasticities are negative. B) income elasticities are positive.
C) cross elasticities are positive. D) cross elasticities are negative.

67) If a rise in the price of good 1 decreases the quantity of good 2 demanded, 67)
A) the cross elasticity of demand is negative. B) good 1 is an inferior good.
C) good 2 is an inferior good. D) the cross elasticity of demand is positive.

68) The cross elasticity of demand between apples and oranges is defined as 68)
A) the price elasticity of demand for apples divided by the price elasticity of demand for oranges.
B) the change in the quantity of apples demanded divided by the change in the quantity of
oranges demanded.
C) the percentage change in the quantity of apples demanded divided by the percentage change
in the price of oranges.
D) the percentage change in the quantity of apples demanded divided by the percentage change
in the quantity of oranges demanded.

69) If the cross elasticity of demand between goods A and B is positive, 69)
A) the demands for A and B are both price elastic.
B) A and B are complements.
C) A and B are substitutes.
D) the demands for A and B are both price inelastic.

11
70) If the cross elasticity of demand between goods A and B is negative, 70)
A) the demands for A and B are both price elastic.
B) A and B are complements.
C) the demands for A and B are both price inelastic.
D) A and B are substitutes.

71) The greater the substitutability between Northwest timber and Southeast timber, the ________ is 71)
the cross elasticity of demand between timber from the two regions and the ________ is the
elasticity of demand for Northwest timber.
A) smaller; smaller B) larger; smaller C) smaller; larger D) larger; larger

72) If goods A and B are complements, 72)


A) the cross elasticity of demand between A and B is negative.
B) the cross elasticity of demand between A and B is positive.
C) their income elasticities of demand are both less than 1.
D) their income elasticities of demand are both greater than 1.

73) If a rise in the price of good B increases the quantity demanded of good A, 73)
A) B is a substitute for A, but A is a complement to B.
B) A is a substitute for B, but B is a complement to A.
C) A and B are complements.
D) A and B are substitutes.

74) If a fall in the price of good A increases the quantity demanded of good B, 74)
A) A and B are substitutes.
B) A and B are complements.
C) B is a substitute for A, but A is a complement to B.
D) A is a substitute for B, but B is a complement to A.

75) The cross elasticity of demand between Coca-Cola and Pepsi-Cola is 75)
A) positive, that is, Coke and Pepsi are complements.
B) negative, that is, Coke and Pepsi are complements.
C) positive, that is, Coke and Pepsi are substitutes.
D) negative, that is, Coke and Pepsi are substitutes.

76) A rise in the price of good A will shift the 76)


A) supply curve of good B rightward if the cross elasticity of demand between A and B is
positive.
B) demand curve for good B rightward if the cross elasticity of demand between A and B is
negative.
C) demand curve for good B rightward if the cross elasticity of demand between A and B is
positive.
D) supply curve of good B rightward if the cross elasticity of demand between A and B is
negative.

12
77) The income elasticity of demand is the percentage change in 77)
A) income divided by the percentage change in price.
B) the quantity demanded divided by the percentage change in income.
C) the price divided by the percentage change in income.
D) income divided by the percentage change in quantity demanded.

78) Demand is income elastic if 78)


A) an increase in income will not affect the quantity demanded.
B) a small percentage increase in income will result in a large percentage increase in quantity
demanded.
C) the good in question has close substitutes.
D) a large percentage increase in income will result in a small percentage increase in quantity
demanded.

79) The income elasticity of demand is high for 79)


A) shelter. B) luxuries. C) clothing. D) food.

80) To say that turnips are inferior goods means that the income elasticity 80)
A) is definitely greater than 1.
B) is negative.
C) is positive but could be greater than or less then (or equal to) 1.
D) is definitely between 0 and 1.

81) An increase in Abigail's income decreases her demand for cassette tapes. For her, cassette tapes are 81)
A) a complement to any good. B) a normal good.
C) an inferior good. D) a substitute good.

82) Goods whose income elasticities are negative are called 82)
A) superior goods. B) inferior goods. C) normal goods. D) complements.

83) A 10 percent increase in income has caused a 5 percent decrease in the quantity demanded. The 83)
income elasticity is
A) 0.5. B) -2.0. C) 2.0. D) -0.5.

84) Deb's income has just risen from $950 per week to $1,050 per week. As a result, she decides to 84)
increase the number of movies she attends each month by 5 percent. Her demand for movies is
A) income inelastic. B) income elastic.
C) represented by a vertical line. D) represented by a horizontal line.

85) Fred's income has just risen from $940 per week to $1,060 per week. As a result, he decides to 85)
purchase 9 percent more steak per week. The income elasticity of Fred's demand for steak is
A) 0.75. B) 1.33. C) 0.90. D) 1.00.

86) Joan's income has just risen from $940 per week to $1,060 per week. As a result, she decides to 86)
purchase 12 percent more lettuce per week. The income elasticity of Joan's demand for lettuce is
A) 1.33. B) 0.90. C) 1.00. D) 0.75.

13
87) A 10 percent increase in income causes the quantity of orange juice demanded to increase from 87)
19,200 to 20,800 gallons. The income elasticity of demand for orange juice is
A) 0.8. B) 1.2. C) 1.0. D) 0.5.

88) A 10 percent increase in income causes the quantity of apple juice demanded to increase from 88)
18,800 to 21,200 gallons. The income elasticity of demand for apple juice is
A) 0.5. B) 1.0. C) 1.2. D) 0.8.

89) The above figure shows a good 89)


A) that is an inferior good over all income ranges.
B) whose income elasticity is greater than 0 but less than 1.
C) that is a normal good over some income ranges and an inferior good over other ranges.
D) whose income elasticity always exceeds 1.0.

90) Of the following, which one is most likely to have a negative income elasticity of demand? 90)
A) shoes B) tennis balls
C) inter-city bus travel D) frozen yogurt

14
91) The above figure shows a good 91)
A) whose income elasticity is greater than 0 but less than 1.
B) that is an inferior good over all income ranges.
C) whose income elasticity always exceeds 1.0.
D) that is a normal good over some income ranges and an inferior good over other ranges.

92) The above figure shows a good 92)


A) whose income elasticity always exceeds 1.0.
B) whose income elasticity is greater than 0 but less than 1.
C) that is an inferior good over all income ranges.
D) that is a normal good over some income ranges and an inferior good over other ranges.

15
93) The increase in the demand for widgets, shown in the figure above, is caused by an increase in the 93)
price of McBoover devices. Therefore,
A) widgets and McBoover devices are substitutes.
B) widgets and McBoover devices are complements.
C) McBoover devices are a normal good.
D) widgets are a normal good.

94) The increase in the demand for widgets, shown in the figure above, is caused by a decrease in the 94)
price of McBoover devices. Therefore,
A) widgets and McBoover devices are substitutes.
B) widgets are a normal good.
C) McBoover devices are a normal good.
D) widgets and McBoover devices are complements.

95) The increase in the demand for widgets, shown in the figure above, is caused by an increase in the 95)
price of McBoover devices from $9 to $11. Therefore, the cross-price elasticity for these two
products is
A) 0.5. B) -2.0. C) 2.0. D) -0.5.

96) The increase in the demand for widgets, shown in the figure above, is caused by a decrease in the 96)
price of McBoover devices from $11 to $9. Therefore, the cross-price elasticity for these two
products is
A) -2.0. B) 2.0. C) -0.5. D) 0.5.

97) The increase in the demand for widgets, shown in the figure above, is caused by an increase in 97)
average incomes. Therefore, widgets
A) are a normal good. B) are elastically demanded.
C) are an inferior good. D) are inelastically demanded.

16
98) The increase in the demand for widgets, shown in the figure above, is caused by an increase in 98)
average incomes from $28,500 per year to $31,500 per year. Therefore, the income elasticity of
demand for widgets is
A) 4. B) 3/4. C) 1/4. D) 4/3.

99) As income rises, the share of income spent on food in the United States 99)
A) rises. B) remains constant at 15 percent.
C) falls. D) remains constant at 33 percent.

100) The elasticity of supply measures the responsiveness of 100)


A) quantity supplied to changes in price. B) quantity demanded to changes in supply.
C) quantity supplied to changes in income. D) quantity supplied to changes in demand.

101) The elasticity of supply measures the sensitivity of 101)


A) supply to changes in costs. B) quantity supplied to a change in price.
C) price to changes in supply. D) quantity supplied to quantity demanded.

102) On most days the price of a rose is $1 and 80 roses are purchased. On Valentine's Day the demand 102)
increases so that the price of a rose rises to $2 and 320 roses are purchased. Therefore, the price
elasticity of
A) demand for roses is about 1.8. B) supply of roses is about 1.8.
C) demand for roses is about 0.55. D) supply of roses is about 0.55.

103) Supply is elastic if 103)


A) a 1 percent change in price causes a larger percentage change in quantity supplied.
B) the good in question is a normal good.
C) the slope of the supply curve is positive.
D) a 1 percent change in price causes a smaller percentage change in quantity supplied.

104) If a 1 percent decrease in the price of a pound of oranges results in a smaller percentage decrease in 104)
the quantity supplied,
A) supply is inelastic. B) demand is inelastic.
C) demand is elastic. D) supply is elastic.

105) If a 1 percent decrease in the price of a pound of squash results in a larger percentage decrease in 105)
the quantity supplied,
A) demand is inelastic. B) demand is elastic.
C) supply is inelastic. D) supply is elastic.

106) If at a given moment, no matter what the price, producers cannot change the quantity supplied, the 106)
momentary supply
A) has infinite elasticity. B) has unit elasticity.
C) does not exist. D) has zero elasticity.

17
107) If a rise in the price of oranges from $7 to $9 a bushel, caused by a shift of the demand curve, 107)
increases the quantity of bushels supplied from 4,500 to 5,500 bushels, the
A) demand for oranges is elastic. B) supply of oranges is elastic.
C) demand for oranges is inelastic. D) supply of oranges is inelastic.

108) If a shift in the demand curve that raises the price of oranges from $7 to $9 a bushel increases the 108)
quantity of oranges supplied from 4,000 bushels to 6,000 bushels, the
A) supply of oranges is elastic. B) supply of oranges is inelastic.
C) demand for oranges is inelastic. D) demand for oranges is elastic.

109) A rise in the price of cabbage from $14 to $18 per bushel, caused by a shift of the demand curve, 109)
increases the quantity supplied from 4,000 to 6,000 bushels. The elasticity of supply is
A) 1.6. B) 1.0. C) 0.6. D) 0.8.

110) If a 5 percent increase in the price results in a 9 percent increase in quantity supplied, the elasticity 110)
of supply is
A) 0.30. B) 0.55. C) 1.80. D) 1.20.

111) If a 5 percent increase in price results in a 3 percent increase in the quantity supplied, the elasticity 111)
of supply is
A) 1.20. B) 0.60. C) 1.66. D) 0.30.

112) A vertical supply curve indicates an elasticity of supply that equals 112)
A) 0. B) infinity. C) 1. D) -1.

113) A horizontal supply curve indicates an elasticity of supply that equals 113)
A) 0. B) infinity. C) 1. D) -1.

114) Suppose a 10 percent increase in the price of textbooks decreases the quantity demanded by 20 114)
percent. The elasticity of demand for textbooks is
A) 0.2. B) 5.0. C) 10.0. D) 2.0.

115) The quantity of new cars increases by 10 percent. If the price elasticity of demand for new cars is 115)
1.25, the price of new cars will fall by
A) 8 percent. B) 10 percent. C) 2.5 percent. D) 12.5 percent.

116) Suppose the price elasticity of demand for oil is 0.1. In order to lower the price of oil by 20 percent, 116)
the quantity of oil supplied must be increased by
A) 20 percent. B) 2 percent. C) 0.2 percent. D) 200 percent.

117) Moving up (to the left) along a linear demand curve, the price elasticity of demand 117)
A) at first increases and then decreases. B) increases.
C) decreases. D) does not change.

118) If the price elasticity of demand for a product equals 1, as its price rises the 118)
A) total revenue increases. B) quantity demanded does not change.
C) total revenue does not change. D) quantity demanded increases.

18
119) A rise in the price of a product lowers the total revenue from the product if the 119)
A) good is an inferior product. B) demand for the product is inelastic.
C) demand for the product is elastic. D) income elasticity of demand exceeds 1.

120) If a 4 percent rise in the price of peanut butter lowers the total revenue received by the producers 120)
of peanut butter by 4 percent, the demand for peanut butter
A) is inelastic. B) is elastic.
C) is unit elastic. D) has an elasticity of 2.0.

121) A product is likely to have a price elasticity of demand that exceeds 1 when 121)
A) its price falls.
B) it is a necessity.
C) it has close substitutes.
D) the percentage of income spent on it decreases.

122) Which of the following is likely to have the smallest price elasticity of demand? 122)
A) a new Ford automobile B) a new automobile
C) a new Ford Mustang D) an automobile

123) A 10 percent decrease in the price of a Pepsi decreases the demand for a Coca-Cola by 50 percent. 123)
The cross elasticity of demand between a Pepsi and Coca-Cola is
A) 5. B) 10. C) 0.20. D) 50.

124) A fall in the price of X from $12 to $8 causes an increase in the quantity of Y demanded from 900 to 124)
1,100 units. What is the cross elasticity of demand between X and Y?
A) 2 B) -0.5 C) -2 D) 0.5

125) A fall in the price of X from $12 to $8 causes an increase in the quantity of Y demanded from 900 to 125)
1,100 units. X and Y are
A) complements. B) normal goods. C) substitutes. D) inferior goods.

126) A 10 percent decrease in income decreases the quantity demanded of compact discs by 3 percent. 126)
The income elasticity of demand for compact discs is
A) 10.0. B) 3.3. C) -0.3. D) 0.3.

19
127) In the figure above, when the price of a disk is $B, total revenue is shown in the graph by area 127)
A) FCDE. B) ADE0. C) AGF0. D) BCF0.

128) The above figure illustrates the demand curve for a good. The good has 128)
A) many substitutes. B) no substitutes.
C) only one substitute. D) only a few substitutes.

20
129) The elasticity of demand along the demand curve shown in the above figure is constant and equal 129)
to 1. Thus,
A) area 0BCF equals area 0AGF. B) area 0BCF equals area 0ADE.
C) area 0BCF equals area FGDE. D) area ABCG equals area 0AGF.

130) The above figure shows a linear (straight-line) demand curve. Start at point A and then moving to 130)
point B and then point C, the price elasticity of demand
A) increases. B) increases and then decreases.
C) decreases and then increases. D) decreases.

21
Answer Key
Testname: UNTITLED2.TST

1) B
2) D
3) C
4) B
5) A
6) A
7) A
8) A
9) A
10) D
11) A
12) C
13) B
14) B
15) D
16) C
17) B
18) C
19) D
20) A
21) A
22) B
23) C
24) A
25) D
26) C
27) D
28) D
29) C
30) C
31) C
32) B
33) A
34) C
35) A
36) B
37) C
38) C
39) C
40) C
41) D
42) A
43) C
44) B
45) B
46) A
47) C
48) C
49) B
50) D
1
Answer Key
Testname: UNTITLED2.TST

51) C
52) A
53) D
54) D
55) B
56) C
57) D
58) D
59) C
60) A
61) C
62) A
63) D
64) B
65) C
66) D
67) A
68) C
69) C
70) B
71) D
72) A
73) D
74) B
75) C
76) C
77) B
78) B
79) B
80) B
81) C
82) B
83) D
84) A
85) A
86) C
87) A
88) C
89) D
90) C
91) A
92) D
93) A
94) D
95) C
96) A
97) A
98) A
99) C
100) A
2
Answer Key
Testname: UNTITLED2.TST

101) B
102) B
103) A
104) A
105) D
106) D
107) D
108) A
109) A
110) C
111) B
112) A
113) B
114) D
115) A
116) B
117) B
118) C
119) C
120) B
121) C
122) D
123) A
124) B
125) A
126) D
127) D
128) B
129) B
130) D

3
e
b ut
tri
is
rd
C H A PT E R

4
o
t,
os

Demand, Supply, and


Market Equilibrium
,p
py
co

LEARNING OUTCOMES
After reading this chapter, you should be able to
t
no

4.1 Describe and explain why buyers and sellers participate 4.4 Define and explain the law of supply.
in markets.
4.5 Discuss shifts in market supply.
4.2 Define and explain the law of demand.
4.6 Explain how the market equilibrium price and quantity
o

4.3 Discuss shifts in market demand. are determined.


D

Master the content.


edge.sagepub.com/sextonexploreecon8e

Ian Dagnall / Alamy Stock Photo

Copyright ©2020 by SAGE Publications, Inc.


This work may not be reproduced or distributed in any form or by any means without express written permission of the publisher.
Chapter 4  •  Demand, Supply, and Market Equilibrium 95  

Every morning fishermen bring in their daily catch. Along the pier, they negotiate with fish
­brokers—sellers find buyers, and buyers find sellers. Supply and demand is without a doubt the
most powerful tool in the economist’s toolbox. It can help explain much of what goes on in the
world and help predict what will happen tomorrow. In this chapter, we will learn about the law of
demand and the law of supply and the factors that can change supply and demand.
We then bring market supply and market demand together to determine equilibrium price and
quantity. We also learn how markets with many buyers and sellers adjust to temporary shortages
and surpluses.

e
ut
4.1 Markets
• What is a market?

b
• Why is it so difficult to define a market?

tri
4.1a Defining a Market

is
Although we usually think of a market as a market: the process
place where some sort of exchange occurs, a of buyers and sellers
exchanging goods and
market is not really a place at all. A market

rd
services
is the process of buyers and sellers exchanging
goods and services. Supermarkets, the New

Spencer Platt/Getty Images


York Stock Exchange, drug stores, roadside

o
stands, garage sales, Internet stores, and restau-
rants are all markets.
Every market is different. That is, the con-
ditions under which the exchange ­ between
t,
Do markets have to be
buyers and sellers takes place can vary. These physical places?
os
­differences make it difficult to precisely d­ efine iStock/skynesher
a market. After all, an incredible variety • The stock market involves many buyers and sellers,
and profit statements and stock prices are readily
of ­ exchange arrangements exist in the real available. New information is quickly understood by
­ ECS
,p

world—organized securities markets, wholesale ­buyers and sellers and is incorporated into the price of
auction markets, foreign exchange m ­ arkets, the stock. When people expect a company to do better economic content
real estate markets, labor markets, and so forth. in the f­uture, the price of the stock rises; when people standards
The ­important point is not what a market looks expect the ­company to do poorly in the future, the price
py

like, but what it does—it facilitates trade. of the stock falls. • Prices send signals
and provide incentives
to buyers and sellers.
4.1b Buyers and Sellers When supply or demand
co

The roles of buyers and sellers in markets are changes, market


important. Buyers, as a group, determine the prices adjust, affecting
demand side of the market. Buyers include incentives. Understanding
the consumers who purchase the goods and the role of prices as
t

services and the firms that buy inputs—labor, signals and incentives
no

capital, and raw materials. Sellers, as a group, helps people anticipate


determine the supply side of the market. Sellers market opportunities
include the firms that produce and sell goods and make better choices
and services and the resource owners who sell as producers and
iStock/NicolasMcComber
o

their inputs to firms—workers who “sell” their consumers.


labor and resource owners who sell raw mate-
D

rials and capital. The interaction of buyers and


competitive mar-
sellers determines market prices and outputs—
ket: a market where
through the forces of supply and demand.
the many buyers and
In the next few chapters, we will focus on sellers have little
• eBay is an Internet auction company that brings ­together
how supply and demand work in a competi- millions of buyers and sellers from all over the world. The market power—each
tive market. A competitive market is one in gains from these mutually beneficial exchanges are large. buyer’s or seller’s
which many buyers and sellers are offering Craigslist also uses the power of the Internet to connect effect on market price
similar products, and no single buyer or seller many buyers and sellers in local markets. • is negligible

Copyright ©2020 by SAGE Publications, Inc.


This work may not be reproduced or distributed in any form or by any means without express written permission of the publisher.
96    PART 2  •  SUPPLY AND DEMAND

can influence the market price. That is, buyers and sellers have little market power. Because many
markets contain a high degree of competitiveness, the lessons of supply and demand can be applied
to many different types of problems.
The supply and demand model is particularly useful in markets such as agriculture, finance,
labor, construction, services, wholesale, and retail.
In short, a model is only as good as how well it explains and predicts. The model of supply
and demand is very good at predicting changes in prices and quantities in many markets, large and
small.

e
b ut
SECTION QUIZ

tri
1. Which of the following is a market?

is
a. a garage sale
b. a restaurant

rd
c. the New York Stock Exchange
d. an eBay auction
e. all of the above

o
2. In a competitive market,
a. there are numerous buyers and sellers.
t,
b. no single buyer or seller can appreciably affect the market price.
c. sellers offer similar products.
os
d. all of the above are true.
3. Buyers determine the _________ side of the market; sellers determine the _________ side of the market.
a. demand; demand
,p

b. demand; supply
c. supply; demand
d. supply; supply
py

1. Why is it difficult to define a market precisely?


co

2. Why do you get your produce at a supermarket rather than directly from farmers?
3. Why do the prices people pay for similar items at garage sales vary more than the prices of similar items in a
­department store?
t

Multiple-choice answers: 1. e 2. d 3. b
no

4.2 Demand
o

• What is the law of demand?


D

• What is an individual demand curve?


law of demand: • What is a market demand curve?
the law stating that
the quantity of a good
or service demanded
4.2a The Law of Demand
varies inversely (neg- Sometimes observed behavior is so pervasive that it is called a law—the law of demand, for
atively) with its price, ­example. According to the law of demand, the quantity of a good or service demanded varies
ceteris paribus ­inversely (negatively) with its price, ceteris paribus. More directly, the law of demand says that,

Copyright ©2020 by SAGE Publications, Inc.


This work may not be reproduced or distributed in any form or by any means without express written permission of the publisher.
Chapter 4  •  Demand, Supply, and Market Equilibrium 97  

other things being equal, when the price (P) of a good or service falls, the quantity demanded
individual ­demand
increases. Conversely, if the price (P) of a good or service rises, the quantity demanded decreases.
schedule: a
P ↑⇒ QD ↓ and P ↓⇒ QD ↑ ­schedule that shows
the relationship
between price and
4.2b Individual Demand quantity demanded
AN INDIVIDUAL DEMAND SCHEDULE
The individual demand schedule shows the relationship between the price of the good and the ECS

e
quantity demanded. For example, suppose Elizabeth enjoys drinking coffee. How many pounds

ut
of coffee would Elizabeth be willing and able to buy at various prices during the year? At a economic content
price of $3 per pound, Elizabeth buys 15 pounds of coffee over the course of a year. If the price standards
is higher, at $4 per pound, she might buy only 10 pounds; if it is lower, say, $1 per pound, she Higher prices for a good

b
might buy 25 pounds of coffee during the year. Elizabeth’s demand for coffee for the year is or service provide the
summarized in the demand schedule shown in Exhibit 1. Elizabeth might not be consciously incentives for buyers to

tri
aware of the quantities that she would purchase at prices other than the prevailing one, but that purchase less. Lower
does not alter the fact that she has a schedule, in the sense that she would have bought various prices for goods or
other quantities had other prices prevailed. It must be emphasized that the schedule is a list of

is
services provide incentives
alternative possibilities. At any one time, only one of the prices will prevail, and thus, a certain to purchase more of the
quantity will be purchased. good or service.

rd
Section 4.2 Section 4.2
Elizabeth’s Demand Schedule exhibit 2
Elizabeth’s Demand Curve
exhibit 1

o
for Coffee t, for Coffee

PRICE OF QUANTITY OF COFFEE


COFFEE (PER DEMANDED (POUNDS
os
$5
Elizabeth’s Demand
POUND) PER YEAR) Curve
4
$5 5
Price of Coffee
(per month)
,p

3
4 10
2
3 15
1
py

2 20

1 25 0 5 10 15 20 25
Quantity of Coffee
co

(pounds per year)

AN INDIVIDUAL DEMAND CURVE


The dots represent various quantities of coffee that Elizabeth
By plotting the different prices and corresponding quantities
would be willing and able to buy at different prices in a given
demanded in Elizabeth’s demand schedule in Exhibit 1 and
t

period. The demand curve shows how the quantity demanded


no

then connecting them, we can create the individual demand


varies inversely with the price of the good when we hold
curve for Elizabeth shown in Exhibit 2. From the curve, we
everything else constant—ceteris paribus. Because of this
can see that when the price is higher, the quantity demanded
inverse relationship between price and quantity demanded, the
is lower, and when the price is lower, the quantity demanded
demand curve is downward sloping.
is higher. The demand curve shows how the quantity of the
o

good demanded changes as its price varies.


D

4.2c What Is a Market Demand Curve? individual demand


curve: a graphical
Although we introduced the concept of the demand curve in terms of the individual, economists
representation that
usually speak of the demand curve in terms of large groups of people—a whole nation, a ­community, shows the inverse
or a trading area. That is, to analyze how the market works, we will need to use market demand. relationship between
As you know, every individual has his or her demand curve for every product. The horizontal sum- price and quantity
ming of the demand curves of many individuals is called the market demand curve. demanded

Copyright ©2020 by SAGE Publications, Inc.


This work may not be reproduced or distributed in any form or by any means without express written permission of the publisher.
98    PART 2  •  SUPPLY AND DEMAND

Section 4.2
exhibit 3
Creating a Market Demand Curve

a. Creating a Market Demand Schedule for Coffee

QUANTITY OF COFFEE DEMANDED (POUNDS PER YEAR)

e
REST OF MARKET
PRICE (PER POUND) PETER + LOIS + QUAHOG = DEMAND

ut
$4 20 + 10 + 2,970 = 3,000

b
$3 25 + 15 + 4,960 = 5,000

tri
b. Creating a Market Demand Curve for Coffee
Peter Lois Rest of Quahog Market Demand

is
$5 $5 $5 $5
Price (per pound)

Price (per pound)


Price (per pound)

Price (per pound)


4 4 4 4
1 1 5

rd
3 3 3 3

DPeter DM
2 2 2 DQ 2
DLois
1 1 1 1

o
0 5 10 15 20 25 0 5 10 15 20 25 0 2,970 4,960 0 3,000 5,000
Quantity of Coffee Quantity of Coffee Quantity of Coffee Quantity of Coffee
(pounds per year) (pounds per year) (pounds per year) (pounds per year)
t,
os

Suppose the consumer group is composed of Peter, Lois, and the rest of their small community,
,p

market demand
curve: the horizontal Quahog, and that the product is still coffee. The effect of price on the quantity of coffee demanded
summation of ­individual by Lois, Peter, and the rest of Quahog is given in the demand schedule and demand curves shown
demand curves in Exhibit 3. At $4 per pound, Peter would be willing and able to buy 20 pounds of coffee per year,
py

Lois would be willing and able to buy 10 pounds, and the rest of Quahog would be willing and
able to buy 2,970 pounds. At $3 per pound, Peter would be willing and able to buy 25 pounds of
coffee per year, Lois would be willing and able to buy 15 pounds, and the rest of Quahog would be
willing and able to buy 4,960 pounds. The market demand curve is simply the (horizontal) sum of
co

the quantities Peter, Lois, and the rest of Quahog demand at each price. That is, at $4, the quantity
demanded in the market would be 3,000 pounds of coffee (20 + 10 + 2,970 = 3,000), and at $3,
the quantity demanded in the market would be 5,000 pounds of coffee (25 + 15 + 4,960 = 5,000).
In Exhibit 4, we offer a more complete set of prices and quantities from the market demand for
t

coffee during the year. Remember, the market demand curve shows the quantities that all the buy-
no

ers in the market would be willing and able to buy at various prices. For example, when the price
of coffee is $2 per pound, consumers in the market collectively would be willing and able to buy
8,000 pounds per year. At $1 per pound, the quantity demanded would be 12,000 pounds per year.
The market demand curve is the negative (inverse) relationship between price and the q ­ uantity
o

demanded, while holding all other factors that affect how much consumers are able and willing to
pay constant, ceteris paribus. For the most part, we are interested in how the market works, so we
D

will primarily use market demand curves.

4.2d Ceteris Paribus and the Law of Demand


When we considered how Elizabeth’s demand for coffee is affected by a change in price, we had to
hold many other things constant, such as her income, her taste, the weather outside, the price of
other things that Elizabeth buys, and so on. This ceteris paribus assumption allows us to focus on
the variable we are interested in, which is the price of coffee.

Copyright ©2020 by SAGE Publications, Inc.


This work may not be reproduced or distributed in any form or by any means without express written permission of the publisher.
Chapter 4  •  Demand, Supply, and Market Equilibrium 99  

Section 4.2
exhibit 4
A Market Demand Curve

a. Market Demand Schedule for Coffee b. Market


b. Market DemandCurve
Demand Curvefor
forCoffee
Coffee

QUANTITY DEMANDED $5
PRICE (PER POUND) (POUNDS PER YEAR)

e
4

Price (per pound)


$5 1,000 Market

ut
Demand Curve
3
4 3,000
2

b
3 5,000
1

tri
2 8,000

1 12,000 0 1 3 5 8 12

is
Quantity of Coffee
(thousands of pounds per year)

rd
The market demand curve shows the quantities that all the buyers in the market would be willing and able to buy at various prices. We find the
market demand curve by adding horizontally the individual demand curves. For example, when the price of coffee is $2 per pound, consumers
in the market collectively would be willing and able to buy 8,000 pounds per year. At $1 per pound, the quantity collectively demanded would be

o
12,000 pounds per year.
t,
The ceteris paribus assumption also holds when we define a particular good. That is, we are
os
assuming that all goods are the same (homogenous). For example, if we are referring to the market
for frozen yogurt, we would assume that the serving of yogurt is the same size and quality. Not a
higher-quality yogurt served in a chocolate-covered waffle cone versus a small scoop in a child-size
cup—those would be two different goods. By allowing something other than the price of yogurt to
,p

change, you would be violating the ceteris paribus assumption.


py
t co
no
o
D

iStock/ewg3D

• Why is gasoline consumption lower in Europe than in the United States? The main reason is price. Because of higher
taxes, gasoline prices are at least twice as high in Europe. Consequently, Europeans on average consume half as much
gasoline—buying smaller cars with better mileage. •

Copyright ©2020 by SAGE Publications, Inc.


This work may not be reproduced or distributed in any form or by any means without express written permission of the publisher.
SECTION QUIZ
1. If the demand for milk is downward sloping, then an increase in the price of milk will result in a(n)

e
a. increase in the demand for milk.
b. decrease in the demand for milk.

ut
c. increase in the quantity of milk demanded.
d. decrease in the quantity of milk demanded.

b
e. decrease in the supply of milk.
2. Which of the following is true?

tri
a. The law of demand states that when the price of a good falls (rises), the quantity demanded rises (falls), ceteris
paribus.

is
b. An individual demand curve is a graphical representation of the relationship between the price and the quantity
demanded.

rd
c. The market demand curve shows the quantity of a good that all buyers in the market would be willing and able
to buy at various prices.
d. All of the above are true.

o
3. Which of the following is true?
a. The relationship between price and quantity demanded is inverse, or negative.
b. The market demand curve is the vertical summation of individual demand curves.
t,
c. A change in a good’s price causes a movement along its demand curve.
d. All of the above are true.
os
e. Answers (a) and (c) are true.
,p

1. What is an inverse relationship?


2. How do lower prices change buyers’ incentives?
3. How do higher prices change buyers’ incentives?
py

4. What is an individual demand schedule?


5. What is the difference between an individual demand curve and a market demand curve?
co

6. Why does the amount of dating on campus tend to decline just before and during final exams?
Multiple-choice answers: 1. d 2. d 3. e
t
no

4.3 Shifts in the Demand Curve


• What is the difference between a change in demand and a change in quantity demanded?
• What are the determinants of demand?
o

• What are substitutes and complements?


• What are normal and inferior goods?
D

• How does the number of buyers affect the demand curve?


• How do changes in taste affect the demand curve?
change in quan- • How do changing expectations affect the demand curve?
tity demanded: a
change in a good’s
own price leads to a 4.3a A Change in Demand versus a Change in Quantity Demanded
change in quantity
­demanded, a move-
Understanding the relationship between price and quantity demanded is so important that econo-
ment along a given mists make a clear distinction between it and the various other factors that can influence ­consumer
demand curve behavior. A change in a good’s own price is said to lead to a change in quantity demanded.

Copyright ©2020 by SAGE Publications, Inc.


This work may not be reproduced or distributed in any form or by any means without express written permission of the publisher.
Chapter 4  •  Demand, Supply, and Market Equilibrium 101  

That is, it “moves you along” a given demand curve. The demand curve is the answer to the ques-
shifts in the
tion “What happens to the quantity demanded when the price of the good changes?” The demand
demand curve: a
curve is drawn under the assumption that all other things are held constant, except the price of change in one of the
the good. However, economists know that price is not the only thing that affects the quantity of a variables, other than
good that people buy. The other variables that influence the demand curve are called determinants the price of the good
of demand, and a change in these other factors lead to shifts in the demand curve. itself, that affects
the willingness of

e
consumers to buy
4.3b Shifts in Demand (“PYNTE”)

ut
There are two ways the demand curve can shift. We say there is an increase in demand when the
curve shifts rightward: At any given price, consumers demand a larger quantity of the good than
before. Or when there is a decrease in demand, there is a leftward shift in the demand curve: At

b
any given price, consumers demand a smaller quantity of the good than before. These shifts are
shown in Exhibit 1.

tri
Several variables can shift the demand curve, but here are some of the most important. It might
be helpful to remember the old English spelling of the word pint—PYNTE. This acronym can help Does a movement along
you remember the five important factors that shift the demand curve for a good or service. a given demand curve

is
illustrate a change in
• Changes in the Prices of Related Goods and Services (P) demand or a change in
• Changes in Income (Y)

rd
quantity demanded?
• Changes in the Number of Buyers (N)
• Changes in Tastes (T) Section 4.3
exhibit 1
Demand Shifts
• Changes in Expectations (E)

4.3c Changes in the Prices of Related Goods


and Services (P)
o
t,
Decrease Increase
In deciding how much of a good or service to buy, consum-
os
in in
Price

ers are influenced by the price of that good or service, a rela- Demand Demand

tionship summarized in the law of demand. However, some-


times consumers are also influenced by the prices of related
,p

goods and services—substitutes and complements. D3 D1 D2


0
SUBSTITUTES Quantity
Substitutes are generally goods for which one could be used
py

Any change that will cause an increase in the quantity of a good


in place of the other. To many, substitutes would include
that consumers want to buy at any given price shifts the demand
muffins and bagels, Crest and Colgate toothpaste, domestic
curve to the right. Any change that will cause a decrease in the
and foreign cars, movie tickets and video streaming, j­ackets
quantity that consumers want to buy at any given price will shift
co

and sweaters, Exxon and Shell gasoline, and Nikes and


the demand curve to the left.
Reeboks.
Two goods are substitutes when an increase (decrease)
in the price of one good causes an increase (decrease) in
the demand for the other good. For example, if an increase in the price of Diet Pepsi (an upward substitutes: goods
t

for which an increase


no

movement along the demand curve for Diet Pepsi) causes an increase in demand for Diet Coke
(a rightward shift of the demand curve for Diet Coke), we would say that for this buyer, the (decrease) in the price
two goods are substitutes. (See the “Use What You’ve Learned” feature on substitute goods.) of one good causes an
increase (decrease)
in the demand for the
other good
o

• Some cities have tried to reduce traffic con-


gestion by lowering the price of substitutes
D

for cars, such as buses and rail ­services. •


iStock/krblokhin

Copyright ©2020 by SAGE Publications, Inc.


This work may not be reproduced or distributed in any form or by any means without express written permission of the publisher.
102    PART 2  •  SUPPLY AND DEMAND

USE WHAT YOU’VE LEARNED

Substitute Goods Answer: In Exhibit 2(a), we see that an increase in the price of Coca-­
Cola—an upward movement along the demand curve for Coca-Cola,
from point A to point B—causes a reduction in the quantity demanded
Question: Can you describe the change we would expect to see in the

e
of Coca-Cola. If the two goods are substitutes, the higher price for
­demand curve for Pepsi if the relative price for Coca-Cola increased ­Coca-Cola will cause an increase in the demand for Pepsi (a rightward

ut
­significantly? shift), as seen in Exhibit 2(b).

b
Section 4.3
exhibit 2
Substitute Goods

tri
a. Market for Coca-Cola b. Market for Pepsi

is
Price of Coca-Cola

Price of Pepsi

rd
P2 B

P1 A

o
Demand D1 D2
0 0
Q2 Q1
t,
Quantity of Coca-Cola Quantity of Pepsi
os
,p

COMPLEMENTS
Two goods are complements if they are used together, such as skis and bindings, peanut butter
py

and jelly, hot dogs and buns, cars and gasoline, and printers and ink cartridges. When an increase
(decrease) in the price of one good causes a decrease (increase) in the demand for another good,
complements: the two goods are called complements. For many people, motorcycles and motorcycle helmets are
goods for which an
co

complements, especially in states that have required helmet laws. So when the price of motorcycles
increase (decrease) in
falls, the quantity of motorcycles demanded will rise—a movement downward along the demand
the price of one good
leads to a decrease curve for motorcycles. As more people buy motorcycles, they will demand more motorcycle hel-
(increase) in the mets—the demand curve for motorcycle helmets shifts to the right. However, most pairs of goods
demand for the other are not closely related. For example, ice cream and housing or cars and pizzas are not closely
t

good related goods.


no
o
D

Copyright ©2020 by SAGE Publications, Inc.


This work may not be reproduced or distributed in any form or by any means without express written permission of the publisher.
Chapter 4  •  Demand, Supply, and Market Equilibrium 103  

USE WHAT YOU’VE LEARNED

Complementary Goods (a ­movement downward along the demand curve, from point A to point
B, called an increase in quantity demanded) and an increase in the
demand for software (a rightward shift). Of course, the opposite is true,

e
Question: If the price of computers fell markedly, what do you think too—an increase in the price of computers will lead to fewer people
would happen to the demand for software? purchasing computers (a movement upward along the demand curve

ut
Answer: If computers and software are complements, the decrease for computers, from point B to point A, called a decrease in quantity
in the price of computers will lead to more computers purchased demanded) and a lower demand for software (a leftward shift).

b
Section 4.3

tri
Complementary Goods
exhibit 3

is
a. Market for Computers b. Market for Software

rd
Price of Computers

Price of Software
P1 A

o
P2 B

Demand D1 D2
0
t, 0
Q1 Q2
os
Quantity of Computers Quantity of Software
,p

4.3d Changes in Income (Y)


py

Why (Y)? The reason is because macroeconomists use the letter (I) for investment, so microeco-
nomists often use the letter (Y) to denote income. Economists have observed that generally the
co

consumption of goods and services is positively related to the income available to consumers.
Empirical studies support the notion that as individuals receive more income, they tend to increase
their purchases of most goods and services. Other things held equal, rising income usually leads
to an increase in the demand for goods (a rightward shift of the demand curve), and decreasing
income usually leads to a decrease in the demand for goods (a leftward shift of the demand curve).
t
no

NORMAL AND INFERIOR GOODS


normal goods:
If demand for a good increases when incomes rise and decreases when incomes fall, the good is if income increases,
called a normal good. Most goods are normal goods. Consumers will typically buy more clothes, the demand for a
pizzas, and trips to the movies as their incomes rise. However, if demand for a good decreases good increases; if
o

when incomes rise or if demand increases when incomes fall, the good is called an inferior good. income decreases, the
For example, as your income rises, you may choose to stay in nice hotels rather than youth h ­ ostels, demand for a good
D

or you may purchase fewer fast-food meals. The term inferior in this sense does not refer to the decreases
­quality of the good in question but shows that demand decreases when income increases and
inferior goods: if
demand increases when income decreases.
income increases, the
Or if people’s incomes rise and they increase their demand for movie tickets, we say that movie demand for a good
tickets are a normal good. But if people’s incomes fall and they increase their demand for bus rides, decreases; if income
we say that bus rides are an inferior good. Whether goods are normal or inferior, the point here is decreases, the demand
that income influences demand—usually positively, but sometimes negatively. for a good increases

Copyright ©2020 by SAGE Publications, Inc.


This work may not be reproduced or distributed in any form or by any means without express written permission of the publisher.
104    PART 2  •  SUPPLY AND DEMAND

USE WHAT YOU’VE LEARNED

Normal and Inferior Goods Answer: Yes. Furniture is generally considered a normal good, so a
rise in ­income will increase the demand for high-quality furniture,
as shown in Exhibit 4(a). However, if Chester sells unfinished, used,

e
Question: Chester Field owns a high-quality furniture shop. If a boom or low-­quality furniture, the demand for his products might fall, as
in the economy occurs (with a higher average income per person and ­higher incomes allow customers to buy furniture that is finished, new,

ut
­fewer people unemployed), can Chester expect to sell more high-­ or of higher quality. Chester’s furniture would then be an inferior good,
quality ­furniture? as shown in Exhibit 4(b).

b
Section 4.3
Normal and Inferior Goods

tri
exhibit 4

is
a. Rising Income and a Normal Good b. Rising Income and an Inferior Good

rd
Price of Furniture

Price of Furniture

D1 D2

o D2 D1
0
t, 0
Quantity of High-Quality Furniture Quantity of Low-Quality Furniture
os
,p
py
t co
no
o
Robert Alexander/Getty Images
D

• In the midst of a recession, is it possible that many people will increase their demand for fast-food restaurants?
It is not only possible—it actually happened in 2009! If declining income causes demand for a good to rise, is it a
normal good or an inferior good? •

Copyright ©2020 by SAGE Publications, Inc.


This work may not be reproduced or distributed in any form or by any means without express written permission of the publisher.
Chapter 4  •  Demand, Supply, and Market Equilibrium 105  

4.3e Changes in the Number of Buyers (N)


The demand for a good or service will vary with the size of the
­potential consumer population. The demand for wheat, for example,
rises as population increases because the added population wants to

iStock/visualspace
consume wheat products, such as bread or cereal. Marketing ­experts,
who closely follow the patterns of consumer behavior regarding
a particular good or service, are usually vitally concerned with the

e
­demographics of the product—the vital statistics of the potential con-
sumer population, ­including size, race, income, and age characteris- • Body piercing and tattoos have risen in popularity in

ut
tics. For example, market researchers for baby food companies keep a recent years. The demand for these services has been
close watch on the birth rate. pushed to the right. According to the Pew Research
­Center, 38 percent of 18- to 29-year-olds have at least

b
one tattoo. •
4.3f Changes in Tastes (T)

tri
The demand for a good or service may increase or decrease with changes in people’s tastes or pref-
erences. When tastes change in favor of a good, more people want to buy the good at any given

is
price—a rightward shift in the demand curve. When tastes change against a good, fewer people
want to buy the good at any given price—a leftward shift in the demand curve.

rd
Changes in taste may be triggered by advertising or promotion, by a news story, by the behavior
of some popular public figure, and so on. Changes in taste are particularly noticeable in apparel.
Skirt lengths, coat lapels, shoe styles, and tie sizes change frequently.
Changes in preferences naturally lead to changes in demand. A person may grow tired of one

o
type of recreation or food and try another type. People may decide they want more organic food;
consequently, we will see more stores and restaurants catering to this change in taste. Changes in
occupation, number of dependents, state of health, and age also tend to alter preferences. The birth
t,
of a baby might cause a family to spend less on recreation and more on food and clothing. Illness
increases the demand for medicine and lessens purchases of other goods. A cold winter i­ncreases
os
the demand for heating oil. Changes in customs and traditions also affect preferences, and the
development of new products draws consumer preferences away from other goods. Compact discs
replaced record albums, just as DVD players replaced VCRs, and DVD players are now being
,p

replaced by Internet streaming services such as Netflix and Hulu. A change in information can
also impact consumers’ demand. For example, a breakout of E. coli or new information about a
defective and/or dangerous product, such as a baby crib, can reduce demand.
py

BUSINESS WATCH
co

The Rise and Fall of Crocs

Remember Crocs, the foam rubber shoes with the holes in the top?
t
no

They became extremely popular in 2006 through 2007. More Crocs


were sold, not because the company lowered the price, but b­ ecause
there was an increase in demand. More Crocs were sold at all
­prices—a rightward shift of the demand curve. Eventually, by 2008,
iStock/NAKphotos

the fad had run its course, and fewer Crocs were sold at all prices—a
o

leftward shift in the demand curve.


D

Copyright ©2020 by SAGE Publications, Inc.


This work may not be reproduced or distributed in any form or by any means without express written permission of the publisher.
106    PART 2  •  SUPPLY AND DEMAND

4.3g Changes in Expectations (E)


Sometimes the demand for a good or service in a given period will increase or decrease because
consumers expect the good to change in price or availability at some future date. If people expect
the future price to be higher, they will purchase more of the good now before the price increase—an
increase in the demand today. If people expect the future price to be lower, they will purchase less
of the good now and wait for the price decrease—a decrease in the demand today. For example, if
you expect the price of computers to fall soon, you may be less willing to buy one today. Or you

e
might buy next year’s Halloween decorations on November 1 during a post-Halloween sale. That
is, expectations of higher prices in the future could increase your demand now.

ut
A change in consumers’ expectations about their future incomes can also shift the demand
curve. For example, if you expect to earn additional income next month, you may be more willing

b
to dip into your current savings to buy something this month. If you expect your income to fall in
the future, you may choose to save more today and reduce your demand for some goods.

tri
4.3h Changes in Demand versus Changes in Quantity Demanded—Revisited

is
Economists put particular emphasis on the impact of a change in the price of a good on consumer
behavior. We are interested in distinguishing between consumer behavior related to the price of a

rd
good itself (movements along a demand curve) and behavior related to changes in other factors
(shifts of the demand curve).
As indicated earlier, if the price of a good changes, it causes a change in quantity demanded,
ceteris paribus. If one of the other factors (determinants) influencing consumer behavior changes,

o
How is a change in it results in a change in demand. The effects of some of the determinants that cause changes in
demand different from demand (shifters) are reviewed in Exhibit 5. For example, there are two different ways to curb
a change in quantity teenage smoking: raise the price of cigarettes (a reduction in the quantity of cigarettes demanded)
t,
demanded? or decrease the demand for cigarettes (a leftward shift in the demand curve for cigarettes). Both
os

Section 4.3 Possible Demand Shifters


exhibit 5
,p
py
Price

Price

Price
co

D1 D2 D1 D2 D2 D1
0 0 0
t

Quantity Quantity Quantity


no

Price of Complement Falls Income Increases (normal good) Income Increases (inferior good)
or Price of Substitute Rises
o Price

Price

Price
D

D1 D2 D1 D2 D1 D2
0 Quantity 0 Quantity 0 Quantity
Increase in the Number of Taste Change in Favor of the Good Future Price Increase Expected
Buyers in the Market

Copyright ©2020 by SAGE Publications, Inc.


This work may not be reproduced or distributed in any form or by any means without express written permission of the publisher.
Chapter 4  •  Demand, Supply, and Market Equilibrium 107  

USE WHAT YOU’VE LEARNED

Changes in Demand versus Changes


in Quantity Demanded

e
Section 4.3
Question: How would you use a graph to demonstrate the two following exhibit 6
Change in Demand versus

ut
­scenarios? (1) Someone buys more pizzas because the price of pizzas Change in Quantity Demanded
has fallen; and (2) a student buys more pizzas because she just received

b
a 20 percent raise at work, giving her additional income.
Answer: In Exhibit 6, the movement from A to B is called an increase in A C

tri
$15
­quantity demanded; the movement from B to A is called a decrease in

Price of Pizzas
A C Change
­quantity demanded. Economists use the phrase “increase or decrease in in Demand

is
­quantity demanded” to describe movements along a given demand B A B Change
10
curve. ­However, the change from A to C is called an increase in demand, in Quantity

rd
Demanded
and the change from C to A is called a decrease in demand. The phrase
“increase or decrease in demand” is reserved for a shift in the whole D1 D2
curve. So if an individual buys more pizzas because the price fell, we call 0 3 5 8
it an increase in quantity demanded. However, if she buys more pizzas

o
Quantity of Pizzas
even at the current price, say $15, we say it is an increase in demand. (per month)
In this case, the increase in income was responsible for the increase in
t,
demand because she chose to spend some of her new income on pizzas.
os
,p

would reduce the amount of smoking. Specifically, to increase the price of cigarettes, the govern-
ment could impose a higher tax on manufacturers. Most of this would be passed on to consumers
in the form of higher prices (more on this in Chapter 6). Or to shift the demand curve leftward, the
py

government could adopt policies to discourage smoking, such as advertising bans and increasing
consumer awareness of the harmful side effects of smoking—disease and premature death.
The following table lists some of the most important variables that affect how much consumers are
willing to buy. Remember, changes in the price of the good itself cause a movement along a given
co

demand curve, resulting in a change in quantity demanded. That is what happens to the quantity
consumers demand when only a good’s price changes and all the other variables that influence buy-
ers are held constant. A change in the other variables shifts the curve, causing a change in demand.
t
no

VARIABLE A CHANGE IN THIS VARIABLE CAUSES


Price of the good itself a movement along the demand curve

Price of related goods and services a shift in the demand curve


o

Income a shift in the demand curve


D

Number of buyers a shift in the demand curve

Tastes a shift in the demand curve

Expectations a shift in the demand curve

Copyright ©2020 by SAGE Publications, Inc.


This work may not be reproduced or distributed in any form or by any means without express written permission of the publisher.
SECTION QUIZ

1. Which of the following would be most likely to increase the demand for jelly?
a. an increase in the price of peanut butter, which is often used with jelly
b. an increase in income; jelly is a normal good

e
c. a decrease in the price of jelly
d. medical research that finds that daily consumption of jelly makes people live 10 years less, on average

ut
2. Which of the following would not cause a change in the demand for cheese?
a. an increase in the price of crackers, which are consumed with cheese

b
b. an increase in the income of cheese consumers
c. an increase in the population of cheese lovers

tri
d. an increase in the price of cheese
3. Whenever the price of Good A decreases, the demand for Good B increases. Goods A and B appear to be

is
a. complements.
b. substitutes.

rd
c. inferior goods.
d. normal goods.
e. inverse goods.

o
4. Whenever the price of Good A increases, the demand for Good B increases as well. Goods A and B appear to be
a. complements.
b. substitutes.
t,
c. inferior goods.
d. normal goods.
os
e. inverse goods.
5. The difference between a change in quantity demanded and a change in demand is that a change in
,p

a. quantity demanded is caused by a change in a good’s own price, while a change in demand is caused by a
change in some other variable, such as income, tastes, or expectations.
b. demand is caused by a change in a good’s own price, while a change in quantity demanded is caused by a
change in some other variable, such as income, tastes, or expectations.
py

c. quantity demanded is a change in the quantity people actually buy, while a change in demand is a change in the
quantity they want to buy.
d. This is a trick question. A change in demand and a change in quantity demanded are the same thing.
co

6. Suppose CNN announces that bad weather in Central America has greatly reduced the number of cocoa bean plants,
and for this reason, the price of chocolate is expected to rise soon. As a result,
a. the current market demand for chocolate will decrease.
b. the current market demand for chocolate will increase.
t
no

c. the current quantity demanded for chocolate will decrease.


d. no change will occur in the current market for chocolate.
7. If incomes are rising, in the market for an inferior good,
a. demand will rise.
o

b. demand will fall.


c. supply will rise.
D

d. supply will fall.

1. What is the difference between a change in demand and a change in quantity demanded?
2. If the price of zucchini increases, causing the demand for yellow squash to rise, what do we call the relationship
between zucchini and yellow squash?

Copyright ©2020 by SAGE Publications, Inc.


This work may not be reproduced or distributed in any form or by any means without express written permission of the publisher.
3. If incomes rise and, as a result, demand for jet skis increases, how do we describe that good?
4. How do expectations about the future influence the demand curve?
5. Would a change in the price of ice cream cause a change in the demand for ice cream? Why or why not?
6. Would a change in the price of ice cream likely cause a change in the demand for frozen yogurt, a substitute?
7. If plane travel is a normal good and bus travel is an inferior good, what will happen to the demand curves for plane
and bus travel if people’s incomes increase?

e
Multiple-choice answers: 1. b 2. d 3. a 4. b 5. a 6. b 7. b

b ut
tri
law of supply: the
4.4 Supply law stating that the
• What is the law of supply? higher (lower) the

is
price of the good, the
• What is an individual supply curve?
greater (smaller) the
• What is a market supply curve? quantity supplied,

rd
ceteris paribus
4.4a The Law of Supply
In a market, the answer to the fundamental question, “What do we produce, and in what quanti- ECS

o
ties?” depends on the interaction of both buyers and sellers. Demand is only half the story. The will-
ingness and ability of sellers to provide goods are equally important factors that must be weighed economic content
by decision makers in all societies. As with demand, the price of the good is an important factor. standards
t,
And just as with demand, factors other than the price of the good are also important to sellers, Higher prices for a
such as the cost of inputs or advances in technology. While behavior will vary among individual
os
good or service provide
sellers, economists expect that, other things being equal, the quantity supplied will vary directly incentives for producers
with the price of the good, a relationship called the law of supply. According to the law of supply, to make or sell more
the higher the price of the good (P), the greater the quantity supplied, and the lower the price (P) of it. Lower prices for a
,p

of the good, the smaller the quantity supplied (Qs), ceteris paribus. good or service provide
incentives for producers
P ↑⇒ Qs ↑ and P ↓⇒ Qs ↓ to make or sell less of it.
py

The relationship described by the law of supply is a direct, or positive, relationship because the
variables move in the same direction.
ECS
4.4b A Positive Relationship between Price and Quantity Supplied
co

economic content
Firms supplying goods and services want to increase their profits, and the higher the price per unit, standards
the greater the profitability generated by supplying more of that good. For example, if you were a An increase in the price
coffee grower, wouldn’t you much rather be paid $5 per pound than $1 per pound, ceteris paribus? of a good or service
t

When the price of coffee is low, the coffee business is less profitable, and less coffee will be enables producers to
no

produced. Some sellers may even shut down, reducing their quantity supplied to zero if the price cover higher costs,
is low enough. ceteris paribus, causing
the quantity supplied to
increase, and vice versa.
4.4c An Individual Supply Curve
o

To illustrate the concept of an individual supply curve, consider the amount of coffee that an
individual seller, Juan Valdés, is willing and able to supply in one year. The law of supply can be
D

individual supply
illustrated, like the law of demand, by a table or graph. Juan’s supply schedule for coffee is shown
curve: a graphical
in Exhibit 1(a). The combinations of price and quantity supplied were then plotted and joined to representation that
create the individual supply curve shown in Exhibit 1(b). Note that the individual supply curve is shows the positive
upward sloping as you move from left to right. At higher prices, there will be a greater quantity relationship between
supplied, other things being equal. That is, existing firms or growers will produce more at higher price and quantity
prices than at lower prices. supplied

Copyright ©2020 by SAGE Publications, Inc.


This work may not be reproduced or distributed in any form or by any means without express written permission of the publisher.
110    PART 2  •  SUPPLY AND DEMAND

Section 4.4
exhibit 1
An Individual Supply Curve

a. Juan’s Supply Schedule for Coffee b. Juan’s Supply


Juan’s Curve
Supply for for
Curve Coffee
Coffee

PRICE (PER QUANTITY SUPPLIED $5


POUND) (POUNDS PER YEAR)

e
4
Juan’s Supply

Price of Coffee
$5 80

(per pound)
Curve

ut
3
4 70
2
3 50

b
1
2 30

tri
1 10 0 10 30 50 70 80
Quantity of Coffee

is
(pounds per year)

rd
market supply
curve: the horizon- 4.4d The Market Supply Curve

o
tal summation of
the supply curves of The market supply curve may be thought of as the horizontal summation of the supply curves
producers; a graphical for all the individual producers in the market. The market supply curve shows how the total
representation of the
t,
quantity supplied varies with the market price of the good, while holding constant all other fac-
quantity of goods and tors that affect how much producers are able and willing to supply. The market supply schedule,
os
services that all the which reflects the total quantity supplied at each price by all of the coffee producers, is shown
individual producers
in Exhibit 2(a). Exhibit 2(b) illustrates the resulting market supply curve for this group of coffee
are willing and able
to supply at various producers.
,p

prices

Section 4.4
exhibit 2
A Market Supply Curve
py

a. Market Supply Schedule for Supply


a. Market Coffee Schedule for Coffee b. Market Supply Curve for Coffee
co

QUANTITY SUPPLIED (POUNDS


Quantity SuppliedPER YEAR) $5
(pounds per year)
PRICE Other Market
4
Price of Coffee

(PER (perPrice OTHER MARKET


(per pound)

pound) Juan 1 Producers 5 Supply 3


POUND) JUAN + PRODUCERS = SUPPLY)
t

Market
$5 80 1 7,920 5 8,000 Supply Curve
no

$5 4 80 +
70 1 7,920
6,930 =5 8,000
7,000 2
3 50 1 4,950 5 5,000
4 1
2 70 +
30 1 6,930
2,970 =5 7,000
3,000
1 10 1 990 5 1,000
3 50 + 4,950 = 5,000
0 1 3 5 7 8
o

2 30 + 2,970 = 3,000 Quantity of Coffee


D

(thousands of pounds per year)


1 10 + 990 = 1,000
The dots on this graph indicate different quantities of coffee that sellers would be willing and able to supply at various prices. The line connecting
those combinations is the market supply curve.

Copyright ©2020 by SAGE Publications, Inc.


This work may not be reproduced or distributed in any form or by any means without express written permission of the publisher.
SECTION QUIZ
1. An upward-sloping supply curve shows that
a. buyers are willing to pay more for particularly scarce products.
b. sellers expand production as the product price falls.

e
c. sellers are willing to increase production of their goods if they receive higher prices for them.
d. buyers are willing to buy more as the product price falls.

ut
2. Along a supply curve,
a. supply changes as price changes.

b
b. quantity supplied changes as price changes.
c. supply changes as technology changes.

tri
d. quantity supplied changes as technology changes.
3. A supply curve illustrates a(n) _________ relationship between _________ and _________.

is
a. direct; price; supply
b. direct; price; quantity demanded

rd
c. direct; price; quantity supplied
d. introverted; price; quantity demanded
e. inverse; price; quantity supplied

o
4. Which of the following is true?
a. The law of supply states that the higher (lower) the price of a good, the greater (smaller) the quantity supplied.
b. The relationship between price and quantity supplied is positive because profit opportunities are greater at higher
t,
prices and because the higher production costs of increased output mean that suppliers will require higher prices.
c. The market supply curve is a graphical representation of the number of goods and services that suppliers are
os
willing and able to supply at various prices.
d. All of the above are true.
,p

1. What are the two reasons why a supply curve is positively sloped?
2. What is the difference between an individual supply curve and a market supply curve?
py

Multiple-choice answers: 1. c 2. b 3. c 4. d
co

4.5 Shifts in the Supply Curve


• What is the difference between a change in supply and a change in quantity supplied?
t

• What are the determinants of supply?


no

• How does the number of suppliers affect the supply curve?


• How does technology affect the supply curve? Why is a change in
• How do taxes affect the supply curve? supply different from
a change in quantity
4.5a A Change in Quantity Supplied versus a Change in Supply supplied?
o

Changes in the price of a good lead to changes in the quantity supplied by sellers, just as changes
D

in the price of a good lead to changes in the quantity demanded by buyers. Similarly, a change in
supply, whether an increase or a decrease, can occur for reasons other than changes in the price
of the product itself, just as changes in demand may be due to factors (determinants) other than
the price of the good. In other words, a change in the price of the good in question is shown as a
movement along a given supply curve, leading to a change in quantity supplied. A change in any
other factor that can affect seller behavior (the seller’s input prices, the prices of related products,
expectations, the number of sellers, and technology) results in a shift in the entire supply curve.
This is called a change in supply.

Copyright ©2020 by SAGE Publications, Inc.


This work may not be reproduced or distributed in any form or by any means without express written permission of the publisher.
112    PART 2  •  SUPPLY AND DEMAND

Section 4.5 4.5b Shifts in Supply (“SPENT”)


exhibit 1
Supply Shifts There are two ways the supply curve can shift. We say there
is an increase in supply when the curve shifts rightward: At
any given price, producers supply a larger quantity of the
S3 S1 S2 good than before. Or when there is a decrease in supply,
there is a leftward shift in the supply curve: At any given
Decrease Increase price, ­producers supply a smaller quantity of the good than
in in

e
before. These shifts are shown in Exhibit 1. We now look
Price

Supply Supply
at some of the possible determinants of supply—factors that

ut
determine the position of the supply curve—in greater depth.
Several variables can shift the supply curve, but here are
some of the most important. It might be helpful to remember

b
the word “SPENT.” This acronym can help you remember
0 the five important factors that shift the supply curve for a

tri
Quantity good or service.
Any change that will cause an increase in the quantity that sellers • Changes in the seller’s input prices (S)

is
want to produce and sell at any given price shifts the supply • Changes in the prices of related goods and services (P)
curve to the right. Any change that will cause a decrease in the • Changes in expectations (E)
quantity that sellers want to produce and sell at any given price • Changes in the number of sellers (N)

rd
will shift the supply curve to the left. • Changes in technology (T)

o
CHANGES IN THE SELLER’S INPUT PRICES (S)
Sellers are strongly influenced by the costs of inputs used in the production process, such as steel
used for automobiles or microchips used in computers. Recall that inputs are used to make out-
t,
puts of goods and services. Inputs, like outputs, also have prices. And if the input prices rise, this
increases the cost of producing the output of a good or service. Consequently, the producer is less
os
willing to supply the final good at any given price, causing the supply curve to shift to the left. For
example, higher labor, materials, energy, or other input costs increase the costs of producing an
automobile, causing the supply curve for automobiles to shift to the left. If input prices fall, pro-
,p

ducing the final good or service is less costly to the seller; thus, the seller is more willing to supply
the good at any given price, so the supply curve shifts to the right.

CHANGES IN THE PRICES OF RELATED


py

GOODS AND SERVICES (P)


The supply of a good increases if the price of one
of its substitutes in production falls; and the sup-
co

ply of a good decreases if the price of one of its


substitutes in production rises. Suppose you own
your own farm, on which you plant cotton and
wheat. One year, the price of wheat falls and
farmers reduce the quantity of wheat supplied, as
t

shown in Exhibit 2(a). What effect does the l­ower


no

price of wheat have on your cotton production?


It increases the supply of cotton. You want to
produce relatively less of the crop that has fallen
in price (wheat) and relatively more of the now
iStock/Mordolff

more attractive other crop (cotton). Cotton and


wheat are substitutes in production because both
D

goods can be produced using the same resourc-


es. Producers tend to substitute the production
• Veterinarians can choose to work on either suburban pets or farm animals; of more profitable products for that of less prof-
they are substitutes in production. Veterinarians have increasingly chosen to
live in metropolitan areas, where they can pursue much more lucrative practic- itable products. So the decrease in the price in the
es specializing in domestic pets. Because the value of working on pets in the wheat market has caused an increase in supply
city has risen, the cost of being a farm veterinarian has increased, shifting the (a rightward shift) in the cotton market, as seen
supply curve for farm veterinarians to the left. • in Exhibit 2(b).

Copyright ©2020 by SAGE Publications, Inc.


This work may not be reproduced or distributed in any form or by any means without express written permission of the publisher.
Chapter 4  •  Demand, Supply, and Market Equilibrium 113  

Section 4.5
exhibit 2
Substitutes and Complements in Production

Substitutions in Production
a. Market for Wheat b. Market for Cotton

Supply

Price of Cotton
S1

Price of Wheat
S2

e
P1

ut
P2

b
0 0
Q2 Q1

tri
Quantity of Wheat Quantity of Cotton
Complements in Production

is
c. Market for Cattle d. Market for Leather

Supply

Price of Leather
S1 S2
Price of Cattle

rd
P2

P1

0
Q1 Q2

o 0
t,
Quantity of Cattle Quantity of Leather
os
If land can be used for either wheat or cotton, a decrease in the price of wheat causes a decrease in the quantity supplied, a movement downward
along the supply curve in Exhibit 2(a). This may cause some farmers to shift out of the production of wheat and into the substitute in production—
cotton—shifting the cotton supply curve to the right, as shown in Exhibit 2(b). In another example, if the price of the complement in production
increases (cattle), it becomes more profitable and as a result cattle ranchers increase the quantity supplied of beef, moving up the supply curve for
,p

beef, as seen in Exhibit 2(c). When cattle ranchers produce more beef, they also produce more leather. Thus, when the price of beef increases, the
supply of the related good, leather, shifts to the right, as seen in Exhibit 2(d).
py

If the price of wheat, a substitute in production, i­ncreases, then that crop becomes more profit-
able. This leads to an increase in the quantity supplied of wheat. Consequently, farmers will shift their
co

resources out of the relatively lower-priced crop (cotton); the result is a decrease in supply of cotton.
Other examples of substitutes in production include automobile producers that have to decide
between producing sedans and pick-ups or construction companies that have to choose between
building single residential houses or commercial buildings. A producer of soccer balls may produce
basketballs if the price of basketballs rises relative to soccer balls. This will increase profitability.
t

Some goods are complements in production. Producing one good does not prevent the production
no

of the other, but actually enables production of the other. For example, leather and beef are comple-
ments in production. Suppose the price of beef rises and, as a result, cattle ranchers increase the quan-
tity supplied of beef, moving up the supply curve for beef, as seen in Exhibit 2(c). When cattle ranchers
produce more beef, they automatically produce more leather. Thus, when the price of beef increases,
o

the supply of the related good, leather, shifts to the right, as seen in Exhibit 2(d). Suppose the price of
beef falls, and as a result, the quantity supplied of beef falls; this leads to a decrease (a leftward shift)
D

in the supply of leather.


Other examples of complements in production where goods are produced simultaneously from
the same resource include a lumber mill that p ­ roduces lumber and sawdust or an oil refinery that
can produce gasoline and heating oil from the same r­esource—crude oil. Another example is that
when dairy farmers produce skim milk, they also produce cream. If the price of skim milk rises, the
dairy ­farmer produces more skim milk (an increase in the quantity supplied) and the supply of cream
increases.

Copyright ©2020 by SAGE Publications, Inc.


This work may not be reproduced or distributed in any form or by any means without express written permission of the publisher.
114    PART 2  •  SUPPLY AND DEMAND

CHANGES IN EXPECTATIONS (E)


Another factor shifting supply is sellers’ expectations. If
producers expect a higher price in the future, they will
supply less now than they ­otherwise would have, prefer-
ring to wait and sell when their goods will be more valu-
able. For e­ xample, if a cotton producer expected the f­ uture
price of cotton to be higher next year, he might decide to
iStock/nightman1965

e
store some of his current production of cotton for next
year when the price will be higher. Oil refiners will often

ut
store some of their spring supply of gasoline for summer
­because gasoline prices typically peak in summer. In addi-
tion, some of the heating oil for the fall is stored to supply

b
• During the global recession of 2007–2009, oil demand dropped sub- it in the winter when h ­ eating oil prices peak. Similarly, if
stantially. Early in 2009, as oil prices bottomed out, a situation known as producers expect now that the price will be lower later,

tri
contango developed, in which a large gap developed between oil prices they will supply more now. That is, expectations of lower
trading on the daily spot market and future-dated oil contracts. Traders
prices can increase supply (shift the supply curve to the
bought oil on the spot market and parked it in tankers until the prices

is
went back up. Floating storage worldwide peaked in April with nearly 90
right) as producers scramble to sell off their inventories
million barrels sitting in oil tankers waiting to be sold. • before the price falls.

rd
CHANGES IN THE NUMBER OF SELLERS (N)
We are normally interested in market demand and supply (because together they determine prices
and quantities) rather than in the behavior of individual consumers and firms. As we discussed ear-
lier in the chapter, the supply curves of individual suppliers can be summed horizontally to create

o
a market supply curve. An increase in the number of sellers leads to an increase in supply, denoted
by a rightward shift in the supply curve. For example, think of the number of gourmet coffee shops
t,
that have sprung up over the last 15 to 20 years, shifting the supply curve of gourmet coffee to the
right. An exodus of sellers has the opposite impact, a decrease in supply, which is indicated by a
os
leftward shift in the supply curve.

CHANGES IN TECHNOLOGY (T)


,p

Technological change can lower the firm’s costs of production through productivity advances.
These changes allow the firm to spend less on inputs and produce the same level of output. Human
py

USE WHAT YOU’VE LEARNED


co

Change in Supply versus Change in Section 4.5


Change in Supply versus
exhibit 3
Quantity Supplied Change in Quantity Supplied
t
no

Question: How would you graph the following two scenarios: (1) the
S1 S2
price of wheat per bushel rises; and (2) good weather causes an
Price of Wheat (per bushel)

­unusually abundant wheat harvest?


Answer: In the first scenario, the price of wheat (per bushel) i­ncreases, B C
o

$10
so the quantity supplied changes (i.e., a movement along the supply A B Change
D

curve). In the second scenario, the good weather causes the supply curve in Quantity
Supplied
for wheat to shift to the right, which is called a change in supply (not A
quantity supplied). A shift in the whole supply curve is caused by one of 5
B C Change
the other variables, not by a change in the price of the good in question. in Supply
As shown in Exhibit 3, the movement from A to B is called an i­ncrease in
quantity supplied, and the movement from B to A is called a d­ ecrease in 0 4 7 11
quantity supplied. However, the change from B to C is called an increase Quantity of Wheat (thousands of bushels per year)
in supply, and the movement from C to B is called a decrease in supply.

Copyright ©2020 by SAGE Publications, Inc.


This work may not be reproduced or distributed in any form or by any means without express written permission of the publisher.
Chapter 4  •  Demand, Supply, and Market Equilibrium 115  

creativity works to find new ways to produce goods and services using fewer or less costly inputs
of labor, natural resources, or capital. That is, changes in production technology, including the way
you make, distribute, and sell a good, can change the cost of production. For example, a new and
better technology can reduce the costs of production, raising the sellers’ willingness to supply the
good or service. Because the firm can now produce the good at a lower cost, it will supply more of
the good at each and every price—the supply curve shifts to the right.

e
4.5c Change in Supply versus Change in Quantity Supplied—Revisited

ut
If the price of the good changes, it leads to a change in the quantity supplied, ceteris paribus. If one
of the other factors influences sellers’ behavior, we say it results in a change in supply. For exam-
ple, if production costs rise because of a wage increase or higher fuel costs, other things remaining

b
constant, we would expect a decrease in supply—that is, a leftward shift in the supply curve.
Alternatively, if some variable, such as lower input prices, causes the costs of production to fall, the

tri
supply curve will shift to the right. Exhibit 4 illustrates the effects of some of the determinants that
cause shifts in the supply curve.
The following table lists some of the most important variables that affect how much producers are

is
willing to produce and sell. Remember, changes in the price of the good itself, when all the other vari-
ables that influence sellers are held constant, cause movements along a given curve, changing quantity

rd
supplied. A change in any of these other variables shifts the supply curve, causing a change in supply.

Section 4.5

o
exhibit 4
Possible Supply Shifts
t,
S2 S1 S1 S2 S1 S2 S1 S2
os
Price

Price

Price

Price
,p

0 0 0 0
Quantity Quantity Quantity Quantity
Input Price (wages) Input Price (fuel) Falls Price Decreases for Producer Expects
py

Increases a Substitute in Production Now that the Price


Will Be Lower Later

S1 S2 S2 S1 S1 S2 S2 S1
co
Price

Price

Price

Price
t
no

0 0 0 0
Quantity Quantity Quantity Quantity
Number of Sellers Producer Expects Now that Productivity Price Increase for a
Increases the Price Will Be Higher Later Rises Substitute in Production
o

VARIABLE A CHANGE IN THIS VARIABLE CAUSES


D

Price of the good itself a movement along the supply curve


Seller’s input prices a shift in the supply curve
Price of related goods and services a shift in the supply curve
Expectations a shift in the supply curve
Number of sellers a shift in the supply curve
Technology a shift in the supply curve

Copyright ©2020 by SAGE Publications, Inc.


This work may not be reproduced or distributed in any form or by any means without express written permission of the publisher.
SECTION QUIZ
1. All of the following factors will affect the supply of shoes except one. Which will not affect the supply of shoes?
a. higher wages for shoe factory workers
b. higher prices for leather

e
c. a technological improvement that reduces waste of leather and other raw materials in shoe production
d. an increase in consumer income

ut
2. The difference between a change in quantity supplied and a change in supply is that a change in
a. quantity supplied is caused by a change in a good’s own price, while a change in supply is caused by a change

b
in some other variable, such as input prices, prices of related goods, expectations, or taxes.
b. supply is caused by a change in a good’s own price, while a change in quantity supplied is caused by a change

tri
in some other variable, such as input prices, prices of related goods, expectations, or taxes.
c. quantity supplied is a change in the quantity people want to sell, while a change in supply is a change in the

is
quantity they actually sell.
d. A change in supply and a change in quantity supplied are the same thing.

rd
3. Antonio’s makes the greatest pizza and delivers it hot to all the dorms around campus. Last week Antonio’s supplier
of pepperoni informed him of a 25 percent increase in price. Which variable determining the position of the supply
curve has changed, and what effect does it have on supply?
a. future expectations; supply decreases

o
b. future expectations; supply increases
c. input prices; supply decreases
d. input prices; supply increases
t,
e. technology; supply increases
os
4. Which of the following is not a determinant of supply?
a. input prices
b. technology
,p

c. tastes
d. expectations
e. the prices of related goods
py

5. A leftward shift in supply could be caused by


a. an improvement in productive technology.
b. a decrease in income.
co

c. some firms leaving the industry.


d. a fall in the price of inputs to the industry.
t

1. What is the difference between a change in supply and a change in quantity supplied?
no

2. If a seller expects the price of a good to rise in the near future, how will that expectation affect the current supply
curve?
3. Would a change in the price of wheat change the supply of wheat? Would it change the supply of corn, if wheat and
o

corn can be grown on the same type of land?


4. If a guitar manufacturer increased its wages to keep its workers, what would happen to the supply of guitars as a
D

result?
5. What happens to the supply of babysitting services in an area when many teenagers get their driver’s licenses at
about the same time?
Multiple-choice answers: 1. d 2. a 3. c 4. c 5. c

Copyright ©2020 by SAGE Publications, Inc.


This work may not be reproduced or distributed in any form or by any means without express written permission of the publisher.
Chapter 4  •  Demand, Supply, and Market Equilibrium 117  

4.6 Market Equilibrium Price and Quantity


• What is the equilibrium price? market equilibri-
• What is the equilibrium quantity? um: the point at which
the market supply and
• What is a shortage?
market ­demand curves
• What is a surplus?
intersect

e
equilibrium price:
4.6a Equilibrium Price and Quantity the price at the inter-
The market equilibrium is found at the point at which the market supply and market demand

ut
section of the market
curves intersect. The price at the intersection of the market supply curve and the market demand supply and demand
curve is called the equilibrium price, and the quantity is called the equilibrium quantity. At the curves; at this price,

b
equilibrium price, the quantity that buyers are willing and able to buy is exactly equal to the the quantity demanded
equals the quantity
quantity that sellers are willing and able to produce. The equilibrium market solution is best

tri
supplied
understood with the help of a simple graph. Let’s return to the coffee example we used in our
earlier discussions of supply and demand. Exhibit 1 combines the market demand curve for cof- equilibrium quan-
fee with the market supply curve. At $3 per pound, buyers are willing to buy 5,000 pounds of tity: the quantity at

is
coffee and sellers are willing to supply 5,000 pounds of coffee. Neither may be “happy” about the intersection of the
the price; the buyers would probably like a lower price and the sellers would probably like a market supply and

rd
higher price. But both buyers and sellers are able to carry out their purchase and sales plans demand curves; at the
at the $3 price. At any other price, either suppliers or demanders would be unable to trade as equilibrium quantity,
the quantity demanded
much as they would like.
equals the quantity

o
supplied
4.6b Shortages and Surpluses
surplus: a situation
What happens when the market price is not equal to the equilibrium price? Suppose the market
t, where quantity sup-
price is above the equilibrium price, as seen in Exhibit 2(a). At $4 per pound, the quantity of coffee plied exceeds quantity
demanded would be 3,000 pounds, but the quantity supplied would be 7,000 pounds. At this price,
os
demanded
a surplus, or excess quantity supplied, would exist. That is, at this price, growers would be willing
to sell more coffee than demanders would be willing to buy. To get rid of the unwanted surplus, shortage: a situation
frustrated sellers have an incentive to cut their price to attract more buyers and cut back on produc- where quantity
,p

tion. Cutting the price will simultaneously increase the quantity demanded and decrease the quan- demanded exceeds
tity supplied. Note that the changes in quantity demanded and quantity supplied are movements quantity supplied
along the supply and demand curves, not shifts in the curves.
This adjustment will continue to reduce the surplus, as long Section 4.6
Market Equilibrium
py

as the price is above the equilibrium price, at $3. exhibit 1


What would happen if the market price of coffee were
below the equilibrium price? As seen in Exhibit 2(b), at $2
co

Supply
per pound, the yearly quantity demanded of 7,000 pounds $5
would be greater than the 3,000 pounds that producers
would be willing to supply at that low price. So at $2 per 4
Price of Coffee
(per pound)

pound, a shortage or excess quantity demanded of 4,000


t

pounds would exist. Some consumers are lucky enough to 3 Equilibrium


Equilibrium
no

find coffee, but others are not able to find any sellers who are price
2
willing to sell them coffee at $2 per pound. Some frustrated
consumers may offer to pay sellers more than $2. In addi- 1 Equilibrium Demand
tion, sellers noticing that there are disappointed consumers quantity
will be more than willing to raise their prices. That is, with
o

0
many buyers chasing few goods, sellers can respond to the 1 2 3 4 5 6 7 8 9 10
D

shortage by raising prices without the fear of losing sales. Quantity of Coffee
(thousands of pounds)
These actions by buyers and sellers cause the market price to
rise. As the market price rises, the quantity demanded falls The equilibrium is found at the intersection of the market supply
and the quantity supplied rises. Notice that these are move- and demand curves. The equilibrium price is $3 per pound, and the
ments along the supply and demand curves that move the equilibrium quantity is 5,000 pounds of coffee. At the equilibrium
market toward equilibrium. The upward pressure on price quantity, the quantity demanded equals the quantity supplied.
continues until equilibrium is reached at $3.

Copyright ©2020 by SAGE Publications, Inc.


This work may not be reproduced or distributed in any form or by any means without express written permission of the publisher.
118    PART 2  •  SUPPLY AND DEMAND

Section 4.6
exhibit 2
Market in Temporary Disequilibrium

a. Excess Quantity Supplied b. Excess Quantity Demanded

4,000 Pound
$5 Surplus $5
Supply Supply

e
4 4
Price of Coffee

Price of Coffee

ut
(per pound)

(per pound)
3 3

b
2 2
Demand Demand

tri
1 Quantity Quantity 1 Quantity 4,000 Pound Quantity
demanded supplied supplied Shortage demanded

0 0

is
3 5 7 3 5 7
Quantity of Coffee Quantity of Coffee
(thousands of pounds) (thousands of pounds)

rd
In (a), the market price is above the equilibrium price. At $4, the quantity supplied (7,000 pounds) exceeds the quantity demanded (3,000
pounds), resulting in a surplus of 4,000 pounds. To get rid of the unwanted surplus, suppliers cut their prices. As prices fall, consumers buy more,
eliminating the surplus and moving the market back to equilibrium. In (b), the market price is below the equilibrium price. At $2, the quantity

o
demanded (7,000 pounds) exceeds the quantity supplied (3,000 pounds), and a shortage of 4,000 pounds is the result. The many frustrated
buyers compete for the existing supply, offering to buy more and driving the price up toward the equilibrium level. Therefore, with both shortages
and surpluses, market prices tend to pull the market back to the equilibrium level.
t,
os

ECS Recall our earlier discussion of Adam Smith’s invisible hand. Here it is in action. Producers
,p

independently decide how much they are going to produce and at what price, and consumers
will show up at websites, garage sales, restaurants, and stores to buy those goods and services.
economic content Sometimes there might be too much supplied, while at other times there might not be enough. But
standards over time, these mistakes will be corrected by the process of adjustment in supply and demand,
py

A market exists when which comprises the invisible hand of the market.
buyers and sellers So whether the price starts off too high or too low, the activities of the many buyers and sell-
interact. This interaction ers will move the market toward equilibrium. Once equilibrium is reached, buyers and sellers are
co

between supply satisfied in their ability to buy and sell at that price, and there is no longer any pressure on prices.
and demand curves How quickly do markets adjust to equilibrium? It depends on the type of market. But in most
determines market prices competitive markets, shortages and surpluses tend to be temporary.
and thereby allocates
scarce goods and
4.6c Don’t Confuse Scarcity and Shortages
t

services.
no

People often confuse scarcity with shortages. Remember, most goods are scarce—desirable but
limited. A shortage occurs when the quantity demanded is greater than the quantity supplied at the
current price. We can eliminate shortages by increasing the price, but we cannot eliminate scarcity.
o
D

Copyright ©2020 by SAGE Publications, Inc.


This work may not be reproduced or distributed in any form or by any means without express written permission of the publisher.
SECTION QUIZ

1. A market will experience a _________ in a situation where quantity supplied exceeds quantity demanded and a
_________ in a situation where quantity demanded exceeds quantity supplied.
a. shortage; shortage

e
b. surplus; surplus
c. shortage; surplus

ut
d. surplus; shortage
2. The price of a good will tend to rise when

b
a. a temporary shortage at the current price occurs (assuming no price controls are imposed).
b. a temporary surplus at the current price occurs (assuming no price controls are imposed).

tri
c. demand decreases.
d. supply increases.

is
3. Which of the following is true?
a. The intersection of the supply and demand curves shows the equilibrium price and equilibrium quantity in a

rd
market.
b. A surplus is a situation where quantity supplied exceeds quantity demanded.
c. A shortage is a situation where quantity demanded exceeds quantity supplied.
d. Shortages and surpluses set in motion actions by many buyers and sellers that will move the market toward the

o
equilibrium price and quantity unless otherwise prevented.
e. All of the above are true.
t,
os
1. How does the intersection of supply and demand indicate the equilibrium price and quantity in a market?
2. What can cause a change in the supply and demand equilibrium?
,p

3. What must be true about the price charged for a shortage to occur?
4. What must be true about the price charged for a surplus to occur?
5. Why do market forces tend to eliminate both shortages and surpluses?
py

6. If tea prices were above their equilibrium level, what force would tend to push tea prices down? If tea prices were
below their equilibrium level, what force would tend to push tea prices up?
co

Multiple-choice answers: 1. d 2. a 3. e
t
no

Visit edge.sagepub.com/sextonexploreecon8e to help you accomplish


your coursework goals in an easy-to-use learning environment.
o

• Mobile-friendly eFlashcards
D

• Chapter summaries with learning objectives


• Mobile-friendly practice quizzes • EXCLUSIVE! Access to full-text SAGE journal articles
• A complete online action plan

Copyright ©2020 by SAGE Publications, Inc.


This work may not be reproduced or distributed in any form or by any means without express written permission of the publisher.
120    PART 2  •  SUPPLY AND DEMAND

INTERACTIVE SUMMARY
Fill in the blanks: 14. According to the law of supply, the higher the price of the
good, the greater the _________, and the lower the price of
1. A(n) _________ is the process of buyers and sellers
the good, the smaller the _________.
_________ goods and services.
15. The quantity supplied is positively related to the price ­because
2. The important point about a market is what it does—it facil-

e
firms supplying goods and services want to ­increase their
itates _________.
_________.

ut
3. _________, as a group, determine the demand side of the
16. An individual supply curve is a graphical representation that
market. _________, as a group, determine the supply side
shows the _________ relationship between the price and
of the market.

b
the quantity supplied.
4. A(n) _________ market consists of many buyers and sell-

tri
17. The market supply curve is a graphical representation of the
ers, no single one of whom can influence the market price.
number of goods and services that sellers are _________
5. According to the law of demand, other things being equal, and _________ to supply at various prices.

is
when the price of a good or service falls, the _________
18. Possible supply determinants (factors that determine the
increases.

rd
­position of the supply curve) are the _________ prices, the
6. An individual _________ curve reveals the different quan- _________ of related goods; _________, the _________
tities of a particular good a person would be willing and able of sellers, and _________.
to buy at various possible prices in a particular time interval,
19. A fall in input prices will _________ the costs of production,

o
other things being equal.
causing the supply curve to shift to the _________.
7. The _________ curve for a product is the horizontal sum-
20. The supply of a good _________ if the price of one of its
ming of the demand curves of the individuals in the market.
t,
substitutes in production falls.
os
8. A change in _________ leads to a change in quantity
21. The supply of a good _________ if the price of one of its
­demanded, illustrated by a(n) _________ demand curve.
substitutes in production rises.
9. A change in demand is caused by changes in any of the
22. The price at the intersection of the market demand curve
,p

other factors (besides the good’s own price) that would


and the market supply curve is called the _________ price,
­affect how much of the good is purchased: the _________,
and the quantity is called the _________ quantity.
_________, the _________ of buyers, _________, and
py

_________. 23. A situation where quantity supplied is greater than quantity


demanded is called a(n) _________.
10. An increase in demand is represented by a(n) _________
shift in the demand curve; a decrease in demand is repre- 24. A situation where quantity demanded is greater than quantity
co

sented by a(n) _________ shift in the demand curve. supplied is called a(n) _________.
11. Two goods are called _________ if an increase in the price 25. At a price greater than the equilibrium price, a(n) _________,
of one causes the demand curve for another good to shift to or excess quantity supplied, would exist. Sellers would be
the _________. willing to sell _________ than demanders would be willing
t
no

to buy. Frustrated suppliers would _________ their price


12. For normal goods an increase in income leads to a(n)
and _________ on production, and consumers would buy
_________ in demand, and a decrease in income leads to
_________, returning the market to equilibrium.
a(n) _________ in demand, other things being equal.
13. An increase in the expected future price of a good or an
o

­increase in expected future income may _________ c­ urrent


D

demand.
equilibrium 23. surplus 24. shortage 25. surplus; more; lower; cut back; more
18. seller’s input; prices; expectations; number of sellers; technology 19. lower; right 20. increases 21. decreases 22. equilibrium;
substitutes; right 12. increase; decrease 13. increase 14. quantity supplied; quantity supplied 15. profits 16. positive 17. willing; able
a good’s price; movement along the 9. prices of related goods; income; number; tastes; expectations 10. rightward; leftward 11.
Answers: 1. market; exchanging 2. trade 3. Buyers; Sellers 4. competitive 5. quantity demanded 6. demand 7. market demand 8.

Copyright ©2020 by SAGE Publications, Inc.


This work may not be reproduced or distributed in any form or by any means without express written permission of the publisher.
Chapter 4  •  Demand, Supply, and Market Equilibrium 121  

KEY TERMS AND CONCEPTS


change in quantity demanded 100 individual supply curve 109 market supply curve 110
competitive market 95 inferior goods 103 normal goods 103
complements 102 law of demand 96 shifts in the demand curve 101
equilibrium price 117 law of supply 109 shortage 117

e
equilibrium quantity 117 market 95 substitutes 101

ut
individual demand curve 97 market demand curve 98 surplus 117
individual demand schedule 97 market equilibrium 117

b
ADDITIONAL SECTION QUIZ ANSWERS

tri
4.1 Markets

is
1. Why is it difficult to define a market precisely? 3. Why do the prices people pay for similar items at
garage sales vary more than the prices of similar
Every market is different. An incredible variety of exchange

rd
items in a department store?
arrangements arise for different types of products, different
­
­degrees of organization, different geographical extents, and so on. Items for sale at department stores are more standardized,
easier to compare, and more heavily advertised, which
2. Why do you get your produce at a supermarket rather

o
makes consumers more aware of the prices at which they
than directly from farmers?
could get a particular good elsewhere, reducing the differ-
Supermarkets act as middlepersons between growers of
t, ences in price that can persist among department stores.
produce and consumers of produce. You hire them to do this Garage sale items are nonstandardized, costly to compare,
os
task for you when you buy produce from them, rather than and not advertised, which means people are often quite
directly from growers, because they conduct those transac- ­unaware of how much a given item could be purchased for
tions at lower costs than you could. (If you could do it more elsewhere, so price differences for similar items at different
,p

cheaply than supermarkets, you would buy directly rather garage sales can be substantial.
than from supermarkets.)

4.2 Demand
py

1. What is an inverse relationship? 5. What is the difference between an individual demand


curve and a market demand curve?
An inverse, or negative, relationship is one where one vari-
co

able changes in the opposite direction from the other—if The market demand curve shows the total quantities of a
one increases, the other decreases. good or service all the buyers as a group are willing to buy
at various possible prices in a particular time interval. The
2. How do lower prices change buyers’ incentives?
market quantity demanded at a given price is just the sum of
t

A lower price for a good means that the opportunity cost to the quantities demanded by each buyer at that price.
no

buyers of purchasing it is lower than before, and self-interest


6. Why does the amount of dating on campus tend to
leads buyers to buy more of it as a result.
decline just before and during final exams?
3. How do higher prices change buyers’ incentives?
The opportunity cost of dating—in this case, the value to
o

A higher price for a good means that the opportunity cost to students of the studying time forgone—is higher just before
buyers of purchasing it is higher than before, and self-interest and during final exams than during most of the rest of an
D

leads buyers to buy less of it as a result. academic term. Because the cost is higher, students do less
4. What is an individual demand schedule? of it.

An individual demand schedule reveals the different quanti-


ties of a good or service a person would be willing to buy at
various possible prices in a particular time interval.

Copyright ©2020 by SAGE Publications, Inc.


This work may not be reproduced or distributed in any form or by any means without express written permission of the publisher.
122    PART 2  •  SUPPLY AND DEMAND

4.3 Shifts in the Demand Curve


1. What is the difference between a change in demand p­ urchases become, and the greater the current demand for
and a change in quantity demanded? that good; people will buy more now when it is expected to
be cheaper, rather than later, when it is expected to be more
A change in demand shifts the entire demand curve, while a
costly.
change in quantity demanded refers to a movement along a
given demand curve, caused by a change in the good’s price. 5. Would a change in the price of ice cream cause a

e
change in the demand for ice cream? Why or why not?
2. If the price of zucchini increases, causing the

ut
demand for yellow squash to rise, what do we call the No. The demand for ice cream represents the different
relationship between zucchini and yellow squash? quantities of ice cream that would be purchased at different
­prices. In other words, it represents the relationship ­between

b
Whenever an increased price of one good increases the
the price of ice cream and the quantity of ice cream
demand for another, the goods are substitutes. The fact

tri
­demanded. Changing the price of ice cream does not change
that some people consider zucchini an alternative to ­yellow
this relationship, so it does not change demand.
squash explains in part why zucchini becomes more c­ ostly.
6. Would a change in the price of ice cream likely cause a

is
Therefore, some people substitute by buying relatively
cheaper yellow squash now instead. change in the demand for frozen yogurt, a substitute?

rd
3. If incomes rise and, as a result, demand for jet skis Yes. Changing the price of ice cream, a substitute for ­frozen
increases, how do we describe that good? yogurt, would change the quantity of frozen yogurt ­demanded
at a given price. This change in price means that the whole
If income rises and, as a result, demand for jet skis ­increases,
relationship between the price and quantity of frozen y­ ogurt

o
we call jet skis a normal good because for most (or normal)
demanded has changed, which means the demand for
goods, we would rather have more of them than less, so an
­frozen yogurt has changed.
increase in income would lead to an increase in demand for
t,
such goods. 7. If plane travel is a normal good and bus travel is an
os
inferior good, what will happen to the demand curves
4. How do expectations about the future influence the
for plane and bus travel if people’s incomes increase?
demand curve?
The demand for plane travel and all other normal goods
Expectations about the future influence the demand curve
,p

will increase if incomes increase, while the demand for bus


because buying a good in the future is an alternative to buy-
travel and all other inferior goods will decrease if incomes
ing it now. Therefore, the higher future prices are ­expected
increase.
to be compared to the present, the less attractive future
py

4.4 Supply
1. What are the two reasons why a supply curve is 2. What is the difference between an individual supply
co

positively sloped? curve and a market supply curve?


A supply curve is positively sloped because when the price The market supply curve shows the total quantities of a good
rises, the good becomes more profitable for sellers, so the all the sellers as a group are willing to sell at various prices
t

quantity supplied rises; when the price falls, it becomes less in a particular time period. The market quantity supplied at a
no

profitable for sellers, and the quantity supplied falls. given price is just the sum of the quantities supplied by each
seller at that price.

4.5 Shifts in the Supply Curve


o

1. What is the difference between a change in supply 2. If a seller expects the price of a good to rise in the
D

and a change in quantity supplied? near future, how will that expectation affect the
­current supply curve?
A change in supply shifts the entire supply curve, while a
change in quantity supplied refers to a movement along a Selling a good in the future is an alternative to selling it now.
given supply curve. Therefore, the higher the expected future price relative to
the current price, the more attractive future sales become,
and the less attractive current sales become. This will lead

Copyright ©2020 by SAGE Publications, Inc.


This work may not be reproduced or distributed in any form or by any means without express written permission of the publisher.
Chapter 4  •  Demand, Supply, and Market Equilibrium 123  

sellers to reduce (shift left) the current supply of that good, 4. If a guitar manufacturer increased its wages to keep
as they want to sell later, when the good is expected to be its workers, what would happen to the supply of
more valuable, rather than now. guitars as a result?
3. Would a change in the price of wheat change the An increase in wages, or any other input price, would de-
­supply of wheat? Would it change the supply of corn, crease (shift left) the supply of guitars, making fewer guitars
if wheat and corn can be grown on the same type of available for sale at any given price, by raising the opportu-
land? nity cost of producing guitars.

e
The supply of wheat represents the different quantities of 5. What happens to the supply of babysitting services

ut
wheat that would be offered for sale at different prices. In in an area when many teenagers get their driver’s
other words, it represents the relationship between the price licenses at about the same time?

b
of wheat and the quantity of wheat supplied. Changing the
When teenagers get their driver’s licenses, their increased
price of wheat does not change this relationship, so it does

tri
mobility expands their alternatives to babysitting substan-
not change the supply of wheat. However, a change in the
tially, raising the opportunity cost of babysitting. This change
price of wheat changes the relative attractiveness of raising
decreases (shifts left) the supply of babysitting services.

is
wheat instead of corn, which changes the supply of corn.

rd
4.6 Market Equilibrium Price and Quantity
1. How does the intersection of supply and demand A ­seller is better off successfully selling at a lower equilib-
indicate the equilibrium price and quantity in a market? rium price than not being able to sell at a higher price (the

o
surplus situation) and a buyer is better off successfully buy-
The intersection of supply and demand indicates the equilib-
ing at a higher equilibrium price than not being able to buy at
rium price and quantity in a market because at higher prices,
a lower price (the shortage situation). Therefore, we expect
sellers would be frustrated by their inability to sell all they
t,
market forces to eliminate both shortages and surpluses.
would like, leading sellers to compete by lowering the price
os
they charge; at lower prices, buyers would be frustrated by 6. If tea prices were above their equilibrium level, what
their inability to buy all they would like, leading buyers to force would tend to push tea prices down? If tea
compete by increasing the price they offer to pay. prices were below their equilibrium level, what force
,p

would tend to push tea prices up?


2. What can cause a change in the supply and demand
equilibrium? If tea prices were above their equilibrium level, sellers would
be frustrated by their inability to sell as much tea as they
Changes in any of the demand curve shifters or the
py

would like at those prices. That is, not all producers could
­supply curve shifters will change the supply and demand
find willing buyers at these high prices. To eliminate the sur-
­equilibrium.
plus, producers would try to attract more buyers by bringing
3. What must be true about the price charged for a
co

the price down. As the price falls, the quantity demanded


shortage to occur? rises and the quantity supplied falls, until the market r­ eaches
The price charged must be less than the equilibrium price, the equilibrium quantity, where quantity demanded and
with the result that buyers would like to buy more at that quantity supplied are equal. If tea prices were below their
t

price than sellers are willing to sell. equilibrium level, buyers would be frustrated by their inability
no

to buy as much tea as they would like at those prices. Buyers


4. What must be true about the price charged for a who could find the good available would bid up the price, and
surplus to occur? this would incent producers to raise their prices. As the price
The price charged must be greater than the equilibrium rises, the quantity demanded falls and the quantity supplied
o

price, with the result that sellers would like to sell more at rises, until the market reaches the equilibrium quantity,
that price than buyers are willing to buy. where quantity demanded and quantity supplied are equal.
D

5. Why do market forces tend to eliminate both shortages


and surpluses?
Market forces tend to eliminate both shortages and s­ urpluses
because of the self-interest of the market participants.

Copyright ©2020 by SAGE Publications, Inc.


This work may not be reproduced or distributed in any form or by any means without express written permission of the publisher.
124    PART 2  •  SUPPLY AND DEMAND

PROBLEMS
1. Is the market for laptop computers local, national, or global?
2. Sid moves from New York City, where he lived in a small condominium, to rural Minnesota, where he buys a big house on five acres
of land. Using the law of demand, what do you think is true of land prices in New York City relative to those in rural Minnesota?
3. The following table shows Hillary’s demand schedule for Cherry Blossom Makeup. Graph Hillary’s demand curve.

e
PRICE (DOLLARS PER OUNCE) QUANTITY DEMANDED (OUNCES PER WEEK)

ut
$15 5 oz.

12 10

b
9 15

tri
6 20

is
3 25

4. The following table shows Cherry Blossom Makeup demand schedules for Hillary’s friends, Barbara and Nancy. If Hillary, Barbara,

rd
and Nancy constitute the whole market for Cherry Blossom Makeup, complete the market demand schedule and graph the market
demand curve.

o
QUANTITY DEMANDED (OUNCES PER WEEK)

PRICE (DOLLARS PER


OUNCE) HILLARY BARBARA
t, NANCY MARKET
os
$15 5 0 15

12 10 5 20
,p

9 15 10 25

6 20 15 30
py

3 25 20 35

5. What would be the effects of each of the following on the demand for hamburger in Hilo, Hawaii? In each case, identify the
­responsible determinant of demand.
co

a. The price of chicken falls.


b. The price of hamburger buns doubles.
t

c. Scientists find that eating hamburger prolongs life.


no

d. The population of Hilo doubles.


6. What would be the effect of each of the following on the demand for Chevrolets in the United States? In each case, identify the
responsible determinant of demand.
o

a. The price of Fords plummets.


D

b. Consumers believe that the price of Chevrolets will rise next year.
c. The incomes of Americans rise.
d. The price of gasoline falls dramatically.

Copyright ©2020 by SAGE Publications, Inc.


This work may not be reproduced or distributed in any form or by any means without express written permission of the publisher.
Chapter 4  •  Demand, Supply, and Market Equilibrium 125  

7. The following graph shows three market demand curves for cantaloupe. Starting at point A,
a. which point represents an increase in quantity demanded?
b. which point represents an increase in demand?
c. which point represents a decrease in demand?
d. which point represents a decrease in quantity demanded?

ut
C

E
A

b
Price

tri
F B

D1 D2

is
D0

Quantity of Cantaloupes per Year

rd
8. Using the demand curve, show the effect of the following events on the market for beef:
a. Consumer income increases.

o
b. The price of beef increases.
c. An outbreak of “mad cow” disease occurs.
t,
d. The price of chicken (a substitute) increases.
os
e. The price of barbecue grills (a complement) increases.
9. Draw the demand curves for the following goods. If the price of the first good listed rises, what will happen to the demand for the
second good, and why?
,p

a. hamburger and ketchup


b. Coca-Cola and Pepsi
py

c. golf clubs and golf balls


10. If the price of ice cream increased,
co

a. what would be the effect on the demand for ice cream?


b. what would be the effect on the demand for frozen yogurt?
11. Using the graph below, answer the following questions.
t

a. What is the shift from D1 to D2 called?


no

b. What is the movement from B to A called?


c. What is the movement from A to B called?
d. What is the shift from D2 to D1 called?
o

P
D

A
P1 C

B
P2

D2
D1
0
Q1 Q2 Q3 Q

Copyright ©2020 by SAGE Publications, Inc.


This work may not be reproduced or distributed in any form or by any means without express written permission of the publisher.
126    PART 2  •  SUPPLY AND DEMAND

12. Felix is a wheat farmer who has two fields he can use to grow wheat. The first field is right next to his house, and the topsoil is
rich and thick. The second field is 10 miles away in the mountains, and the soil is rocky. At current wheat prices, Felix produces
from the field next to his house because the market price for wheat is just high enough to cover his costs of production, including
a reasonable profit. What would have to happen to the market price of wheat for Felix to have the incentive to produce from the
second field?
13. Show the impact of each of the following events on the oil market.

e
a. OPEC becomes more effective in limiting the supply of oil.

ut
b. OPEC becomes less effective in limiting the supply of oil.
c. The price for natural gas (a substitute for heating oil) rises.

b
d. New oil discoveries occur in Alaska.

tri
e. Electric and hybrid cars become subsidized and their prices fall.
14. The following table shows the supply schedule for Rolling Rock Oil Co. Plot Rolling Rock’s supply curve on a graph.

is
PRICE (DOLLARS QUANTITY SUPPLIED
PER BARREL) (BARRELS PER MONTH)

rd
$5 10,000

10 15,000

o
15 20,000 t,
20 25,000

25 30,000
os
15. The following table shows the supply schedules for Rolling Rock and two other petroleum companies. Armadillo Oil and Pecos
Petroleum. Assuming these three companies make up the entire supply side of the oil market, complete the market supply
,p

­schedule and draw the market supply curve on a graph.

QUANTITY SUPPLIED (BARRELS PER MONTH)


py

PRICE (DOLLARS
PER BARREL) ROLLING ROCK ARMADILLO OIL PECOS ­PETROLEUM MARKET
$5 10,000 8,000 2,000 _________
co

10 15,000 10,000 5,000 _________


15 20,000 12,000 8,000 _________
20 25,000 14,000 11,000 _________
t
no

25 30,000 16,000 14,000 _________

16. If the price of corn rose,


a. what would be the effect on the supply of corn?
o

b. what would be the effect on the supply of wheat?


D

17. Using the following graph, answer the below questions:


a. What is the shift from S1 to S2 called?
b. What is the movement from A to B called?
c. What is the movement from B to A called?
d. What is the shift from S2 to S1 called?

Copyright ©2020 by SAGE Publications, Inc.


This work may not be reproduced or distributed in any form or by any means without express written permission of the publisher.
Chapter 4  •  Demand, Supply, and Market Equilibrium 127  


P S1
S2
B
P2
C

A
P1

e
0
Q1 Q2 Q3 Q

ut
18. What would be the effect of each of the following on the supply of salsa in the United States? In each case, identify the responsible
determinant of supply.

b
a. Tomato prices skyrocket.

tri
b. Congress places a 26 percent tax on salsa.

is
c. Ed Scissorhands introduces a new, faster vegetable ­chopper.
d. J. Lo, Beyoncé, and Adam Sandler each introduce a new brand of salsa.

rd
19. What would be the effects of each of the following on the supply of coffee worldwide? In each case, identify the responsible
­determinant of supply.
a. Freezing temperatures wipe out half of Brazil’s coffee crop.

o
b. Wages of coffee workers in Latin America rise as unionization efforts succeed.
t,
c. Indonesia offers big subsidies to its coffee producers.
d. Genetic engineering produces a super coffee bean that grows faster and needs less care.
os
e. Coffee suppliers expect prices to be higher in the future.
20. The following graph shows three market supply curves for cantaloupe. Compared with point A, which point represents
,p

a. an increase in quantity supplied?


b. an increase in supply?
py

c. a decrease in quantity supplied?


d. a decrease in supply?
co


S0
S1
B
E S2
t

A C
Price
no

D
o

Quantity of Cantaloupes per Year


D

Copyright ©2020 by SAGE Publications, Inc.


This work may not be reproduced or distributed in any form or by any means without express written permission of the publisher.
128    PART 2  •  SUPPLY AND DEMAND

21. The following table shows the hypothetical monthly demand and supply schedules for cans of macadamia nuts in Hawaii.
a. What is the equilibrium price of macadamia nuts in Hawaii?
b. At a price of $7 per can, is there equilibrium, a surplus, or a shortage? If it is a surplus or shortage, how large is it?
c. At a price of $10, is there equilibrium, a surplus, or a shortage? If it is a surplus or shortage, how large is it?

QUANTITY QUANTITY

e
PRICE ­DEMANDED (CANS) ­SUPPLIED (CANS)

ut
$6 700 100

7 600 200

b
8 500 300

tri
9 400 400

is
10 300 500

rd
22. When asked about the reason for a lifeguard shortage that threatened to keep one-third of the city’s beaches closed for the sum-
mer, the Deputy Parks Commissioner of New York responded that “Kids seem to want to do work that’s more in tune with a career.
Maybe they prefer carpal tunnel syndrome to sunburn.” What do you think is causing the shortage? What would you advise the
Deputy Parks Commissioner to do to alleviate the shortage?

o
23. If a price is above the equilibrium price, explain the forces that bring the market back to the equilibrium price and quantity. If a price
is below the equilibrium price, explain the forces that bring the market back to the equilibrium price and quantity.
t,
24. The market for baseball tickets at your college stadium, which seats 2,000, is the following:
os
QUANTITY
PRICE DEMANDED QUANTITY SUPPLIED
,p

$2 4,000 2,000

4 2,000 2,000
py

6 1,000 2,000

8 500 2,000
co

a. What is the equilibrium price?


b. What is unusual about the supply curve?
c. At what price(s) would a shortage occur?
t
no

d. At what price(s) would a surplus occur?


e. Suppose that the addition of new students (all big baseball fans) next year will add 1,000 to the quantity demanded at each
price. What will this increase do to next year’s demand curve? What is the new equilibrium price?
o
D

Copyright ©2020 by SAGE Publications, Inc.


This work may not be reproduced or distributed in any form or by any means without express written permission of the publisher.
Chapter 4  •  Demand, Supply, and Market Equilibrium 129  

25. Assume the following information for the demand and supply curves for good Z.

DEMAND SUPPLY
QUANTITY QUANTITY
PRICE DEMANDED PRICE SUPPLIED
$10 10 $1 10

e
9 20 2 15

ut
8 30 3 20

7 40 4 25

b
6 50 5 30

tri
5 60 6 35

is
4 70 7 40

3 80 8 45

rd
2 90 9 50

1 100 10 55

o
a. Draw the corresponding supply and demand curves.
b. What are the equilibrium price and quantity traded?
t,
c. Would a price of $9 result in a shortage or a surplus? How large?
os
d. Would a price of $3 result in a shortage or a surplus? How large?
e. If the demand for Z increased by 15 units at every price, what would the new equilibrium price and quantity traded be?
,p

f. Given the original demand for Z, if the supply of Z were increased by 15 units at every price, what would the new equilibrium
price and quantity traded be?
py
t co
no
o
D

Copyright ©2020 by SAGE Publications, Inc.


This work may not be reproduced or distributed in any form or by any means without express written permission of the publisher.
Exam Four - Sample Questions
Chapters 12-14

MULTIPLE CHOICE. Choose the one alternative that best completes the statement or answers the question.

1) What is the difference between perfect competition and monopolistic competition?


A) Perfect competition has a large number of small firms while monopolistic competition does not.
B) In perfect competition, firms produce identical goods, while in monopolistic competition, firms produce
slightly different goods.
C) Perfect competition has no barriers to entry, while monopolistic competition does.
D) Perfect competition has barriers to entry while monopolistic competition does not.

2) The market type known as perfect competition is


A) almost free from competition and firms earn large profits.
B) highly competitive and firms find it impossible to earn an economic profit in the long run.
C) dominated by fierce advertising campaigns.
D) marked by firms continuously trying to change their products so that consumers prefer their product to
their competitors' products.

3) Which of the following market types has all firms selling products so identical that buyers do not care from
which firm they buy?
A) perfect competition B) oligopoly
C) monopolistic competition D) monopoly

4) Perfect competition is characterized by all of the following EXCEPT


A) well-informed buyers and sellers with respect to prices.
B) a large number of buyers and sellers.
C) no restrictions on entry into or exit from the industry.
D) considerable advertising by individual firms.

5) Which of the following is the best example of a perfectly competitive market?


A) diamonds B) athletic shoes C) soft drinks D) farming

6) Which of the following market types has the fewest number of firms?
A) perfect competition B) monopoly
C) monopolistic competition D) oligopoly

7) Which of the following market types has a large number of firms that sell similar but slightly different
products?
A) perfect competition B) oligopoly
C) monopolistic competition D) monopoly

8) Which of the following market types has only a few competing firms?
A) perfect competition B) monopolistic competition
C) monopoly D) oligopoly

1
9) In a perfectly competitive market, the type of decision a firm has to make is different in the short run than in
the long run. Which of the following is an example of a perfectly competitive firm's short-run decision?
A) what price to charge buyers for the product
B) whether or not to enter or exit an industry
C) the profit-maximizing level of output
D) how much to spend on advertising and sales promotion

10) In perfect competition, a firm maximizes profit in the short run by deciding
A) how much output to produce. B) whether or not to enter a market.
C) what price to charge. D) how much capital to use.

11) In a perfectly competitive market, the type of decision a firm has to make is different in the short run than in
the long run. Which of the following is an example of a perfectly competitive firm's long-run decision?
A) what price to charge buyers for the product
B) how much to spend on advertising and sales promotion
C) the profit-maximizing level of output
D) whether or not to enter or exit an industry

12) A price-taking firm


A) cannot influence the price of the product it sells.
B) talks to rival firms to determine the best price for all of them to charge.
C) sets the product's price to whatever level the owner decides upon.
D) asks the government to set the price of its product.

13) A large number of sellers all selling an identical product implies which of the following?
A) horizontal market supply curves
B) large losses by all sellers
C) the inability of any seller to change the price of the product
D) market chaos

14) Perfectly competitive firms are price takers because


A) each firm is very large. B) there are no good substitutes for their goods.
C) many other firms produce identical products. D) their demand curves are downward sloping.

15) If demand for a seller's product is perfectly elastic, which of the following is correct?
A) There is no incentive to sell at a price below the market price.
B) It will not sell any output at all if it tries to price its product above the market price.
C) There are a very large number of perfect substitutes for the seller's product.
D) All of the above answers are correct.

2
16) One of the requirements for a monopoly is that
A) the product cannot be produced by small firms.
B) there are several close substitutes for the product.
C) there is a unique product with no close substitutes.
D) products are high priced.

17) A monopoly is a market with


A) no barriers to entry. B) many substitutes. C) many suppliers. D) one supplier.

18) Firms face competition when the good they produce


A) is in a market with natural barriers to entry. B) is unique.
C) is in a market with legal barriers to entry. D) has a close substitute.

19) Which of the following statements is correct?


A) The market demand and the firm's demand are the same for a monopoly.
B) Monopolies have perfectly inelastic demand for the product sold.
C) Monopolies are guaranteed to earn an economic profit.
D) All of the above are correct.

20) Which describes a barrier to entry?


A) anything that protects a firm from the arrival of new competitors
B) a government regulation that bars a monopoly from earning an economic profit
C) something that establishes a barrier to expanding output
D) firms already in the market incurring economic losses so that no new firm wants to enter the market

21) A barrier to entry is


A) an open door.
B) the economic term for diseconomies of scale.
C) illegal in most markets.
D) anything that protects a firm from the arrival of new competitors.

22) Which of the following would create a natural monopoly?


A) requirement of a government license before the firm can sell the good or service
B) technology enabling a single firm to produce at a lower average cost than two or more firms
C) an exclusive right granted to supply a good or service
D) ownership of all the available units of a necessary input

23) If the technology for producing a good enables one firm to meet the entire market demand at a lower price than
two or more firms could, then that firm has
A) a legal barrier to entry. B) a natural monopoly.
C) increasing average total costs. D) patented the market.

24) Which of the following goods is the best example of a natural monopoly?
A) natural gas B) diamonds C) a patented good D) first-class mail

3
25) Which of the following is the best example of a natural monopoly?
A) owning the only licensed taxicab in town
B) the United States Postal Service
C) ownership of the only ferry across Puget Sound for twenty miles
D) the cable television company in your hometown

26) Which barrier to entry is an exclusive right granted to the author or composer of a literary, musical, dramatic
or artistic work?
A) government license B) patent C) public franchise D) copyright

27) Patents
A) stimulate innovation.
B) encourage the invention of new products and production methods.
C) are exclusive rights granted to the inventor of a product or service.
D) All of the above answers are correct.

28) Which of the following is NOT correct about patents?


A) Patents stimulate innovation.
B) A patent is a barrier to entry.
C) Patents enable a firm to be a permanent monopoly.
D) Patents encourage invention of new products.

29) Recently in a small city, building contractors lobbied the city council to pass a law requiring all people
working on residential dwellings be licensed by the city. Why would the contractors lobby for this
requirement?
A) to reduce the cost of building dwellings
B) There is no good explanation for this type of lobbying.
C) to guarantee that work on dwellings is of high quality
D) to create a legal barrier to entry

30) Ownership of a necessary input creates what type of barrier to entry?


A) natural barrier to entry B) a public franchise
C) a government license D) legal barrier to entry

31) An industry with a large number of firms, differentiated products, and free entry and exit is called
A) oligopoly. B) monopoly.
C) monopolistic competition. D) perfect competition.

32) In monopolistic competition, each firm supplies a small part of the market. This occurs because
A) there are barriers to entry. B) firms produce differentiated products.
C) there are no barriers to entry. D) there are a large number of firms.

33) In monopolistic competition, the products of different sellers are assumed to be


A) similar but slightly different. B) identical perfect substitutes.
C) either identical or differentiated. D) unique without any close or perfect substitutes.

4
34) Which of the following is different about perfect competition and monopolistic competition?
A) Firms in monopolistic competition compete on their product's price as well as its quality and marketing.
B) In monopolistic competition, entry into the industry is unblocked.
C) Perfect competition has a large number of independently acting sellers.
D) Only firms in monopolistic competition can earn an economic profit in the short run.

35) In an industry with a large number of firms,


A) collusion is impossible.
B) one firm will dominate the market.
C) each firm will produce a large quantity, relative to market demand.
D) competition is eliminated.

36) Which of the following is an example of a monopolistically competitive industry?


A) wheat farming B) colleges and universities
C) the local electricity producer D) the domestic automobile producing industry

37) All of the following are examples of product differentiation in monopolistic competition EXCEPT
A) new and improved packaging.
B) lower price.
C) acceptance of more credit cards than the competition.
D) location of the retail store.

38) A differentiated product has


A) many perfect substitutes. B) close but not perfect substitutes.
C) no close substitutes. D) no substitutes of any kind.

39) As the degree of product differentiation increases among the products sold in a monopolistically competitive
industry, which of the following occurs?
A) The cost of production falls.
B) The amount of marketing expenditures decreases for each firm.
C) The demand curve for each seller's product becomes more horizontal.
D) Each seller's demand becomes more inelastic.

40) Marketing consists of what?


A) selling at a lower price than rivals sell for B) producing more output to lower average costs
C) advertising and packaging D) None of the above answers are correct.

41) Firms use marketing to


A) influence a consumer's buying decision.
B) convince customers that their product is worth its price.
C) persuade buyers that their product is superior to others.
D) All of the above answers are correct.

5
42) If a monopolistically competitive seller can convince buyers that its product is of better quality and value than
products sold by rival firms,
A) demand increases. B) the firm gains more control over its price.
C) demand becomes more inelastic. D) all of the above occur.

43) If you have found the percentage of the value of sales accounted for by the four largest firms in an industry, you
have found the
A) elasticity of supply value. B) Herfindahl-Hirschman Index.
C) elasticity of demand value. D) four-firm concentration ratio.

44) Which of the following four-firm concentration ratios would be the best indication of a perfectly competitive
industry?
A) 100 percent B) 78 percent C) 0.25 percent D) 31 percent

45) Which of the following four-firm concentration ratios is consistent with monopolistic competition?
A) 0 percent B) 25 percent C) 100 percent D) 75 percent

6
Answer Key
Testname: EXAM FOUR SAMPLE QUESTIONS.TST

1) Answer: B
2) Answer: B
3) Answer: A
4) Answer: D
5) Answer: D
6) Answer: B
7) Answer: C
8) Answer: D
9) Answer: C
10) Answer: A
11) Answer: D
12) Answer: A
13) Answer: C
14) Answer: C
15) Answer: D
16) Answer: C
17) Answer: D
18) Answer: D
19) Answer: A
20) Answer: A
21) Answer: D
22) Answer: B
23) Answer: B
24) Answer: A
25) Answer: D
26) Answer: D
27) Answer: D
28) Answer: C
29) Answer: D
30) Answer: D
31) Answer: C
32) Answer: D
33) Answer: A
34) Answer: A
35) Answer: A
36) Answer: B
37) Answer: B
38) Answer: B
39) Answer: D
40) Answer: C

1
Answer Key
Testname: EXAM FOUR SAMPLE QUESTIONS.TST

41) Answer: D
42) Answer: D
43) Answer: D
44) Answer: C
45) Answer: B

2
ec2ex1.rvw

Practice Questions

Economics 120 M. Babcock


Exam 1 Spring 2007

1. The law of increasing costs states that:


a) if the prices of all the resources devoted to the production of goods increase, the
cost of producing any particular good will increase at the same rate.
b) if the sum of the costs of producing a particular good rises by a specified
percentage, the price of that good must rise by a greater relative amount.
c) the sum of the costs of producing a particular good cannot rise above the current
market price of that good.
d) if society wants to produce more of a particular good, it must sacrifice larger and
larger amounts of other goods to do so.

2. The fundamental problem of economics is:


a) to achieve a more equitable distribution of money income in order to mitigate
poverty.
b) the scarcity of productive resources relative to material wants.
c) the establishment of prices which accurately reflect the relative scarcities of
products and resources.
d) to establish a democratic political framework for the provision of social goods and
services.
e) to establish an equitable system of personal and business taxation.

3. Assume that a change in government policy results in the increased production of both
consumer goods and investment goods. It can be concluded that:
a) the law of increasing costs does not apply in this society.
b) this economy's production possibilities curve is convex (bowed inward) as viewed
from the origin.
c) the economy's production possibilities curve has been shifted to the left as a result
of the policy decision.
d) the economy was suffering from unemployment and/or the inefficient use of
resources before the policy change.

4. Which of the following is not an illustration of the notion of opportunity cost?


a) The production of more military goods means fewer resources are available for
civilian goods.
b) The land a Kansas farmer plants in wheat is not available for corn production.
c) Resources devoted to consumer goods production are not available for capital
goods production.
d) If I buy a pizza, I will not be able to afford a movie.
e) A growing economy can produce more consumer goods and more capital goods at
the same time.
5. Which of the following will tend to shift the production possibilities curve to the right?
a) an increase in the unemployment rate from 6 to 8 percent
b) a decline in the efficiency with which the present labor force is allocated
c) a decrease in the unemployment rate from 8 to 6 percent
d) a technological advance which allows farmers to produce more output from given
inputs
e) none of the above

6. Which of the following statements is incorrect?


a) If supply declines and demand remains constant, equilibrium price will rise.
b) If demand decreases and supply increases, equilibrium price will rise.
c) If supply increases and demand decreases, equilibrium price will fall.
d) If demand increases and supply decreases, equilibrium price will rise.

7. In which of the following instances will the effect upon equilibrium price be
indeterminant, that is, dependent upon the magnitude of the given shifts in supply and
demand?
a) demand rises and supply is constant
b) supply rises and demand falls
c) demand rises and supply falls
d) supply falls and demand remains constant
e) demand rises and supply rises

8. If the supply and demand curves for a product both decrease, we can say that equilibrium:

a) quantity and equilibrium price must both decline.


b) quantity must decline, but equilibrium price may either rise, fall, or remain
unchanged.
c) price must fall, but equilibrium quantity may either rise, fall, or remain
unchanged.
d) quantity must fall and equilibrium price must rise.

9. If the supply of a product decreases and the demand for that product simultaneously
increases, we can conclude that equilibrium:
a) quantity must decrease, but equilibrium price may either rise, fall, or remain
unchanged.
b) price and equilibrium quantity must both decline.
c) price and equilibrium quantity must both increase.
d) price must rise and equilibrium quantity must fall.
e) price must rise, but equilibrium quantity may either rise, fall, or remain
unchanged.
10. Assuming competitive markets with typical supply and demand curves, which of the
following statements is correct?
a) An increase in demand with no change in supply will result in a decline in sales.
b) An increase in demand with no change in supply will result in an increase in sales.

c) An increase in supply with no change in demand will result in a decline in sales.


d) An increase in supply with no change in demand will result in an increase in price.
e) An increase in supply with a decrease in demand will result in an increase in
price.

11. In moving along a stable demand curve which of the following is NOT held constant?
a) the prices of substitute goods
b) the prices of complementary goods
c) consumer incomes
d) price expectations
e) the price of the product for which the demand curve is relevant

12. In moving along a stable supply curve which of the following is NOT held constant?
a) the prices of the resources used in producing this good
b) the price of the product for which the supply curve is relevant
c) techniques used in producing this product
d) expectations about the future price of the product
e) the number of firms producing this good

13. If products A and B are complements and the price of B decreases:


a) the demand for A will decline and the demand for B will increase.
b) the demand for A will increase and the amount of B demanded will increase.
c) the amount of B purchased will increase, but the demand curve for A will not
shift.
d) the demand curves for both A and B will shift to the left.

14. If products C and D are close substitutes, an increase in the price of C will:
a) shift the demand curves of both products to the left.
b) shift the demand curves of both products to the right.
c) shift the demand curve of D to the right.
d) shift the demand curve of C to the left and the demand curve of D to the right.
e) tend to cause the price of D to fall.

15. Which of the following is NOT a basic criticism of the price system?
a) The price system limits individual freedom by imposing resource and product
prices which buyers and sellers cannot control.
b) The price system does not account for external costs and benefits.
c) The price system is permissive of recession and/or inflation.
d) Income inequality causes the price system to result in the production of luxuries
for the rich at the expense of necessities for the poor.
Resource Possible production techniques
Resource Prices #1 #2 #3 #4 #5

Land $4 2 4 2 4 4
Labor 3 1 2 4 1 3
Capital 3 5 2 3 1 2
Entrepreneurial ability 2 3 1 1 4 1

Use this figure to answer question 16

The figure illustrates all available techniques by which 20 units of a given commodity can
be produced.

16. Given the indicated resource prices, the economically most efficient production
technique(s) will be technique(s):
a) #1.
b) #2 and #4.
c) #3.
d) #1 and #3.
e) #5.

17. From society's point of view the economic function of profits and losses is to:
a) reallocate resources from less-desired to more-desired uses.
b) contribute to a more equal distribution of income.
c) perpetuate full employment and price level stability.
d) promote the equal distribution of real assets and wealth.

18. One drawback to the proprietorship form of business organization is:


a. lack of personal discretion in management.
b. separation of ownership and management.
c. limited access to capital.
d. stifling of individual initiative.

19. Partnership is an improvement over proprietorship in that:


a. partnerships are not subject to unlimited liability.
b. partnerships provide a stable, unchanging form of organization.
c. partnerships bring in added capital and expertise.
d. decision making in a partnership is smoother and quicker.

20. The primary advantage of a corporation is:


a. limited liability.
b. double taxation.
c. the closeness between ownership and management.
d. limited access to resources.
26. The opportunity cost of moving from point A to point B is:
a. 18 million doughnuts.
b. 2 million doughnuts.
c. 6 million glasses of milk.
d. 1 million glasses of milk.

27. If an economy uses inefficient methods of production, then:


a. the economy is sacrificing goods and services it could otherwise have.
b. the economy will need a larger government sector.
c. the economy at least will have more people employed.
d. the economy will be on its production possibilities curve, but will be producing
primarily one good or the other rather than nearly equal amounts of each good.

28. Increasing opportunity cost occurs because:


a. workers become less productive as they work longer hours.
b. resources are specialized.
c. prices rise as additional output is produced.
d. unemployment falls as output increases.

29. Which of the following best expresses the law of increasing costs ?
a. The more units of a good someone buys, the more money that must be paid for
them.
b. The more units of a good someone buys, the higher will be the price charged.
c. The more units of a good a country produces, the greater is the opportunity cost of
an extra unit.
d. The more resources a country has, the more costly will be imports.

30. How do consumers signal firms to increase production of a good?


a. by bidding up the price of the good.
b. by contacting government agencies.
c. by writing letters to the company.
d. by voting for political candidates of their choice.

31. Other things being equal, when the price of a good rises:
a. quantity demanded will fall.
b. quantity demanded will rise.
c. demand will decrease.
d. demand will increase.
Chapter 01 - The Investment Environment

Chapter 01
The Investment Environment

Multiple Choice Questions

1. In 2007, ____________ was the most significant real asset of U.S. households in terms of
total value.
A. consumer durables
B. automobiles
C. real estate
D. mutual fund shares
E. bank loans

See Table 1.1.

Difficulty: Easy

2. In 2007, ____________ was the least significant financial asset of U.S. households in terms
of total value.
A. real estate
B. mutual fund shares
C. debt securities
D. life insurance reserves
E. pension reserves

See Table 1.1.

Difficulty: Easy

1-1
Chapter 01 - The Investment Environment

3. In 2007, ____________ was the most significant asset of U.S. households in terms of total
value.
A. real estate
B. mutual fund shares
C. debt securities
D. life insurance reserves
E. pension reserves

See Table 1.1.

Difficulty: Easy

4. In 2007, ____________ was the most significant liability of U.S. households in terms of
total value.
A. credit cards
B. mortgages
C. bank loans
D. student loans
E. other debt

See Table 1.1.

Difficulty: Easy

5. The largest component of domestic net worth in 2007 was ____________.


A. non-residential real estate
B. residential real estate
C. inventories
D. consumer durables
E. equipment and software

See Table 1.2.

Difficulty: Moderate

1-2
Chapter 01 - The Investment Environment

6. In 2007, ____________ was the most significant real asset of U.S. nonfinancial businesses
in terms of total value.
A. equipment and software
B. inventory
C. real estate
D. trade credit
E. marketable securities

See Table 1.4.

Difficulty: Easy

7. In 2007, ____________ was the least significant real asset of U.S. nonfinancial businesses
in terms of total value.
A. equipment and software
B. inventory
C. real estate
D. trade credit
E. marketable securities

See Table 1.4.

Difficulty: Easy

8. In 2007, ____________ was the least significant liability of U.S. nonfinancial businesses in
terms of total value.
A. bonds and mortgages
B. bank loans
C. inventories
D. trade debt
E. marketable securities

See Table 1.4.

Difficulty: Easy

1-3
Chapter 01 - The Investment Environment

9. In terms of total value, the most significant liability of U.S. nonfinancial businesses in 2007
was _______.
A. bank loans
B. bonds and mortgages
C. trade debt
D. other loans
E. marketable securities.

See Table 1.4.

Difficulty: Easy

10. In 2007, ____________ was the most significant financial asset of U.S. nonfinancial
businesses in terms of total value.
A. cash
B. trade credit
C. trade debt
D. inventory
E. marketable securities

See Table 1.4.

Difficulty: Easy

11. The material wealth of a society is equal to the sum of _________.


A. all financial assets
B. all real assets
C. all financial and real assets
D. all physical assets
E. none of the above

Financial assets do not directly contribute the productive capacity of the economy.

Difficulty: Easy

1-4
Chapter 01 - The Investment Environment

12. ____________ of an investment bank.


A. Citigroup is an example
B. Merrill Lynch is an example
C. Goldman is an example
D. B and C are each examples
E. Each of the above is an example

Merrill Lynch, Citigroup, and Goldman are all examples of investment banks.

Difficulty: Easy

13. _______ are financial assets.


A. Bonds
B. Machines
C. Stocks
D. A and C
E. A, B and C

Machines are real assets; stocks and bonds are financial assets.

Difficulty: Easy

14. An example of a derivative security is ______.


A. a common share of General Motors
B. a call option on Mobil stock
C. a commodity futures contract
D. B and C
E. A and B

The values of B and C are derived from that of an underlying financial asset; the value of A is
based on the value of the firm only.

Difficulty: Easy

1-5
Chapter 01 - The Investment Environment

15. _______ was the first to introduce mortgage pass-through securities.


A. Chase Manhattan
B. Citicorp
C. FNMA
D. GNMA
E. None of the above

GNMA introduced mortgage pass through securities in 1970.

Difficulty: Easy

16. A bond issue is broken up so that some investors will receive only interest payments while
others will receive only principal payments, which is an example of ________.
A. bundling
B. credit enhancement
C. unbundling
D. financial engineering
E. C and D

Unbundling is one of many examples of financial engineering that offer more alternatives to
the investor.

Difficulty: Easy

17. An example of a primitive security is __________.


A. a common share of General Motors
B. a call option on Mobil stock
C. a call option on a stock of a firm based in a Third World country
D. a U.S. government bond
E. A and D

A primitive security's return is based only upon the earning power of the issuing agency, such
as stock in General Motors and the U.S. government.

Difficulty: Easy

1-6
Chapter 01 - The Investment Environment

18. The ____________ refers to the potential conflict between management and shareholders
due to management's control of pecuniary rewards as well as the possibility of incompetent
performance by managers.
A. agency problem
B. diversification problem
C. liquidity problem
D. solvency problem
E. regulatory problem

The agency problem describes potential conflict between management and shareholders. The
other problems are those of firm management only.

Difficulty: Easy

19. _________ financial asset(s).


A. Buildings are
B. Land is a
C. Derivatives are
D. U.S. Agency bonds are
E. C and D

A and B are real assets.

Difficulty: Easy

20. The value of a derivative security _______.


A. depends on the value of the related primitive security
B. can only cause increased risk.
C. is unrelated to the value of the related primitive security
D. has been enhanced due the recent misuse and negative publicity regarding these
instruments
E. is worthless today

Of the factors cited above, only A affects the value of the derivative and/or is a true statement.

Difficulty: Easy

1-7
Chapter 01 - The Investment Environment

21. Money market funds were a financial innovation partly inspired to circumvent _______.
A. Regulation B, which is still in existence
B. Regulation D
C. DIDMCA
D. Regulation M
E. Regulation Q, which is no longer in existence

Regulation Q limited the amount of interest that banks could pay to depositors; money market
funds were not covered by Regulation Q and thus could pay a higher rate of interest. Although
Regulation Q no longer exists, money market funds continue to be popular.

Difficulty: Easy

22. __________ are a way U.S. investor can invest in foreign companies.
A. ADRs
B. IRAs
C. SDRs
D. GNMAs
E. Krugerrands

Only ADRs represent an indirect investment in a foreign company.

Difficulty: Easy

23. _______ are examples of financial intermediaries.


A. Commercial banks
B. Insurance companies
C. Investment companies
D. Credit unions
E. All of the above

All are institutions that bring borrowers and lenders together.

Difficulty: Easy

1-8
Chapter 01 - The Investment Environment

24. Financial intermediaries exist because small investors cannot efficiently ________.
A. diversify their portfolios
B. gather all relevant information
C. assess credit risk of borrowers
D. advertise for needed investments
E. all of the above.

The individual investor cannot efficiently and effectively perform any of the tasks above
without more time and knowledge than that available to most individual investors.

Difficulty: Easy

25. Firms that specialize in helping companies raise capital by selling securities are called
________.
A. commercial banks
B. investment banks
C. savings banks
D. credit unions
E. all of the above.

An important role of investment banks is to act as middlemen in helping firms place new
issues in the market.

Difficulty: Easy

26. Financial assets ______.


A. directly contribute to the country's productive capacity
B. indirectly contribute to the country's productive capacity
C. contribute to the country's productive capacity both directly and indirectly
D. do not contribute to the country's productive capacity either directly or indirectly
E. are of no value to anyone

Financial assets indirectly contribute to the country's productive capacity because these assets
permit individuals to invest in firms and governments. This in turn allows firms and
governments to increase productive capacity.

Difficulty: Easy

1-9
Chapter 01 - The Investment Environment

27. The sale of a mortgage portfolio by setting up mortgage pass-through securities is an


example of ________.
A. credit enhancement
B. securitization
C. unbundling
D. derivatives
E. none of the above

The financial asset is secured by the mortgages backing the instrument.

Difficulty: Easy

28. Corporate shareholders are best protected from incompetent management decisions by
A. the ability to engage in proxy fights.
B. management's control of pecuniary rewards.
C. the ability to call shareholder meetings.
D. the threat of takeover by other firms.
E. one-share / one-vote election rules.

Proxy fights are expensive and seldom successful, and management may often control the
board or own significant shares. It is the threat of takeover of underperforming firms that has
the strongest ability to keep management on their toes.

Difficulty: Moderate

29. The national net worth of the U.S. in 2007 was _________.
A. $15.411 trillion
B. $26.431 trillion
C. $42.669 trillion
D. $48.038 trillion
E. $70.983 trillion

See Table 1.2.

Difficulty: Moderate

1-10
Chapter 01 - The Investment Environment

30. In 2007, _______ of the assets of U.S. households were financial assets as opposed to
tangible assets.
A. 20.4%
B. 34.2%
C. 61.1%
D. 71.7%
E. 82.5%

See Table 1.1.

Difficulty: Moderate

31. Investment bankers perform the following role(s) ___________.


A. market new stock and bond issues for firms
B. provide advice to the firms as to market conditions, price, etc
C. design securities with desirable properties
D. all of the above
E. none of the above

Investment bankers perform all of the roles described above for their clients.

Difficulty: Easy

32. Theoretically, takeovers should result in ___________.


A. improved management
B. increased stock price
C. increased benefits to existing management of taken over firm
D. A and B
E. A, B, and C

Theoretically, when firms are taken over, better managers come in and thus increase the price
of the stock; existing management often must either leave the firm, be demoted, or suffer a
loss of existing benefits.

Difficulty: Easy

1-11
Chapter 01 - The Investment Environment

33. Important trends changing the contemporary investment environment are


A. globalization.
B. securitization.
C. information and computer networks.
D. financial engineering.
E. all of the above

All of these are examples of important trends in the contemporary investment environment.

Difficulty: Easy

34. The means by which individuals hold their claims on real assets in a well-developed
economy are
A. investment assets.
B. depository assets.
C. derivative assets
D. financial assets.
E. exchange-driven assets

Financial assets allocate the wealth of the economy. Example: it is easier for an individual to
own shares of an auto company than to own an auto company directly.

Difficulty: Easy

35. Which of the following financial assets made up the greatest proportion of the financial
assets held by U.S. households?
A. pension reserves
B. life insurance reserves
C. mutual fund shares
D. debt securities
E. personal trusts

See Table 1.1.

Difficulty: Moderate

1-12
Chapter 01 - The Investment Environment

36. Which of the following are mechanisms that have evolved to mitigate potential agency
problems?
I) compensation in the form of the firm's stock options
II) hiring bickering family members as corporate spies
III) underperforming management teams being forced out by boards of directors
IV) security analysts monitoring the firm closely
V) takeover threats
A. II and V
B. I, III, and IV
C. I, III, IV, and V
D. III, IV, and V
E. I, III, and V

All but the second option have been used to try to limit agency problems.

Difficulty: Moderate

37. Commercial banks differ from other businesses in that both their assets and their liabilities
are mostly
A. illiquid.
B. financial.
C. real.
D. owned by the government.
E. regulated.

See Table 1.3.

Difficulty: Easy

1-13
Chapter 01 - The Investment Environment

38. Which of the following is true about GNMA pass-throughs?


I) They aggregate individual home mortgages into heterogeneous pools.
II) The purchaser of a GNMA receives monthly interest and principal payments received from
payments made on the pool.
III) The banks that originated the mortgages maintain ownership of them.
IV) The banks that originated the mortgages continue to service them.
A. II, III, and IV
B. I, II, and IV
C. II and IV
D. I, III, and IV
E. I, II, III, and IV

III is not correct because the bank no longer owns the mortgage investments.

Difficulty: Moderate

39. Although derivatives can be used as speculative instruments, businesses most often use
them to
A. attract customers.
B. appease stockholders.
C. offset debt.
D. hedge.
E. enhance their balance sheets.

Firms may use forward contracts and futures to protect against currency fluctuations or
changes in commodity prices. Interest-rate options help companies control financing costs.

Difficulty: Easy

1-14
Chapter 01 - The Investment Environment

40. An ETF
A. limits the diversification potential of investors who hold it.
B. may be traded only in the primary market.
C. is linked directly to the value of a composite index of futures contracts.
D. must be earned as a performance bonus within a corporation rather than purchased.
E. tracks the performance of an index of share returns for a particular country or industry
sector.

ETFs allow investors to trade portfolios in the secondary markets.

Difficulty: Moderate

41. A country ETF


A. invests in real estate in the country.
B. invests in small country businesses.
C. is linked directly to the value of a composite index of commodity futures contracts.
D. is not very popular and is only used by speculators.
E. tracks the performance of an index of share returns for a particular country.

Country ETFs allow investors to trade portfolios that mimic foreign indices in the secondary
markets.

Difficulty: Moderate

1-15
Chapter 01 - The Investment Environment

42. During the period between 2000 and 2002, a large number of scandals were uncovered.
Most of these scandals were related to
I) Manipulation of financial data to misrepresent the actual condition of the firm.
II) Misleading and overly optimistic research reports produced by analysts.
III) Allocating IPOs to executives as a quid pro quo for personal favors.
IV) Greenmail.
A. II, III, and IV
B. I, II, and IV
C. II and IV
D. I, III, and IV
E. I, II, and III

I, II, and III are all mentioned as causes of recent scandals.

Difficulty: Moderate

43. A disadvantage of using stock options to compensate managers is that


A. it encourages mangers to undertake projects that will increase stock price.
B. it encourages managers to engage in empire building.
C. it can create an incentive for mangers to manipulate information to prop up a stock price
temporarily, giving them a chance to cash out before the price returns to a level reflective of
the firms true prospects.
D. all of the above.
E. none of the above.

A is a desired characteristic. B is not necessarily a good or bad thing in and of itself. C creates
an agency problem.

Difficulty: Easy

1-16
Chapter 01 - The Investment Environment

44. A fixed-income security pays ____________.


A. a fixed level of income for the life of the owner
B. a fixed stream of income or a stream of income that is determined according to a specified
formula for the life of the security
C. a variable level of income for owners on a fixed income
D. a fixed or variable income stream at the option of the owner
E. none of the above

Only answer B is correct.

Difficulty: Easy

45. Money market securities ____________.


A. are short term
B. pay a fixed income
C. are highly marketable
D. generally very low risk
E. all of the above

All answers are correct.

Difficulty: Easy

46. Financial assets permit all of the following except ____________.


A. consumption timing
B. allocation of risk
C. separation of ownership and control
D. elimination of risk
E. all of the above

Financial assets do not allow risk to be eliminated. However, they do permit allocation of risk,
consumption timing, and separation of ownership and control.

Difficulty: Moderate

1-17
Chapter 01 - The Investment Environment

47. The Sarbanes-Oxley Act ____________.


A. requires corporations to have more independent directors
B. requires the firm's CFO to personally vouch for the firm's accounting statements
C. prohibits auditing firms from providing other services to clients
D. A and B are correct.
E. A, B, and C are correct.

The Sarbanes-Oxley Act does all of the above.

Difficulty: Moderate

48. Asset allocation refers to ____________.


A. choosing which securities to hold based on their valuation
B. investing only in "safe" securities
C. the allocation of assets into broad asset classes
D. bottom-up analysis
E. all of the above

Asset allocation refers to the allocation of assets into broad asset classes.

Difficulty: Moderate

49. Which of the following portfolio construction methods starts with security analysis?
A. Top-down
B. Bottom-up
C. Middle-out
D. Buy and hold
E. Asset allocation

Bottom-up refers to using security analysis to find securities that are attractively priced. Top-
down refers to using asset allocation as a starting point.

Difficulty: Moderate

1-18
Chapter 01 - The Investment Environment

50. Which of the following portfolio construction methods starts with asset allocation?
A. Top-down
B. Bottom-up
C. Middle-out
D. Buy and hold
E. Asset allocation

Bottom-up refers to using security analysis to find securities that are attractively priced.

Difficulty: Moderate

Short Answer Questions

51. Discuss the agency problem in detail.

Managers are the agents of the shareholders, and should act on their behalf to maximize
shareholder wealth (the value of the stock). A conflict (the agency conflict) arises when
managers take self-interested actions to the detriment of shareholders. The roles of the board
of directors selected by the shareholders are to oversee management and to minimize agency
problems. However, often these boards are figureheads, and individual shareholders do not
own large enough blocks of the shares to override management actions. One potential
resolution of an agency problem occurs when inefficient management actions cause the price
of the stock to be depressed. The firm may then become a takeover target. If the acquisition is
successful, managers may be replaced and potentially, stockholders benefit.
Feedback: The question is designed to ascertain that the student understands the corporate
relationship between shareholders, management, and the board of directors. In addition, this
problem has been addressed extensively in recent years, both in the popular financial press
during the mergers and acquisitions mania of the 1980s, and in the academic literature as
agency theory.

Difficulty: Moderate

1-19
Chapter 01 - The Investment Environment

52. Discuss the similarities and differences between real and financial assets.

Real assets represent the productive capacity of the firm, and appear as assets on the firm's
balance sheet. Financial assets are claims against the firm, and thus appear as liabilities on the
firm's balance sheet. On the other hand, financial assets are listed on the asset side of the
balance sheet of the individuals who own them. Thus, when financial statements are
aggregated across the economy, the financial assets cancel out, leaving only the real assets,
which directly contribute to the productive capacity of the economy. Financial assets
contribute indirectly only.
Feedback: The purpose of this question is to ascertain if the student understands the difference
between real and financial assets, both in the aggregate balance sheet context and the relative
contribution of the two types of assets to the productive capacity of the economy.

Difficulty: Moderate

53. Discuss the euro in relation to its impact on globalization. How is it currently used and
what are the plans for its future use?

The euro was introduced in 1999 as a new currency and has replaced the currencies of twelve
participating countries so there will be one common European currency in the participating
countries. A common currency is expected to facilitate global trade and encourage the
integration of markets across national boundaries.
Feedback: The purpose of this question is to test the student's understanding of the use and
impact of the euro.

Difficulty: Moderate

1-20
Chapter 01 - The Investment Environment

54. Discuss the following ongoing trends as they relate to the field of investments:
globalization, financial engineering, securitization, and computer networks.

Globalization offers a wider array of investment choices than what would be available to
investors who could only choose domestic securities. As efficient communication technology
has become available, globalization of markets has been significantly enhanced. There are
many mechanisms by which one country's investors can hold foreign companies' securities.
Some examples are ADRs, WEBS, and direct purchase of foreign securities.
Securitization refers to aggregating underlying financial assets, such as mortgages, into pools
and then offering a security that represents a claim on these underlying assets. Examples are
GNMAs. Securitization allows investors to hold partial ownership in financial assets that
would otherwise be beyond their reach (e.g., mortgages).
Financial engineering involves bundling or unbundling. Bundling involves combining
separate securities together into one composite security. Examples are combining primitive
and derivative securities, and combining three primitive securities such as common stock,
preferred stock, and bonds. Unbundling is the opposite - two or more security classes are
created by separating a composite security into parts.
Computer networks have permitted online trading, online information dissemination and
automated trade crossing. Each of these major breakthroughs has significant implications for
investments.
Feedback: The purpose of this question is to test the student's understanding of the major
trends that impact the field of investments.

Difficulty: Moderate

1-21
BA Economics - II Semester–Core Course
MICRO ECONOMICS -II
Multiple Choice Questions
1. The value of the inputs owned and used by the firm in its own production processes is
a. Implicit cost b. Marginal cost
c. Sunk cost d. Explicit cost
2. MC at any level of output is given by
a. Slope of TC curve b. Slope of TVC curve
c. Slope of either TC or TVC d. Slope of TFC
3. If a firm’s average cost is Rs.32 at 6 units of output and Rs.34 at 7 unit, which one among
the following is the marginal cost of producing the 7th unit
a. 46 b. 2
c. 36 d. 42
4. Cost function expresses the relationship between
a. Cost and price b. Output and cost
c. Input and cost d. Output and input
5. A firm’s average total cost is Rs.110, its average variable cost is Rs.100 and its output is 50
units. What is its total fixed cost?
a. Less than 300 b. Between 400 to 600
c. More than 600 d. Firm has no fixed cost
6. The cost that cannot be recovered once spent
a. Accounting cost b. Fixed cost
c. Implicit cost d. Sunk cost
7. The saucer-type of modern Short run Average Variable Cost (SAVC) represents
a. Excess capacity b. managerial costs
c. Load factors d. Reserve capacity
8. The Long run Average Cost curve (LAC) in modern cost theory is roughly
a. U shaped b. Saucer shaped
c. L shaped d. Rectangular hyperbola
9. Under increasing returns to scale, which of the following is the nature of the long run
average cost curve?
a. Downward sloping b. Upward rising
c. Parallel to output axis d. Identical to short run average cost curve
10. Which of the following has a U shape?
a. Average fixed cost curve b. Total cost curve
c. Average variable cost curve d. Total variable cost curve
11. For a perfectly competitive firm, the optimum level of output is given by the point where
a. MR equals MC b. MR equals AC
c. MR exceeds MC by the greatest amount d. MR equal MC and MC is rising
12. Which of the following most closely approximates the perfectly competitive model?
a. Cigarette industry b. Rice farming
c. Newspaper d. Automobile
13. Short run supply curve of perfectly competitive market is given by
a. Rising portion of MC curve
b. Rising portion of MC curve over and above its AVC curve
c. Rising portion of MC curve over and above its AC curve
d. Rising portion of MC curve over and above its MR curve
14. When the perfectly competitive firm and industry are both in long run equilibrium?
a. P = MR = SMC = LMC b. P = MR = SAC = LAC
c. P = MR = lowest point on the LAC curve d. All the above
15. Total profits are maximized under perfect competition where
a. TR exceed TC by the greatest amount b. TR equals TC and TC is rising
c. TR equal to TC d. TC exceed TR by the greatest amount
16. Under increasing cost industries, long run supply curve is
a. Negatively sloped b. Horizontal
c. Positively sloped d. any of the above
17. If factor prices and factor quantities move in opposite direction, we have
a. A constant cost industry b. An increasing cost industry
c. A decreasing cost industry d. Any of the above
18. At shutdown point
a. P = AVC b. TR = TVC
c. The total losses of the firm equal TFC d. all the above
19. In short run equilibrium, a perfect competitive firm will
a. Incur loss b. No loss No profit situation
c. Maximizing its profit d. Any of above is possible
20. The following are the characteristics of perfect competition except
a. Government regulations b. Product homogeneity
c. Large number of sellers d. Free and exit
21. Monopsony is a market structure which is characterized by
a. Single buyer b. Single seller
c. Few buyers d. Single buyer and single seller
22. A market with only one seller and only one buyer is called
a. Duopoly b. Monopsony
c. Bilateral monopoly d. Oligopoly
23. In short run, the monopolist
a. Breaks even b. Incurs a loss
c. Makes a profit d. Any of the above
24. The Lerner index is a measure of
a. A firm’s monopoly power b. Peak- load pricing
c. An industry’s monopoly power d. Collusion in the oligopoly market
25. Under monopoly MR is always
a. Less than AR b. Equals to AR
c. Greater than AR d. Any of the above
26. In long run a pure monopolist can make profits because of
a. High prices b. Blocked entry
c. Low cost d. Advertising
27. Maximum exploitation of each buyer in the interest of seller is possible under
a. First degree price discrimination b. Second degree price discrimination
c. Third degree price discrimination d. All the above
28. An international price discrimination is
a. Bundling b. Dumping
c. Discounting d. Off-loading
29. If the monopolist incurs losses in short run, then in long run
a. He will go out of the business b. He will stay in the business
c. He will break even d. Any of above is possible
30. If the demand curve for the product is identical in two separate markets, then by
practicing third degree price discrimination, the monopolist
a. Will increase profits b. Cannot increase profits
c. Will charge different price in different market d. Any of the above is possible
31. The cost incurred by the firm in hiring labour
a. Explicit cost b. Marginal cost
c. Implicit cost d. Social cost
32. Supply curve is backward bending for
a. Land b. Labour
c. Capital d. Organization
33. A discriminating monopolist will charge a lower price in the market in which the price
elasticity of demand
a. Unity b. Lower
c. Greater d. Any of above
34. A Cartel aims at maximizing
a. Individual profit b. Industrial profit
c. Share of output of market d. Good will of members
35. The kinked demand curve is reflected in a discontinuity in
a. TR curve b. MC curve
c. MR curve d. AR curve
36. Price discrimination under monopoly possible only when
a. Price elasticity of demand is same in different market
b. Markets should have coordination
c. Factors of production among markets should be freely mobile
d. price elasticity of demand should be different among markets
37. Kinked demand curve of an Oligopolist is
a. Elastic in upper portion but inelastic in lower portion
b. Inelastic in upper portion but elastic in lower portion
c. Elastic throughout
d. Inelastic throughout
38. AFC curve will always be
a. Rectangular hyperbola b. U shaped
c. Horizontal d. Downward sloping
39. Implicit cost of a factor of production is determined by its
a. Sunk cost b. Variable cost
c. Fixed cost d. Opportunity cost
40. Economic cost include both
a. Explicit cost and implicit cost b. Fixed cost and variable cost
c. Explicit cost and prime cost d. Money cost and sunk cost
41. The U shape of MC curve reflects
a. Economies of scale b. Law of increasing returns
c. Reserve capacity d. Law of variable proportion
42. Under monopolistic competition
a. Few firms selling a differentiated product
b. Many firms selling a homogeneous product
c. Few firms selling a homogeneous product
d. Many firms selling a differentiated product
43. Supply curve of land for the economy as a whole is
a. Perfectly elastic b. Perfectly inelastic
c. Inelastic d. Elastic
44. Envelope curve is
a. Long run marginal cost curve b. Long run average cost curve
c. Total cost curve d. None of the above
45. In long run, which factor of production is fixed?
a. Labour b. Capital
c. building d. none of the above
46. Under monopolist competition, in long run firm may incur
a. Profit b. Loss
c. Normal profit d. Any of above
47. The process of distinguishing a product or service from others to make it more attractive
a. Price competition b. Selling cost
c. Non price competition d. Product differentiation
48. The kinked demand curve hypothesis was put forward by
a. Alfred Marshall b. Hall and Hitch
c. Paul. M. Sweezy d. Jean Robbinson
49. A monopolist practicing dumping, charges
a. High price abroad and low price in home market
b. Low price abroad and high price in home market
c. Same price in both abroad and home market
d. Any of above is possible
50. Perfect price discrimination is
a. First degree price discrimination b. Second degree price discrimination
c. Third degree price discrimination d. Dumping
51. By imposing a lump sum tax, firm under perfect composition face
a. Upward shift in AC curve in long run b. Upward shift in AC curve in short run
c. Upward shift in MC curve in short run d. Upward shift in MC curve in long run
52. Under constant cost industry, the long run average supply curve is
a. Downward sloping b. Upward sloping
c. Horizontal d. Vertical
53. In which of the following markets firms are interdependent?
a. Monopoly b. Monopolistic competition
c. Perfect competition d. Oligopoly
54. Under oligopoly, the price rigidity is the result of
a. Product differentiation b. Interdependence among the firms
c. Kinked demand curve d. Cartelization of firms
55. As the number of sellers in an oligopoly grows larger, an oligopoly market look like a
a. Monopoly b. Perfect competition
c. Duopoly d. Monopolistic competition
56. According to Chamberlin, selling cost curve is
a. U shaped b. Rectangular hyperbola
c. L shaped d. Inverse S shape
57. A monopolist always produces
a. Unitary portion of demand curve b. Less elastic portion of demand curve
c. More elastic portion of demand curve d. Any of above
58. Under price contol, the government set a maximum price at the level where
a. SMC curve cut MR from below b. SMC curve cut the demand curve
c. SMC curve AR curve d. SMC = MR
59. A monopolist able to shift burden of which regulation to consumers
a. lump sum tax b. price control
c. Per unit tax d. All the above
60. The Xerox Corporation requires consumers to purchase its papers along with its machine.
This is an example of
a. Tying b. Bundling
c. Two part tariffs d. mark up pricing
61. Cost functions are derived from
a. Demand functions b. Production functions
c. Supply functions d. Revenue functions
62. The U shape of the average total cost curve reflects
a. LDMU b. The Law of Variable Proportions
c. Consumer’s Surplus d. Reserve capacity
63. The total fixed cost is a
a. horizontal straight line b. vertical
c. hyperbola d. U shaped
64. When AC minimum in short run
a. AC < MC b. AC > MC
c. AC = MC d. any of above is possible
65. The shape of TVC and TC are
a. rectangular hyperbola b. inverse ‘S’ shape
c. horizontal straight line d. L shaped
66. The cost expressed not in terms of money but in terms of efforts of workers undergone for
making the commodity
a. Opportunity cost b. Real cost
c. Sacrifice cost d. Implicit cost
67. The MC curve cuts the AC curve at
a. The maximum point b. The initial Point
c. The minimum Point d. any point
68. The minimum point of ATC is at ……………. position of the minimum point of AVC
a. right b. left
c. same d. all of above can be
69. If the long run cost curve shifts down wards it is an indication of
a. technological progress b. lower factor prices
c. both of these d. reserve capacity
70. The U shape of the LAC reflects
a. Law of Variable proportions b. Laws of returns to scale
c. reserve capacity d. none of these
71. Price discrimination is an essential feature of
a. Perfect competition b. Oligopoly
c. Duopoly d. monopoly
72. Under monopoly the slope of AR curve is:
a. Upward sloping b. downward sloping
c. horizontal d. None of these
73. Monopoly market is :
a. Single seller market b. single buyer market
c. single buyer and seller d. None
74. In a monopsony market there is:
a. Single seller b. single buyer
c. Two sellers d. two buyers
75. Third degree price discrimination occurs when the monopolist charges different prices for
the same commodity in different
a. Markets b. places
c. continents d. countries
76. Price discrimination is possible:
a. Under any market form b. only under monopoly
c. Only under monopolistic completion d. Only in perfect competition
77. Monopolist maximizes profit at the point where
a. MC = AC b. MC = MR
c. AC = AR d. MR = A
78. At the point of equilibrium of a monopolist MC cuts MR curve
a. From below b. from above
c. at point of equality of AC and AR d. None
79. A multiplant monopolist maximizes his profit at the point where:
a. MR = MC1 b. MR = MC2
c. MR1 = MR2 d. MR = MC1 = MC2
80. For a firm with monopoly power
a. Price equals MC b. Price is less than MC
c. Price exceeds MC d. None
81. Railways is an example of :
a. Simple monopoly b. differentiated monopoly
c. Natural monopoly d. Monopsony
82. The dual pricing system of charging high price during peak time and low price during of
peak time is called
a. Double pricing b. Dual pricing
c. kinked pricing d. peak load pricing
83. Selling more than one product at a single price
a. Dumping b. Bundling
c. Discounting d. Off loading
84. The market structure which have very large number of sellers selling Identical products is
called
a. Perfect competition b. Monopoly
c. Monopolistic competition d. Oligopoly
85. The marker structure with Perfect mobility of factors and products is called
a. Perfect competition b. Monopoly
c. Monopolistic competition d. Oligopoly
86. The marker structure with Perfect knowledge is called
a. Perfect competition b. Monopoly
c. Monopolistic competition d. Oligopoly
87. The condition of perfect competition is fulfilled when
a. Sellers are large in number b. Buyers are large in number
c. Commodity produced is homogenous d. All the above
88. The following are conditions of perfect competition except
a. Sellers are large in number b. Buyers are large in number
c. Commodity produced is homogenous d. Commodity produced is differentiated
89. The following are conditions of perfect competition except
a. Strong barriers to entry b. Sellers are large in number
c. Commodity produced is homogenous d. Buyers are large in number
90. The following are conditions of perfect competition except
a. Sellers are large in number b. Single buyer
c. Commodity produced is homogenous d. Freedom to Entry and exit
91. The condition of short run equilibrium under perfect competition is
a. MC=MR b. AC=MR
c. AC=AR d. AR=Selling cost
92. The large number of firms producing the same commodity ensure that the individual firm
has no control over
a. Price of the commodity b. The quantity of the commodity
c. Both of the above d. None of the above
93. Individual firm has no control on the price of the commodity in the market is a condition
of
a. Perfect competition b. Monopoly
c. Monopolistic competition d. Bilateral monopoly
94. In a Perfect competitive market
a. Firm is the price giver and the industry is a price taker
b. Firm is the price taker and the industry is a price giver
c. Both are price makers
d. Both are price takers
95. One of the essential conditions of perfect competition is
a. Product Differentiation
b. Multiplicity of prices for identical product at any one time
c. Many sellers and few buyers
d. Only one price for identical goods at any one time
96. Under perfect market conditions the individual firm in the industry has -------------------
control over the price of the product.
a. Some b. Full
c. No d. None of the above
97. The condition of short run equilibrium under perfect competition is
a. MC=MR b. MC cuts MR from below
c. MC is rising when it cuts AR d. All the above
98. The marker structure which have large number of sellers selling differentiated product is
called
a. Perfect competition b. Monopoly
c. Monopolistic competition d. Oligopoly
99. The marker structure in which number of sellers is small with interdependence is called
a. Perfect competition b. Monopoly
c. Monopolistic competition d. Oligopoly
100. The cost incurred to alter the position or slope of demand curve is known as
a. Marginal cost b. Selling cost
c. Alternate cost d. Additional cost
101. The condition of short run equilibrium under monopolistic competition is
a. MC=MR b. AC=MR
c. AC=AR d. AR=MR
102. Kinked demand curve explain which of the following features of oligopoly
a. Selling cost b. Price rigidity
c. Non price competition d. Product differentiation
103. Demand curve of a firm under monopolistic competition is
a. Parallel to X axis b. Parallel to Y axis
c. Downward slopping d. Upward slopping
104. Which of the following is not a feature of monopolistic competition?
a. Homogenous product b. Large number of firms
c. Freedom to entry and exit d. Differentiated product
105. In the short run, a monopolistically competitive firm can have
a. Abnormal profit b. loss
c. Normal profit d. Any of the above are possible
106. Selling cost is a feature of
a. Monopolistic competition b. Perfect competition
c. Monopoly d. Bilateral monopoly
107. Refrigerator company is an example of
a. Oligopoly b. Perfect competition
c. Monopoly d. Bilateral monopoly
108. Which of the following is a form collusive oligopoly
a. Bilateral monopoly b. Monopoly
c. cartel d. Kinked Oligopoly
109. OPEC is an example of
a. Bilateral monopoly b. Monopoly
c. cartel d. Kinked Oligopoly
110. In the long run, which of the following is applicable to a firm under monopolistic
competition
a. AR = AC b. AR > AC
c. AR < AC d. AR = MC
111. A discriminating monopolist will charge a higher price from which group of customers?
a. Group with more elastic b. Group with less elastic
c. Group with Unitary Elastic d. Group with Infinitely Elastic
112. Perfect price discrimination means that every customer ____________
a. buys the same amount b. pays the same price
c. contributes the same revenue d. pays what she thinks the product is worth
113. Supernormal profit refers to
a. High proportion of net profit
b. Minimum necessary profit to induce an entrepreneur to remain in business
c. Unexpectedly high Profit
d. Residual surplus

Answers

1 A 16 C 31 A 46 C 61 A 76 B 91 A 106 A
2 C 17 C 32 B 47 D 62 B 77 B 92 C 107 A
3 A 18 D 33 C 48 C 63 A 78 A 93 A 108 C
4 B 19 D 34 B 49 B 64 C 79 D 94 D 109 C
5 B 20 A 35 C 50 A 65 B 80 C 95 D 110 A
6 D 21 A 36 D 51 B 66 B 81 C 96 C 111 B
7 D 22 C 37 A 52 C 67 C 82 D 97 D 112 D
8 C 23 D 38 A 53 D 68 A 83 B 98 C 113 A
9 A 24 A 39 D 54 B 69 C 84 A 99 D
10 C 25 A 40 A 55 D 70 B 85 A 100 B
11 D 26 B 41 D 56 A 71 D 86 A 101 A
12 B 27 A 42 D 57 C 72 B 87 D 102 B
13 B 28 B 43 B 58 B 73 A 88 D 103 C
14 D 29 D 44 B 59 C 74 B 89 A 104 A
15 A 30 B 45 D 60 A 75 A 90 B 105 D

Prepared by:
Smt. Fasnabanu M.K
Assistant Professor on Contract,
School of Distance Education, University of Calicut
Answers to Multiple Choice Questions For Monopolistic Competition

Click on the correct answer for detailed help.

1. Suppose local taverns (bars) and the legal profession are both characterized as monopolistic
competition with free entry, so in long run equilibrium economic profits will be the same in the two
industries. Suppose a study shows that, even in this setting, on average, lawyers have much higher
incomes than tavern owners. How can we explain this seeming inconsistency?
a. lawyers define profit differently than tavern owners do.
b. lawyers are better at investing money than tavern owners so the forgone interest on their
investment is higher than it is for tavern owners.
c. Attorneys don't have to pay for liquor licenses and deal with health inspectors and so their costs
of doing business will always be lower.
d. Being a tavern owner is more fun than being a lawyer,and tavern owners don't need to pay
themselves as large a salary since their opportunity costs are zero.
e. Due to the higher level of education of the typical attorney relative to the typical bar owner, their
forgone salary is higher. Thus, to make zero profit a lawyer must have a higher income than a
tavern owner.
Next Question

2. Consider a monopolistically competitive firm. From the point of view of remaining firms, as firms leave
the industry we can think of this as a:
a. shift out in each individual firm's demand curve.
b. shift back in each individual firm's MC curve.
c. shift up in each individual firm's AC curve.
d. shift back in each individual firm's supply curve.
e. shift back in each individual firm's MR curve.
Next Question

3. Which of the following describes long run equilibrium for a firm in monopolistic competition with free
entry?
a. Marginal Revenue = Average Total Cost, Price > Marginal Cost.
b. Price > Average Total Cost, Price = Marginal Cost.
c. Price > Average Total Cost, Marginal Revenue = Marginal Cost.
d. Price = Average Total Cost, Marginal Revenue > Marginal Cost.
e. Price = Average Total Cost, Price > Marginal Cost.
Next Question

Figure 1
4. For the monopolistically competitive firm depicted in Figure 1 the profit maximizing quantity and price
are:
a. Q = 19, P = 30
b. Q = 15, P = 27
c. Q = 15, P = 20
d. Q = 25, P = 15
e. Q = 25, P = 18
Next Question

5. As depicted, this firm is earning an economic profit of:


a. 105
b. 75
c. zero
d. -105
e. -75
Next Question

6. Compared with the results in Figure 1, in the long run, in a market with free entry, the firm will:
a. Charge a higher price and produce a larger quantity
b. Charge a higher price and produce a smaller quantity
c. Charge a lower price and produce a smaller quantity
d. Charge a lower price and produce a larger quantity
e. The long run result is the same as original result
Next Question

7. Suppose bars in the town of Chapel Arbor can be thought of as a monopolistically competitive industry.
Suppose that a group of well meaning citizens is campaigning to have no more new liquor licenses
issued in Chapel Arbor, meaning that no new bars can be opened unless a license is purchased from an
existing bar. Suppose also that demand has increased such that, if there are no new licenses, the
typical bar owner can expect to earn $5,000 profit per year. If the interest rate is 5% what is the most
money the typical bar owner would be willing to contribute to make certain that the group is successful
in making sure no new licenses are issued. (Hint: they would be willing to contribute an amount almost
as large as the value of a license).
a. just under $50,000.
b. just under $25,000.
c. just under $500,000.
d. just under $20,000.
e. just under $100,000.
Next Question

8. Suppose we imagine that bars in Madistin can be characterized as a monopolistically competitive


industry with free entry. Suppose that the legal drinking age is lowered to 19. Our best short run
prediction is that: (Unfortunately there seems to be evidence that such changes lead to increased
numbers of alcohol related highway deaths)
a. bars will increase prices to keep out the new drinkers.
b. bars will experience a reduction in profit due to the increased costs of serving more patrons.
c. bars will become more profitable due to the increase in demand.
d. more bars will be opened.
e. many bars will close.
Next Question

9. Again, consider the lowering of the drinking age as stated in the prior problem. Our best long run
prediction for such an industry is that:
a. due to the reduction in profitability, bars will start to close reducing demand for the remaining
bars.
b. due to the increase in profitability more bars will be opened reducing demand for remaining bars.
c. due to the reduction in profitability, bars will start to close increasing demand for the remaining
bars.
d. due to the increase in profitability more bars will be opened increasing demand for remaining
bars.
e. there will be no change in the number of bars in the long run.
Next Question

10. Once more considering the effect of a reduction in the drinking age on the bar business. Comparing a
new long run equilibrium with the old we expect:
a. More bars, but profits are back to zero.
b. Fewer bars but profits are back to zero.
c. The same number of bars but profits are higher.
d. Fewer bars but profits are higher.
e. More bars, but with positive profits.
Next Question

11. Suppose that lowering the drinking age is accompanied with a strict limit on liquor licenses in Madistin
so that no more liquor licenses can be had in the Madistin area. However, if a bar owners choose to go
out of business she may sell her license. Suppose the typical bar owner could earn $35,000 annually in
her next best job but earns $55,000 as a bar owner after the drinking age change. If typical
investments pays 10% roughly what would a liquor license sell for?
a. 550,000.
b. $5.5 million.
c. $200,000.
d. $2 million.
e. $20,000.
Start Over
© 1995-2004 OnLineTexts.com - All Rights Reserved
Economics 103 Spring 2012: Multiple choice review questions for final exam. Exam will cover chapters on perfect
competition, monopoly, monopolistic competition and oligopoly up to the Nash equilibrium

MULTIPLE CHOICE. Choose the one alternative that best completes the statement or answers the question.

1) What is the difference between perfect competition and monopolistic competition? 1)


A) Perfect competition has a large number of small firms while monopolistic competition
does not.
B) In monopolistic competition, firms produce identical goods, while in perfect competition,
firms produce slightly different goods.
C) Perfect competition has no barriers to entry, while monopolistic competition does.
D) In perfect competition, firms produce identical goods, while in monopolistic competition,
firms produce slightly different goods.
E) Perfect competition has barriers to entry while monopolistic competition does not.

2) In a perfectly competitive market, the type of decision a firm has to make is different in the short 2)
run than in the long run. Which of the following is an example of a perfectly competitive firm's
short-run decision?
A) the profit-maximizing level of output
B) whether or not to change its plant size
C) how much to spend on advertising and sales promotion
D) what price to charge buyers for the product
E) whether or not to enter or exit an industry

3) The firm's over-riding objective is to 3)


A) maximize economic profit.
B) avoid an economic loss.
C) maximize total revenue.
D) maximize normal profit.
E) earn a normal profit.

4) The price charged by a perfectly competitive firm is 4)


A) higher the more the firm produces.
B) different than the price charged by competing firms.
C) the same as the market price.
D) indeterminate.
E) lower the more the firm produces.

5) A profit-maximizing output for a single-price monopoly is determined by the intersection of the 5)


________ curves and the profit-maximizing price is found on the ________ curve.
A) total revenue and total cost, total revenue
B) marginal cost and marginal revenue; marginal revenue
C) demand and supply; supply
D) marginal cost and marginal revenue; demand
E) marginal cost and average total cost; demand

6) A single-price monopoly has marginal revenue and marginal cost equal to $19 at 15 units of 6)
output where the price on the demand curve is $38. At this output, average total cost is $15.
What is the total profit earned?
A) $225 B) $570 C) $19 D) $285 E) $345

1
7) Rate of return regulation is designed to allow a natural monopoly to 7)
A) underestimate its average cost.
B) earn zero normal profit.
C) earn an economic profit.
D) earn a normal profit.
E) compete with any firm entering the market.

8) Which of the following is true about monopolistic competition but false about perfect 8)
competition?
A) Firms can earn an economic profit in the short run.
B) There are a large number of independently acting sellers.
C) There are no barriers to entry.
D) Firms compete on their product's price as well as its quality and marketing.
E) Firms cannot earn an economic profit in the long run.

9) What does monopolistic competition have in common with monopoly? 9)


A) mutual interdependence
B) the ability to collude with respect to price
C) a large number of firms
D) a downward-sloping demand curve
E) barriers to entry

10) Firms in monopolistic competition have demand curves that are 10)
A) U-shaped.
B) horizontal.
C) downward sloping.
D) vertical.
E) upward sloping.

2
11) Kevin owns a personal training gymnasium in Orlando. The above figure shows the demand 11)
and cost curves for his firm, which competes in a monopolistically competitive market. Kevin
will train how many clients per day?
A) between 2 and 4
B) 6
C) 10
D) 4
E) None of the above answers is correct.

12) Kevin owns a personal training gymnasium in Orlando. The above figure shows the demand 12)
and cost curves for his firm, which competes in a monopolistically competitive market. What
price will Kevin charge per session?
A) $20 B) $80 C) $60 D) $100 E) $40

13) In monopolistic competition, the products of different sellers are 13)


A) similar but slightly different.
B) unique without any close or perfect substitutes.
C) perfect substitutes.
D) identical.
E) either identical or differentiated.

14) When a monopolistically competitive firm's demand curve shifts leftward, what happens to its 14)
marginal revenue curve?
A) It disappears.
B) Nothing, the marginal revenue curve is unchanged.
C) It shifts leftward.
D) It shifts rightward.
E) None of the above is correct because the effect on the marginal revenue curve depends on
whether the demand was initially elastic or inelastic.

3
15) Firms in an oligopoly 15)
i. are independent of each others' actions.
ii. can each influence the market price.
iii. charge a price equal to marginal revenue.
A) i only
B) i and iii
C) ii only
D) iii only
E) i, ii, and iii

16) When oligopolies seek to operate as a single-price monopoly, the firms produce at the point 16)
where:
A) MR = MC.
B) P = MR.
C) P < ATC.
D) P = MC.
E) MC = ATC.

17) A cartel is a collusive agreement among a number of firms that is designed to 17)
A) expand output and lower prices but not to a predatory level.
B) expand output and lower prices to a predatory level.
C) restrict output and raise prices.
D) expand output and raise prices.
E) restrict output and lower prices to a predatory level.

18) When oligopolies operate like firms in perfect competition, the firms produce at the point where 18)
the
A) price exceeds the average total cost by the greatest amount.
B) price exceeds the marginal cost by the greatest amount.
C) marginal cost equals the average total cost.
D) price is less than the marginal cost.
E) marginal cost equals the price.

19) If one firm in a duopoly increases its production by one unit beyond the monopoly output, that 19)
firm's profit ________, the other firm's profit ________, and the total profit of the duopoly
________.
A) increases; increases; increases
B) increases; decreases; decreases
C) does not change; does not change; does not change
D) increases; does not change; increases
E) increases; decreases; does not change

20) A Nash equilibrium is defined as 20)


A) each player taking the action that is best for all the players.
B) forming a cartel with strong penalties for cheaters.
C) earning zero economic profit in the long run.
D) relying on other game players to realize the benefit of cooperation.
E) each player taking the best possible action given the action of the other player.

4
21) In a prisoners' dilemma game, in the Nash equilibrium 21)
A) neither player gets his or her best outcome.
B) both players get their best outcome.
C) one player gets his or her best outcome and the other player does not.
D) collusion would not alter the outcome.
E) Either answer A or C might be correct depending on whether the players communicate
with each other or do not communicate with each other.

22) In the prisoners' dilemma, each player is ________ regardless of the other player's actions. 22)
A) better off confessing
B) forced to confess
C) better off denying
D) forced to deny
E) going to go free

23) In an oligopoly, output is 23)


A) greater than the output in perfect competition.
B) somewhere between the output in monopoly and that in perfect competition outcomes.
C) in all circumstances the same as the output in perfect competition.
D) less than the output in monopoly.
E) in all circumstances the same as the output in monopoly.

24) Which of the following is true? In the above figure, if the market is 24)
A) a monopoly, output will be Q3 and price will be P3.
B) perfect competition, output will be Q3 and price will be P3.
C) perfect competition, output will be Q1 and price will be P1.
D) a monopoly, output will be Q1 and price will be P3.
E) perfect competition, output will be Q2 and price will be P2.

5
25) In the above figure, the output of an oligopoly will range between 25)
A) Q2 and Q3.
B) 0 and Q1.
C) Q1 and Q2.
D) Q1 and Q3.
E) 0 and Q2.

26) A Nash equilibrium in the duopoly game 26)


A) means that one player has greater market power.
B) will always lead to equilibrium in which the firms' total profit is the largest.
C) can occur only if firms cooperate with each other.
D) means that a firm must be able to determine its actions and the actions of its competitor.
E) occurs when each player takes the best possible action regardless of the strategy chosen by
other firms.

27) The major dilemma facing Boeing and Airbus is the 27)
A) fact that neither will respond to the behavior of the other.
B) fact that if each firm separately tries to maximize its profit, it might wind up with less
profit that otherwise.
C) fact that when they collude to maximize their profit, the other firm's profit might be larger
than its profit.
D) certainty surrounding the reaction of each firm to the behavior of the other firm.
E) competition from other firms that drives their economic profit to zero.

6
28) The above figure shows the market demand curve for long-distance telephone calls. Suppose 28)
the marginal cost of a long-distance telephone call is 2¢ a minute for a call no matter how many
minutes of calls are made and there are 3 firms in the industry. If the firms in the industry
operate as perfect competitors, there are ________ minutes of calls made per hour.
A) between 0 and 3 million
B) more than 3 million and less than or equal to 5 million
C) more than 5 million and less than or equal to 7 million
D) more than 9 million
E) more than 7 million and less than or equal to 9 million

7
29) The figure above shows the market demand curve and the ATC curve for a firm. If all firms in 29)
the market have the same ATC curve, the lowest price at which a firm could stay in business in
the long run is ________ per unit and the quantity demanded in the market at that price is
________ units per hour.
A) $20; 2,000
B) $10; 4,000
C) $20; 8,000
D) $10; 8,000
E) $20; 4,000

30) The figure above shows the market demand curve and the ATC curve for a firm. If all firms in 30)
the market have the same ATC curve, the efficient scale for one firm is ________ units per hour.
A) 2,000
B) 4,000
C) 8,000
D) 10,000
E) more than 10,000

31) Which of the following is correct? 31)


A) In the long run, a firm in monopolistic competition earns zero economic profit and its
price is equal to the minimum average total cost.
B) In the long run, a firm in monopolistic competition can earn an economic profit because of
product differentiation.
C) A firm in perfect competition operates at maximum average total cost in the long run.
D) In the long run, a firm in monopolistic competition maximizes its profit at a point where
price is equal to average total cost but the average total cost is not minimized.
E) A firm in monopolistic competition does not have excess capacity in the long run.

8
32) A cartel is most likely to occur in 32)
A) perfect competition as firms compete by reducing cost.
B) oligopoly as firms compete to lower price and increase their own profits.
C) monopolistic competition where firms collude to increase profits.
D) monopoly because it faces no competition.
E) oligopoly as firms act together to raise prices and increase profits.

33) When firms in monopolistic competition are making an economic profit, firms will 33)
A) enter the industry, and demand will decrease for the original firms.
B) exit the industry, and demand will increase for the firms that remain.
C) enter the industry and then will exit the industry.
D) enter the industry, and demand will increase for the original firms.
E) exit the industry, and demand will decrease for the firms that remain.

34) Herb's Inc. has a large share of its market and is tempted to collude with the few firms that are in 34)
its market. Herb's operates in
A) a perfectly competitive market.
B) collusively protected market.
C) a monopoly market.
D) a monopolistically competitive market.
E) an oligopoly.

35) A cartel is 35)


A) a market structure with a large number of small firms.
B) a market with only two firms.
C) a group of firms acting together to raise price, decrease output, and increase economic
profit.
D) a market structure with a small number of large firms.
E) another name for an oligopoly.

36) For a firm in monopolistic competition, the efficient scale is the amount of output at which 36)
________ is a minimum.
A) marginal cost
B) fixed cost
C) average total cost
D) average variable cost
E) average fixed cost

37) A firm in monopolistic competition ________ influence its price and ________ influence the 37)
market average price.
A) cannot; cannot
B) can; only in the short run can
C) can; cannot
D) cannot; can
E) can; can

9
38) At a long-run equilibrium in monopolistic competition, price equals 38)
A) marginal cost but not marginal revenue.
B) average total cost.
C) marginal revenue and marginal cost.
D) marginal revenue but not marginal cost.
E) zero.

39) In monopolistic competition there are ________ barriers to entry, so therefore in the long run, 39)
economic profit ________.
A) no; is substantial
B) many; equals zero
C) no; equals zero
D) many; is substantial
E) many; might be earned depending on the degree of product differentiation

40) The major difference between monopolistic competition and monopoly is 40)
A) monopoly is a price setter and a firm in monopolistic competition is a price taker.
B) how the quantity of output is determined.
C) only a monopoly can earn an economic profit in the long run.
D) only a firm in monopolistic competition can earn an economic profit in the short run.
E) only firms in monopolistic competition are protected by barriers to entry.

41) If a monopolistically competitive seller's marginal cost is $3.56, the firm will decrease its output 41)
if
A) its marginal revenue is less than $3.56.
B) its marginal revenue is equal to $3.56.
C) its marginal revenue is more than $3.56.
D) its average total cost is equal to $4.00.
E) Both answers B and D are correct.

42) In monopolistic competition, profit is maximized by producing so that marginal revenue 42)
A) equals price.
B) equals average total cost but not marginal cost.
C) equals marginal cost and which are less than price.
D) equals marginal cost and equals price.
E) is negative.

43) The absence of barriers to entry in monopolistic competition means that in the long run firms 43)
A) earn either an economic profit or zero economic profit.
B) earn zero economic profit.
C) incur an economic loss.
D) earn an economic profit.
E) earn either zero economic profit or suffer an economic loss.

10
44) With an average cost pricing rule, the quantity produced by the natural monopoly is ________ 44)
the quantity produced with a marginal cost pricing rule.
A) less than
B) greater than
C) not comparable to
D) equal to
E) greater than in the long run and less than in the short run than

45) Because of the number of firms in monopolistic competition 45)


A) each firm has a large market share.
B) it is possible for the firms to collude.
C) one firm has the ability to dictate market conditions.
D) each firm must carefully monitor what its competitors do.
E) no one firm can dominate the market.

46) If a large number of firms are competing, the market could be 46)
A) monopolistic competition or monopoly.
B) perfect competition or monopoly.
C) oligopoly or monopoly.
D) perfect competition or monopolistic competition.
E) monopolistic competition or oligopoly.

47) With a natural monopoly 47)


A) no regulation is necessary because it is a natural monopoly.
B) regulation takes the form of breaking the company into several competing firms.
C) regulation takes the form of forcing the company out of business.
D) regulation can take the form of average cost pricing to allow coverage of costs.
E) regulation takes the form of forcing competition from new firms.

48) If a natural monopoly is regulated using 48)


A) a total cost pricing rule, the firm will exit the industry.
B) a marginal cost pricing rule, the firm maximizes its profit.
C) an average cost pricing rule, the firm maximizes its profit.
D) a marginal cost pricing rule, the firm incurs an economic loss.
E) an average cost pricing rule, the firm incurs an economic loss.

49) Price cap regulation is defined as regulation that 49)


A) imposes a price ceiling on the regulated firm.
B) is essentially the same as rate of return regulation.
C) uses average cost pricing to ensure costs are covered.
D) uses marginal cost pricing to ensure efficient output.
E) encourages firms to exaggerate costs to increase profits.

50) The process of price cap regulation includes which of the following? 50)
i. a price ceiling.
ii. marginal cost pricing.
iii. average cost pricing
A) i and ii B) i and iii C) ii only D) ii and iii E) i only

11
51) The above figure represents the market for cable television in Oakland, Florida. Time Warner 51)
Communications (TWC) is the sole provider of cable television to the residents of this Central
Florida community. If TWC is left unregulated, what is the price of cable television in Oakland?
A) $10 B) $20 C) $50 D) $30 E) $40

52) The above figure represents the market for cable television in Oakland, Florida. Time Warner 52)
Communications (TWC) is the sole provider of cable television to the residents of this Central
Florida community. If TWC is left unregulated, how many households in Oakland are served?
A) 40,000 B) 50,000 C) 10,000 D) 30,000 E) 20,000

53) If a regulatory agency sets the price equal to marginal cost for a natural monopoly, the 53)
A) price is the same as the unregulated monopoly price.
B) firm earns an economic profit, though not the maximum economic profit.
C) firm earns a normal profit.
D) firm earns the maximum economic profit.
E) government might have to provide a subsidy to the firm to keep it in business.

54) Capture theory is 54)


A) a model about perfect competition.
B) a theory that explains behavior of competitive firms.
C) an economic theory of regulation.
D) the theory that regulators capture firms' attention by dictating a very low price.
E) the same as the public interest theory.

55) A monopoly creates a deadweight loss because the monopoly 55)


A) sets a price that is too low.
B) produces less than the efficient quantity.
C) produces more than the efficient quantity.
D) does not maximize profit.
E) earns a normal profit.

12
56) Which of the following explains why the marginal cost pricing rule results in an economic loss 56)
for a natural monopoly?
A) The ATC curve is downward sloping throughout the relevant range, therefore the MC is
lower than the ATC.
B) The MC is constant and equal to price.
C) Because output is determined by setting MC equal to the price, consumer surplus is
maximized.
D) The demand curve is downward sloping, therefore price falls as quantity increases.
E) The firm's MR is always less than its price.

57) ________ natural monopolies is a commonly used, potential solution to the problems presented 57)
by natural monopolies.
A) Giving incentives to firms to become
B) Breaking up firms that are
C) Regulating
D) Refusing to grant patents to
E) Outlawing price discrimination by

58) With perfect price discrimination, the level of output 58)


A) is the same as the amount produced by any monopoly that price discriminates.
B) equals the amount produced by a single-price monopoly.
C) is the same as the amount produced in a perfectly competitive market.
D) exceeds the efficient quantity.
E) is unknown.

59) Comparing a perfectly competitive market to a single-price monopoly with the same costs, we 59)
see that
A) the monopoly market always is more efficient in the use of resources.
B) the monopoly market achieves efficiency in resource use while perfectly competitive
market does not.
C) both markets are equally efficient in their use of resources.
D) the perfectly competitive market achieves efficiency in resource use while the monopoly
market does not.
E) None of the above answers is correct because comparing a perfectly competitive market to
a monopoly is impossible.

60) When a perfectly competitive industry is taken over by a monopoly, some consumer surplus is 60)
transferred to the monopolist in the form of
A) taxes.
B) marginal cost.
C) deadweight loss.
D) economic profit.
E) average variable cost.

13
61) With price discrimination, a monopoly 61)
A) produces less output than if it does not price discriminate.
B) converts consumer surplus into deadweight loss.
C) converts producer surplus into economic profit.
D) can charge a single price to all customers.
E) converts consumer surplus into economic profit.

62) A price-discriminating monopoly 62)


A) cannot offer discounts.
B) cannot control the price of its product.
C) sells a larger quantity than it would if it were a single-price monopoly.
D) is illegal.
E) makes a smaller economic profit than it would if it were a single-price monopoly.

63) In the above figure, a perfectly competitive market will have a price of ________ and a 63)
single-price monopoly will have a price of ________.
A) P1 and quantity of Q1; P2 and quantity of Q2
B) P2 and quantity of Q2; P3 and quantity of Q1
C) P2 and quantity of Q1; P1 and quantity of Q1
D) P2 and quantity of Q2; P1 and quantity of Q1
E) P3 and quantity of Q3; P1 and quantity of Q1

64) A single-price monopoly transfers 64)


A) economic profit to the government.
B) consumer surplus to producers.
C) producer surplus to consumers.
D) economic profit to deadweight loss.
E) economic profit to consumers.

14
65) The figure above shows the demand curve, marginal revenue curve, and marginal cost curve. 65)
The amount of consumer surplus when the market has a monopoly producer is
A) ace. B) bcef. C) bcd. D) abf. E) acd.

66) The figure above shows the demand curve, marginal revenue curve, and marginal cost curve. 66)
The amount of consumer surplus when the market has a monopoly producer is ________ and
the amount of consumer surplus when the market is perfectly competitive is ________.
A) abf; ace B) ace; bcd C) ace; abf D) abf; bcd E) bcd; ace

67) Compared to a perfectly competitive market, a single-price monopoly sets 67)


A) a higher price.
B) a lower price.
C) the same price.
D) a price that might be higher, lower, or the same depending on whether the monopoly's
marginal revenue curve lies above, below, or on its demand curve.
E) a price that might be higher, lower, or the same depending on whether the monopoly's
marginal cost curve lies above, below, or on its marginal revenue curve.

68) Compared to a perfectly competitive industry, a single-price monopoly produces 68)


A) the same output.
B) more output.
C) less output.
D) some amount that might be more, less, or the same depending on whether the monopoly's
marginal revenue curve lies above, below, or on its demand curve.
E) some amount that might be more, less, or the same depending on whether the monopoly's
marginal cost curve lies above, below, or on its marginal revenue curve.

15
69) Mark owns a cattle ranch near Hugo, Oklahoma. Mark is currently producing beef at an output 69)
level where marginal revenue exceeds marginal cost. In order to maximize his profit, Mark
should
A) decrease his output.
B) shut down his ranch.
C) increase his output.
D) not change his output.
E) probably change his output, but more information is needed to determine if he should
increase, decrease, or not change it.

70) When compared to a perfectly competitive market, a single-price monopoly with the same costs 70)
produces ________ output and charges ________ price.
A) a smaller; a lower
B) a larger; a lower
C) a smaller; a higher
D) a smaller; the same
E) the same; a higher

71) Suppose the Busy Bee Café is the monopoly producer of hamburgers in Hugo, Oklahoma. The 71)
above figure represents the demand, marginal revenue, and marginal cost curves for this
establishment. What quantity will the Busy Bee produce to maximize its profit?
A) 20 hamburgers per hour
B) 50 hamburgers per hour
C) 10 hamburgers per hour
D) 0 hamburgers per hour.
E) 30 hamburgers per hour

16
Price(dollars) Quantity(units)
6 1
5 2
4 3
3 4
2 5
1 6

72) The above table gives the demand schedule for a monopoly. The demand is elastic at all prices 72)
between
A) $3 and $1.
B) $5 and $1.
C) $4 and $3.
D) $6 and $1.
E) $6 and $4.

73) The above table gives the demand schedule for a monopoly. The demand is inelastic over the 73)
entire price range between
A) $6 and $4.
B) $6 and $1.
C) $3 and $1.
D) $4 and $3.
E) $5 and $1.

74) If the Boston Red Sox baseball team is currently charging a ticket price where its demand is 74)
inelastic, then the Red Sox's marginal revenue is
A) positive.
B) zero.
C) undefined.
D) maximized.
E) negative.

Quantity Price
(units) (dollars per unit)
1 8
2 7
3 6
4 5
5 4
6 3

75) The table above gives the demand for a monopolist's output. What is the total revenue in when 3 75)
units of output are produced?
A) $18 B) $20 C) $21 D) $6

76) The table above gives the demand for a monopolist's output. What is the marginal revenue 76)
when output is increased from 5 to 6 units?
A) $18 B) -$2 C) $4 D) $3

17
77) The demand curve facing a single-price monopoly is 77)
A) the same as only the marginal revenue curve.
B) the same as both the marginal revenue curve and the marginal cost curve.
C) below the marginal revenue curve.
D) above the marginal revenue curve.
E) the same as only the marginal cost curve.

78) A single-price monopoly can sell 10 units of its product at a price of $45 each but to sell 11 units, 78)
the monopoly must cut the price to $44. What is the marginal revenue of the extra unit sold?
A) $484 B) $450 C) $34 D) -$1 E) $44

79) A single-price monopoly faces a linear demand curve. If the marginal revenue for the second 79)
unit is $20, then the marginal revenue for the
A) third unit is also $20.
B) third unit is less than $20.
C) first unit is less than $20.
D) third unit is more than $20.
E) more information is needed to determine if the marginal revenue for the third unit is more
than, less than, or equal to $20.

80) For a single-price monopoly, price is 80)


A) greater than marginal revenue.
B) equal to marginal revenue.
C) less than marginal revenue because the firm must lower its price in order to sell another
unit of output.
D) less than marginal revenue because the firm cannot increase its total revenue when the
demand curve is downward sloping.
E) equal to zero because the firm is not a price taker.

81) For a monopoly, marginal revenue is equal to 81)


A) the price of the product.
B) the amount people buy between two prices.
C) the amount people buy at a given price.
D) the change in total revenue brought about by a one-unit increase in quantity sold.
E) the price multiplied by the quantity sold.

82) A single-price monopoly 82)


A) is able to raise its price as high as it wants and consumers must still buy from it because it
is a monopoly.
B) can lower its price for only a few select consumers if it wants to increase its sales.
C) must practice price discrimination.
D) must lower the price for all customers if it wants to increase its sales.
E) will set its price equal to a consumer's willingness to pay.

18
83) In order for a hotel to successfully price discriminate so that senior citizens are given a discount, 83)
the hotel must be able to
A) lower its prices to younger customers too.
B) prevent senior citizens from reselling their rooms to younger customers.
C) offset the economic loss from charging senior citizens a lower price by lowering the
marginal cost of renting rooms to senior citizens.
D) shift its demand curve rightward.
E) determine if a senior citizen can pay a higher price.

84) A price-discriminating monopoly is a monopoly that 84)


A) has a license to sell the product.
B) sells its output at a single price to all of its customers.
C) illegally charges different customers different prices for the good it produces.
D) sells different units of a good or service at different prices.
E) has control over the resources used to produce the product.

85) A single-price monopoly 85)


A) sets a single price for all consumers.
B) asks each consumer what single price they would be willing to pay.
C) sets a single, different price for each consumer.
D) sells each unit of its output for the single, highest price that the buyer of that unit is
willing to pay.
E) sets a single, different price for each of two different groups.

86) Which of the following statements is correct? 86)


A) Because a monopoly is the only firm in the market, its marginal revenue curve must be the
same as the market demand curve.
B) Monopolies are guaranteed to earn an economic profit.
C) The market demand and the firm's demand are the same for a monopoly.
D) Monopolies have perfectly inelastic demand for the product sold.
E) Because a monopoly is the only firm in the market, its supply curve is the same as the
market demand curve.

87) Patents 87)


A) remove legal barriers to entry.
B) are prohibited in the United States.
C) are a legal barrier to entry.
D) decrease the incentive to innovate.
E) create economies of scale.

19
88) A natural monopoly's average cost curve 88)
i. intersects the demand curve while the average cost curve slopes downward.
ii. reaches its minimum before it intersects the demand curve.
iii. intersects the demand curve below the intersection of the marginal cost curve and the
demand curve.
A) i, ii, and iii.
B) ii only.
C) i and iii.
D) iii only.
E) i only.

89) For a natural monopoly, economies of scale 89)


A) as well as constant returns to scale and diseconomies of scale exist along the long-run
average cost curve at least until it crosses the market demand curve..
B) are totally absent.
C) and diseconomies of scale exist along the long-run average cost curve at least until it
crosses the market demand curve.
D) lead to a legal barrier to entry.
E) exist along the long-run average cost curve at least until it crosses the market demand
curve.

90) A natural monopoly 90)


A) occurs when one firm controls a natural resource.
B) arises when one firm can meet the entire market demand at a lower average total cost than
two or more firms.
C) arises as a result of legal barriers to entry.
D) Both answers A and B are correct.
E) Both answers A and C are correct.

91) A natural barrier to entry is defined as a barrier that arises because of 91)
A) technology that allows economies of scale over the entire relevant range of output.
B) patents or licenses that exclude others from producing a good or service.
C) many firms producing the good and thereby allowing choice for all consumers.
D) one firm owning a key natural resource.
E) anticompetitive practices by a firm that keep other firms from producing.

92) A monopoly 92)


A) must determine the price it will charge.
B) cannot price discriminate because such a pricing strategy is illegal in the United States.
C) faces extensive competition from firms making close substitutes.
D) has no control over the price it must charge.
E) Both answers B and C are correct.

20
93) One of the requirements for a monopoly is that 93)
A) products are high priced.
B) there is a unique product with no close substitutes.
C) there are several close substitutes for the product.
D) there is no barrier to entry.
E) the product cannot be produced by small firms.

94) A monopoly is a market with 94)


A) no barriers to entry.
B) many substitutes.
C) one supplier.
D) many suppliers each producing an identical product.
E) many suppliers each producing a slightly different product.

95) Technology reduces the average cost of production, so in the long run 95)
i. perfectly competitive firms produce at a lower average cost.
ii. the market price of the good falls.
iii. firms with older plants either exit the market or adopt the new technology.
A) i and ii.
B) i only.
C) iii only.
D) i and iii.
E) i, ii, and iii.

96) When a firm adopts new technology, generally its 96)


A) cost curves are unaffected.
B) cost curves shift downward.
C) production permanently decreases.
D) supply curve shifts leftward.
E) cost curves shift upward.

97) Suppose a perfectly competitive market is in long-run equilibrium with a price of $12. Then 97)
there is a permanent increase in demand. As a result, in the short run the market price ________
and in the long run the number of firms ________ and the price is ________ the price was in the
short run.
A) falls; decreases; is equal to
B) rises; does not change; lower than
C) rises; increases; higher than
D) rises; increases; lower than
E) rises; does not change; is equal to

98) Keith is a perfectly competitive carnation grower. The market price is $2 per dozen carnations. 98)
Keith's average total cost to grow carnations is $2.50 per dozen. In the long run, Keith will
A) continue to earn an economic profit.
B) raise his price to more than $2.50 per dozen carnations.
C) raise his price to $2.50 per dozen carnations.
D) exit the industry if the price and his costs do not change.
E) incur an economic loss.

21
99) In the long run, existing firms exit a perfectly competitive market 99)
A) only if economic profits are zero.
B) only if they incur an economic loss.
C) if they earn a positive economic profit.
D) if they either earn only a normal profit or if they incur an economic loss.
E) if normal profits are greater than zero.

100) Suppose a perfectly competitive market is in short-run equilibrium. Firms that are incurring a 100)
________ economic loss ________.
A) persistent; exit the industry and shift the market supply curve rightward
B) temporary; decrease their production but definitely stay open
C) persistent; exit the industry and shift the market supply curve leftward
D) temporary; exit the industry
E) persistent; increase their output to increase their profit

101) Suppose a perfectly competitive market is in long-run equilibrium and then there is a 101)
permanent increase in the demand for that product. The new long-run equilibrium will have
A) a permanent decrease in supply.
B) fewer firms in the market.
C) the same number of firms in the market.
D) probably a different number of firms, but it is not possible to determine if there will be
more or fewer firms.
E) more firms in the market.

102) The cranberry market is perfectly competitive. Reports that consuming cranberries can lead to 102)
improved health result in a permanent increase in the demand for cranberries and an immediate
upward jump in the price of cranberries. As time passes, the price of cranberries ________ and
the initial firms' economic ________.
A) rises still higher; loss will be eliminated
B) rises still higher; profit will not change
C) falls; profit will not change
D) falls; loss will be increased
E) falls; profit will be eliminated

103) In the long run, a perfectly competitive firm 103)


A) makes zero economic profit.
B) makes an economic profit.
C) can make an economic profit, zero economic profit, or incur an economic loss.
D) incurs an economic loss.
E) can make either an economic profit or a normal profit.

104) Juan's Software Service Company is in a perfectly competitive market. Juan has total fixed cost of 104)
$25,000, average variable cost for 1,000 service calls is $45, and marginal revenue is $75. Juan's
makes 1,000 service calls a month. What is his economic profit?
A) $25,000 B) $45,000 C) $75,000 D) $5,000 E) $50,000

22
105) A perfectly competitive firm definitely earns an economic profit in the short run if price is 105)
A) equal to average total cost.
B) greater than average total cost.
C) greater than average variable cost.
D) equal to marginal cost.
E) greater than marginal cost.

106) In the short run, a perfectly competitive firm 106)


A) must make zero economic profit.
B) must make an economic profit.
C) None of the above answers is correct.
D) must incur an economic loss.
E) might make an economic profit, an economic loss, or a normal profit.

107) The above figure shows a perfectly competitive firm. If the market price is $20 per unit, the firm 107)
A) will stay open to produce and will earn a normal profit.
B) will definitely shut down to minimize its losses.
C) will stay open to produce and will incur an economic loss.
D) might shut down but more information is needed about the fixed cost.
E) will stay open to produce and will earn an economic profit.

23
108) The above figure shows a perfectly competitive firm. If the market price is $15, the firm 108)
A) is earning an economic profit.
B) is incurring an economic loss.
C) is earning a normal profit.
D) might shut down but more information is needed about the AVC.
E) will immediately shut down.

109) A perfectly competitive firm is producing 50 units of output and selling at the market price of 109)
$23. The firm's average total cost is $20. What is the firm's total cost?
A) $20 B) $150 C) $23 D) $1,000 E) $1,150

110) Suppose that marginal revenue for a perfectly competitive firm is $20 . When the firm produces 110)
10 units, its marginal cost is $20, its average total cost is $22, and its average variable cost is $17.
Then to maximize its profit in the short run, the firm
A) must decrease its output to increase its profit.
B) should shut down.
C) must increase its output to increase its profit.
D) should not change its production because it is already maximizing its profit and is earning
a normal profit.
E) should stay open and incur an economic loss of $20.

111) Peter's Pencils is a perfectly competitive company producing pencils. Suppose Peter is 111)
producing 1,000 pencils an hour. If the total cost of 1,000 pencils is $500, the market price per
pencil is $2, and the marginal cost is $2, then Peter
A) has an economic profit because marginal revenue is equal to marginal cost at this output
level.
B) should decrease his output to increase his profit.
C) is not maximizing his profit but is earning a normal profit anyway.
D) should increase his output to increase his profit.
E) is maximizing his profit and is earning an economic profit.

24
112) In the short run, a perfectly competitive firm can experience which of the following? 112)
i. an economic profit
ii. an economic loss but it continues to stay open
iii. an economic loss equal to its total fixed cost when it shuts down
A) i and ii
B) i, ii, and iii
C) i and iii
D) only i
E) ii and iii

113) For a perfectly competitive corn grower in Nebraska, the marginal revenue curve is 113)
A) the same as its demand curve.
B) upward sloping.
C) downward sloping.
D) vertical at the profit maximizing quantity of production.
E) U-shaped.

114) Suppose a perfectly competitive firm's minimum average variable cost is $3 when it produces 114)
50. If the price is $2 and the firm's marginal cost is $2, the firm should
A) continue to produce 50.
B) continue to operate, but to determine the amount of production needs more information
than is given.
C) continue to produce, but produce less than 50.
D) shut down.
E) continue to produce, but produce more than 50.

115) A perfectly competitive firm will continue to operate in the short run when the market price is 115)
below its average total cost if the
A) marginal cost is minimized.
B) price is at least equal to the minimum average variable cost.
C) price is also less than the minimum average variable cost.
D) marginal revenue is greater than marginal cost.
E) total fixed costs are less than total revenue.

116) A perfectly competitive firm will shutdown when the price is just below the minimum point on 116)
the
A) marginal revenue curve.
B) average fixed cost curve.
C) average total cost curve.
D) average variable cost curve.
E) marginal cost curve.

25
117) The above figure illustrates a perfectly competitive firm. If the market price is $40 a unit, to 117)
maximize its profit (or minimize its loss) the firm should
A) produce more than 10 and less than 30 units.
B) produce more than 30 units and less than 40 units..
C) produce 40 units.
D) shut down.
E) produce 30 units.

118) In a perfectly competitive industry, when a firm is producing so that its total revenue equals its 118)
total cost, the firm is
A) definitely not maximizing its profit.
B) earning zero economic profit, that is, earning a normal profit.
C) incurring an economic loss.
D) earning an economic profit.
E) None of the above answers is correct because the relationship between total revenue and
total cost has nothing to do with the firm's profit or loss.

119) For a perfectly competitive firm, marginal revenue is 119)


A) equal to the change in profit from selling one more unit.
B) less than the price.
C) equal to the price.
D) undefined because the firm's demand curve is horizontal.
E) greater than the price.

26
120) If the market price of a product is $14 and all sellers are price takers, then which of the following 120)
is correct?
A) Each seller can earn more total revenue by raising the price he or she charges above $14.
B) The demand curve for each seller's product is a downward-sloping straight line.
C) Each seller's total revenue is graphed as an upside-down U-shaped curve.
D) The demand curve for each seller's product is a downward-sloping but not necessarily a
straight line.
E) Each seller's total revenue line is graphed as an upward-sloping straight line.

121) For the perfectly competitive broccoli producers in California, the market demand curve for 121)
broccoli is
A) a horizontal line.
B) nonexistent.
C) downward sloping.
D) the same as the demand curve each firm faces.
E) upward sloping.

27
Answer Key
Testname: S12_103_FINAL_MC_REVIEW

1) D
2) A
3) A
4) C
5) D
6) E
7) D
8) D
9) D
10) C
11) D
12) C
13) A
14) C
15) C
16) A
17) C
18) E
19) B
20) E
21) A
22) A
23) B
24) E
25) C
26) E
27) B
28) E
29) D
30) A
31) D
32) E
33) A
34) E
35) C
36) C
37) C
38) B
39) C
40) C
41) A
42) C
43) B
44) A
45) E
46) D
47) D
48) D
49) A
28
Answer Key
Testname: S12_103_FINAL_MC_REVIEW

50) E
51) D
52) E
53) E
54) C
55) B
56) A
57) C
58) C
59) D
60) D
61) E
62) C
63) D
64) B
65) D
66) A
67) A
68) C
69) C
70) C
71) A
72) E
73) C
74) E
75) A
76) B
77) D
78) C
79) B
80) A
81) D
82) D
83) B
84) D
85) A
86) C
87) C
88) E
89) E
90) B
91) A
92) A
93) B
94) C
95) E
96) B
97) D
98) D
29
Answer Key
Testname: S12_103_FINAL_MC_REVIEW

99) B
100) C
101) E
102) E
103) A
104) D
105) B
106) E
107) A
108) A
109) D
110) E
111) E
112) B
113) A
114) D
115) B
116) D
117) C
118) B
119) C
120) E
121) C

30
Chapter 11 Perfect Competition - Sample Questions

MULTIPLE CHOICE. Choose the one alternative that best completes the statement or answers the question.

1) Perfect competition is an industry with 1)


A) a few firms producing identical goods.
B) many firms producing goods that differ somewhat.
C) a few firms producing goods that differ somewhat in quality.
D) many firms producing identical goods.

2) In a perfectly competitive industry, there are 2)


A) many buyers and many sellers.
B) many sellers, but there might be only one or two buyers.
C) many buyers, but there might be only one or two sellers.
D) one firm that sets the price for the others to follow.

3) In perfect competition, the product of a single firm 3)


A) is sold to different customers at different prices.
B) has many perfect complements produced by other firms.
C) has many perfect substitutes produced by other firms.
D) is sold under many differing brand names.

4) In perfect competition, restrictions on entry into an industry 4)


A) do not exist. B) apply to labor but not to capital.
C) apply to both capital and labor. D) apply to capital but not to labor.

5) In perfect competition, 5)
A) there are significant restrictions on entry.
B) each firm can influence the price of the good.
C) there are few buyers.
D) all firms in the market sell their product at the same price.

6) The price elasticity of demand for any particular perfectly competitive firm's output is 6)
A) less than 1. B) equal to zero. C) infinite. D) 1.

7) The demand for wheat from farm A is perfectly elastic because wheat from farm A is a(n) 7)
A) perfect complement to wheat from farm B. B) perfect substitute for wheat from farm B.
C) normal good. D) inferior good.

8) In perfect competition, the elasticity of demand for the product of a single firm is 8)
A) 0. B) infinite.
C) 1. D) between 0 and 1.

1
9) In perfect competition, the elasticity of demand for the product of a single firm is 9)
A) infinite, because many other firms produce identical products.
B) zero, because many other firms produce identical products.
C) zero, because the firm produces a unique product.
D) infinite, because the firm produces a unique product.

10) In perfect competition, an individual firm 10)


A) has a price elasticity of supply equal to one.
B) faces unitary elasticity of demand.
C) has a price elasticity of supply equal to infinity.
D) faces infinitely elastic demand.

11) If Steve's Apple Orchard, Inc. is a perfectly competitive firm, the demand for Steve's apples has 11)
A) elasticity equal to the price of apples. B) unitary elasticity.
C) infinite elasticity. D) zero elasticity.

12) In a perfectly competitive industry, the price elasticity of demand for the market demand is 12)
________ and the price elasticity of demand for an individual firm's demand is ________.
A) infinite; less than infinite B) infinite; infinite
C) less than infinite; less than infinite D) less than infinite; infinite

13) A perfectly competitive firm's demand curve is 13)


A) perfectly inelastic.
B) the same as the market demand curve.
C) downward sloping.
D) the same as the firm's marginal revenue curve.

14) The market for fish is perfectly competitive. So, the price elasticity of demand for fish from a single 14)
fishery
A) is sometimes greater than and sometimes less than the elasticity of demand for fish overall.
B) is greater than the elasticity of demand for fish overall.
C) is less than the elasticity of demand for fish overall.
D) equals the elasticity of demand for fish overall.

15) In perfect competition, the price of the product is determined where the industry 15)
A) elasticity of supply equals the industry elasticity of demand.
B) supply curve and industry demand curve intersect.
C) fixed cost is zero.
D) average variable cost equals the industry average total cost.

16) Economists assume that a perfectly competitive firm's objective is to maximize its 16)
A) revenue. B) economic profit. C) output price. D) quantity sold.

2
17) Total economic profit is 17)
A) total revenue minus total opportunity cost.
B) marginal revenue minus marginal cost.
C) total revenue divided by total cost.
D) marginal revenue divided by marginal cost.

18) The economic profit of a perfectly competitive firm 18)


A) is less than its total revenue.
B) is greater than its total revenue.
C) equals its total revenue.
D) is less than its total revenue if its supply curve is inelastic and is greater than its total revenue
if its supply curve is elastic.

19) In perfect competition, a firm that maximizes its economic profit will sell its good 19)
A) below the market price.
B) above the market price.
C) below the market price if its supply curve is inelastic and above the market price if its supply
curve is elastic.
D) at the market price.

20) The above figure shows a firm's total revenue line. The firm must be in a market with 20)
A) monopolistic competition. B) monopoly.
C) perfect competition. D) oligopoly.

21) For a perfectly competitive firm, curve A in the above figure is the firm's 21)
A) average fixed cost curve. B) average variable cost curve.
C) total revenue curve. D) total fixed cost curve.

3
22) The figure above portrays a total revenue curve for a perfectly competitive firm. Curve A is straight 22)
because the firm
A) has perfect information. B) wants to maximize its profits.
C) is a price taker. D) faces constant returns to scale.

23) The figure above portrays a total revenue curve for a perfectly competitive firm. The firm's 23)
marginal revenue from selling a unit of output
A) equals $1.00. B) equals $2.00.
C) equals $0.50. D) cannot be determined.

24) The figure above portrays a total revenue curve for a perfectly competitive firm. The price of the 24)
product in this industry
A) equals $1.00. B) equals $2.00.
C) equals $0.50. D) cannot be determined.

25) In the above figure showing a perfectly competitive firm's total revenue line, the firm's marginal 25)
revenue
A) does not change as output increases. B) falls as output increases.
C) rises as output increases. D) cannot be determined.

Quantity Price
5 $15
6 $15
7 $15

26) In the above table, if the firm sells 5 units of output, its total revenue is 26)
A) $30. B) $15. C) $75. D) $90.

27) In the above table, if the quantity sold by the firm rises from 5 to 6, its marginal revenue is 27)
A) $15. B) $75. C) $90. D) $30.

28) In the above table, if the quantity sold by the firm rises from 6 to 7, its marginal revenue is 28)
A) $90. B) $30. C) $105. D) $15.

29) In perfect competition, the marginal revenue of an individual firm 29)


A) equals the price of the product.
B) is positive but less than the price of the product.
C) exceeds the price of the product.
D) is zero.

30) In the case of a perfectly competitive firm, the 30)


A) firm's marginal revenue exceeds the price of the product.
B) change in the firm's total revenue equals the price of the product multiplied by the change in
quantity sold.
C) firm's marginal revenue is less than average revenue.
D) price of the product falls sharply when the quantity the firm sells doubles.

4
31) In perfect competition, the firm's marginal revenue curve 31)
A) cuts its demand curve from above, going from left to right.
B) always lies below its demand curve.
C) cuts its demand curve from below, going from left to right.
D) is the same as its demand curve.

32) At a firm's break-even point, definitely its 32)


A) marginal revenue equals its average fixed cost.
B) marginal revenue equals its average variable cost.
C) total revenue equals its total opportunity cost.
D) marginal revenue exceeds its marginal cost.

33) When Sidney's Sweaters, Inc. makes exactly zero economic profit, Sidney, the owner, 33)
A) makes an income equal to his best alternative forgone income.
B) will boost output.
C) will shut down in the short run.
D) is taking a loss.

34) The break-even point is defined as occurring at an output rate at which 34)
A) total cost is minimized.
B) total revenue equals total opportunity cost.
C) economic profit is maximized.
D) marginal revenue equals marginal cost.

Output Total Revenue Total Cost


0 $0 $25
1 $30 $49
2 $60 $69
3 $90 $91
4 $120 $117
5 $150 $147
6 $180 $180

35) In the above table, the price of the product is 35)


A) $30. B) $150. C) $147. D) $180.

36) In the above table, the firm 36)


A) must be in a perfectly competitive industry, because its marginal revenue is constant.
B) cannot be in a perfectly competitive industry, because its short-run economic profits are
greater than zero.
C) cannot be in a perfectly competitive industry, because its long-run economic profits are
greater than zero.
D) must be in a perfectly competitive industry, because its marginal cost curve eventually rises.

37) In the above table, the marginal revenue from the fourth unit of output is 37)
A) $180. B) $147. C) $150. D) $30.

5
38) In the above table, if the firm produces 2 units of output, it will make an economic 38)
A) loss of $60. B) profit of $60. C) loss of $9. D) profit of $9.

Output Total Cost


(balloons per hour) (dollars per hour)
0 $4.00
1 $7.00
2 $8.00
3 $12.50
4 $17.20
5 $22.00
6 $29.00
39) In the above table, the firm's total fixed cost of production is 39)
A) $29.00. B) $4.00. C) $3.00. D) $7.00.

40) In the above table, the average fixed cost at 4 units of output is 40)
A) $4.80. B) $4.70. C) $1.00. D) $4.50.

41) In the above table, the average variable cost at 2 units of output is 41)
A) $4.00. B) $2.00. C) $1.00. D) $4.80.

42) In the above figure, by increasing its output from Q1 toQ2, the firm 42)
A) increases its profit. B) increases its marginal revenue.
C) reduces its marginal revenue. D) decreases its profit.

43) In the above figure, by increasing its output from Q2 to Q3, the firm 43)
A) increases its marginal revenue. B) reduces its marginal revenue.
C) decreases its profit. D) increases its profit.

6
44) The above figure illustrates a firm's total revenue and total cost curves. Which one of the following 44)
statements is FALSE?
A) At output Q1 the firm makes zero economic profit.
B) At an output above Q3 the firm incurs an economic loss.
C) Economic profit is the vertical distance between the total revenue curve and the total cost
curve.
D) At output Q2 the firm incurs an economic loss.

45) The feature of the above figure that indicates that the firm is a perfectly competitive firm is the 45)
A) fact that the total cost and total revenue curves are farthest apart at output is Q2.
B) shape of the total revenue curve.
C) fact that the total cost and total revenue curves cross twice.
D) shape of the total cost curve.

46) In the above figure, the firm is making an economic loss at 46)
A) point a. B) points b and d.
C) points a, b, and d. D) point c.

47) In the above figure, the firm is breaking even at points 47)
A) a and d. B) b and d. C) c and d. D) a and c.

48) In the above figure, when the firm produces output corresponding to point c, the firm's marginal 48)
cost
A) is less than its marginal revenue. B) equals its average revenue.
C) exceeds its marginal revenue. D) equals its marginal revenue.

7
49) For a perfectly competitive firm, in a diagram with quantity on the horizontal axis and both total 49)
revenue and total cost on the vertical axis, the firm's ________ is a straight line ________.
A) total cost curve; through the origin B) total revenue curve; with zero slope
C) total cost curve; with zero slope D) total revenue curve; through the origin

50) A perfectly competitive firm maximizes its profit by producing the output at which its marginal 50)
cost equals its
A) average variable cost. B) marginal revenue.
C) average total cost. D) average fixed cost.

51) For a firm in perfect competition, a diagram shows quantity on the horizontal axis and both the 51)
firm's marginal cost (MC) and its marginal revenue (MR) on the vertical axis. The firm's
profit-maximizing quantity occurs at the point where the
A) MC curve intersects the MR curve from above, going from left to right.
B) slope of the MC curve is zero.
C) MC curve intersects the MR curve from below, going from left to right.
D) MC and MR curves are parallel.

52) A firm will expand the amount of output it produces as long as its 52)
A) average total revenue exceeds its average variable cost.
B) marginal revenue exceeds its marginal cost.
C) marginal cost exceeds its marginal revenue.
D) average total revenue exceeds its average total cost.

53) A perfectly competitive firm is producing at the point where its marginal cost equals its marginal 53)
revenue. If the firm boosts its output, its total revenue will ________ and its profit will ________.
A) fall; fall B) fall; rise C) rise; rise D) rise; fall

54) A perfectly competitive firm is producing at the point where its marginal cost equals its marginal 54)
revenue. If the firm boosts its output, its revenue will
A) rise and its total variable cost will rise, but not by as much.
B) fall but its total variable cost will rise.
C) fall and its total variable cost will fall, but not by as much.
D) rise and its total variable cost will rise even more.

55) A perfectly competitive firm's marginal revenue exceeds its marginal cost at its current output. To 55)
increase its profit, the firm will
A) increase its output. B) raise its price.
C) decrease its output. D) lower its price.

56) A perfectly competitive firm's marginal cost exceeds its marginal revenue at its current output. To 56)
increase its profit, the firm will
A) increase its output. B) raise its price.
C) lower its price. D) decrease its output.

8
57) A perfectly competitive firm is producing more than the profit-maximizing amount of its product. 57)
You can conclude that its
A) marginal revenue is less than the price of the product.
B) total cost exceeds its total revenue.
C) average total cost exceeds the price of the product.
D) marginal cost exceeds the price of the product.

58) The costs incurred even when no output is produced are called 58)
A) fixed costs. B) external costs. C) variable costs. D) marginal costs.

59) A firm's shutdown point is the quantity and price at which the firm's total revenue just equals its 59)
A) marginal cost. B) total variable cost.
C) total cost. D) total fixed cost.

60) It definitely pays a firm to shut down if the price of its product is 60)
A) below its minimum average variable cost. B) above its maximum variable cost.
C) above its minimum average variable cost. D) below its minimum total cost.

61) The owners definitely will shut down a perfectly competitive firm if the price of its good falls 61)
below its minimum
A) average marginal cost. B) wage rate.
C) average variable cost. D) average total cost.

62) A firm that shuts down and produces no output incurs a loss equal to its 62)
A) marginal costs. B) total fixed costs.
C) total variable costs. D) marginal revenue.

63) By producing less, a firm can reduce 63)


A) its variable costs but not its fixed costs.
B) its fixed costs and its variable costs.
C) its fixed costs but not its variable costs.
D) neither its variable costs nor its fixed costs.

64) The shutdown point occurs at the level of output for which the ________ is at its minimum. 64)
A) marginal cost B) total cost
C) average fixed cost D) average variable cost

65) A competitive firm is more likely to shut down during a recession, when the demand for its 65)
product declines, than during an economic expansion, because during the recession it might be
unable to cover its
A) external costs.
B) depreciation due to machinery becoming obsolete.
C) variable costs.
D) fixed costs.

9
66) If the price of its product falls below the minimum point on the AVC curve, the best a perfectly 66)
competitive firm can do is to
A) shut down and incur a loss equal to its total variable cost.
B) shut down and incur a loss equal to its total fixed cost.
C) keep producing and incur a loss equal to its total variable cost.
D) keep producing and incur a loss equal to its total fixed cost.

67) If the price of its product just equals the average variable cost of production for a competitive firm, 67)
A) total revenue equals total variable cost and the firm's loss equals total fixed cost.
B) total revenue equals total fixed cost and the firm's loss equals total variable cost.
C) total variable cost equals total fixed cost.
D) total fixed cost is zero.

Output Total cost


(tons of rice per year) (dollars per ton)
0 $1,000
1 $1,200
2 $1,600
3 $2,200
4 $3,000
5 $4,000

68) Based on the table above which shows Chip's costs, if rice sells for $600 a ton, Chip's 68)
profit-maximizing output is
A) less than one ton. B) between one and two tons.
C) between two and three tons. D) between three and four tons.

69) Based on the table above which shows Chip's costs, if rice sells for $600 a ton, Chip will 69)
A) stay open because he earns an economic profit.
B) stay open because the price is above his minimum average variable cost.
C) shut down because the price is below his minimum average variable cost.
D) shut down because he incurs an economic loss.

70) Based on the table above which shows Chip's costs, if rice sells for $600 a ton, Chip 70)
A) earns an economic profit, but should shut down in the short run.
B) incurs an economic loss, but should stay open in the short run.
C) incurs an economic loss and should shut down in the short run.
D) earns an economic profit and should stay open in the short run.

71) Based on the table above which shows Chip's costs, if Chip shuts down in the short run, his total 71)
cost will be
A) $1,200. B) $4,000. C) $1,000. D) $0.

72) Based on the table above which shows Chip's costs, if Chip shuts down in the short run, his 72)
economic loss will be
A) $1,000. B) $1,200. C) $0. D) $4,000.

10
73) In the above figure, if the price is P1, the firm will produce 73)
A) where ATC equals P1. B) where MC equals P1.
C) nothing. D) where MC equals ATC.

74) In the above figure, if the price is P1, the firm maximizes its profit by producing 74)
A) where ATC equals P1. B) nothing.
C) where MC equals P1. D) where MC equals ATC.

75) In the above figure, if the firm increases its output from Q1 to Q2, it will 75)
A) increase its profit. B) reduce its marginal revenue.
C) decrease its profit. D) increase its marginal revenue.

76) In the above figure, if the firm increases its output from Q2 to Q1, it will 76)
A) reduce its marginal revenue. B) increase its profit.
C) increase its marginal revenue. D) decrease its profit.

77) In the above figure, if the price is P1, the firm is 77)
A) incurring an economic loss. B) shut down.
C) breaking even. D) making an economic profit.

78) In the above figure, if the firm produced Q1, the firm's economic profit is ________ than if it 78)
produced Q2 and ________ than if it produced Q3.
A) more; less B) less; more C) more; more D) less; less

79) In the above figure, if the firm produced Q3, the firm's economic profit is ________ than if it 79)
produced Q1 and ________ than if it produced Q2.
A) more; less B) more; more C) less; more D) less; less

11
80) A perfectly competitive firm will have an economic profit of zero if, at its profit-maximizing 80)
output, its marginal revenue equals its
A) marginal cost. B) average variable cost.
C) average total cost. D) average fixed cost.

81) The figure above shows short-run cost curves for a perfectly competitive firm. If the price of the 81)
product is $8, in the short run the firm will
A) incur an economic loss.
B) earn an economic profit.
C) earn a normal profit.
D) None of the above answers is correct because more information is needed to determine the
firm's profit or loss.

82) The figure above shows short-run cost curves for a perfectly competitive firm. If the price of the 82)
product is $8 and the firm does not shut down, the firm's output in the short run
A) will be 0.
B) will be 10 or higher.
C) will be between 0 and 10.
D) cannot be determined without more information.

83) The short-run supply curve for a perfectly competitive firm is its 83)
A) marginal cost curve above the horizontal axis.
B) average cost curve above the horizontal axis.
C) average cost curve above its shutdown point.
D) marginal cost curve above its shutdown point.

12
84) The short-run supply curve for a perfectly competitive firm is its marginal cost curve 84)
A) below its shutdown point. B) above the horizontal axis.
C) everywhere. D) above its shutdown point.

85) The short-run supply curve for a perfectly competitive firm is its marginal cost curve above the 85)
minimum point on the
A) average variable cost curve. B) demand curve.
C) average total cost curve. D) average fixed cost curve.

86) A perfectly competitive firm's supply curve is made up of its marginal cost curve at all points 86)
above its minimum
A) average variable cost curve. B) average total cost curve.
C) average fixed cost curve. D) price.

87) The firm's supply curve is its 87)


A) marginal cost curve, at all points above the minimum average fixed cost curve.
B) marginal revenue curve, at all points above the minimum average total cost curve.
C) marginal cost curve, at all points above the minimum average variable cost curve.
D) marginal revenue curve, at all points above the minimum average revenue curve.

88) The figure represents a firm in a perfectly competitive market. The firm will shut down if price falls 88)
below
A) P2. B) P1. C) P3. D) P4.

89) The figure represents a firm in a perfectly competitive market. If the firm does not shut down, the 89)
least amount of output that it will produce is
A) 10 units. B) 8 units. C) 5 units. D) less than 5 units.

13
90) The figure represents a firm in a perfectly competitive market. If the price rises from P3 to P4 then 90)
output will increase by
A) 3 units. B) 0 units. C) 1 unit. D) 2 units.

91) The figure above represents a firm in a perfectly competitive market. The firm's supply curve is the 91)
curved line linking
A) point c to point e and continuing on past point e along the ATC curve.
B) point b to point f and stopping at point f.
C) point a to point c and stopping at point c.
D) point b to point d and continuing on past point d along the MC curve.

92) In a perfectly competitive industry, the industry supply curve is the sum of the 92)
A) average total cost curves of all the individual firms.
B) supply curves of all the individual firms.
C) average variable cost curves of all the individual firms.
D) average fixed cost curves of all the individual firms.

93) If there are 1,000 rutabaga farms, all perfectly competitive, an increase in the price of fertilizer used 93)
for growing rutabagas will
A) have no effect on the total quantity of rutabagas supplied, because each farm's supply curve is
a vertical line.
B) reduce the total quantity of rutabagas supplied, because each farm's supply curve is a
horizontal line and will shift upward.
C) have no effect on the total quantity of rutabagas supplied, because no farm has enough market
power to raise the price.
D) decrease the total quantity of rutabagas supplied, because each farm's supply curve shifts
leftward.

14
94) In the above figure, if the price is P1, the firm is 94)
A) earning a normal profit.
B) incurring an economic loss.
C) earning enough revenue to pay all of its opportunity costs.
D) making an economic profit.

95) Suppose the cost curves in the above figure apply to all firms in the industry. Then, if the initial 95)
price is P1, in the long run the market
A) supply will decrease. B) supply will increase.
C) demand will decrease. D) demand will increase.

96) Suppose the cost curves in the above figure apply to all firms in the industry. If the initial price is 96)
P1, firms are
A) making an economic profit and some firms will leave the industry.
B) incurring an economic loss and some firms will leave the industry.
C) making an economic profit and some firms will enter the industry.
D) incurring an economic loss and some firms will enter the industry.

97) New reports indicate that eating turnips helps people remain healthy. The news shifts the demand 97)
curve for turnips rightward. In response, new farms enter the turnip industry. During the period in
which the new farms are entering, the price of a turnip ________ and the profit of each existing
firm ________.
A) falls; rises B) rises; falls C) rises; rises D) falls; falls

98) If firms exit an industry, the 98)


A) profits of the remaining firms decrease. B) industry supply curve shifts leftward.
C) price of the product falls. D) output of the industry increases.

15
99) As firms leave an industry because they are incurring an economic loss, the economic loss of each 99)
remaining firm
A) increases and the price of the product rises.
B) decreases and the price of the product falls.
C) decreases and the price of the product rises.
D) increases and the price of the product falls.

100) In a perfectly competitive industry, a permanent decrease in demand initially brings a lower price, 100)
economic
A) profit, and entry into the industry. B) profit, and exit from the industry.
C) loss, and entry into the industry. D) loss, and exit from the industry.

101) In the above figure, the firm's initial average total cost curve is SRAC with an initial marginal cost 101)
curve of SRMC. The price of the product is P1. In the short run the firm will produce output equal
to the amount
A) Q2. B) Q1. C) Q4. D) Q3.

102) In the above figure, the firm's initial average total cost curve is SRAC. If the price is P1., in the long 102)
run the firm will
A) retain the same plant size. B) expand its plant size.
C) exit the industry. D) reduce its plant size.

103) In the above figure when the firm has reached its long-run equilibrium position, it will produce 103)
output equal to the amount
A) Q4. B) Q3. C) Q2. D) Q1.

16
104) If the cost curves shown in the above figure apply to all firms in the industry and the initial price is 104)
P1, in the long run the price will be
A) greater than P1. B) zero. C) equal to P1. D) less than P1.

105) In a perfectly competitive industry, a permanent increase in demand initially brings a higher price, 105)
economic
A) profit, and entry into the industry. B) profit, and exit from the industry.
C) loss, and entry into the industry. D) loss, and exit from the industry.

106) In the long run, fixed costs are 106)


A) zero and variable costs are zero. B) zero and variable costs are positive.
C) positive and variable costs are positive. D) positive and variable costs are zero.

107) In the long run, the economic profits of a firm in a perfectly competitive industry 107)
A) will equal zero. B) will be below zero.
C) will be above zero. D) can be above, below, or equal to zero.

108) Assuming long-run external diseconomies exist, when demand increases in a perfectly competitive 108)
market, in the long run, the price of the product
A) falls below the initial price (before the increase in demand) and the quantity decreases.
B) equals the initial price (before the increase in demand) and the quantity increases.
C) equals the initial price (before the increase in demand) and the quantity decreases.
D) rises above the initial price (before the increase in demand) and the quantity increases.

109) Assuming long-run external economies exist, when demand increases in a perfectly competitive 109)
market, in the long run, the price of the product
A) rises above the initial price (before the increase in demand) and the quantity increases.
B) equals the initial price (before the increase in demand) and the quantity increases.
C) falls below the initial price (before the increase in demand) and the quantity increases.
D) equals the initial price (before the increase in demand) and the quantity decreases.

110) In a perfectly competitive market, if there are no external economies or diseconomies, an increase 110)
in demand
A) raises average cost in the long run. B) lowers the price in the long run.
C) leaves the price the same in the long run. D) raises the price in the long run.

111) If there are external economies, as demand increases, 111)


A) output decreases in the long run.
B) the price falls in the long run.
C) the price rises in the long run.
D) firms exit from the industry in the long run.

17
112) External economies are factors beyond the control of an individual firm that ________ as the total 112)
industry output increases.
A) raise its marginal revenue B) raise its costs
C) lower its costs D) lower its profit

113) A long-run supply curve for a perfectly competitive industry can slope upward because of 113)
A) external economies. B) economic profit.
C) external diseconomies. D) diminishing marginal returns.

114) In the above figure, the industry short-run supply curve shifts from S0 to S2 as the 114)
A) wage rate falls. B) number of firms increases.
C) external economies rise. D) number of firms decreases.

115) The curve LS0 in the above figure is the long-run supply curve of a perfectly competitive industry. 115)
As the demand curve shifts rightward, the industry exhibits
A) external economies.
B) neither external economies nor external diseconomies.
C) external diseconomies.
D) technological advancement.

116) Congestion of airports and airspace causes the airline industry to experience external 116)
A) diseconomies and have a long-run supply curve with positive slope.
B) economies and have a long-run supply curve with positive slope.
C) diseconomies and have a long-run supply curve with negative slope.
D) economies and have a long-run supply curve with negative slope.

18
117) Assuming long-run external economies exist, when demand increases in a perfectly competitive 117)
market, in the long run the average total cost curve for a typical firm
A) shifts upward. B) shifts downward.
C) is no longer U-shaped. D) stays the same.

118) If the slope of the long-run supply curve for a perfectly competitive industry is positive, the 118)
industry experiences
A) internal economies. B) external economies.
C) external diseconomies. D) internal diseconomies.

119) If the slope of the long-run supply curve for a perfectly competitive industry is negative, the 119)
industry experiences
A) external economies. B) external diseconomies.
C) internal diseconomies. D) internal economies.

120) The gains from trade that go to households are called 120)
A) consumer surplus. B) income.
C) profits. D) producer surplus.

121) Among the obstacles to the efficient allocation of resources are all of the following EXCEPT 121)
A) competition. B) monopoly.
C) external benefits. D) external costs.

122) An example of an external cost is 122)


A) the damage created by a tornado.
B) pollution.
C) the price that a consumer pays for a new car.
D) the price that a firm pays for a consultant's advice.

123) Which of the following characterizes a perfectly competitive industry? 123)


A) Each firm produces a product slightly different from that of its competitors.
B) The industry demand curve is vertical.
C) The demand for each individual firm is perfectly elastic.
D) Each firm sets a different price.

124) Paul runs a shop that sells printers. Paul is a perfect competitor and can sell each printer for a price 124)
of $300. The marginal cost of selling one printer a day is $200; the marginal cost of selling a second
printer is $250; and the marginal cost of selling a third printer is $350. To maximize his profit, Paul
should sell
A) two printers a day. B) more than three printers a day.
C) three printers a day. D) one printer a day.

19
125) Because of a decrease in the wage rate it must pay, a perfectly competitive firm's marginal costs 125)
decrease but its demand curve stays the same. As a result, the firm
A) decreases the amount of output it produces and lowers its price.
B) increases the amount of output it produces and lowers it price.
C) increases the amount of output it produces and does not change its price.
D) decreases the amount of output it produces and raises its price.

126) For prices above the minimum average variable cost, a perfectly competitive firm's supply curve is 126)
A) the same as its average variable cost curve.
B) horizontal at the market price.
C) the same as its marginal cost curve.
D) vertical at zero output.

127) A perfectly competitive firm is definitely earning an economic profit when 127)
A) P > ATC. B) P > AVC. C) P < ATC. D) MR < MC.

128) In the short run, a perfectly competitive firm can 128)


A) earn a normal profit.
B) incur an economic loss.
C) earn an economic profit.
D) earn an economic profit, earn a normal profit, or incur an economic loss.

129) Suppose firms in a perfectly competitive industry are suffering an economic loss. Over time, 129)
A) some firms leave the industry, so the price falls and the economic loss decreases.
B) some firms leave the industry, so the price rises and the economic loss decreases.
C) other firms enter the industry, so the price falls and the economic loss decreases.
D) other firms enter the industry, so the price rises and the economic loss decreases.

130) As firms enter a perfectly competitive industry, 130)


A) the price falls and the existing firms' economic profits do not change.
B) the price falls and the existing firms' economic profits decrease.
C) the price falls and the existing firms' economic losses do not change.
D) the price rises and the existing firms' economic profits decrease.

131) In the long run, a perfectly competitive firm can 131)


A) earn an economic profit, earn a normal profit, or incur an economic loss.
B) earn an economic profit.
C) incur an economic loss.
D) earn a normal profit.

20
132) The demand for a product produced in a perfectly competitive market permanently increases. In 132)
the short run the price
A) rises and each firm produces less output.
B) does not change because each firm produces more output.
C) rises and each firm produces more output.
D) does not change as new firms enter the industry.

133) If there are external diseconomies in an industry, in the long run, after a permanent increase in 133)
demand, the price
A) will be the same as it was initially before the increase in demand.
B) will be lower than it was initially before the increase in demand.
C) may be higher or lower than it was initially before the increase in demand, depending on
whether or not the firms are earning an economic profit.
D) will be higher than it was initially before the increase in demand.

134) To which of the following situations does the term "external diseconomies" apply? 134)
A) Increases in an industry's output reduce the costs of the firms in an industry.
B) The firm's ATC curve slopes upward as the firm produces more output.
C) The firm's MC curve falls as more output is produced.
D) Increases in an industry's output raise the costs of the firms in an industry.

135) The above figure shows the total revenue curve for Dizzy Discs. The demand curve for CD's sold 135)
by Dizzy Discs
A) has positive slope. B) has negative slope.
C) is horizontal. D) is vertical.

21
136) In the figure above, a firm is operating at point A on the graph. At point A, the firm's average cost 136)
curve
A) is horizontal. B) has negative slope.
C) is vertical. D) has positive slope.

137) Carol's Candies is producing 150 boxes of candy a day. Carol's marginal revenue and marginal cost 137)
curves are shown in the figure above. To increase her profit, Carol should
A) decrease output to increase profit.
B) maintain the current level of output to maximize profit.
C) increase output to increase profit.
D) Not enough information is given to determine if Carol should increase, decrease, or not
change her level of output.

22
138) Joe's Shiny Shoes is a firm that operates in a perfectly competitive market. The figure above shows 138)
Joe's cost and revenue curves. If the number of firms in the shoe market decreases, Joe will
A) decrease his production. B) have an MR curve with positive slope.
C) have an MR curve with negative slope. D) increase his production.

23
Answer Key
Testname: UNTITLED3.TST

1) D
2) A
3) C
4) A
5) D
6) C
7) B
8) B
9) A
10) D
11) C
12) D
13) D
14) B
15) B
16) B
17) A
18) A
19) D
20) C
21) C
22) C
23) B
24) B
25) A
26) C
27) A
28) D
29) A
30) B
31) D
32) C
33) A
34) B
35) A
36) A
37) D
38) C
39) B
40) C
41) B
42) A
43) C
44) D
45) B
46) A
47) B
48) D
49) D
50) B
1
Answer Key
Testname: UNTITLED3.TST

51) C
52) B
53) D
54) D
55) A
56) D
57) D
58) A
59) B
60) A
61) C
62) B
63) A
64) D
65) C
66) B
67) A
68) C
69) B
70) B
71) C
72) A
73) B
74) C
75) A
76) D
77) D
78) B
79) D
80) C
81) A
82) C
83) D
84) D
85) A
86) A
87) C
88) A
89) B
90) C
91) D
92) B
93) D
94) B
95) A
96) B
97) D
98) B
99) C
100) D
2
Answer Key
Testname: UNTITLED3.TST

101) A
102) B
103) B
104) D
105) A
106) B
107) A
108) D
109) C
110) C
111) B
112) C
113) C
114) B
115) C
116) A
117) B
118) C
119) A
120) A
121) A
122) B
123) C
124) A
125) C
126) C
127) A
128) D
129) B
130) B
131) D
132) C
133) D
134) D
135) C
136) A
137) A
138) D

3
Chapter 12 Monopoly - Sample Questions

MULTIPLE CHOICE. Choose the one alternative that best completes the statement or answers the question.

1) Unregulated monopolies 1)
A) cannot change the market quantity.
B) can influence the market quantity and price.
C) cannot incorporate.
D) take the market price as given.

2) The following are key features of a monopoly EXCEPT 2)


A) diseconomies of scale. B) no close substitutes.
C) influence over price. D) barriers to entry.

3) Which of the following statements about a monopoly is FALSE? 3)


A) A monopoly is the only supplier of the good.
B) Monopolies have no barriers to entry or exit.
C) The good produced by a monopoly has no close substitutes.
D) None of the above; that is, all of the above answers are true statements about a monopoly.

4) Which of the following is LEAST likely to be a monopoly? 4)


A) the sole owner of an occupational license
B) a pharmaceutical company with a patent on a drug
C) a store in a large shopping mall
D) the holder of a public franchise

5) A public franchise is 5)
A) an exclusive right granted to an inventor of a product.
B) a government issued license required to practice a profession.
C) a unique source of raw materials.
D) an exclusive right granted to a firm to supply a good or service.

6) Public franchises create monopolies by restricting 6)


A) entry. B) demand. C) prices. D) profit.

7) A patent grants 7)
A) a guarantee of quality to consumers.
B) an exclusive right to an inventor of a product.
C) the right to practice a profession.
D) control over a unique source or supply of raw materials.

8) Patents create monopolies by restricting 8)


A) prices. B) profit. C) entry. D) demand.

9) Patents are ________ barriers to entry and public franchises are ________ barriers to entry. 9)
A) legal; legal B) legal; natural C) natural; natural D) natural; legal

1
10) A defining characteristic of a natural monopoly is that 10)
A) it exists because of legal barriers to entry.
B) it has no close substitutes.
C) its average total cost curve slopes downward as it intersects the demand curve.
D) its demand curve slopes downward.

11) An industry in which one firm can supply the entire market at a lower price than two or more 11)
firms can is called a
A) legal monopoly. B) single-price monopoly.
C) natural monopoly. D) price-discriminating monopoly.

12) Which of the following is true of a natural monopoly? 12)


A) The firm can supply the entire market at a lower cost than could two or more firms.
B) Its average total cost curve slopes upward as it intersects the demand curve.
C) The firm is not protected by any barrier to entry.
D) Economies of scale exist to only a very low level of output.

13) A market in which competition and entry are restricted by the granting of a public franchise, 13)
government license, patent, or copyright is called a
A) price-discriminating monopoly. B) single-price monopoly.
C) natural monopoly. D) legal monopoly.

14) A single-price monopoly charges the same price 14)


A) even if the demand curve shifts.
B) to all customers.
C) even if its cost curves shift.
D) and the price equals the firm's marginal revenue.

15) All of the following are examples of price discrimination EXCEPT 15)
A) lower ticket prices for matinee performances.
B) buy-one-get-one-free offers.
C) "early bird specials" at a restaurant.
D) "buy now, pay later" payment options.

16) Total revenue equals 16)


A) total cost minus profit.
B) price times quantity sold.
C) marginal revenue times quantity sold.
D) the area between the demand curve and the marginal revenue curve.

17) For a monopoly, the industry demand curve is the firm's 17)
A) profit function. B) marginal revenue curve.
C) supply curve. D) demand curve.

2
18) Monopolists 18)
A) face downward sloping demand curves. B) are price takers.
C) have no short-run fixed costs. D) maximize revenue, not profits.

19) The marginal revenue curve for a single-price monopoly 19)


A) lies below its demand curve. B) is horizontal.
C) lies above its demand curve. D) coincides with its demand curve.

20) For a single-price monopoly, marginal revenue is ________ when demand is elastic and is ________ 20)
when demand is inelastic.
A) negative; positive B) positive; positive
C) positive; negative D) negative; negative

21) If the price elasticity of demand is greater than 1, a monopoly's 21)


A) marginal revenue is zero.
B) total revenue decreases when the firm lowers its price.
C) marginal revenue is negative.
D) total revenue increases when the firm lowers its price.

22) If the price elasticity of demand is less than 1, a monopoly's 22)


A) marginal revenue is undefined.
B) total revenue decreases when the firm lowers its price.
C) total revenue increases when the firm lowers its price.
D) marginal revenue is zero.

23) If the demand for its product is elastic, a monopoly's 23)


A) total revenue is unchanged when the firm lowers its price.
B) total revenue decreases when the firm lowers its price.
C) marginal revenue is zero.
D) marginal revenue is positive.

24) If the demand for its product is inelastic, a monopoly's 24)


A) marginal revenue is negative.
B) total revenue is unchanged when the firm lowers its price.
C) total revenue increases when the firm lowers its price.
D) marginal revenue is equal to zero.

25) A monopoly firm expands its output and lowers its price. The firm finds that its total revenue falls. 25)
Hence, the firm is producing in the
A) inelastic range of its supply curve. B) elastic range of its supply curve.
C) elastic range of its demand curve. D) inelastic range of its demand curve.

3
26) The figure above shows a monopoly firm's demand curve. If the price and quantity of haircuts 26)
move from point t to point r, the monopoly's
A) marginal revenue will decrease. B) total revenue will fall.
C) total revenue will remain the same. D) total revenue will rise.

27) The figure above shows a monopoly firm's demand curve. If the price and quantity of haircuts 27)
move from point t to point u, the monopoly's
A) total revenue will remain the same. B) total revenue will fall.
C) marginal revenue will increase. D) total revenue will rise.

28) The figure above shows a monopoly firm's demand curve. At point t 28)
A) demand is inelastic. B) demand is elastic.
C) demand is unit elastic. D) total revenue is at a minimum.

29) The figure above shows a monopoly firm's demand curve. The monopoly's total revenue is at its 29)
maximum when the firm produces at point
A) t. B) u. C) x. D) r.

30) The figure above shows a monopoly firm's demand curve. The monopoly's total revenue is zero at 30)
point
A) x. B) t. C) u. D) r.

31) The figure above shows a monopoly firm's demand curve. At point u in the figure, the demand 31)
facing the monopoly is
A) less than the supply. B) inelastic.
C) unit elastic. D) elastic.

4
32) An unregulated monopoly will 32)
A) produce in the elastic range of its demand curve.
B) flood the market with goods to deter entry.
C) produce only where marginal revenue is zero.
D) produce in the inelastic range of its demand curve.

33) An unregulated monopoly finds that its marginal cost exceeds its marginal revenue. In order to 33)
increase its profit, the firm will
A) lower its price and increase its output.
B) raise its price and increase its output.
C) raise its price and decrease its output.
D) continue to produce this level of output because any change will lower its profit.

34) The figure above shows a monopoly's total revenue and total cost curves. The monopoly's 34)
economic profit is positive if it produces between
A) 0 and 20 units. B) 5 and 20 units. C) 0 and 15 units. D) 0 and 5 units.

35) The figure above shows a monopoly's total revenue and total cost curves. The monopoly's 35)
economic profit is zero if it produces
A) 15 units of output. B) 5 or 20 units of output.
C) 0 units of output. D) none of the above

36) The figure above shows a monopoly's total revenue and total cost curves. The monopoly's 36)
economic profit is maximized when it produces
A) 5 units of output. B) 20 units of output.
C) 0 units of output. D) 15 units of output.

5
37) The figure above shows a monopoly's total revenue and total cost curves. The monopoly's marginal 37)
revenue equals its marginal cost when it produces
A) 5 units of output. B) 15 units of output.
C) 20 units of output. D) 0 units of output.

38) The monopoly with the TR and TC curves shown in the figure above will produce 38)
A) 5 units of output. B) 20 units of output.
C) 15 units of output. D) 0 units of output.

39) For the unregulated, single-price monopoly shown in the figure above, when its profit is 39)
maximized, output will be
A) 4 units per year and the price will be $6. B) 6 units per year and the price will be $4.
C) 4 units per year and the price will be $4. D) None of the above answers is correct.

40) The unregulated, single-price monopoly shown in the figure above will produce where its demand 40)
A) equals its ATC curve. B) is inelastic.
C) is elastic. D) equals its MC curve.

41) The unregulated, single-price monopoly shown in the figure above has a total economic profit of 41)
A) $4. B) $16. C) $24. D) $8.

6
42) The unregulated, single-price monopoly shown in the figure above will sell 42)
A) 50 tickets. B) 30 tickets.
C) less than 30 tickets. D) 100 tickets.

43) An unregulated, single-price monopoly is shown in the figure above. If fixed cost is $20, the 43)
monopoly's total costs when it is maximizing its profit will be
A) $30. B) $40. C) $140. D) $80

44) An unregulated, single-price monopoly is shown in the figure above. If fixed cost is $20, the 44)
monopoly's total economic profit when it is maximizing its profit will be
A) $0. B) $50. C) negative. D) $25.

45) The monopoly illustrated in the figure above is unregulated and charges a single price. The 45)
deadweight loss created by the monopoly is
A) $90.00. B) $0. C) $22.50. D) $45.00.

46) Unregulated monopolies can often earn an economic profit in the long run because 46)
A) they have high costs.
B) barriers to entry prevent competing firms from entering the market.
C) they receive government subsidies.
D) the risks of running a monopoly are high.

47) Compared to a single-price monopoly, a perfectly competitive industry produces 47)


A) more output and has a higher price. B) more output and has a lower price.
C) less output and has a higher price. D) less output and has a lower price.

7
48) Which of the following statements is true? 48)
A) A perfectly competitive industry produces more output and charges the same price as a
single-price monopoly.
B) A perfectly competitive industry produces less output but charges a lower price than a
single-price monopoly.
C) A perfectly competitive industry produces less output and charges the same price as a
single-price monopoly.
D) A perfectly competitive industry produces more output and charges a lower price than a
single-price monopoly.

49) The fundamental reason a single-price monopoly creates a deadweight loss is that it 49)
A) restricts output. B) raises variable cost.
C) raises fixed cost. D) reduces the elasticity of demand.

50) The unregulated, single-price monopolist illustrated in the figure above has a total revenue of 50)
A) $8.00 per day. B) $36.00 per day. C) $16.00 per day. D) $40.00 per day.

51) The unregulated, single-price monopolist illustrated in the figure above has a total cost of 51)
A) $16.00 per day. B) $40.00 per day. C) $32.00 per day. D) $8.00 per day.

52) The unregulated, single-price monopolist illustrated in the figure above earns an economic profit 52)
of
A) $8.00 per day. B) zero. C) $10.00 per day. D) $40.00 per day.

53) The unregulated, single-price monopolist illustrated in the figure above will produce 53)
A) 6 units per day. B) 9 units per day. C) 0 units per day. D) 4 units per day.

8
54) In the figure above, compared to a perfectly competitive industry with the same costs, a 54)
single-price, unregulated monopoly will decrease production by
A) 2 units per day. B) 4 units per day. C) 6 units per day. D) zero.

55) The unregulated, single-price monopolist illustrated in the figure above will set a price of 55)
A) $6.00 per unit. B) $8.00 per unit. C) $10.00 per unit. D) $2.00 per unit.

56) In the figure above, compared to a perfectly competitive industry with the same costs, a 56)
single-price, unregulated monopoly will raise the price by
A) $8.00 per unit. B) $6.00 per unit. C) $4.00 per unit. D) $2.00 per unit.

57) In the figure above, the deadweight loss created if the industry changes from perfectly competitive 57)
to a single-price, unregulated monopoly is
A) zero. B) $36.00 per day. C) $8.00 per day. D) $24.00 per day.

58) In the figure above, the redistribution from the consumers to the producer if the firm is a 58)
single-price, unregulated monopoly rather than a perfectly competitive industry is
A) $16.00 per day. B) $32.00 per day. C) $8.00 per day. D) zero.

59) In the figure above, the single-price, unregulated monopoly produces 59)
A) less than 20 units per day. B) 40 or more units per day.
C) 20 units per day. D) between 20 and 40 units per day.

60) If the industry in the above figure was perfectly competitive, the level of output would 60)
A) exceed the single-price monopoly level of output by 20 units.
B) be less than the single-price monopoly level of output.
C) be the same as the single-price monopoly level of output.
D) exceed the single-price monopoly level of output by 60 units.

9
61) In the figure above, the efficient amount of output is 61)
A) 40 units. B) 60 units. C) 20 units. D) 80 units.

62) The output produced by the single-price, unregulated monopoly in the above figure is 62)
A) efficient because marginal costs equals marginal revenue.
B) efficient because profit is maximized.
C) inefficient because too little is produced.
D) inefficient because too much is produced.

63) In the figure above, the single-price, unregulated monopoly sets a price of 63)
A) $40 per unit. B) $60 per unit. C) $80 per unit. D) $0 per unit.

64) Consumer surplus is 64)


A) equal to the price minus the marginal cost.
B) less in the case of a single-price monopoly than in the case of a perfectly competitive industry.
C) zero for a single-price monopolist.
D) positive in the case of a monopolist practicing perfect price discrimination.

65) In comparison with a perfect competition, a single-price monopolist with the same costs 65)
A) generates a larger consumer surplus and a larger economic profit.
B) generates a smaller consumer surplus but a larger economic profit.
C) generates a larger consumer surplus and a smaller economic profit.
D) generates a smaller consumer surplus and a smaller economic profit.

66) Compared to a competitive industry, a monopoly transfers 66)


A) consumer surplus to producers.
B) producer surplus to consumers.
C) deadweight loss away from producers to consumers.
D) deadweight loss away from consumers to producers.

67) Any attempt to capture a consumer surplus, a producer surplus, or an economic profit is called 67)
A) efficiency gain. B) profit-maximizing.
C) rent-seeking. D) price discriminating.

68) Efforts by a firm to obtain a monopoly 68)


A) are called price taking. B) are called price discrimination.
C) raise consumer surplus. D) are called rent seeking.

69) Activity aimed at creating artificial barriers to entry to a particular market 69)
A) improves competition. B) is rent seeking.
C) has no social cost. D) improves the economy's efficiency.

70) Rent seeking is devoted to the creation of 70)


A) more elastic demand. B) monopolies.
C) human capital. D) competitive industries.

10
71) Rent seeking through lobbying 71)
A) results in perfectly competitive industries. B) uses up resources.
C) results in perfect price discrimination. D) reduces deadweight loss.

72) The value of resources devoted to rent seeking will 72)


A) reduce consumer surplus. B) equal the monopoly's economic profits.
C) raise output to an efficient level. D) reduce deadweight loss.

73) Price discrimination 73)


A) is more likely for services than for goods that can be stored.
B) is common in perfectly competitive markets.
C) is illegal because it always violates antitrust laws.
D) works only if all groups of demanders have the same price elasticity of demand for the
product.

74) A price discriminating monopolist charges lower prices to customers with 74)
A) higher average willingness-to-pay. B) lower average willingness-to-pay.
C) lower supply elasticities. D) higher supply elasticities.

75) Monopolists are able to practice price discrimination because 75)


A) they have constant marginal cost.
B) of differing price elasticities of supply.
C) they have constant average cost.
D) of differing average willingness-to-pay among consumers.

76) The more perfectly a monopoly can price discriminate, the 76)
A) smaller its output and the higher its profits.
B) larger its output and the higher its profits.
C) larger its output and the lower its profits.
D) smaller its output and the lower its profits.

77) Which of the following occurs with both perfectly price discriminating and single-price 77)
monopolies?
A) The level of output is inefficient.
B) Deadweight loss is created.
C) There is a redistribution of consumer surplus to the monopoly.
D) All consumer surplus goes to the monopoly.

78) In the case of a perfectly price-discriminating monopoly, there is no 78)


A) transfer of consumer surplus to the producer.
B) deadweight loss.
C) long-run economic profit.
D) short-run economic profit.

11
Demand Schedule Facing a
Perfectly Price Discriminating Firm
Price
(dollars) Quantity Sold
8 0
7 1
6 2
5 3
4 4
3 5
2 6
1 7

79) Using the demand schedule in the above table, if the firm's marginal cost is constant at $3.00, 79)
output for a perfect price discriminating monopolist is
A) 2 units. B) 4 units. C) 5 units. D) 3 units.

80) Using the demand schedule in the above table, the marginal revenue for the perfectly price 80)
discriminating monopolist from the sale of the third unit of output is
A) $6. B) $5. C) $4. D) $3.

81) Using the demand schedule in the table above, the total revenue a perfectly price discriminating 81)
monopolist receives from selling 5 units of output is
A) $18. B) $5. C) $15. D) $25

82) If the monopoly illustrated in the figure above could engage in perfect price discrimination, then 82)
each buyer would pay
A) $2.00. B) $3.50. C) $3.00. D) a different price.

12
83) If the monopoly illustrated in the figure above could engage in perfect price discrimination, then 83)
the lowest ticket price would be
A) $3.50. B) $3.00. C) $1.00. D) $2.00.

84) If the monopoly illustrated in the figure above could engage in perfect price discrimination, then it 84)
would sell
A) 60 tickets. B) 50 tickets. C) 30 tickets. D) 100 tickets.

85) If the monopoly illustrated in the figure above could engage in perfect price discrimination, then 85)
total revenue collected by the firm would be
A) $110. B) $210. C) $120. D) $310.

86) In the figure above, what is the loss of consumer surplus if the firm is a perfectly 86)
price-discriminating monopoly instead of a perfectly competitive industry?
A) $22.50 B) $90.00 C) $0 D) $45.00

87) If the monopoly illustrated in the figure above could engage in perfect price discrimination, the 87)
deadweight loss would be
A) $22.50. B) $250.00. C) $0. D) $90.00.

88) In the figure above, the elasticity of demand facing the monopoly equals one when it produces 88)
________ output.
A) k B) h
C) j D) none of the above

89) In the figure above, a single-price unregulated monopoly will set price 89)
A) a. B) b. C) c. D) d.

13
90) In the figure above, a single-price unregulated monopoly will produce at output 90)
A) k. B) j.
C) h. D) none of the above

91) In the figure above, the transfer of consumer surplus from consumers to the producer caused by 91)
production under a single-price monopoly instead of perfect competition is the area of
A) trapezoid beic. B) rectangle begd. C) rectangle befc. D) triangle abe.

92) In the figure above, consumer surplus at the price that maximizes the profit for an unregulated, 92)
single-price monopolist is the area of
A) triangle eig. B) triangle abe. C) rectangle 0hgd. D) rectangle 0heb.

93) In the figure above, the deadweight loss from production under a single-price monopoly instead 93)
of perfect competition is the area of
A) triangle aic. B) triangle aeb. C) triangle eig. D) triangle eif.

94) In the figure above, a perfectly price-discriminating monopoly will maximize profit by producing 94)
at output
A) h. B) k.
C) j. D) none of the above

95) In the figure above, the total revenue of a perfectly price-discriminating monopolist at the 95)
profit-maximizing output is equal to the area of
A) 0dgh. B) 0beij. C) aci. D) 0aij.

96) When an increase in a firm's output of a good or service brings a decrease in the average total cost 96)
of producing it, the firm is experiencing
A) diseconomies of scale. B) economies of scale.
C) economies of scope. D) diminishing returns.

97) Economies of scope arise when 97)


A) an increase in output causes average total cost to fall.
B) doubling inputs causes output to more than double.
C) high profit allows a company to undertake research and development.
D) an increase in the range of goods produced causes average total cost to fall.

98) When an increase in the range of goods produced brings a decrease in the average total cost of 98)
production, the firm is experiencing
A) diminishing returns. B) economies of scale.
C) economies of scope. D) diseconomies of scale.

99) Which of the following is NOT a possible gain to society from a monopoly? 99)
A) The monopoly may induce innovation.
B) The monopoly may achieve economies of scope.
C) The monopoly may create rent seeking.
D) The monopoly may achieve economies of scale.

14
100) Which of the following statements regarding a marginal-cost pricing rule is incorrect? 100)
A) It is efficient.
B) It allows the firm to earn a normal profit.
C) It maximizes total surplus in a regulated industry.
D) It sets price equal to marginal cost.

101) Which of the following statements regarding average-cost pricing rule is incorrect? 101)
A) The firm earns a normal profit.
B) It is efficient.
C) More output is produced than if the firm maximized profit.
D) It sets price equal to average total cost.

102) In a small town, Marilyn's Christmas Tree Lot has a monopoly on sales of Christmas trees. In order 102)
to increase her sales from 100 trees to 101 trees, she must drop the price of all of her trees from $20
to $19. What is the marginal revenue?
A) $20 B) $19 C) negative $81 D) $2000

103) A single-price monopoly 103)


A) eliminates all the consumer surplus.
B) charges all consumers the lowest price that they want to pay for each unit purchased.
C) produces less output than it would if it could discriminate.
D) creates a smaller deadweight loss than it would if it could discriminate.

104) Because of a decrease in labor costs, a monopoly finds that its marginal cost and average total cost 104)
have decreased. The monopoly will
A) raise its price and decrease the quantity it produces.
B) raise its price and increase the quantity it produces.
C) lower its price and increase the quantity it produces.
D) lower its price and decrease the quantity it produces.

105) If a monopoly is producing at an output level at which marginal revenue exceeds marginal cost, in 105)
order to increase its profit it will
A) raise its price and decrease its output. B) lower its price and increase its output.
C) raise its price and increase its output. D) lower its price and decrease its output.

106) Compared to a single-price monopoly, the output of a perfectly competitive industry with the 106)
same costs
A) is more than the monopoly's output.
B) is less than the monopoly's output.
C) could be more than, less than, or equal to the monopoly's output.
D) is the same as the monopoly's output.

15
107) Compared to a single-price monopoly, the price charged by a competitive industry with the same 107)
costs
A) could be higher than, lower than, or the same as the monopoly's price.
B) is higher than the monopoly's price.
C) is the same as the monopoly's price.
D) is lower than the monopoly's price.

108) If a perfectly competitive industry becomes a monopoly and the costs do not change, which of the 108)
following allocation of costs and benefits applies?
A) The producer and society benefit, but consumers are harmed.
B) The producer and society are harmed, but consumers benefit.
C) The producer is harmed, but consumers and society benefit.
D) The producer benefits, but consumers and society are harmed.

109) Consumer surplus is largest for 109)


A) a single-price monopoly.
B) a perfectly competitive industry.
C) any price-discriminating monopoly.
D) a perfectly price-discriminating monopoly.

110) Which of the following may be a gain to society from monopoly? 110)
A) Monopolies may be able to generate economies of scale.
B) Monopolies may earn an economic profit in the long run.
C) Monopolies may be able to price discriminate, thereby boosting consumer surplus.
D) Monopolists do not waste resources trying to innovate.

16
111) La Bella Pizza is the only pizza place on Pepper Island. The figure above shows La Bella Pizza's 111)
demand curve, marginal revenue curve, and marginal cost curve. At La Bella Pizza's
profit-maximizing output, its annual total revenue is
A) $312,000. B) $624,000. C) $168,000. D) $336,000.

112) The figure above shows the demand curve facing Sue's Surfboards, the sole renter of surfboards on 112)
Big Wave Island. Sue's Surfboards currently rents 15 surfboards an hour. Sue's total revenue from
the 15 surfboards is
A) $10. B) $225. C) $300. D) $150.

17
113) Sue's Surfboards is the sole renter of surfboards on Big Wave Island. Sue's demand and marginal 113)
revenue curves are illustrated in the figure above. The change in the total revenue from renting the
15th surfboard is
A) $20. B) $0. C) $10. D) $15.

114) The figure above shows the demand and marginal revenue curves facing Sue's Surfboards, the sole 114)
renter of surfboards on Big Wave Island. If Sue is renting 25 surfboards an hour so that the
marginal revenue is negative, then Sue's Surfboards
A) must face a unit elastic demand for surfboard rentals.
B) can increase its profit by increasing the number of rentals.
C) must face an elastic demand for surfboard rentals.
D) must face an inelastic demand for surfboard rentals.

115) Bob's Books is the only bookstore in town. The figure above shows the demand curve for books 115)
and Bob's Books' marginal revenue curve and marginal cost curve. Bob's Books maximizes its
profit and sets the price of a book equal to ________ and has total annual revenue of ________.
A) $20, $60,000 B) $30; $60,000 C) $10; $40,000 D) $40; $40,000

18
Answer Key
Testname: UNTITLED4.TST

1) B
2) A
3) B
4) C
5) D
6) A
7) B
8) C
9) A
10) C
11) C
12) A
13) D
14) B
15) D
16) B
17) D
18) A
19) A
20) C
21) D
22) B
23) D
24) A
25) D
26) B
27) B
28) C
29) A
30) A
31) B
32) A
33) C
34) B
35) B
36) D
37) B
38) C
39) A
40) C
41) D
42) B
43) D
44) D
45) C
46) B
47) B
48) D
49) A
50) D
1
Answer Key
Testname: UNTITLED4.TST

51) C
52) A
53) D
54) A
55) C
56) C
57) C
58) A
59) C
60) A
61) A
62) C
63) B
64) B
65) B
66) A
67) C
68) D
69) B
70) B
71) B
72) B
73) A
74) B
75) D
76) B
77) C
78) B
79) C
80) B
81) D
82) D
83) D
84) A
85) B
86) B
87) C
88) A
89) B
90) C
91) C
92) B
93) C
94) C
95) D
96) B
97) D
98) C
99) C
100) B
2
Answer Key
Testname: UNTITLED4.TST

101) B
102) C
103) C
104) C
105) B
106) A
107) D
108) D
109) B
110) A
111) A
112) D
113) B
114) D
115) B

3
Economics 101 – Lec 3 Student Name: __________________
Elizabeth Kelly ID Number: ____________________
Fall 2000 Section Number: ________________
Midterm #3 / Version #1 TA Name: ______________________
December 4, 2000
TF+MC _____
Problem _____ NOTE: This information and the
VERSION 1 Total ________ similar information on the bubble
sheet are worth 2 points.

DO NOT BEGIN WORKING


UNTIL THE INSTRUCTOR TELLS YOU TO DO SO.
READ THESE INSTRUCTIONS FIRST.

You have the class period of 50 minutes to complete the exam. The exam consists of three parts: 5
true/false questions, 12 multiple choice questions, and 1 problem. Each true/false question is worth
2 points for a total of 10 points, each multiple choice question is worth 3 points for a total of 36
points, and the problem is worth 12 points. Please answer all true/false questions and multiple
choice questions on the coding sheet with a #2 pencil. Choose the best answer. Answer the problem
on the exam booklet. Please make sure you write legibly and are clear about the solution to the
problem. No calculators are allowed. Please fill out the blanks at the top of the exam booklet.
There are 10 pages in this exam booklet.

How to fill in the coding sheet:


1. Print your last name, first name, and middle initial in the spaces marked “Last Name,” “First
Name,” and “MI.” Fill in the corresponding bubbles below.
2. Print your student ID number in the space marked “Identification Number.” Fill in the
corresponding bubbles below.
3. Write your discussion section number under “Special Codes” spaces ABC, and fill in the
bubbles. You can find the discussion numbers below on this page.
4. Write your version number under “Special Codes” space D, and fill in the bubble.

Note that filling in the bubble sheet correctly, and providing the information required in the
box on the upper right corner of this page is worth 2 points.

• If you have any questions during the exam, stay seated and raise your hand.
• When you are finished, please get up quietly and bring your code sheet and this exam booklet to
the place indicated by the instructors.

DISCUSSION SECTIONS:

Disc 381 9:55W 215 Ingraham Oya Ardic Disc 390 2:25R 6228 Soc Sci Seungmoon Choi
Disc 382 11:00W 115 Ingraham Oya Ardic Disc 391 3:30R 1412 Sterling Seungmoon Choi
Disc 383 12:05W 6102 Soc Sci Tsung-Sheng Tsai Disc 392 9:55F B333 Van Vleck Seungmoon Choi
Disc 384 1:20W 6310 Soc Sci Tsung-Sheng Tsai Disc 393 9:55F 122 Ingraham Tsung-sheng Tsai
Disc 385 2:25W 55 Bascom Zhonghua Wu Disc 395 12:05F B129 Van Vleck Seungmoon Choi
Disc 386 3:30W 6314 Soc Sci Zhonghua Wu Disc 396 11:00F 14 Ingraham Seungmoon Choi
Disc 387 11:00R 6322 Soc Sci Zhonghua Wu Disc 397 11:00F B305 Van Vleck Tsung-Sheng Tsai
Disc 389 12:05R 114 Ingraham Zhonghua Wu Disc 398 12:05F 23 Ingraham Tsung-Sheng Tsai

1
PART I: TRUE/FALSE QUESTIONS (2 points each: Allow 5 minutes)

On the coding sheet, darken choice A if you think the statement is TRUE, and darken choice B if
you think the statement is FALSE.

Question 1: It is possible for a firm to experience diminishing marginal returns in the long-run.

A. True
B. False

Question 2: A monopolist profit maximizes by producing that level of output where marginal
revenue equals marginal cost and charging a price greater than the average total cost of production.

A. True
B. False

Question 3: The market described by the following graph is consistent with the market being an
oligopoly.

P P
LRATC D

$30

$20

1,000 10,000 Q 15,000 20,000 Q

Firm Market
A. True
B. False

Question 4: In the long-run a firm will always earn zero economic profit, but it could have a
positive accounting profit.

A. True
B. False

Question 5: If the total cost function is given by TC = 10,000 + 10Q + 85 Q , the associated total
fixed cost and total variable functions are given by: TFC = 10,000 and TVC = 10Q + 85 Q
respectively.

A. True
B. False

2
PART II: MULTIPLE CHOICE QUESTIONS (3 points each: allow 24 minutes)

Question 6: K-Mart and Wal-Mart are considering establishing either 1 or 2 stores in a town. The
payoff matrix, which gives the profits of each company, is as follows (profits are in million dollars):

Wal-Mart
One Two

One 10 12
K-Mart 10 7
4 5
Two 6 5

The upper triangle of each box corresponds to Wal-Mart’s profits whereas the lower triangle shows
K-Mart’s profits. What is the equilibrium outcome in this game? K-Mart builds _______ store(s)
and Wal-Mart builds _________ store(s).

A. (1, 1).
B. (2, 1).
C. (1, 2).
D. (2, 2).
E. There is no equilibrium in this game.

Question 7: Suppose the demand and supply in a perfectly competitive market are as follows:

Demand: P = 20 – 0.005Q
Supply: P = 0.005Q

A representative firm in the perfectly competitive market has a marginal cost, MC = 10Q. In
the short run, the profit-maximizing output is _________ units for an individual firm and the
equilibrium price is __________ .

A. (10,10)
B. (10,100)
C. (1,10)
D. (1,100)
E. (0.1,10)

3
Question 8:

P
MC
ATC
AVC

$16
$12
$10

7 10 Q

The figure above shows the cost structure of a representative firm in a perfectly competitive market.
Suppose the current market equilibrium price is $12. Which of the following statements is FALSE?

A. The marginal revenue curve for the firm is P = $12.


B. There is a negative economic profit in the short run.
C. The long-run equilibrium price is $16.
D. In the short run, this firm will shut down the business and leave the market.
E. In the long run, if there are 100 firms staying in this market, the equilibrium quantity will be
1,000 units.

Question 9: Which of the following is TRUE?

A. A monopolistically competitive firm does not produce at its minimum ATC in the long-run.
B. A monopolistically competitive firm cannot successfully maintain positive economic profits
in the long-run.
C. Barriers to entry make it possible for monopolies to earn positive economic profits in the
long-run.
D. (A) and (C).
E. (A), (B) and (C).

Question 10: A monopolist has cost function TC = 10 + 2Q. Demand in this market is given by the
equation Q = 14 – P. If this monopolist can charge only a single price, its profit in the short run will
be:

A. -8
B. 10
C. 26
D. 34
E. 46

4
Question 11: Which of the following statements is NOT a characteristic of a perfectly competitive
market?

A. The equilibrium output is both allocatively and productively efficient.


B. While firms in a perfectly competitive market can make positive, zero, or negative economic
profits in the short-run, they have to make zero economic profits in the long-run.
C. Marginal revenue for a perfectly competitive firm is equal to the market price because firms
in a perfectly competitive market are price takers.
D. In the short-run, a perfectly competitive firm will continue production as long as it can cover
its total variable costs.
E. There are many firms in a perfectly competitive market that produce differentiated products.

Question 12: A regulated natural monopolist practicing average cost pricing

A. makes zero economic profit.


B. produces an allocatively inefficient level of output.
C. produces the largest quantity possible while still enabling the firm to cover its total costs.
D. all of the above.
E. none of the above.

Question 13: Which of the following is an example of perfect price discrimination?

A. lower air fares for customers willing to stay over a Saturday night.
B. discount movie tickets for children and senior citizens.
C. discounts for seats with an obstructed view in a theater.
D. higher phone rates on weekdays than on weekends.
E. none of the above.

Question 14: Which of the following statements about cost functions is TRUE?

A. If there is an increase in total fixed costs (TFC), average variable costs (AVC) will increase.
B. A firm that has increasing average costs as output expands experiences increasing returns to
scale.
C. Marginal cost is not affected by changes in fixed costs.
D. As the level of output increases, the difference between average total cost (ATC) and
average variable cost (AVC) widens.
E. If marginal cost is less than average variable cost, then the average variable cost is
increasing.

5
Question 15: A monopolist sells a single product and its consumers fall into two groups, group A
and group B. The marginal cost of producing the good is constant at $2 and is equal for both groups.
The two graphs below depict the demand curves and the marginal revenue curves for each of the
two groups:

P P

$8 A’s demand $8 B’s demand

$6
$5 A’s marginal revenue
$3.5 B’s marginal revenue

$2 MC $2 MC

1.5 3 Q 1.5 3 Q

Group A Group B

Suppose that the monopolist can distinguish between the two groups and engages in price
discrimination. Which of the following is true if the monopolist maximizes its profits?

A. The price charged to group A is greater than the price charged to group B. The quantity sold
to group A is greater than the quantity sold to group B.
B. The price charged to group A equals the price charged to group B. The quantity sold to
group A equals the quantity sold to group B.
C. The price charged to group A is greater than the price charged to group B. The quantity sold
to group A is less than the quantity sold to group B.
D. The price charged to group A is less than the price charged to group B. The quantity sold to
group A is greater than the quantity sold to group B.
E. The price charged to group A is less than the price charged to group B. The quantity sold to
group A is less than the quantity sold to group B.

Question 16: Which of the following statements about market structures is TRUE?

A. All unregulated firms, regardless of the type of the market they are in, are productively
efficient.
B. A perfect price discriminating monopoly is allocatively efficient.
C. Monopolistically competitive firms do not operate at the minimum cost per unit in the long-
run.
D. Oligopolistic markets are characterized by strategic interactions.
E. All of the above

6
Question 17: The following table provides information about long-run average cost curve of a firm:

Q LRAC
100 50
200 43
300 39
400 39
500 44
600 52

Which of the following statements is TRUE about this firm?

A. For the first 300 units of output, this firm has decreasing returns to scale.
B. For output levels between 300 and 400, the firm has constant returns to scale.
C. For output levels greater than 400, the firm has increasing returns to scale.
D. This cost structure is an example of natural monopoly.
E. All of the above

7
PART III: PROBLEM (12 points: allow 21 minutes)

Please write legibly. Make sure to show ALL your work. You may not receive any credit if you fail
to show your work. Put your answers for each part in the respective blanks provided.

Suppose a monopoly faces the following demand curve for its product:

100

CS
P
DWL MR

MC
20 D

Q 50 100 Q

The equations for the demand curve and the marginal revenue curve are given as follows:
D: P = 100 − Q
MR: MR = 100 − 2Q

From the demand curve the monopolist obtains its marginal revenue curve, which has been also
drawn in the graph above. Note that both curves are linear.

A. (3 points) Suppose the monopolist has constant marginal costs of production given by
MC = 20 and total costs given by TC = 20Q + 1000 . Using this information and the
information above on the graph mark:
• The monopolist’s quantity Q
• The monopolist’s price P
• Draw the MC curve
Then, numerically calculate:
• The monopolist’s optimal quantity (Q) = ___40 units_
• The price the monopolist will charge (P) = __$60__

Profit maximization condition: MR = MC è 100-2Q = 20 è Q = 40 units


Use the demand curve to find the price that the monopolist will charge for Q = 40:
P = 100 – Q = 100 – 40 è P = $60

8
B. (3 points) Find the consumer surplus that is associated with the monopolist’s optimal
price/quantity combination. On the graph:
• label the consumer surplus for the monopolist
Calculate:
• the dollar value of the consumer surplus if the firm is a monopolist equals __$800_ .
• the dollar value of the consumer surplus if this was a perfectly competitive industry
equals ___$3200__ .

Consumer Surplus in Monopoly:


CS = (40)(40)/2 = $800

If the market was perfectly competitive, price would be $20 and equilibrium quantity would
be 80 units (solve for the point where MC crosses the demand curve).

Consumer Surplus in Perfect Competition:


CSPC = (80)(80)/2 = $3200

C. (3 points)
• Discuss the efficiency properties of the monopolist’s optimal price/quantity
combination.
• On the graph label the area that corresponds to dead-weight loss.
• The dollar value of the dead-weight loss equals ___$600____ .

The monoplist’s optimal output level is allocatively inefficient. Too few of the good is
produced. This creates a deadweight loss.

DWL = (40)(40)/2 = $800

9
D. (3 points)
• Suppose now that the government decides to auction off a license to be the
monopoly provider of this good for one year. Using all the information given above,
decide how much the monopolist that we have analyzed in parts (a), (b), and (c)
would be willing to pay for this license.

The profits available fro the firm that will have the monopoly rights equal:
Profits = TR - TC
TR = P.Q = (60)(40) = $2400
TC = 20Q +1000 = (20)(40) +1000 = $1800
Profits = 2400 – 1800 = $600

Since there will be many firms bidding for the monopoly rights, the winning bid for the
license in the auction will be equal to the potential profits.

• Suppose the government gives consumers the auction proceeds (thus augmenting
their consumer surplus). Rank the following three choices according to consumers’
preferences where 1 is the best choice from the consumers’ point of view and 3 is the
worst choice from their point of view:
__3_ Firm is a monopoly
__1_ Market is perfectly competitive
__2_ Monopolist’s rights are auctioned off with consumers receiving this
payment.
Make sure you support your argument with a numerical analysis using the
information above and your answer for part (B)

Consumer surplus when the monopoly rights are auctioned:


CSMR = Consumer Surplus in Monopoly (from part (B)) + Price paid for the license
= $800 + $600 = $1400

A. ______
B. ______
C. ______
D. ______

Total: ______

10
Results for Quiz
Close this window

You correctly answered 2 questions for a score of 10 percent.


20-1. If there is excess capacity, the transfer price is often

a. market price
10% b. opportunity cost plus incremental cost
c. variable cost or variable cost plus profit
d. full cost plus profit
e. a or b

Your choice of "b" was incorrect. The correct answer is "c".

20-2. A dual transfer pricing system is set up where

a. the two sides cannot agree on a price and the difference between
the two sides is absorbed by the home office
b. a ready market price is not available and the two sides must come up
with an agreeable price
c. the buyer buys at variable cost and the seller only sells at full cost
d. the two sides agree to use a cost basis for transfer pricing
e. none of the above

Your choice of "b" was incorrect. The correct answer is "a".

20-3. If variable cost is used in transfer pricing, it is preferable to use

a. standard variable cost because the buyer does not wish to be


stuck with unknowns
b. standard variable cost because the seller does not wish to pass along
the variations in cost
c. actual variable cost because the buyer is well-advised to deal with the
real rather than anticipated costs
d. actual variable costs because the seller is well-advised to deal with the
real rather than anticipated costs
e. none of the above.

You answered correctly!

20-4. The objective(s) of transfer pricing are

a. to motivate managers
b. to provide an incentive for managers to make decisions consistent with
the firm's goals
c. to provide a basis for fairly rewarding the managers
d. all of the above
e. b and c

Your choice of "b" was incorrect. The correct answer is "d".


20-5. The basic methods used in transfer pricing are

a. variable or full costs


b. dual prices
c. market price or negotiated price
d. all of the above
e. b and c

Your choice of "a" was incorrect. The correct answer is "d".

20-6. To minimize taxes, some multinational companies set high transfer prices

a. when goods are shipped from low tax countries to other low tax
countries
b. when goods are shipped from low tax countries to high tax
countries
c. when goods are shipped from high tax countries to low tax countries
d. a or b
e. a or c

Your choice of "a" was incorrect. The correct answer is "b".

20-7. Market pricing approach in transfer pricing

a. helps to preserve unit autonomy


b. provides incentive for the selling unit to be competitive with outside
suppliers
c. has arm's-length standard desired by taxing authorities
d. may be the most practical approach when there is significant conflict
e. all except d

Your choice of "a" was incorrect. The correct answer is "e".

20-8. Variable costing method of transfer pricing is

a. easy to implement
b. intuitive and easily understood
c. more logical when there is excess capacity
d. all of the above
e. a and b

Your choice of "a" was incorrect. The correct answer is "d".

20-9. Use this data to respond to questions 9 through 15. Canada-US Multinational
transfers 500 units of TV's from Canada to the US. The TV's cost $280 a unit (60% of
which is variable cost). The market price for these TV's is at $390 each. The Canada
division's incremental tax rate is at 30% whereas, the US taxes are at 55%. The US
subsidiary incurs a cost of $24 per set and sells the units for $430 a unit. Canada
division's profit after tax using market transfer prices amounts to

a. $36,000
b. $38,500
c. $42,100
d. $44,800
e. none of the above
Your choice of "c" was incorrect. The correct answer is "b".

20-10. US division's profit after tax using total cost for transfer pricing amounts to

a. $20,350
b. $24,350
c. $26,350
d. $28,350
e. none of the above

Your choice of "b" was incorrect. The correct answer is "d".

20-11. Canada division's profit after tax using variable costs for transfer pricing
amounts to

a. $39,200
b. $-39,200
c. $53,550
d. $-53,550
e. none of the above

Your choice of "c" was incorrect. The correct answer is "b".

20-12. US division's profit after tax using variable costs plus a mark-up of 30% for
transfer pricing amounts to

a. $42,210
b. -$42,210
c. $21,560
d. $-21,560
e. none of the above

You answered correctly!

20-13. Canada division's profit after tax using full costs plus a mark up of 20%
amounts to

a. $15,750
b. $19,600
c. $29,600
d. $35,350
e. none of the above

Your choice of "c" was incorrect. The correct answer is "b".

20-14. Tax for the company with the use of market price in transfer pricing amounts to

a. $20,900
b. $27,650
c. $34,650
d. $42,350
e. $48,650

Your choice of "c" was incorrect. The correct answer is "a".

20-15. Tax for the company with the use of full cost as transfer price amounts to
a. $20,900
b. $27,650
c. $34,650
d. $42,350
e. $48,650

Your choice of "b" was incorrect. The correct answer is "c".

20-16. Use this data to respond to questions 16 through 18. N&R Company transfers
a product from division N to division R. Variable cost of this product is anticipated to
be $40 a unit and total fixed costs amount to $8,000. A total of 100 units are
anticipated to be produceD. Actual cost, however, amounts to $50 for variable costs.
Fixed costs were same as budget. However, actual output was twice as many.
Budgeted cost per unit amounts to

a. $90
b. $92
c. $115
d. $120
e. none of the above

Your choice of "a" was incorrect. The correct answer is "d".

20-17. The transfer price based on budgeted variable costs plus 130% markup
amounts to

a. $90
b. $92
c. $115
d. $120
e. none of the above

Your choice of "c" was incorrect. The correct answer is "b".

20-18. The transfer price based on actual full cost plus 30% markup amounts to

a. $117
b. $140
c. $150
d. $156
e. none of the above

Your choice of "c" was incorrect. The correct answer is "a".

20-19. The forecasted market price was $150 a unit. The actual price was $140. It
was anticipated that marketing and packaging costs which general customers require
and internal transfer does not require amounts to $12. The actual non-required costs
per unit amounts to $11. The negotiated transfer price based on budget should be

a. $126
b. $129
c. $135
d. $138
e. none of the above
Your choice of "c" was incorrect. The correct answer is "d".

20-20. Transfer pricing is used in

a. centralized companies
b. only international companies
c. only domestic firms
d. only companies that need to minimize foreign taxation
e. none of the above

Your choice of "a" was incorrect. The correct answer is "e".

Close this window

To email a summary of your quiz score to your instructor, fill in the form and click the 'Email'
button. Both the 'FROM' and 'TO' fields must be completed. A CC copy of the email message
will also be sent to your address.
FROM: TO:

Your name: Your instructor's name:

Your email address: Your instructor's email address:

Optional comments:

Email the results


Exam Four - Sample Questions
Chapters 12-14

MULTIPLE CHOICE. Choose the one alternative that best completes the statement or answers the question.

1) What is the difference between perfect competition and monopolistic competition?


A) Perfect competition has a large number of small firms while monopolistic competition does not.
B) In perfect competition, firms produce identical goods, while in monopolistic competition, firms produce
slightly different goods.
C) Perfect competition has no barriers to entry, while monopolistic competition does.
D) Perfect competition has barriers to entry while monopolistic competition does not.

2) The market type known as perfect competition is


A) almost free from competition and firms earn large profits.
B) highly competitive and firms find it impossible to earn an economic profit in the long run.
C) dominated by fierce advertising campaigns.
D) marked by firms continuously trying to change their products so that consumers prefer their product to
their competitors' products.

3) Which of the following market types has all firms selling products so identical that buyers do not care from
which firm they buy?
A) perfect competition B) oligopoly
C) monopolistic competition D) monopoly

4) Perfect competition is characterized by all of the following EXCEPT


A) well-informed buyers and sellers with respect to prices.
B) a large number of buyers and sellers.
C) no restrictions on entry into or exit from the industry.
D) considerable advertising by individual firms.

5) Which of the following is the best example of a perfectly competitive market?


A) diamonds B) athletic shoes C) soft drinks D) farming

6) Which of the following market types has the fewest number of firms?
A) perfect competition B) monopoly
C) monopolistic competition D) oligopoly

7) Which of the following market types has a large number of firms that sell similar but slightly different
products?
A) perfect competition B) oligopoly
C) monopolistic competition D) monopoly

8) Which of the following market types has only a few competing firms?
A) perfect competition B) monopolistic competition
C) monopoly D) oligopoly

1
9) In a perfectly competitive market, the type of decision a firm has to make is different in the short run than in
the long run. Which of the following is an example of a perfectly competitive firm's short-run decision?
A) what price to charge buyers for the product
B) whether or not to enter or exit an industry
C) the profit-maximizing level of output
D) how much to spend on advertising and sales promotion

10) In perfect competition, a firm maximizes profit in the short run by deciding
A) how much output to produce. B) whether or not to enter a market.
C) what price to charge. D) how much capital to use.

11) In a perfectly competitive market, the type of decision a firm has to make is different in the short run than in
the long run. Which of the following is an example of a perfectly competitive firm's long-run decision?
A) what price to charge buyers for the product
B) how much to spend on advertising and sales promotion
C) the profit-maximizing level of output
D) whether or not to enter or exit an industry

12) A price-taking firm


A) cannot influence the price of the product it sells.
B) talks to rival firms to determine the best price for all of them to charge.
C) sets the product's price to whatever level the owner decides upon.
D) asks the government to set the price of its product.

13) A large number of sellers all selling an identical product implies which of the following?
A) horizontal market supply curves
B) large losses by all sellers
C) the inability of any seller to change the price of the product
D) market chaos

14) Perfectly competitive firms are price takers because


A) each firm is very large. B) there are no good substitutes for their goods.
C) many other firms produce identical products. D) their demand curves are downward sloping.

15) If demand for a seller's product is perfectly elastic, which of the following is correct?
A) There is no incentive to sell at a price below the market price.
B) It will not sell any output at all if it tries to price its product above the market price.
C) There are a very large number of perfect substitutes for the seller's product.
D) All of the above answers are correct.

2
16) One of the requirements for a monopoly is that
A) the product cannot be produced by small firms.
B) there are several close substitutes for the product.
C) there is a unique product with no close substitutes.
D) products are high priced.

17) A monopoly is a market with


A) no barriers to entry. B) many substitutes. C) many suppliers. D) one supplier.

18) Firms face competition when the good they produce


A) is in a market with natural barriers to entry. B) is unique.
C) is in a market with legal barriers to entry. D) has a close substitute.

19) Which of the following statements is correct?


A) The market demand and the firm's demand are the same for a monopoly.
B) Monopolies have perfectly inelastic demand for the product sold.
C) Monopolies are guaranteed to earn an economic profit.
D) All of the above are correct.

20) Which describes a barrier to entry?


A) anything that protects a firm from the arrival of new competitors
B) a government regulation that bars a monopoly from earning an economic profit
C) something that establishes a barrier to expanding output
D) firms already in the market incurring economic losses so that no new firm wants to enter the market

21) A barrier to entry is


A) an open door.
B) the economic term for diseconomies of scale.
C) illegal in most markets.
D) anything that protects a firm from the arrival of new competitors.

22) Which of the following would create a natural monopoly?


A) requirement of a government license before the firm can sell the good or service
B) technology enabling a single firm to produce at a lower average cost than two or more firms
C) an exclusive right granted to supply a good or service
D) ownership of all the available units of a necessary input

23) If the technology for producing a good enables one firm to meet the entire market demand at a lower price than
two or more firms could, then that firm has
A) a legal barrier to entry. B) a natural monopoly.
C) increasing average total costs. D) patented the market.

24) Which of the following goods is the best example of a natural monopoly?
A) natural gas B) diamonds C) a patented good D) first-class mail

3
25) Which of the following is the best example of a natural monopoly?
A) owning the only licensed taxicab in town
B) the United States Postal Service
C) ownership of the only ferry across Puget Sound for twenty miles
D) the cable television company in your hometown

26) Which barrier to entry is an exclusive right granted to the author or composer of a literary, musical, dramatic
or artistic work?
A) government license B) patent C) public franchise D) copyright

27) Patents
A) stimulate innovation.
B) encourage the invention of new products and production methods.
C) are exclusive rights granted to the inventor of a product or service.
D) All of the above answers are correct.

28) Which of the following is NOT correct about patents?


A) Patents stimulate innovation.
B) A patent is a barrier to entry.
C) Patents enable a firm to be a permanent monopoly.
D) Patents encourage invention of new products.

29) Recently in a small city, building contractors lobbied the city council to pass a law requiring all people
working on residential dwellings be licensed by the city. Why would the contractors lobby for this
requirement?
A) to reduce the cost of building dwellings
B) There is no good explanation for this type of lobbying.
C) to guarantee that work on dwellings is of high quality
D) to create a legal barrier to entry

30) Ownership of a necessary input creates what type of barrier to entry?


A) natural barrier to entry B) a public franchise
C) a government license D) legal barrier to entry

31) An industry with a large number of firms, differentiated products, and free entry and exit is called
A) oligopoly. B) monopoly.
C) monopolistic competition. D) perfect competition.

32) In monopolistic competition, each firm supplies a small part of the market. This occurs because
A) there are barriers to entry. B) firms produce differentiated products.
C) there are no barriers to entry. D) there are a large number of firms.

33) In monopolistic competition, the products of different sellers are assumed to be


A) similar but slightly different. B) identical perfect substitutes.
C) either identical or differentiated. D) unique without any close or perfect substitutes.

4
34) Which of the following is different about perfect competition and monopolistic competition?
A) Firms in monopolistic competition compete on their product's price as well as its quality and marketing.
B) In monopolistic competition, entry into the industry is unblocked.
C) Perfect competition has a large number of independently acting sellers.
D) Only firms in monopolistic competition can earn an economic profit in the short run.

35) In an industry with a large number of firms,


A) collusion is impossible.
B) one firm will dominate the market.
C) each firm will produce a large quantity, relative to market demand.
D) competition is eliminated.

36) Which of the following is an example of a monopolistically competitive industry?


A) wheat farming B) colleges and universities
C) the local electricity producer D) the domestic automobile producing industry

37) All of the following are examples of product differentiation in monopolistic competition EXCEPT
A) new and improved packaging.
B) lower price.
C) acceptance of more credit cards than the competition.
D) location of the retail store.

38) A differentiated product has


A) many perfect substitutes. B) close but not perfect substitutes.
C) no close substitutes. D) no substitutes of any kind.

39) As the degree of product differentiation increases among the products sold in a monopolistically competitive
industry, which of the following occurs?
A) The cost of production falls.
B) The amount of marketing expenditures decreases for each firm.
C) The demand curve for each seller's product becomes more horizontal.
D) Each seller's demand becomes more inelastic.

40) Marketing consists of what?


A) selling at a lower price than rivals sell for B) producing more output to lower average costs
C) advertising and packaging D) None of the above answers are correct.

41) Firms use marketing to


A) influence a consumer's buying decision.
B) convince customers that their product is worth its price.
C) persuade buyers that their product is superior to others.
D) All of the above answers are correct.

5
42) If a monopolistically competitive seller can convince buyers that its product is of better quality and value than
products sold by rival firms,
A) demand increases. B) the firm gains more control over its price.
C) demand becomes more inelastic. D) all of the above occur.

43) If you have found the percentage of the value of sales accounted for by the four largest firms in an industry, you
have found the
A) elasticity of supply value. B) Herfindahl-Hirschman Index.
C) elasticity of demand value. D) four-firm concentration ratio.

44) Which of the following four-firm concentration ratios would be the best indication of a perfectly competitive
industry?
A) 100 percent B) 78 percent C) 0.25 percent D) 31 percent

45) Which of the following four-firm concentration ratios is consistent with monopolistic competition?
A) 0 percent B) 25 percent C) 100 percent D) 75 percent

6
Answer Key
Testname: EXAM FOUR SAMPLE QUESTIONS.TST

1) Answer: B
2) Answer: B
3) Answer: A
4) Answer: D
5) Answer: D
6) Answer: B
7) Answer: C
8) Answer: D
9) Answer: C
10) Answer: A
11) Answer: D
12) Answer: A
13) Answer: C
14) Answer: C
15) Answer: D
16) Answer: C
17) Answer: D
18) Answer: D
19) Answer: A
20) Answer: A
21) Answer: D
22) Answer: B
23) Answer: B
24) Answer: A
25) Answer: D
26) Answer: D
27) Answer: D
28) Answer: C
29) Answer: D
30) Answer: D
31) Answer: C
32) Answer: D
33) Answer: A
34) Answer: A
35) Answer: A
36) Answer: B
37) Answer: B
38) Answer: B
39) Answer: D
40) Answer: C

1
Answer Key
Testname: EXAM FOUR SAMPLE QUESTIONS.TST

41) Answer: D
42) Answer: D
43) Answer: D
44) Answer: C
45) Answer: B

2
Sample Questions on the chapter Factor Pricing for 4th Semester.

MULTIPLE CHOICE QUESTIONS.SET :1

Choose the one alternative that best completes the statement

1) What is meant by derived demand? A) The demand is derived in beginning economics classes.

B) The demand is derived from income, tastes, etc. of sellers.

C) The demand is derived from the price elasticity of demand.

D) The demand for a resource is derived from the demand for what it can
produce.

2) The value of marginal product is calculated by multiplying the ____ by the ____.

A) marginal product; price of the product

B) quantity of labor; wage rate

C) wage; price of the product

D) total product; wage rate

3) To decide the quantity of a factor of production to hire, a firm compares the ____ to the ____.

A) factor's price; average factor price

B) factor's price; value of marginal product

C) factor's output; average factor output

D) factor's output; value of marginal product

4) A profit-maximizing firm hires labor up to the point where

A) the wage times the quantity of labor equals the marginal product.

B) price equals the wage.

C) price equals the quantity of labor.

D) the wage equals value of marginal product.

5) A company finds that the value of marginal product for the current level of employment is Rs.530 and the wage rate is
Rs.450. Which of the following is correct?

A) Too much labor is currently employed and the firm will lay off some
workers.

B) The firm should shut down and produce nothing.

C) The firm is employing the profit-maximizing quantity of labor.


D) Too few workers are currently employed and the firm will hire more
workers.

6) A firm's demand for labor curve is also –

A) its value of marginal product curve.

B) the supply of labor curve.

C) the demand curve for the good it produces.

D) its marginal cost curve.

7) For a firm selling its product in a competitive market, what causes the demand curve for any factor of production to be
downward sloping?

A) diminishing marginal product as more of the factor is used

B) more demand for greater quantities

C) falling product price necessary to sell a larger output

D) None of the above answers are correct.

8) If the price of a firm's product falls, the firm's demand for labor curve

A) shifts leftward.

B) does not shift and there is no movement along it.

C) does not shift but there is a movement along it.

D) shifts rightward.

9) If the wage rate increases, a firm's demand for labor curve

A) shifts leftward.

B) does not shift and there is no movement along it.

C) shifts rightward.

D) does not shift but there is a movement along it.

10) An increase in the price of the firm's output leads to a

A) movement upward along the demand for labor curve.

B) rightward shift of the demand for labor curve.

C) movement downward along the demand for labor curve.

D) leftward shift of the demand for labor curve.

Answer Key :1) Answer: D 2) Answer: A 3) Answer: B 4) Answer: D 5) Answer: D 6) Answer: A 7) Answer: A 8) Answer: A 9)
Answer: D 10) Answer: B
Multiple-Choice Questions SET 2:

1. The marginal physical product of capital is the addition to

a. total revenue generated by adding a unit of loanable funds to production

b. total physical product generated by adding a unit of capital to production

c. total revenue generated by adding a unit of capital to production

d. total physical product divided by a unit of capital

2. A unit of capital equipment, once employed,

a. generates a rate of return equal to the marginal physical product of capital

b. is unalterable in the short run

c. can be altered only by borrowing more loanable funds

d. completely substitutes for labor

3. To economists, rent is the

a. amount of money paid to occupy an apartment, house, or other fixed property

b. difference between what a person pays for something and what it is worth

c. difference between what a resource receives in payment and the cost of bringing the resource into production

d. difference between what a resource is paid in the market and what the resource would be paid if it received the full value of
its marginal revenue product

4. If one argues that land rent is strictly demand determined, then the underlying assumption must be that

a. the supply of land is fixed and of the same quality

b. the supply of land is variable and of unequal quality

c. the demand for land is perfectly elastic and increasing

d. the demand for land is perfectly inelastic and falling

5. The calculation of ________________ is based on the assumption that land is a free good, whereas _____________________
is calculated based on the assumption that the cost of bringing land into production varies.

a. location rent; differential rent

b. differential land rent; land rent

c. differential land rent; location rent

d. land rent; differential land rent

6. Location rent arises because land that is located closer to the market has

a. higher fertility than land farther away


b. lower transportation costs than land farther away

c. easier access to important breakthroughs in technology

d. access to a better labor force

7. Implicit costs of an entrepreneur's labor time and capital refer to

a. the entrepreneur's opportunity costs

b. the total costs incurred by an entrepreneur in doing business

c. the entrepreneurial risks associated with a business venture

d. normal plus economic profit

8. If land is situated so that it earns zero location rent, then the

a. land must be of lower quality than other acres that earn location rent

b. wages of workers farming the land must be higher than average

c. supply price of the land is equal to the transportation costs

d. cost of transporting goods to market is zero

9. The present value of a property can be calculated as the

a. price at which quantity demanded of the property equals quantity supplied

b. minimum price set by an auctioneer at which bidding is to start

c. annual income generated by the property divided by the interest rate

d. annual income generated by the property multiplied by the interest rate

10. As a country’s population increases, we expect that

a. the supply price of land will decrease

b. land rent will increase, but differential land rent and location rent will decrease

c. land rent, differential land rent, and location rent will decrease

d. land rent, differential land rent, and location rent will increase

Answers of Multiple-Choice Questions :1.b,2 b,3c,4a,5d,6b,7a,8c,9c,10d.


Test Name : CA-3 Test ECO531 Q2045 and Q2042
Name : Dhawal Raj - 12009740.st@lpu.in
Test Start Time Marks Scored Test Status
2021-02-08 10:40:21 18.0 / 30.0 Passed

Total Questions Attempted Questions Correct Questions

30 24 18

Incorrect Questions Skipped Questions Pending Evaluation

6 6 0

List of Sections

Section 1 Marks per question : 1 Marks Scored : 18.0

Q No. Q. Type Status Marks Comment  

1 1.0 - Hide Answer


Multiple 
Choice -
Single
Answer

Monopolistically competitive firms most frequently do which of the following?

Compete in pricing wars with other firms in the industry

 Enjoy monopoly pricing power

Merge and consolidate into oligopolistic groupings

Advertise the traits that make their product identifiable

2 0.0 - Hide Answer


Multiple 
Choice -
Single
Answer

Oligopolies can end up looking like competitive markets if the number of firms is 

small and they all cooperate 

large and they all cooperate 

large and they do not cooperate

small and they do not cooperate 


3 1.0 - Hide Answer
Multiple 
Choice -
Single
Answer

In perfect competition, the elasticity of demand for the product of a single firm is

infinite, because many other firms produce identical products.

zero, because the firm produces a unique product.

infinite, because the firm produces a unique product.

zero, because many other firms produce identical products.

4 1.0 - Hide Answer


Multiple 
Choice -
Single
Answer

In an industry with a large number of firms.

collusion is impossible. 

each firm will produce a large quantity, relative to market demand.

competition is eliminated.

one firm will dominate the market.

5 1.0 - Hide Answer


Multiple 
Choice -
Single
Answer

If a monopolistically competitive seller can convince buyers that its product is of better quality and value than products sold by rival firms.

demand increases.

the firm gains more control over its price.

demand becomes more inelastic.

all of the above occur.

6 1.0 - Hide Answer


Multiple 
Choice -
Single
Answer
 A monopoly is able to make greater profits than a perfectly competitive firm because:

the monopolist fixes prices in cooperation with its rivals

antitrust legislation protects the monopolist from regulation

monopolistic firms are always larger than competitive firms

barriers to entry in the monopolist's market prevent competition

7 1.0 - Hide Answer


Multiple 
Choice -
Single
Answer

Which of the following industries is most likely to be monopolistically competitive?

The automobile industry 

The airline industry 

 Restaurant industry

The steel industry 

8 1.0 - Hide Answer


Multiple 
Choice -
Single
Answer

A pricing practice that requires buyers to purchase packages of different goods and does not make the goods available separately is called

           

value pricing.

a two-part tariff.
skimming.

 bundling.

9 1.0 - Hide Answer


Multiple 
Choice -
Single
Answer

If an imperfectly competitive firm is producing a level of output where marginal cost is equal to marginal revenue, marginal revenue is below average
variable cost, and price is equal to average total cost, then the firm:

should shut down

should decrease output, but should not shut down

should increase output

None of the above is appropriate

10 1.0 - Hide Answer


Multiple 
Choice -
Single
Answer

 A firm that is selling a product at or below cost on foreign markets in order to drive foreign producers out of business is engaging in:

 international price discrimination.

predatory dumping.

sporadic dumping

persistent dumping.

11 1.0 - Hide Answer


Multiple 
Choice -
Single
Answer

Which of the following best describes an oligopoly?


  many monopolistically competitive firms

 a former monopoly that has been broken up by

 a government-granted franchise or monopoly

  a few firms sharing monopoly power

12 1.0 - Hide Answer


Multiple 
Choice -
Single
Answer

 A cartel that gives each member the exclusive right to operate in a particular geographic area is :

        

market-sharing cartel

centralized cartel.

price leadership cartel.

None of the above is correct

13 1.0 - View Answer


Multiple 
Choice -
Single
Answer

Which describes a barrier to entry?  

anything that protects a firm from the arrival of new competitors 

something that establishes a barrier to expanding output

 firms already in the market incurring economic losses so that no new firm wants to enter the market

a government regulation that bars a monopoly from earning an economic profit

14 0.0 - Hide Answer


Multiple 
Choice -
Single
Answer
Answer

Which of the following is a form of non-price competition?

Advertising

Quality of service

Product quality

All of the above 

15 0.0 - Hide Answer


Multiple 
Choice -
Single
Answer

A firm that is engaging in price discrimination will            

charge a higher price to consumers with a higher price elasticity of demand.

earn lower profits than a similar firm that does not engage in price discrimination.

generally be a perfectly competitive firm.

charge a higher price to consumers with a lower price elasticity of demand.

16 0.0 - Hide Answer


Multiple 
Choice -
Single
Answer

Which of the following affect the success of the cartelization?

          

Number of firms in the industry

Cost structure

All of these

Nature of product

17 0.0 - Hide Answer


Multiple 
Choice -
Single
Single
Answer

As the degree of product differentiation increases among the products sold in a monopolistically competitive industry, which of the following occurs?

The cost of production falls.

The demand curve for each seller's product becomes more horizontal.

Each seller's demand becomes more inelastic.

The amount of marketing expenditures decreases for each firm.

18 0.0 - Hide Answer


Multiple 
Choice -
Single
Answer

 Under loss leader pricing agreement it requires buyers of a product to

            

  

purchase another product needed in the use of the first product.

purchase a minimum number of units.

 refrain from exporting the product to certain countries.

All of the above are true of a tying agreement.

19 1.0 - Hide Answer


Multiple 
Choice -
Single
Answer

Setting a high price when a product is first introduced and then gradually lowering its price over time is referred to as

          

value pricing.

price lining.

prestige pricing.

skimming.

20 1.0 - Hide Answer


Multiple 
Choice -
Single
Answer

A Monopolistically competitive firm advertises in order to:      

Shift the demand curve for its product to the left

Make its product more similar to its competitors

Increases externalities in the long run.

Make the demand for its product less price elastic

21 1.0 - Hide Answer


Multiple 
Choice -
Single
Answer

The kinked demand curve model assumes that

firms match price increases, but not price cut

demand is more elastic for price cuts than for price increases

changes in marginal cost can never lead to changes in market price

None of the above is correct

22 1.0 - Hide Answer


Multiple 
Choice -
Single
Answer

Recently in a small city, building contractors lobbied the city council to pass a law requiring all people working on residential dwellings be licensed by
the city. Why would the contractors lobby for this requirement?

to reduce the cost of building dwellings

to guarantee that work on dwellings is of high quality

to create a legal barrier to entry

There is no good explanation for this type of lobbying.

23 0.0 - Hide Answer


Multiple 
Choice -
Multiple
Answers
Which of the following statements is true for both monopolistically competitive and oligopolistic industries?

It is impossible for new firms to enter the industries.

Producers cannot benefit from knowing other firms' plans.

Firms have some degree of control over prices.

Collusion and the creation of cartels is common for both.

24 1.0 - Hide Answer


Multiple 
Choice -
Single
Answer

Which of the following characterizes monopolistic competition?

Many interdependent firms sell a homogeneous product.

 Many firms produce a particular type of product, but each maintains some independent control over its own price. 

A few firms produce all of the market supply of a good. 

A few firms produce a particular type of product.

25 1.0 - Hide Answer


Multiple 
Choice -
Single
Answer

In perfect competition, the marginal revenue of an individual firm

equals the price of the product.

exceeds the price of the product.

is zero

is positive but less than the price of the product.

26 0.0 - Hide Answer


Multiple 
Choice -
Single
Answer

If an industry is comprised of four firms and their market shares are 40% 30% 20% and 10% then such firms will fall under which market
If an industry is comprised of four firms and their market shares are 40%, 30%, 20%, and 10%, then  such firms will fall under which market
formation: 

 Monopoly

 Monopolistic competition

 Oligopoly

 Perfect competition

27 0.0 - Hide Answer


Multiple 
Choice -
Single
Answer

If the market demand curve for a commodity has a negative slope then the market structure must be

perfect competition.

imperfect competition.

The market structure cannot be determined from the information given.

monopoly.

28 0.0 - Hide Answer


Multiple 
Choice -
Single
Answer

 According to the kinked demand curve model, a firm will assume that rival firms will

keep their rates of production constant

 match price cuts but not price increases. 

match price increases but not price cuts.

keep their prices constant

29 0.0 - Hide Answer


Multiple 
Choice -
Single
Answer

 A differentiated product has

many perfect substitutes.

no close substitutes.

no substitutes of any kind.

close but not perfect substitutes.

30 0.0 - Hide Answer


Multiple 
Choice -
Single
Answer

 A pricing practice that involves charging a fixed fee plus a per unit price for a good or service is referred to as       

bundling.

a two-part tariff.

 first degree price discrimination.

skimming.

©
Name Econ Unit 3 Test

Part 1: Multiple Choice: Choose the best answer for each question (2 points each)
1. Which of the following statements is true for both monopolistically 9. Which of the following would most likely be a monopoly?
competitive and oligopolistic industries?
A) An appliance store B) A supermarket
A) It is impossible for new firms to enter the industries. C) An electricity provider D) A dentist's office
B) Collusion and the creation of cartels is common. 10. Monopolistically competitive firms most frequently do which of the
C) Producers cannot benefit from knowing other firms' plans. following?
D) Firms have some degree of control over prices.
A) Compete in pricing wars with other firms in the industry
2. Which of the following best describes an oligopoly?
B) Advertise the traits that make their product identifiable
A) many monopolistically competitive firms C) Enjoy monopoly pricing power
B) a few firms sharing monopoly power D) Merge and consolidate into oligopolistic groupings
C) a former monopoly that has been broken up by the government 11. When the government grants an exclusive patent to one firm, that
D) a government-granted franchise or monopoly firm enjoys
3. Collusion most frequently occurs in industries that are A) productive efficiency B) antitrust regulation
A) oligopolistic C) monopoly powers D) collusive prices
B) monopolistically competitive 12. Which of these could be considered a government-created barrier to
C) monopolistic market entry?
D) perfectly competitive
A) Discretionary spending
4. If a few firms share most of an entire industry's revenues, the market B) Antitrust legislation
structure is most likely C) Patents and copyrights
A) monopolistically competitive D) Progressive income taxes
B) an oligopoly 13. The basic difference between copyrights and patents is that
C) perfectly competitive
A) copyrights never expire, while patents always have an
D) a monopoly
expiration date
5. All of the following can help break a monopoly EXCEPT B) copyrights are used to protect ideas, while patents protect
A) increased barriers to entry processes and products
B) changing consumer tastes C) copyrights are awarded by private entities, while patents are
awarded by the government
C) international competition
D) copyrights always last longer than patents
D) technology and innovation
14. A patent is given to a firm to protect that firm's
6. Which of the following best describes a successful monopolist?
A) monopoly power over a new process or product
A) The only buyer of a resource or type of labor
B) research and development investment
B) The only seller of a difficult-to-substitute product
C) ability to profit from its discoveries
C) The only buyer of a consumer product
D) all of the above
D) The only seller of a non-essential product
15. If a proprietorship fails, who is responsible for the firm's debts?
7. A monopoly is able to make greater profits than a perfectly
competitive firm because A) The workers B) The sole proprietor
A) the monopolist fixes prices in cooperation with its rivals C) The shareholders D) The public
B) barriers to entry in the monopolist's market prevent 16. Which of the following is a disadvantage faced by sole proprietors?
competition
A) Limited decision-making power
C) antitrust legislation protects the monopolist from regulation
B) Full responsibility for the business' debts
D) monopolistic firms are always larger than competitive firms
C) Demands placed on them by shareholders
8. Which of the following is most likely to be observed in a D) "Double taxation" of profits through corporate and dividend
monopolistically competitive market? taxes
A) Standardized, homogenous products 17. A proprietorship shared between multiple people is known as
B) Collusion and price-fixing between firms
A) a partnership B) a franchise
C) Government antitrust oversight
C) an oligopoly D) a conglomerate
D) Non-price competition, such as advertising

Page 1 Version 1
18. In general, a company whose shareholders collect dividends must 23. A vertical merger is
be
A) the combination of two firms that specialize in different
A) publicly traded B) a private bank stages of the same supply chain
C) taking a loss D) earning a profit B) the combination of two firms from completely unrelated
19. A corporation distributes profits to its many part-owners by industries
C) a combination of two firms that were previously in
A) raising the interest rate B) offering stock competition with each other
C) paying dividends D) issuing bonds D) a combination of two firms that are not corporations
20. Which of the following organizations is most likely to sell stock? 24. If a corporation that makes rubber combines with a corporation that
A) Sole proprietorship B) Partnership makes tires, this action would most likely be considered
C) Corporation D) Labor union A) a horizontal merger B) a conglomerate merger
21. A business that is related to an established corporation and shares C) a stock merger D) a vertical merger
the same trademarks, brands, and business model is known as a 25. The personal income earned by the sale of stock is known as
A) merger B) franchise A) preferred stock B) dividends
C) proprietorship D) entrepreneurship C) mutual fund income D) capital gains
22. Firm X and Firm Y were previously in direct competition, but now
they plan to merge. This combination would be considered a
A) horizontal merger B) vertical merger
C) complementary merger D) conglomerate merger

Page 2 Version 1
Answer Key
Unit 3 2015

1. D
2. B
3. A
4. B
5. A
6. B
7. B
8. D
9. C
10. B
11. C
12. C
13. B
14. D
15. B
16. B
17. A
18. D
19. C
20. C
21. B
22. A
23. A
24. D
25. D

Page 3 Version 1
Exam Four - Sample Questions
Chapters 12-14

MULTIPLE CHOICE. Choose the one alternative that best completes the statement or answers the question.

1) What is the difference between perfect competition and monopolistic competition?


A) Perfect competition has a large number of small firms while monopolistic competition does not.
B) In perfect competition, firms produce identical goods, while in monopolistic competition, firms produce
slightly different goods.
C) Perfect competition has no barriers to entry, while monopolistic competition does.
D) Perfect competition has barriers to entry while monopolistic competition does not.

2) The market type known as perfect competition is


A) almost free from competition and firms earn large profits.
B) highly competitive and firms find it impossible to earn an economic profit in the long run.
C) dominated by fierce advertising campaigns.
D) marked by firms continuously trying to change their products so that consumers prefer their product to
their competitors' products.

3) Which of the following market types has all firms selling products so identical that buyers do not care from
which firm they buy?
A) perfect competition B) oligopoly
C) monopolistic competition D) monopoly

4) Perfect competition is characterized by all of the following EXCEPT


A) well-informed buyers and sellers with respect to prices.
B) a large number of buyers and sellers.
C) no restrictions on entry into or exit from the industry.
D) considerable advertising by individual firms.

5) Which of the following is the best example of a perfectly competitive market?


A) diamonds B) athletic shoes C) soft drinks D) farming

6) Which of the following market types has the fewest number of firms?
A) perfect competition B) monopoly
C) monopolistic competition D) oligopoly

7) Which of the following market types has a large number of firms that sell similar but slightly different
products?
A) perfect competition B) oligopoly
C) monopolistic competition D) monopoly

8) Which of the following market types has only a few competing firms?
A) perfect competition B) monopolistic competition
C) monopoly D) oligopoly

1
9) In a perfectly competitive market, the type of decision a firm has to make is different in the short run than in
the long run. Which of the following is an example of a perfectly competitive firm's short-run decision?
A) what price to charge buyers for the product
B) whether or not to enter or exit an industry
C) the profit-maximizing level of output
D) how much to spend on advertising and sales promotion

10) In perfect competition, a firm maximizes profit in the short run by deciding
A) how much output to produce. B) whether or not to enter a market.
C) what price to charge. D) how much capital to use.

11) In a perfectly competitive market, the type of decision a firm has to make is different in the short run than in
the long run. Which of the following is an example of a perfectly competitive firm's long-run decision?
A) what price to charge buyers for the product
B) how much to spend on advertising and sales promotion
C) the profit-maximizing level of output
D) whether or not to enter or exit an industry

12) A price-taking firm


A) cannot influence the price of the product it sells.
B) talks to rival firms to determine the best price for all of them to charge.
C) sets the product's price to whatever level the owner decides upon.
D) asks the government to set the price of its product.

13) A large number of sellers all selling an identical product implies which of the following?
A) horizontal market supply curves
B) large losses by all sellers
C) the inability of any seller to change the price of the product
D) market chaos

14) Perfectly competitive firms are price takers because


A) each firm is very large. B) there are no good substitutes for their goods.
C) many other firms produce identical products. D) their demand curves are downward sloping.

15) If demand for a seller's product is perfectly elastic, which of the following is correct?
A) There is no incentive to sell at a price below the market price.
B) It will not sell any output at all if it tries to price its product above the market price.
C) There are a very large number of perfect substitutes for the seller's product.
D) All of the above answers are correct.

2
16) One of the requirements for a monopoly is that
A) the product cannot be produced by small firms.
B) there are several close substitutes for the product.
C) there is a unique product with no close substitutes.
D) products are high priced.

17) A monopoly is a market with


A) no barriers to entry. B) many substitutes. C) many suppliers. D) one supplier.

18) Firms face competition when the good they produce


A) is in a market with natural barriers to entry. B) is unique.
C) is in a market with legal barriers to entry. D) has a close substitute.

19) Which of the following statements is correct?


A) The market demand and the firm's demand are the same for a monopoly.
B) Monopolies have perfectly inelastic demand for the product sold.
C) Monopolies are guaranteed to earn an economic profit.
D) All of the above are correct.

20) Which describes a barrier to entry?


A) anything that protects a firm from the arrival of new competitors
B) a government regulation that bars a monopoly from earning an economic profit
C) something that establishes a barrier to expanding output
D) firms already in the market incurring economic losses so that no new firm wants to enter the market

21) A barrier to entry is


A) an open door.
B) the economic term for diseconomies of scale.
C) illegal in most markets.
D) anything that protects a firm from the arrival of new competitors.

22) Which of the following would create a natural monopoly?


A) requirement of a government license before the firm can sell the good or service
B) technology enabling a single firm to produce at a lower average cost than two or more firms
C) an exclusive right granted to supply a good or service
D) ownership of all the available units of a necessary input

23) If the technology for producing a good enables one firm to meet the entire market demand at a lower price than
two or more firms could, then that firm has
A) a legal barrier to entry. B) a natural monopoly.
C) increasing average total costs. D) patented the market.

24) Which of the following goods is the best example of a natural monopoly?
A) natural gas B) diamonds C) a patented good D) first-class mail

3
25) Which of the following is the best example of a natural monopoly?
A) owning the only licensed taxicab in town
B) the United States Postal Service
C) ownership of the only ferry across Puget Sound for twenty miles
D) the cable television company in your hometown

26) Which barrier to entry is an exclusive right granted to the author or composer of a literary, musical, dramatic
or artistic work?
A) government license B) patent C) public franchise D) copyright

27) Patents
A) stimulate innovation.
B) encourage the invention of new products and production methods.
C) are exclusive rights granted to the inventor of a product or service.
D) All of the above answers are correct.

28) Which of the following is NOT correct about patents?


A) Patents stimulate innovation.
B) A patent is a barrier to entry.
C) Patents enable a firm to be a permanent monopoly.
D) Patents encourage invention of new products.

29) Recently in a small city, building contractors lobbied the city council to pass a law requiring all people
working on residential dwellings be licensed by the city. Why would the contractors lobby for this
requirement?
A) to reduce the cost of building dwellings
B) There is no good explanation for this type of lobbying.
C) to guarantee that work on dwellings is of high quality
D) to create a legal barrier to entry

30) Ownership of a necessary input creates what type of barrier to entry?


A) natural barrier to entry B) a public franchise
C) a government license D) legal barrier to entry

31) An industry with a large number of firms, differentiated products, and free entry and exit is called
A) oligopoly. B) monopoly.
C) monopolistic competition. D) perfect competition.

32) In monopolistic competition, each firm supplies a small part of the market. This occurs because
A) there are barriers to entry. B) firms produce differentiated products.
C) there are no barriers to entry. D) there are a large number of firms.

33) In monopolistic competition, the products of different sellers are assumed to be


A) similar but slightly different. B) identical perfect substitutes.
C) either identical or differentiated. D) unique without any close or perfect substitutes.

4
34) Which of the following is different about perfect competition and monopolistic competition?
A) Firms in monopolistic competition compete on their product's price as well as its quality and marketing.
B) In monopolistic competition, entry into the industry is unblocked.
C) Perfect competition has a large number of independently acting sellers.
D) Only firms in monopolistic competition can earn an economic profit in the short run.

35) In an industry with a large number of firms,


A) collusion is impossible.
B) one firm will dominate the market.
C) each firm will produce a large quantity, relative to market demand.
D) competition is eliminated.

36) Which of the following is an example of a monopolistically competitive industry?


A) wheat farming B) colleges and universities
C) the local electricity producer D) the domestic automobile producing industry

37) All of the following are examples of product differentiation in monopolistic competition EXCEPT
A) new and improved packaging.
B) lower price.
C) acceptance of more credit cards than the competition.
D) location of the retail store.

38) A differentiated product has


A) many perfect substitutes. B) close but not perfect substitutes.
C) no close substitutes. D) no substitutes of any kind.

39) As the degree of product differentiation increases among the products sold in a monopolistically competitive
industry, which of the following occurs?
A) The cost of production falls.
B) The amount of marketing expenditures decreases for each firm.
C) The demand curve for each seller's product becomes more horizontal.
D) Each seller's demand becomes more inelastic.

40) Marketing consists of what?


A) selling at a lower price than rivals sell for B) producing more output to lower average costs
C) advertising and packaging D) None of the above answers are correct.

41) Firms use marketing to


A) influence a consumer's buying decision.
B) convince customers that their product is worth its price.
C) persuade buyers that their product is superior to others.
D) All of the above answers are correct.

5
42) If a monopolistically competitive seller can convince buyers that its product is of better quality and value than
products sold by rival firms,
A) demand increases. B) the firm gains more control over its price.
C) demand becomes more inelastic. D) all of the above occur.

43) If you have found the percentage of the value of sales accounted for by the four largest firms in an industry, you
have found the
A) elasticity of supply value. B) Herfindahl-Hirschman Index.
C) elasticity of demand value. D) four-firm concentration ratio.

44) Which of the following four-firm concentration ratios would be the best indication of a perfectly competitive
industry?
A) 100 percent B) 78 percent C) 0.25 percent D) 31 percent

45) Which of the following four-firm concentration ratios is consistent with monopolistic competition?
A) 0 percent B) 25 percent C) 100 percent D) 75 percent

6
Answer Key
Testname: EXAM FOUR SAMPLE QUESTIONS.TST

1) Answer: B
2) Answer: B
3) Answer: A
4) Answer: D
5) Answer: D
6) Answer: B
7) Answer: C
8) Answer: D
9) Answer: C
10) Answer: A
11) Answer: D
12) Answer: A
13) Answer: C
14) Answer: C
15) Answer: D
16) Answer: C
17) Answer: D
18) Answer: D
19) Answer: A
20) Answer: A
21) Answer: D
22) Answer: B
23) Answer: B
24) Answer: A
25) Answer: D
26) Answer: D
27) Answer: D
28) Answer: C
29) Answer: D
30) Answer: D
31) Answer: C
32) Answer: D
33) Answer: A
34) Answer: A
35) Answer: A
36) Answer: B
37) Answer: B
38) Answer: B
39) Answer: D
40) Answer: C

1
Answer Key
Testname: EXAM FOUR SAMPLE QUESTIONS.TST

41) Answer: D
42) Answer: D
43) Answer: D
44) Answer: C
45) Answer: B

2
ec2ex1.rvw

Practice Questions

Economics 120 M. Babcock


Exam 1 Spring 2007

1. The law of increasing costs states that:


a) if the prices of all the resources devoted to the production of goods increase, the
cost of producing any particular good will increase at the same rate.
b) if the sum of the costs of producing a particular good rises by a specified
percentage, the price of that good must rise by a greater relative amount.
c) the sum of the costs of producing a particular good cannot rise above the current
market price of that good.
d) if society wants to produce more of a particular good, it must sacrifice larger and
larger amounts of other goods to do so.

2. The fundamental problem of economics is:


a) to achieve a more equitable distribution of money income in order to mitigate
poverty.
b) the scarcity of productive resources relative to material wants.
c) the establishment of prices which accurately reflect the relative scarcities of
products and resources.
d) to establish a democratic political framework for the provision of social goods and
services.
e) to establish an equitable system of personal and business taxation.

3. Assume that a change in government policy results in the increased production of both
consumer goods and investment goods. It can be concluded that:
a) the law of increasing costs does not apply in this society.
b) this economy's production possibilities curve is convex (bowed inward) as viewed
from the origin.
c) the economy's production possibilities curve has been shifted to the left as a result
of the policy decision.
d) the economy was suffering from unemployment and/or the inefficient use of
resources before the policy change.

4. Which of the following is not an illustration of the notion of opportunity cost?


a) The production of more military goods means fewer resources are available for
civilian goods.
b) The land a Kansas farmer plants in wheat is not available for corn production.
c) Resources devoted to consumer goods production are not available for capital
goods production.
d) If I buy a pizza, I will not be able to afford a movie.
e) A growing economy can produce more consumer goods and more capital goods at
the same time.
5. Which of the following will tend to shift the production possibilities curve to the right?
a) an increase in the unemployment rate from 6 to 8 percent
b) a decline in the efficiency with which the present labor force is allocated
c) a decrease in the unemployment rate from 8 to 6 percent
d) a technological advance which allows farmers to produce more output from given
inputs
e) none of the above

6. Which of the following statements is incorrect?


a) If supply declines and demand remains constant, equilibrium price will rise.
b) If demand decreases and supply increases, equilibrium price will rise.
c) If supply increases and demand decreases, equilibrium price will fall.
d) If demand increases and supply decreases, equilibrium price will rise.

7. In which of the following instances will the effect upon equilibrium price be
indeterminant, that is, dependent upon the magnitude of the given shifts in supply and
demand?
a) demand rises and supply is constant
b) supply rises and demand falls
c) demand rises and supply falls
d) supply falls and demand remains constant
e) demand rises and supply rises

8. If the supply and demand curves for a product both decrease, we can say that equilibrium:

a) quantity and equilibrium price must both decline.


b) quantity must decline, but equilibrium price may either rise, fall, or remain
unchanged.
c) price must fall, but equilibrium quantity may either rise, fall, or remain
unchanged.
d) quantity must fall and equilibrium price must rise.

9. If the supply of a product decreases and the demand for that product simultaneously
increases, we can conclude that equilibrium:
a) quantity must decrease, but equilibrium price may either rise, fall, or remain
unchanged.
b) price and equilibrium quantity must both decline.
c) price and equilibrium quantity must both increase.
d) price must rise and equilibrium quantity must fall.
e) price must rise, but equilibrium quantity may either rise, fall, or remain
unchanged.
10. Assuming competitive markets with typical supply and demand curves, which of the
following statements is correct?
a) An increase in demand with no change in supply will result in a decline in sales.
b) An increase in demand with no change in supply will result in an increase in sales.

c) An increase in supply with no change in demand will result in a decline in sales.


d) An increase in supply with no change in demand will result in an increase in price.
e) An increase in supply with a decrease in demand will result in an increase in
price.

11. In moving along a stable demand curve which of the following is NOT held constant?
a) the prices of substitute goods
b) the prices of complementary goods
c) consumer incomes
d) price expectations
e) the price of the product for which the demand curve is relevant

12. In moving along a stable supply curve which of the following is NOT held constant?
a) the prices of the resources used in producing this good
b) the price of the product for which the supply curve is relevant
c) techniques used in producing this product
d) expectations about the future price of the product
e) the number of firms producing this good

13. If products A and B are complements and the price of B decreases:


a) the demand for A will decline and the demand for B will increase.
b) the demand for A will increase and the amount of B demanded will increase.
c) the amount of B purchased will increase, but the demand curve for A will not
shift.
d) the demand curves for both A and B will shift to the left.

14. If products C and D are close substitutes, an increase in the price of C will:
a) shift the demand curves of both products to the left.
b) shift the demand curves of both products to the right.
c) shift the demand curve of D to the right.
d) shift the demand curve of C to the left and the demand curve of D to the right.
e) tend to cause the price of D to fall.

15. Which of the following is NOT a basic criticism of the price system?
a) The price system limits individual freedom by imposing resource and product
prices which buyers and sellers cannot control.
b) The price system does not account for external costs and benefits.
c) The price system is permissive of recession and/or inflation.
d) Income inequality causes the price system to result in the production of luxuries
for the rich at the expense of necessities for the poor.
Resource Possible production techniques
Resource Prices #1 #2 #3 #4 #5

Land $4 2 4 2 4 4
Labor 3 1 2 4 1 3
Capital 3 5 2 3 1 2
Entrepreneurial ability 2 3 1 1 4 1

Use this figure to answer question 16

The figure illustrates all available techniques by which 20 units of a given commodity can
be produced.

16. Given the indicated resource prices, the economically most efficient production
technique(s) will be technique(s):
a) #1.
b) #2 and #4.
c) #3.
d) #1 and #3.
e) #5.

17. From society's point of view the economic function of profits and losses is to:
a) reallocate resources from less-desired to more-desired uses.
b) contribute to a more equal distribution of income.
c) perpetuate full employment and price level stability.
d) promote the equal distribution of real assets and wealth.

18. One drawback to the proprietorship form of business organization is:


a. lack of personal discretion in management.
b. separation of ownership and management.
c. limited access to capital.
d. stifling of individual initiative.

19. Partnership is an improvement over proprietorship in that:


a. partnerships are not subject to unlimited liability.
b. partnerships provide a stable, unchanging form of organization.
c. partnerships bring in added capital and expertise.
d. decision making in a partnership is smoother and quicker.

20. The primary advantage of a corporation is:


a. limited liability.
b. double taxation.
c. the closeness between ownership and management.
d. limited access to resources.
26. The opportunity cost of moving from point A to point B is:
a. 18 million doughnuts.
b. 2 million doughnuts.
c. 6 million glasses of milk.
d. 1 million glasses of milk.

27. If an economy uses inefficient methods of production, then:


a. the economy is sacrificing goods and services it could otherwise have.
b. the economy will need a larger government sector.
c. the economy at least will have more people employed.
d. the economy will be on its production possibilities curve, but will be producing
primarily one good or the other rather than nearly equal amounts of each good.

28. Increasing opportunity cost occurs because:


a. workers become less productive as they work longer hours.
b. resources are specialized.
c. prices rise as additional output is produced.
d. unemployment falls as output increases.

29. Which of the following best expresses the law of increasing costs ?
a. The more units of a good someone buys, the more money that must be paid for
them.
b. The more units of a good someone buys, the higher will be the price charged.
c. The more units of a good a country produces, the greater is the opportunity cost of
an extra unit.
d. The more resources a country has, the more costly will be imports.

30. How do consumers signal firms to increase production of a good?


a. by bidding up the price of the good.
b. by contacting government agencies.
c. by writing letters to the company.
d. by voting for political candidates of their choice.

31. Other things being equal, when the price of a good rises:
a. quantity demanded will fall.
b. quantity demanded will rise.
c. demand will decrease.
d. demand will increase.
Chapter 4 - Elasticity - Sample Questions

MULTIPLE CHOICE. Choose the one alternative that best completes the statement or answers the question.

1) The slope of a demand curve depends on 1)


A) the units used to measure quantity but not the units used to measure price.
B) the units used to measure price and the units used to measure quantity.
C) the units used to measure price but not the units used to measure quantity.
D) neither the units used to measure price nor the units used to measure quantity.

2) The price elasticity of demand depends on 2)


A) the units used to measure price but not the units used to measure quantity.
B) the units used to measure price and the units used to measure quantity.
C) the units used to measure quantity but not the units used to measure price.
D) neither the units used to measure price nor the units used to measure quantity.

3) The price elasticity of demand measures 3)


A) the slope of a budget curve.
B) how often the price of a good changes.
C) the responsiveness of the quantity demanded to changes in price.
D) how sensitive the quantity demanded is to changes in demand.

4) When the quantity of coal supplied is measured in kilograms instead of pounds, the demand for 4)
coal becomes
A) more elastic. B) neither more nor less elastic.
C) less elastic. D) undefined.

5) The price elasticity of demand equals 5)


A) the percentage change in the quantity demanded divided by the percentage change in the
price.
B) the change in the quantity demanded divided by the change in price.
C) the percentage change in the price divided by the percentage change in the quantity
demanded.
D) the change in the price divided by the change in quantity demanded.

6) If a rightward shift of the supply curve leads to a 6 percent decrease in the price and a 5 percent 6)
increase in the quantity demanded, the price elasticity of demand is
A) 0.83. B) 0.30. C) 0.60. D) 1.20.

7) A 10 percent increase in the quantity of spinach demanded results from a 20 percent decline in its 7)
price. The price elasticity of demand for spinach is
A) 0.5. B) 20.0. C) 2.0. D) 10.0.

8) A 20 percent increase in the quantity of pizza demanded results from a 10 percent decline in its 8)
price. The price elasticity of demand for pizza is
A) 2.0. B) 10.0. C) 0.5. D) 20.0.

1
9) Suppose a rise in the price of peaches from $5.50 to $6.50 per bushel decreases the quantity 9)
demanded from 12,500 to 11,500 bushels. The price elasticity of demand is
A) 0.5. B) 1000.0. C) 2.0. D) 1.0.

10) A fall in the price of lemons from $10.50 to $9.50 per bushel increases the quantity demanded from 10)
19,200 to 20,800 bushels. The price elasticity of demand is
A) 1.25. B) 1.20. C) 8.00. D) 0.80.

11) A fall in the price of cabbage from $10.50 to $9.50 per bushel increases the quantity demanded from 11)
18,800 to 21,200 bushels. The price elasticity of demand is
A) 1.20. B) 0.80. C) 8.00. D) 1.25.

12) Suppose that the quantity of root beer demanded declines from 103,000 gallons per week to 97,000 12)
gallons per week as a consequence of a 10 percent increase in the price of root beer. The price
elasticity of demand is
A) 1.66. B) 6.00. C) 0.60. D) 1.40.

13) The price elasticity of demand is 5.0 if a 10 percent increase in the price results in a ________ 13)
decrease in the quantity demanded.
A) 10 percent B) 50 percent C) 2 percent D) 5 percent

14) A shift of the supply curve of oil raises the price of oil from $9.50 a barrel to $10.50 a barrel and 14)
reduces the quantity demanded from 41 million to 39 million barrels a day. The price elasticity of
demand for oil is
A) 2 million barrels a day per dollar. B) 0.5.
C) $1 per 2 million barrels a day. D) 2.0.

Price Quantity demanded


(dollars per bushel) (bushels)
8 2,000
7 4,000
6 6,000
5 8,000
4 10,000
3 12,000

15) The table above gives the demand schedule for snow peas. The price elasticity of demand between 15)
$6.00 and $7.00 per bushel is
A) 1.0. B) 5.0. C) 2.0. D) 2.6.

16) The table above gives the demand schedule for snow peas. If the price of snow peas falls from 16)
$4.00 to $3.00 a bushel, total revenue will
A) increase because demand is elastic in this range.
B) increase because demand is inelastic in this range.
C) decrease because demand is inelastic in this range.
D) decrease because demand is elastic in this range.

2
17) The table above gives the demand schedule for snow peas. The demand curve for snow peas is a 17)
straight line and so the elasticity of demand is
A) lower at higher prices. B) higher at higher prices.
C) 1 at all prices. D) the same at all prices but not 1.

Price Quantity demanded


(dollars per bushel) (bushels)
A 10 0
B 8 4
C 6 8
D 4 12
E 2 16

18) The table above gives the demand schedule for peas. As you move from point A to point B, the 18)
price elasticity of demand equals
A) 0.50. B) 0.11. C) 9.09. D) 0.22.

19) The table above gives the demand schedule for peas. As you move from point C to point D, the 19)
price elasticity of demand is
A) 3.00. B) elastic. C) 0.75. D) unit elastic.

20) The table above gives the demand schedule for peas. Which of the following statements correctly 20)
describes the price elasticity of demand?
A) The price elasticity of demand is larger at point A than at point B.
B) The price elasticity of demand is constant because the slope is constant.
C) The price elasticity of demand increases moving from point A to point B to point C to point D
to point E.
D) The price elasticity of demand is larger at point D than at point A.

21) If demand is price elastic, 21)


A) a 1 percent decrease in the price leads to an increase in the quantity demanded that exceeds 1
percent.
B) a 1 percent increase in the price leads to an increase in the quantity demanded that exceeds 1
percent.
C) the price is very sensitive to any shift of the supply curve.
D) a 1 percent decrease in the price leads to a decrease in the quantity demanded that is less than
1 percent.

22) The price elasticity of demand can range between 22)


A) negative one and one. B) zero and infinity.
C) zero and one. D) negative infinity and infinity.

23) Demand is perfectly inelastic when 23)


A) the good in question has perfect substitutes.
B) shifts in the supply curve results in no change in price.
C) shifts of the supply curve results in no change in quantity demanded.
D) shifts of the supply curve results in no change in the total revenue from sales.

3
24) If the price elasticity is between 0 and 1, demand is 24)
A) inelastic. B) elastic. C) perfectly elastic. D) unit elastic.

25) Demand is inelastic if 25)


A) a large change in quantity demanded results in a small change in price.
B) the price elasticity of demand is greater than 1.
C) the quantity demanded is very responsive to changes in price.
D) the price elasticity of demand is less than 1.

26) A good with a vertical demand curve has a demand with 26)
A) infinite elasticity. B) unit elasticity.
C) zero elasticity. D) varying elasticity.

27) The demand curve in the figure above illustrates the demand for a product with 27)
A) unit price elasticity of demand at all prices.
B) a price elasticity of demand that is different at all prices.
C) infinite price elasticity of demand.
D) zero price elasticity of demand at all prices.

28) When the price elasticity of demand for a good equals 28)
A) 0, the demand curve is horizontal. B) 1, the demand curve is vertical.
C) 1, the demand curve is horizontal. D) 0, the demand curve is vertical.

29) A straight-line demand curve along which the price elasticity of demand equals 0 is one that 29)
A) forms a 45 degree angle with the vertical axis.
B) is horizontal.
C) is vertical.
D) forms a 60 degree angle with the horizontal axis.

4
30) The demand for movies is unit elastic if 30)
A) any increase in the price leads to a 1 percent decrease in the quantity demanded.
B) a 5 percent decrease in the price leads to an infinite increase in the quantity demanded.
C) a 5 percent increase in the price leads to a 5 percent decrease in the quantity demanded.
D) a 5 percent increase in the price leads to a 5 percent increase in total revenue.

31) Unit elastic demand 31)


A) means that the ratio of a change in the quantity demanded to a change in the price equals 1.
B) will be vertical.
C) means that the ratio of a percentage change in the quantity demanded to a percentage change
in the price equals 1.
D) will be horizontal.

32) A good with a horizontal demand curve has a demand 32)


A) with an income elasticity of demand of 0.
B) with a price elasticity of demand of infinity.
C) for which there are no substitute.
D) with a price elasticity of demand of 0.

33) The demand curve in the figure above illustrates a product whose demand has a price elasticity of 33)
demand equal to
A) infinity. B) zero at all prices.
C) a different amount at different prices. D) one at all prices.

5
34) The demand curve in the figure above illustrates the demand for a product with 34)
A) zero price elasticity of demand at all prices.
B) a price elasticity of demand that is different at all prices.
C) unit price elasticity of demand at all prices.
D) infinite price elasticity of demand.

35) On a linear demand curve that intersects both axes, 35)


A) the elasticity decreases as the price falls and quantity increases.
B) the elasticity is less than 1.00 at all prices.
C) the elasticity equals 1.00 at all prices.
D) the elasticity exceeds 1.00 at all prices.

36) On a straight-line downward-sloping demand curve, the maximum elasticity of demand occurs 36)
A) where it intersects the supply curve. B) at its vertical intercept.
C) at its horizontal intercept. D) at its midpoint.

37) A straight-line demand curve with negative slope intersects the horizontal axis at 100 tons per 37)
week. At the midpoint on the demand curve (corresponding to 50 tons per week) the price
elasticity of demand is
A) greater than 1.0. B) 0.5. C) 1.0. D) 0.

6
38) The figure above illustrates a linear demand curve. By comparing the price elasticity in the $2 to $4 38)
price range with the elasticity in the $8 to $10 range, you can conclude that the elasticity is
A) the same in both price ranges.
B) greater in the $8 to $10 range when the price rises but greater in the $2 to $4 range when the
price falls.
C) greater in the $8 to $10 range.
D) greater in the $2 to $4 range.

39) The figure above illustrates a linear demand curve. If the price falls from $8 to $6, 39)
A) the quantity demanded will increase by less than 20 percent.
B) total revenue will remain unchanged.
C) total revenue will increase.
D) total revenue will decrease.

40) The figure above illustrates a linear demand curve. In the range from $8 to $6, 40)
A) the demand is unit elastic.
B) the demand is price inelastic.
C) the demand is price elastic.
D) more information is needed to determine if the demand is price elastic, unit elastic, or
inelastic.

41) The figure above illustrates a linear demand curve. If the price falls from $6 to $4, 41)
A) total revenue will decrease.
B) total revenue will increase.
C) quantity demanded will increase by more than 100 percent.
D) total revenue will remain unchanged.

7
42) The figure above illustrates a linear demand curve. In the price range from $8 to $6, demand is 42)
________ and in the price range $4 to $2, demand is ________.
A) elastic; inelastic B) inelastic; inelastic
C) elastic; elastic D) inelastic; elastic

43) The figure above illustrates a linear demand curve. If the price rises from $6 to $8 demand is 43)
________ and if the price falls from $8 to $6 demand is ________.
A) inelastic; inelastic B) elastic; inelastic
C) elastic; elastic D) inelastic; elastic

44) The demand curve in the figure above illustrates the demand for a product with 44)
A) zero price elasticity of demand at all prices.
B) a price elasticity of demand that is different at all prices.
C) unit price elasticity of demand at all prices.
D) infinite price elasticity of demand.

45) A straight-line demand curve with negative slope intersects the horizontal axis at 200 tons per 45)
week. The point on the demand curve at which the price elasticity of demand is 1 corresponds to a
quantity demanded
A) that would be negative if a negative quantity demanded were possible.
B) of 100 tons.
C) of 0 tons.
D) of 200 tons.

8
46) Demand is inelastic if 46)
A) a leftward shift of the supply curve raises the total revenue.
B) the good in question has close substitutes.
C) the smaller angle between the vertical axis and the demand curve is less than 45 degrees.
D) large shifts of the supply curve lead to only small changes in price.

47) Demand is unit elastic when 47)


A) a shift of the supply curve leads to no change in price.
B) the slope of the demand curve is -1.
C) a change in the price of the product leads to no change in the total revenue.
D) a shift of the supply curve leads to an equal shift of the demand curve.

48) Producers' total revenue will decrease if 48)


A) the price rises and demand is inelastic.
B) income increases and the good is a normal good.
C) the price rises and demand is elastic.
D) income falls and the good is an inferior good.

49) Producers' total revenue will increase if 49)


A) income falls and the good is a normal good.
B) the price rises and demand is inelastic.
C) the price rises and demand is elastic.
D) income increases and the good is an inferior good.

50) If the demand for a good is unit elastic, 50)


A) a 5 percent increase in price results in a 5 percent increase in total revenue.
B) the demand curve is a straight line with slope of -1.
C) a 5 percent increase in price results in a 5 percent decrease in total revenue.
D) a 5 percent increase in price does not change total revenue.

51) A shift of the supply curve of oil raises the price from $10 a barrel to $30 a barrel and reduces the 51)
quantity demanded from 40 million to 23 million barrels a day. You can conclude that the
A) supply of oil is elastic. B) supply of oil is inelastic.
C) demand for oil is inelastic. D) demand for oil is elastic.

52) A shift of the supply curve of oil raises the price from $10 a barrel to $15 a barrel and reduces the 52)
quantity demanded from 40 million to 15 million barrels a day. You can conclude that the
A) demand for oil is elastic. B) supply of oil is elastic.
C) supply of oil is inelastic. D) demand for oil is inelastic.

53) A leftward shift of the supply curve of cookies raises the price of a cookie from 10 cents to 20 cents 53)
and decreases the quantity demanded from 700,000 to 500,000. You can conclude that
A) the supply of cookies is elastic. B) the supply of cookies is inelastic.
C) the demand for cookies is elastic. D) the demand for cookies is inelastic.

9
54) The demand for a good is elastic if 54)
A) a decrease in its price results in a decrease in total revenue.
B) the good is a necessity.
C) an increase in its price results in an increase in total revenue.
D) an increase in its price results in a decrease in total revenue.

55) If a price decrease results in your expenditure on a good decreasing, your demand must be 55)
A) unit. B) inelastic. C) linear. D) elastic.

56) An increase in subway fares in New York City will boost your expenditures on subway rides if 56)
A) the supply of subway rides is elastic. B) the supply of subway rides is inelastic.
C) your demand for subway rides is inelastic. D) your demand for subway rides is elastic.

57) The more substitutes available for a product, 57)


A) the larger is its income elasticity of demand.
B) the smaller is its income elasticity of demand.
C) the smaller is its price elasticity of demand.
D) the larger is its the price elasticity of demand.

58) Of the following, demand is likely to be the least elastic for 58)
A) Toyota automobiles. B) compact disc players.
C) Ford automobiles. D) toothpicks.

59) Of the following, demand is likely to be the least elastic for 59)
A) pink grapefruit. B) iceberg lettuce.
C) insulin for diabetics. D) diamonds.

60) The demand for food is most elastic in countries 60)


A) with low income levels. B) that are highly urbanized.
C) with intermediate income levels. D) with high income levels.

61) The demand for Honda Accords is 61)


A) probably inelastic and less elastic than the demand for automobiles.
B) probably elastic but less elastic than the demand for automobiles.
C) probably elastic and more elastic than the demand for automobiles.
D) probably inelastic but more elastic than the demand for automobiles.

62) The route from Dallas to Mexico City is served by more than one airline. The demand for tickets 62)
from American Airlines for that route is probably
A) elastic and more elastic than the demand for all tickets for that route.
B) inelastic and less elastic than the demand for all tickets for that route.
C) elastic but less elastic than the demand for all tickets for that route.
D) inelastic but more elastic than the demand for all tickets for that route.

10
63) The elasticity of demand for Gateway computers is probably 63)
A) elastic and smaller than the elasticity of demand for computers overall.
B) inelastic and smaller than the elasticity of demand for computers overall.
C) inelastic but larger than the elasticity of demand for computers overall.
D) elastic and larger than the elasticity of demand for computers overall.

64) Aglets are the metal or plastic tips on shoelaces that make it easier to lace your shoes. The demand 64)
for aglets is probably
A) perfectly elastic. B) inelastic.
C) elastic but not perfectly elastic. D) unit elastic.

65) The cross elasticity of demand measures the responsiveness of the quantity demanded of a 65)
particular good to changes in the prices of
A) its complements but not its substitutes.
B) its substitutes but not its complements.
C) its substitutes and its complements.
D) neither its substitutes nor its complements.

66) If goods are complements, definitely their 66)


A) income elasticities are negative. B) income elasticities are positive.
C) cross elasticities are positive. D) cross elasticities are negative.

67) If a rise in the price of good 1 decreases the quantity of good 2 demanded, 67)
A) the cross elasticity of demand is negative. B) good 1 is an inferior good.
C) good 2 is an inferior good. D) the cross elasticity of demand is positive.

68) The cross elasticity of demand between apples and oranges is defined as 68)
A) the price elasticity of demand for apples divided by the price elasticity of demand for oranges.
B) the change in the quantity of apples demanded divided by the change in the quantity of
oranges demanded.
C) the percentage change in the quantity of apples demanded divided by the percentage change
in the price of oranges.
D) the percentage change in the quantity of apples demanded divided by the percentage change
in the quantity of oranges demanded.

69) If the cross elasticity of demand between goods A and B is positive, 69)
A) the demands for A and B are both price elastic.
B) A and B are complements.
C) A and B are substitutes.
D) the demands for A and B are both price inelastic.

11
70) If the cross elasticity of demand between goods A and B is negative, 70)
A) the demands for A and B are both price elastic.
B) A and B are complements.
C) the demands for A and B are both price inelastic.
D) A and B are substitutes.

71) The greater the substitutability between Northwest timber and Southeast timber, the ________ is 71)
the cross elasticity of demand between timber from the two regions and the ________ is the
elasticity of demand for Northwest timber.
A) smaller; smaller B) larger; smaller C) smaller; larger D) larger; larger

72) If goods A and B are complements, 72)


A) the cross elasticity of demand between A and B is negative.
B) the cross elasticity of demand between A and B is positive.
C) their income elasticities of demand are both less than 1.
D) their income elasticities of demand are both greater than 1.

73) If a rise in the price of good B increases the quantity demanded of good A, 73)
A) B is a substitute for A, but A is a complement to B.
B) A is a substitute for B, but B is a complement to A.
C) A and B are complements.
D) A and B are substitutes.

74) If a fall in the price of good A increases the quantity demanded of good B, 74)
A) A and B are substitutes.
B) A and B are complements.
C) B is a substitute for A, but A is a complement to B.
D) A is a substitute for B, but B is a complement to A.

75) The cross elasticity of demand between Coca-Cola and Pepsi-Cola is 75)
A) positive, that is, Coke and Pepsi are complements.
B) negative, that is, Coke and Pepsi are complements.
C) positive, that is, Coke and Pepsi are substitutes.
D) negative, that is, Coke and Pepsi are substitutes.

76) A rise in the price of good A will shift the 76)


A) supply curve of good B rightward if the cross elasticity of demand between A and B is
positive.
B) demand curve for good B rightward if the cross elasticity of demand between A and B is
negative.
C) demand curve for good B rightward if the cross elasticity of demand between A and B is
positive.
D) supply curve of good B rightward if the cross elasticity of demand between A and B is
negative.

12
77) The income elasticity of demand is the percentage change in 77)
A) income divided by the percentage change in price.
B) the quantity demanded divided by the percentage change in income.
C) the price divided by the percentage change in income.
D) income divided by the percentage change in quantity demanded.

78) Demand is income elastic if 78)


A) an increase in income will not affect the quantity demanded.
B) a small percentage increase in income will result in a large percentage increase in quantity
demanded.
C) the good in question has close substitutes.
D) a large percentage increase in income will result in a small percentage increase in quantity
demanded.

79) The income elasticity of demand is high for 79)


A) shelter. B) luxuries. C) clothing. D) food.

80) To say that turnips are inferior goods means that the income elasticity 80)
A) is definitely greater than 1.
B) is negative.
C) is positive but could be greater than or less then (or equal to) 1.
D) is definitely between 0 and 1.

81) An increase in Abigail's income decreases her demand for cassette tapes. For her, cassette tapes are 81)
A) a complement to any good. B) a normal good.
C) an inferior good. D) a substitute good.

82) Goods whose income elasticities are negative are called 82)
A) superior goods. B) inferior goods. C) normal goods. D) complements.

83) A 10 percent increase in income has caused a 5 percent decrease in the quantity demanded. The 83)
income elasticity is
A) 0.5. B) -2.0. C) 2.0. D) -0.5.

84) Deb's income has just risen from $950 per week to $1,050 per week. As a result, she decides to 84)
increase the number of movies she attends each month by 5 percent. Her demand for movies is
A) income inelastic. B) income elastic.
C) represented by a vertical line. D) represented by a horizontal line.

85) Fred's income has just risen from $940 per week to $1,060 per week. As a result, he decides to 85)
purchase 9 percent more steak per week. The income elasticity of Fred's demand for steak is
A) 0.75. B) 1.33. C) 0.90. D) 1.00.

86) Joan's income has just risen from $940 per week to $1,060 per week. As a result, she decides to 86)
purchase 12 percent more lettuce per week. The income elasticity of Joan's demand for lettuce is
A) 1.33. B) 0.90. C) 1.00. D) 0.75.

13
87) A 10 percent increase in income causes the quantity of orange juice demanded to increase from 87)
19,200 to 20,800 gallons. The income elasticity of demand for orange juice is
A) 0.8. B) 1.2. C) 1.0. D) 0.5.

88) A 10 percent increase in income causes the quantity of apple juice demanded to increase from 88)
18,800 to 21,200 gallons. The income elasticity of demand for apple juice is
A) 0.5. B) 1.0. C) 1.2. D) 0.8.

89) The above figure shows a good 89)


A) that is an inferior good over all income ranges.
B) whose income elasticity is greater than 0 but less than 1.
C) that is a normal good over some income ranges and an inferior good over other ranges.
D) whose income elasticity always exceeds 1.0.

90) Of the following, which one is most likely to have a negative income elasticity of demand? 90)
A) shoes B) tennis balls
C) inter-city bus travel D) frozen yogurt

14
91) The above figure shows a good 91)
A) whose income elasticity is greater than 0 but less than 1.
B) that is an inferior good over all income ranges.
C) whose income elasticity always exceeds 1.0.
D) that is a normal good over some income ranges and an inferior good over other ranges.

92) The above figure shows a good 92)


A) whose income elasticity always exceeds 1.0.
B) whose income elasticity is greater than 0 but less than 1.
C) that is an inferior good over all income ranges.
D) that is a normal good over some income ranges and an inferior good over other ranges.

15
93) The increase in the demand for widgets, shown in the figure above, is caused by an increase in the 93)
price of McBoover devices. Therefore,
A) widgets and McBoover devices are substitutes.
B) widgets and McBoover devices are complements.
C) McBoover devices are a normal good.
D) widgets are a normal good.

94) The increase in the demand for widgets, shown in the figure above, is caused by a decrease in the 94)
price of McBoover devices. Therefore,
A) widgets and McBoover devices are substitutes.
B) widgets are a normal good.
C) McBoover devices are a normal good.
D) widgets and McBoover devices are complements.

95) The increase in the demand for widgets, shown in the figure above, is caused by an increase in the 95)
price of McBoover devices from $9 to $11. Therefore, the cross-price elasticity for these two
products is
A) 0.5. B) -2.0. C) 2.0. D) -0.5.

96) The increase in the demand for widgets, shown in the figure above, is caused by a decrease in the 96)
price of McBoover devices from $11 to $9. Therefore, the cross-price elasticity for these two
products is
A) -2.0. B) 2.0. C) -0.5. D) 0.5.

97) The increase in the demand for widgets, shown in the figure above, is caused by an increase in 97)
average incomes. Therefore, widgets
A) are a normal good. B) are elastically demanded.
C) are an inferior good. D) are inelastically demanded.

16
98) The increase in the demand for widgets, shown in the figure above, is caused by an increase in 98)
average incomes from $28,500 per year to $31,500 per year. Therefore, the income elasticity of
demand for widgets is
A) 4. B) 3/4. C) 1/4. D) 4/3.

99) As income rises, the share of income spent on food in the United States 99)
A) rises. B) remains constant at 15 percent.
C) falls. D) remains constant at 33 percent.

100) The elasticity of supply measures the responsiveness of 100)


A) quantity supplied to changes in price. B) quantity demanded to changes in supply.
C) quantity supplied to changes in income. D) quantity supplied to changes in demand.

101) The elasticity of supply measures the sensitivity of 101)


A) supply to changes in costs. B) quantity supplied to a change in price.
C) price to changes in supply. D) quantity supplied to quantity demanded.

102) On most days the price of a rose is $1 and 80 roses are purchased. On Valentine's Day the demand 102)
increases so that the price of a rose rises to $2 and 320 roses are purchased. Therefore, the price
elasticity of
A) demand for roses is about 1.8. B) supply of roses is about 1.8.
C) demand for roses is about 0.55. D) supply of roses is about 0.55.

103) Supply is elastic if 103)


A) a 1 percent change in price causes a larger percentage change in quantity supplied.
B) the good in question is a normal good.
C) the slope of the supply curve is positive.
D) a 1 percent change in price causes a smaller percentage change in quantity supplied.

104) If a 1 percent decrease in the price of a pound of oranges results in a smaller percentage decrease in 104)
the quantity supplied,
A) supply is inelastic. B) demand is inelastic.
C) demand is elastic. D) supply is elastic.

105) If a 1 percent decrease in the price of a pound of squash results in a larger percentage decrease in 105)
the quantity supplied,
A) demand is inelastic. B) demand is elastic.
C) supply is inelastic. D) supply is elastic.

106) If at a given moment, no matter what the price, producers cannot change the quantity supplied, the 106)
momentary supply
A) has infinite elasticity. B) has unit elasticity.
C) does not exist. D) has zero elasticity.

17
107) If a rise in the price of oranges from $7 to $9 a bushel, caused by a shift of the demand curve, 107)
increases the quantity of bushels supplied from 4,500 to 5,500 bushels, the
A) demand for oranges is elastic. B) supply of oranges is elastic.
C) demand for oranges is inelastic. D) supply of oranges is inelastic.

108) If a shift in the demand curve that raises the price of oranges from $7 to $9 a bushel increases the 108)
quantity of oranges supplied from 4,000 bushels to 6,000 bushels, the
A) supply of oranges is elastic. B) supply of oranges is inelastic.
C) demand for oranges is inelastic. D) demand for oranges is elastic.

109) A rise in the price of cabbage from $14 to $18 per bushel, caused by a shift of the demand curve, 109)
increases the quantity supplied from 4,000 to 6,000 bushels. The elasticity of supply is
A) 1.6. B) 1.0. C) 0.6. D) 0.8.

110) If a 5 percent increase in the price results in a 9 percent increase in quantity supplied, the elasticity 110)
of supply is
A) 0.30. B) 0.55. C) 1.80. D) 1.20.

111) If a 5 percent increase in price results in a 3 percent increase in the quantity supplied, the elasticity 111)
of supply is
A) 1.20. B) 0.60. C) 1.66. D) 0.30.

112) A vertical supply curve indicates an elasticity of supply that equals 112)
A) 0. B) infinity. C) 1. D) -1.

113) A horizontal supply curve indicates an elasticity of supply that equals 113)
A) 0. B) infinity. C) 1. D) -1.

114) Suppose a 10 percent increase in the price of textbooks decreases the quantity demanded by 20 114)
percent. The elasticity of demand for textbooks is
A) 0.2. B) 5.0. C) 10.0. D) 2.0.

115) The quantity of new cars increases by 10 percent. If the price elasticity of demand for new cars is 115)
1.25, the price of new cars will fall by
A) 8 percent. B) 10 percent. C) 2.5 percent. D) 12.5 percent.

116) Suppose the price elasticity of demand for oil is 0.1. In order to lower the price of oil by 20 percent, 116)
the quantity of oil supplied must be increased by
A) 20 percent. B) 2 percent. C) 0.2 percent. D) 200 percent.

117) Moving up (to the left) along a linear demand curve, the price elasticity of demand 117)
A) at first increases and then decreases. B) increases.
C) decreases. D) does not change.

118) If the price elasticity of demand for a product equals 1, as its price rises the 118)
A) total revenue increases. B) quantity demanded does not change.
C) total revenue does not change. D) quantity demanded increases.

18
119) A rise in the price of a product lowers the total revenue from the product if the 119)
A) good is an inferior product. B) demand for the product is inelastic.
C) demand for the product is elastic. D) income elasticity of demand exceeds 1.

120) If a 4 percent rise in the price of peanut butter lowers the total revenue received by the producers 120)
of peanut butter by 4 percent, the demand for peanut butter
A) is inelastic. B) is elastic.
C) is unit elastic. D) has an elasticity of 2.0.

121) A product is likely to have a price elasticity of demand that exceeds 1 when 121)
A) its price falls.
B) it is a necessity.
C) it has close substitutes.
D) the percentage of income spent on it decreases.

122) Which of the following is likely to have the smallest price elasticity of demand? 122)
A) a new Ford automobile B) a new automobile
C) a new Ford Mustang D) an automobile

123) A 10 percent decrease in the price of a Pepsi decreases the demand for a Coca-Cola by 50 percent. 123)
The cross elasticity of demand between a Pepsi and Coca-Cola is
A) 5. B) 10. C) 0.20. D) 50.

124) A fall in the price of X from $12 to $8 causes an increase in the quantity of Y demanded from 900 to 124)
1,100 units. What is the cross elasticity of demand between X and Y?
A) 2 B) -0.5 C) -2 D) 0.5

125) A fall in the price of X from $12 to $8 causes an increase in the quantity of Y demanded from 900 to 125)
1,100 units. X and Y are
A) complements. B) normal goods. C) substitutes. D) inferior goods.

126) A 10 percent decrease in income decreases the quantity demanded of compact discs by 3 percent. 126)
The income elasticity of demand for compact discs is
A) 10.0. B) 3.3. C) -0.3. D) 0.3.

19
127) In the figure above, when the price of a disk is $B, total revenue is shown in the graph by area 127)
A) FCDE. B) ADE0. C) AGF0. D) BCF0.

128) The above figure illustrates the demand curve for a good. The good has 128)
A) many substitutes. B) no substitutes.
C) only one substitute. D) only a few substitutes.

20
129) The elasticity of demand along the demand curve shown in the above figure is constant and equal 129)
to 1. Thus,
A) area 0BCF equals area 0AGF. B) area 0BCF equals area 0ADE.
C) area 0BCF equals area FGDE. D) area ABCG equals area 0AGF.

130) The above figure shows a linear (straight-line) demand curve. Start at point A and then moving to 130)
point B and then point C, the price elasticity of demand
A) increases. B) increases and then decreases.
C) decreases and then increases. D) decreases.

21
Answer Key
Testname: UNTITLED2.TST

1) B
2) D
3) C
4) B
5) A
6) A
7) A
8) A
9) A
10) D
11) A
12) C
13) B
14) B
15) D
16) C
17) B
18) C
19) D
20) A
21) A
22) B
23) C
24) A
25) D
26) C
27) D
28) D
29) C
30) C
31) C
32) B
33) A
34) C
35) A
36) B
37) C
38) C
39) C
40) C
41) D
42) A
43) C
44) B
45) B
46) A
47) C
48) C
49) B
50) D
1
Answer Key
Testname: UNTITLED2.TST

51) C
52) A
53) D
54) D
55) B
56) C
57) D
58) D
59) C
60) A
61) C
62) A
63) D
64) B
65) C
66) D
67) A
68) C
69) C
70) B
71) D
72) A
73) D
74) B
75) C
76) C
77) B
78) B
79) B
80) B
81) C
82) B
83) D
84) A
85) A
86) C
87) A
88) C
89) D
90) C
91) A
92) D
93) A
94) D
95) C
96) A
97) A
98) A
99) C
100) A
2
Answer Key
Testname: UNTITLED2.TST

101) B
102) B
103) A
104) A
105) D
106) D
107) D
108) A
109) A
110) C
111) B
112) A
113) B
114) D
115) A
116) B
117) B
118) C
119) C
120) B
121) C
122) D
123) A
124) B
125) A
126) D
127) D
128) B
129) B
130) D

3
Economic Analysis for Business Decisions
Multiple Choice Questions
Unit-2: Demand Analysis

1. The law of demand states that an increase in the price of a good:

a. Increases the supply of that good.


b. Decreases the quantity demanded for that good.
c. Increases the quantity supplied of that good.
d. None of these answers.

2. The law of supply states that an increase in the price of a good:

a. None of these answers.


b. Increases the quantity supplied of that good.
c. Decreases the demand for that good.
d. Decreases the quantity demanded for that good.

3. If an increase in consumer incomes leads to a decrease in the demand for camping equipment,
then camping equipment is:
a. A normal good. b. None of these answers. c. An inferior good.
d. A substitute good e. A complementary good.

4. Which of the following shifts the demand for watches to the right?

a. An increase in the price of watches b. None of these answers


c. A decrease in the price of watch batteries if watch batteries and watches are
complements
d. A decrease in consumer incomes if watches are a normal good

5. If the price of a good is above the equilibrium price,

a. There is a surplus and the price will rise.


b. There is a shortage and the price will fall.
c. There is a shortage and the price will rise.
d. The quantity demanded is equal to the quantity supplied and the price remains unchanged.
e. There is a surplus and the price will fall.

6.. If the price of a good is equal to the equilibrium price,

a. There is a shortage and the price will fall.


b. The quantity demanded is equal to the quantity supplied and the price remains
unchanged.
c. There is a surplus and the price will rise. d. There is a shortage and the price will rise.
e. There is a surplus and the price will fall.
7. An inferior good is one for which an increase in income causes a(n)

a. Decrease in supply. b. Increase in demand.


c. Increase in supply. d. Decrease in demand.

8. If a small percentage increase in the price of a good greatly reduces the quantity demanded for
that good, the demand for that good is

a. Income inelastic. b. Price inelastic. c. Price elastic.


d. Unit price elastic. e. Income elastic.

9. The price elasticity of demand is defined as


a. The percentage change in the quantity demanded divided by the percentage change in income.
b. The percentage change in income divided by the percentage change in the quantity demanded.
c. The percentage change in the quantity demanded of a good divided by the percentage
change in the price of that good.
d. None of these answers.
e. The percentage change in price of a good divided by the percentage change in the quantity
demanded of that good.

10. In general, a flatter demand curve is more likely to be:

a. Price elastic. b. Unit price elastic. c. None of these answers. d. Price inelastic.

11. In general, a steeper supply curve is more likely to be

a. Price elastic. b. None of these answers. c. Unit price elastic. d. Price inelastic.

12. Which of the following would cause a demand curve for a good to be price inelastic?
a. The good is a luxury. b. There are a great number of substitutes for the good.
c. The good is a necessity. d. The good is an inferior good.

13. If the cross-price elasticity between two goods is negative, the two goods are likely to be:

a. Substitutes. b. Complements. c. Necessities. d. Luxuries.

14. If there is excess capacity in a production facility, it is likely that the firm's supply curve is:

a. Price inelastic. b. None of these answers. C. Unit price elastic. d. Price elastic.

15. If the income elasticity of demand for a good is negative, it must be:

a. An elastic good. b. An inferior good c. A normal good. d. A luxury good.


16. If consumers think that there are very few substitutes for a good, then

a. Supply would tend to be price elastic.


b. None of these answers.
c. Demand would tend to be price inelastic.
d. Demand would tend to be price elastic.
e. Supply would tend to be price inelastic

17. The sensitivity of the change in quantity demanded to a change in price is called:

a. Income elasticity. b. Cross-elasticity. c. Price elasticity of demand.


d. Coefficient of elasticity.

18. A product that is similar to another, and can be consumed in place of it is called

a. A normal good. b. An inferior good. c. A complementary good. d. A substitute good.

19. Two goods are _____________ if the quantity consumed of one increases when the price of
the other decreases.

a. Normal b. Superior c. Complementary d. Substitute

20. If input prices increase, all else equal,

a. quantity supplied will decrease. b. supply will increase. c. supply will decrease.
d. demand will decrease.

21. Which of the following would decrease the supply of wheat?

a. A decrease in the price of pesticides.


b. An increase in the demand for wheat.
c. A rise in the price of wheat.
d. An increase in the price of corn.
e. None of the above

22. If the price elasticity of demand for a good is .75, the demand for the good can be
described as:
a. Normal. B. Elastic. C. Inferior d. Inelastic.

23. When the price of a product is increased 10 percent, the quantity demanded decreases 15
percent. In this range of prices, demand for this product is:

a. Elastic b. Inelastic. C. Unitary elastic d. Perfectly elastic


24. Cross elasticity of demand is:
a. Negative for complementary goods b. Negative for substitute goods.
c. Unitary for inferior goods d. Positive for inferior goods.
25. A 3 percent increase in the price of tea causes a 6 percent increase in the demand for coffee.
The cross elasticity of demand for coffee with respect to the price of tea is:
a. -0.5 b. +0.5 c. -2.0 d. +2.0

26. The price elasticity of demand for any particular perfectly competitive firm's output is

a. Less than 1 b. Equal to zero c. Infinite d. 1

27.The economic profit of a perfectly competitive firm


a. Is less than its total revenue b. Is greater than its total revenue
c. Equals its total revenue
d. Is less than its total revenue if its supply curve is inelastic and is greater than its total revenue
if its supply curve is elastic.

28. When Sidney's Sweaters, Inc. makes exactly zero economic profit, Sidney, the owner

a. Makes an income equal to his best alternative forgone income.


b. Will boost output
c. Will shut down in the short run.
d. Is taking a loss

29. A perfectly competitive firm's marginal cost exceeds its marginal revenue at its current
output. To Increase its profit, the firm will

a. Increase its output b. raise its price c. lower its price d. decrease its output.

30. Demand for a commodity refers to

a. Desire for the commodity b. Need for the commodity


c. Quantity demanded of that commodity
d. Quantity of the commodity demanded at a certain price during any particular of time

31. Demand for a commodity depends on

a. Price of that commodity b. Price of related goods c. Income d. All the above

32. Law of demand establishes

a. Inverse relationship between price and quantity


b. Positive relationship between price and quantity
c. Both d. none
33. All but one of the following are assumed to remain the same while drawing an individual’s
demand curve for a commodity. Which one is it ?

a. The preference of the individual b. His monetary income


c. The price of the good under consideration d. The prices of other good

34. Demand for a product should have the following pre-requisite

a. Ability to buy b. Willingness c. Need d. All of these

35. Cross elasticity of demand between two perfect substitutes will be

a. Law b. High c. Zero d. Infinity

36. Market demand is aggregation of individual demand

a. Vertically b. Horizontally c. Both (a) and (b) d. None

37. A single point on the demand curve shows

a. Demand and supply relationship b. Price and supply relationship


c. Price and demand relationship d. None of these

38. The fall in the price of one commodity leads to fall in demand for other commodity and vice
versa for

a. Substitutes b. Complementary c. Both (a) and (b) d. None of the above

39. Decrease or fall in the price of commodity leads to increase in demand because of

a. Substitution Effect, i.e., Relatively cheaper than related goods


b. Income Effect, i.e., Consumer become better off
c. Both (a) and (b) d. None of these

40. Generally the demand curve has

a. A slope downward from left to right


b. A negative slope right c. Both of these d. None of these

41. The law of demand can be derived with the help of


a. Law of D.M.U. (Diminishing Marginal Utility)
b. Law of EMU (Equi-Marginal Utility)
c. Any of these two d. None of these

42. The value of elasticity of demand ranges from


a. Zero to one b. One to infinity c. Zero to infinity d. None of these
43. When the quantity demanded of goods increases by a larger percentage as compared with the
income of the consumer, income elasticity of demand is high

a. Unitary Income Elasticity b. Low-Income Elasticity


c. Zero-Income Elasticity d. High-Income Elasticity

44. The relation between price and supply in law of supply is

a. Direct b. Inverse c. Proportional d. None of these

45. Under perfect competition, price of the product

a. Can be controlled b. Cannot be controlled c. Can be controlled within certain limit


d. None of the above

46. Perfect competition is a market situation where we have

a. A single seller b. Two sellers c. Large number of sellers d. Few sellers

47. The firm can achieve equilibrium when its

a. MC = MR b. MC = AC c. MR = AR d. MR = AC

48. A firm and industry are one and the same under

a. Perfect competition b. Duopoly c. Oligopoly d. Monopoly

49. Homogeneity of product is characteristic of

a. Monopoly b. Oligopoly c. Perfect competition d. None of the above

50. In case of perfect competition, elasticity will be

a. O b. 2 c. 3 d. Infinity

51. Under which of the following forms of market structure a firm does has no control over the
price of its product?

a. Monopoly b. Monopolistic competition c. Oligopoly d. Perfect competition

52. Who is the price-leader under oligopoly ?

a. any unit with efficient production capabilities


b. there is no firm that can be termed as price leader under oligopoly
c. the largest firm d. the smallest firm
53. In this one firm assumes the role of price leader and fixes the price of the product for the
entire industry

a. Price leadership b. Cartel c. Kinked demand curve d. None of these

54. It is the art of translating into quantitative terms (rupees and paise) the value of the product
or a unit of a service to customers at point in time

a. Costing b. Pricing c. Profit analysis d. All of the Above

55. For a monopoly, the industry demand curve is the firm's

a. Profit function b. marginal revenue curve c. supply curve d. demand curve

56. If the price elasticity of demand is greater than 1, a monopoly's

a. Marginal revenue is zero


b. Total revenue decreases when the firm lowers its price
c. Marginal revenue is negative
d. Total revenue increases when the firm lowers its price

57. Consumer surplus is

a. Equal to the price minus the marginal cost


b. Less in the case of a single-price monopoly than in the case of a perfectly competitive
industry
c. Zero for a single-price monopolist
d. Positive in the case of a monopolist practicing perfect price discrimination.

58. An increase in income will:

a. Lead to a movement along the demand curve


b. Shift the supply curve c. Shift the demand curve d. Lead to an extension of demand

59. A shift in supply will have a bigger effect on price than output if demand is

a. Income elastic b. Income inelastic c. Price elastic d. Price inelastic

60. Assuming a downward sloping demand curve and upward sloping supply curve, a higher
equilibrium price may be caused by:
a. A fall in demand b. An increase in supply
c. Improvements in production technology d. An increase in demand

61. A movement along the supply curve may be caused by:


a. A change in technology b. A change in the number of producers
c. A shift in demand d. A change in costs
62. A subsidy paid to producers

a. Shifts the supply curve b. Shifts the demand curve


c. Leads to a contraction in supply d. Leads to an extension of supply

63. Which would definitely not be an example of price discrimination?

a. A theater charges children less than adults for a movie.


b. Universities charge higher tuition for out-of-state residents.
c. A doctor charges for services according to the income of patients.
d. An electric power company charges less for electricity used during off-peak hours
when production costs are lower.

64. If shortly after Kellogg’s Company has announced price increases on its ready-to-eat cereals, the
other cereal manufacturers announce identical price increases on their products, this is likely to be:

a. The essence of competition


b. A cartel c. Price leadership d. a coincidence.

65. Price discrimination is

a. Charging different prices to different customers because it costs the firm more to serve some
customers than others.
b. Changing the firm’s price frequently to respond to market conditions
c. Charging different prices to different customers when the price differences are not based on
cost differences.
d. Charging the same price to all customers

66. Monopolies and oligopolies are:

a. Price takers, as are competitive firms


b. Price takers, in contrast to competitive firms which are price makers
c. Price makers, in contrast to competitive firms which are price takers.
d. Price makers, as are competitive firms.

67. Concentration ratios may:

a. Overstate the extent of competition because they ignore imported products


b. Understate the extent of competition because they ignore imported products
c. Either overstate or understate the extent of competition because they ignore imported products.
d. None of the above.
68. If new firms enter a monopolistically competitive market, the demand curves for the existing
firms will:

a. Shift to the left and become more price inelastic.


b. Shift to the left and there will be no change in price elasticity.
c. Shift to the left and become more price elastic.
d. Shift to the right and there will be no change in price elasticity.

69. In the long run, monopolistically competitive firms maximize profit at the output where:

a. They earn zero economic profit.


b. P = MC.
c. Marginal cost = the minimum of the long-run average total cost curve.
d. All of the above.

70. Suppose that an economy wants to eliminate the resource waste associated with excess capacity
in monopolistically competitive markets. Which of the following actions would achieve this
goal?

a. Allow monopolistically competitive firms to create more significant barriers to entry.


b. Encourage more competition in monopolistically competitive markets.
c. Require all the firms in a given monopolistically competitive market to produce identical
products.
d. Require all the firms in a given monopolistically competitive market to charge the same price.

71. Which best describes a demand curve?

a. The quantity consumers would like to buy in an ideal world


b. The quantity consumers are willing to sell
c. The quantity consumers are willing and able to buy at each and every income all other things
unchanged
d. The quantity consumers are willing and able to buy at each and every price all other
things unchanged

72. A fall in price:

a. Will cause an inward shift of demand b. Will cause an outward shift of supply
c. Leads to a movement along a demand curve d. Leads to a higher level of production

73. Demand for a normal product may shift outwards if:

a. Price decreases b. The price of a substitute rises


c. The price of a complement rises d. Income falls
74. According to the law of diminishing marginal utility:

a. Utility is at a maximum with the first unit


b. Increasing units of consumption increase the marginal utility
c. Marginal product will fall as more units are consumed
d. Total utility will rise at a falling rate as more units are consumed

75. If marginal utility is zero:

a. Total utility is zero


b. An additional unit of consumption will decrease total utility
c. An additional unit of consumption will increase total utility
d. Total utility is maximized

76. A decrease in income should:

a. Shift demand for an inferior product inwards


b. Shift demand for an inferior product outwards
c. Shift supply for an inferior product outwards
d. Shift supply for an inferior product inwards

77. An increase in the price of a complement for product A would:

a. Shift demand for product A outwards


b. Shift demand for product A inwards
c. Shift supply for product A outwards
d. Shift supply for product A inwards

78. An increase in price, all other things unchanged, leads to:

a. Shift demand outwards b. Shift demand inwards


c. A contraction of demand d. An extension of demand

79. If a product is a Veblen good:

a. Demand is inversely related to income b. Demand is inversely related to price


c. Demand is directly related to price
d. Demand is inversely related to the price of substitutes

80. If a product is an inferior good:

a. Demand is inversely related to income b. Demand is inversely related to price


c) Demand is directly related to price
d) Demand is directly related to the price of substitutes
81. Average income increases from £20,000 p.a. to £22,000 p.a. Quantity demanded per year
increases from 5000 to 6000 units. Which of the following is correct?

a) Demand is price inelastic b) The good is inferior


c) Income elasticity is -2 d) The product is normal

82. The price decreases from £2,000 to £1,800. Quantity demanded per year increases from 5000
to 6000 units. Which of the following is correct?

a. The price elasticity of demand is -2 b. The good is inferior


c. Income elasticity is + 0.5 d. Income elasticity is + 2

83. If the price elasticity of demand is unit then a fall in price:

a. Reduces revenue b. Leaves revenue unchanged


c) Increases revenue d. Reduces costs

84. If the cross elasticity of demand is -2:

a. The products are substitutes and demand is cross price elastic


b. The products are substitutes and demand is cross price inelastic
c. The products are complements and demand is cross price elastic
d. The products are complements and demand is cross price inelastic

85. The income elasticity is +2 and income increases by 20%. Sales were 5000 units, what will
they be now?

a. 3000 b. 7000 c. 5500 d. 4500

86. The price elasticity of demand is a negative number this means:

a. Demand is price elastic b. Demand is price inelastic


c. The demand curve is downward sloping
d. An increase in income will reduce the quantity demanded

87. If demand is price inelastic:

a. An increase in price must raise profits b. An increase in price decreases revenue


c) An increase in price increases revenue d. A decrease in price reduces sales

88. For an inferior good:

a. The price elasticity of demand is negative; the income elasticity of demand is negative.
b. The price elasticity of demand is positive; the income elasticity of demand is negative.
c. The price elasticity of demand is negative; the income elasticity of demand is positive.
d. The price elasticity of demand is positive; the income elasticity of demand is positive.
89. For a normal good:

a. The price elasticity of demand is negative; the income elasticity of demand is negative.
b. The price elasticity of demand is positive; the income elasticity of demand is negative.
c. The price elasticity of demand is negative; the income elasticity of demand is positive.
d. The price elasticity of demand is positive; the income elasticity of demand is positive.

90. Which of the following characterizes monopolistic competition?

a. Many interdependent firms sell a homogeneous product.


b. A few firms produce a particular type of product.
c. Many firms produce a particular type of product, but each maintains some independent
control over its own price.
d. A few firms produce all of the market supply of a good.

91. Monopolistically competitive industries are characterized by:

a. Low concentration ratios. c. Independent production decisions.


b. Low entry barriers. d. All of the above.

92. In monopolistic competition, a firm:

a. Has no market power.


b. Captures significant economies of scale
c. Has a downward-sloping demand curve.
d. Has a standardized product that all firms produce.

93. If there are many firms in an industry producing goods that are similar but slightly different,
this is an example of:

a. Perfect competition. B. Monopolistic competition. C. Oligopoly. D. Monopoly.

94.Large cities typically have many drug stores which have different qualities of service and
selections of product. The drug store market in big cities can best be classified as:

a. A competitive market. b. Monopolistic competition. C. Oligopoly. d. Monopoly

95..A major difference between monopoly and monopolistic competition is:

a. One maximizes profits by setting MR equal to MC, and the other does not.
b. The number of firms in the market.
c. One type of firm has market power, and the other does not.
d. One has a downward-sloping demand curve, and the other does not.
96. Entry into a market characterized by monopolistic competition is generally:

a. Entirely blocked by existing firms.


b. Very easy because few barriers exist.
c. As difficult as in oligopoly.
d. More difficult than entry into monopolized markets.

97. Which of the following characterizes monopolistic competition?

a. Price leadership b. Product differentiation c. Price discrimination. d. Economies of scale.

98. Product differentiation refers to:

a. Features that make one product appear different from competing products in the same
market.
b. Different prices for the same product in a certain market.
c. The selling of identical products in different markets.
d. The charging of different prices for the same product in different markets.

99. Which of the following is an example of product differentiation?

a. Two bars of soap differ only in their label, but consumers pay 50 paisa more for the label
they recognize.
b. Sugar can be made from sugar beets or sugar cane which consumers cannot differentiate when
looking at sugar.
c. Consumers substitute vans in place of cars because vans accommodate more passengers.
d. Some sawmills specialize in producing softwood and others specialize in producing hardwood,
but the two types of wood are used for very different purposes.

100. Perfect competition and monopolistic competition are best distinguished by:

a. The degree of product differentiation.


b. The long-run economic profits that are expected.
c. The number of firms in the market. d. The ease of entry and exit.

101. A monopolistically competitive firm can raise its price somewhat without fear of great
change in unit sales because:

a. The demand for its product is typically very price-elastic.


b. Its demand curve is horizontal.
c. Of product differentiation and brand loyalty.
d. Of the gap in its marginal revenue curve.
102. A monopolistically competitive firm can raise its price somewhat without fear of great
change in unit sales because of:

a. Brand loyalty c. Inelastic demand.


b. Economies of scale. d.Large market shares of firms in the market.

103. When a monopolistically competitive firm advertises, it is attempting to increase:

a. The demand and decrease the price elasticity of demand for its product.
b. The demand and increase the price elasticity of demand for its product.
c. Long-run profits. d. Market demand.

104. Brand loyalty usually makes the demand curve for a product:

a. More price elastic. b. Less price elastic. c. Unitary elastic. d. More income elastic.

105. Both monopoly and monopolistic competition:

a. Maximize profit where MR = MC. c.Use advertising to differentiate their product.


b. Have high concentration ratios. d. Have high barriers to entry.

106. A monopolistically competitive firm maximizes profits or minimizes losses in the short run
by:

a. Using marginal cost pricing.


b. Producing output at the level where ATC is minimized.
c. Producing output at the level where price equals ATC.
d. Producing output at the level where MC = MR.

107. In monopolistic competition, a firm's demand is tangent to the long-run average cost curve
in the long run because:

a. Barriers to entry are very high.


b. Entry eliminates economic profit, and exit eliminates losses.
c. Advertising is ineffective in differentiating a product.
d. All of the above.

108. The demand for wheat from farm A is perfectly elastic because wheat from farm A is a(n)

a. Perfect complement to wheat from farm B.


b. Perfect substitute for wheat from farm B.
c. Normal good. d. Inferior good.

109. The price elasticity of demand for any particular perfectly competitive firm's output is

a. Less than 1. b. Equal to zero. c. Infinite. d. 1.


110. The market for fish is perfectly competitive. So, the price elasticity of demand for fish from
a single fishery

a. Sometimes greater than and sometimes less than the elasticity of demand for fish overall.
b. Greater than the elasticity of demand for fish overall.
c. Less than the elasticity of demand for fish overall.
d. Equals the elasticity of demand for fish overall.

111. In perfect competition, the price of the product is determined where the industry

a. Elasticity of supply equals the industry elasticity of demand.


b. Supply curve and industry demand curve intersect.
c. Fixed cost is zero.
d. Average variable cost equals the industry average total cost.

112. Total economic profit is

a. Total revenue minus total opportunity cost.


b. Marginal revenue minus marginal cost.
c. Total revenue divided by total cost.
d. Marginal revenue divided by marginal cost.

113. The economic profit of a perfectly competitive firm

a. Less than its total revenue. b. Greater than its total revenue. c. Its total revenue.
d. Less than its total revenue if its supply curve is inelastic and is greater than its total revenue if
its supply curve is elastic.

114. When Sidney's Sweaters, Inc. makes exactly zero economic profit, Sidney, the owner,

a. Makes an income equal to his best alternative forgone income.


b. Will boost output. c. Will shut down in the short run. d. Is taking a loss

115. Law of Demand explains relation between quantity demanded and …..

a. Quantity supplied b. Price c) Need d) Want

116. Income Elasticity established relation between Income and ……


a. Price b) Supply c) Quantity Demanded d) Income

117. Market Equilibrium relates to …….

a. Demand & Supply b. Production and Raw Material c. Demand and Price d. Supply and
Production
Chapter 3 - Demand and Supply - Sample Questions
Answers are at the end fo this file
MULTIPLE CHOICE. Choose the one alternative that best completes the statement or answers the question.

1) A relative price is 1)
A) the ratio of one price to another.
B) the difference between one price and another.
C) the slope of the supply curve.
D) the slope of the demand curve.

2) If the price of a candy bar is $1 and the price of a fast food meal is $5, 2)
A) the money price of a fast food meal is 1/5 of a candy bar.
B) the money price of a candy bar is 1/5 of a fast food meal.
C) the relative price of a fast food meal is 5 candy bars.
D) the relative price of a candy bar is 5 fast food meals.

3) If the price of a hot dog is $2 and the price of a hamburger is $4, 3)


A) the money price of a hamburger is 2 hot dogs.
B) the money price of a hot dog is 2 hamburgers.
C) the relative price of a hot dog is 1/2 of a hamburger.
D) the relative price of a hamburger is 1/2 of a hot dog.

4) The opportunity cost of good A in terms of good B is equal to the 4)


A) ratio of the price of good B to the price of good A.
B) ratio of the price of good A to the price of good B.
C) price of good A minus the price of good B.
D) price of good B minus the price of good A.

5) The opportunity cost of a hot dog in terms of hamburgers is 5)


A) the price of a hot dog minus the price of a hamburger.
B) the ratio of the slope of the supply curve for hot dogs to the slope of the supply curve for
hamburgers.
C) the ratio of the slope of the demand curve for hot dogs to the slope of the demand curve for
hamburgers.
D) the ratio of the price of a hot dog to the price of a hamburger.

6) Wants, as opposed to demands, 6)


A) depend on the price.
B) are the goods the consumer plans to acquire.
C) are the unlimited desires of the consumer
D) are the goods the consumer has acquired.

1
7) Demands differ from wants in that 7)
A) wants require a plan to acquire a good but demands require no such plan.
B) demands are unlimited, whereas wants are limited by income.
C) wants imply a decision about which demands to satisfy, while demands involve no specific
plan to acquire the good.
D) demands reflect a decision about which wants to satisfy and a plan to buy the good, while
wants are unlimited and involve no specific plan to acquire the good.

8) Scarcity guarantees that 8)


A) wants will exceed demands. B) demands will be equal to wants.
C) demands will exceed wants. D) most demands will be satisfied.

9) The quantity demanded is 9)


A) the amount of a good that consumers plan to purchase at a particular price.
B) independent of the price of the good.
C) independent of consumers' buying plans.
D) always equal to the equilibrium quantity.

10) The law of demand states that, other things remaining the same, the higher the price of a good, the 10)
A) smaller is the demand for the good.
B) smaller is the quantity of the good demanded.
C) larger is the quantity of the good demanded.
D) larger is the demand for the good.

11) The law of demand implies that, other things remaining the same, 11)
A) as the demand for cheeseburgers increases, the price of a cheeseburger will fall.
B) as the price of a cheeseburger rises, the quantity of cheeseburgers demanded will decrease.
C) as income increases, the quantity of cheeseburgers demanded will increase.
D) as the price of a cheeseburger rises, the quantity of cheeseburgers demanded will increase.

12) The law of demand states that the quantity of a good demanded varies 12)
A) inversely with its price.
B) directly with population.
C) directly with income.
D) inversely with the price of substitute goods.

13) Which of the following is consistent with the law of demand? 13)
A) A decrease in the price of a gallon of milk causes a decrease in the quantity of milk demanded.
B) An increase in the price of a soda causes a decrease in the quantity of soda demanded.
C) An increase in the price of a tape causes an increase in the quantity of tapes demanded.
D) A decrease in the price of juice causes no change in the quantity of juice demanded.

2
14) The law of demand implies that if nothing else changes, there is 14)
A) a linear relationship between price of a good and the quantity demanded.
B) a positive relationship between the price of a good and the quantity demanded.
C) a negative relationship between the price of a good and the quantity demanded.
D) an exponential relationship between price of a good and the quantity demanded.

15) Which of the following influences people's buying plans and varies moving along a demand curve? 15)
A) preferences B) the price of the good
C) income D) the prices of related goods

16) The law of demand states that 16)


A) a decrease in the price of a good shifts the demand curve leftward.
B) other things remaining the same, the higher the price of a good, the smaller is the quantity
demanded.
C) other thing remaining the same, the higher the price of a good, the larger is the quantity
demanded.
D) an increase in the price of a good shifts the demand curve leftward.

17) The law of demand implies that demand curves 17)


A) shift leftward whenever the price rises. B) shift rightward whenever the price rises.
C) slope down. D) slope up.

18) Each point on the demand curve reflects 18)


A) the highest price consumers are willing and able to pay for that particular unit of a good.
B) the highest price sellers will accept for all units they are producing.
C) the lowest-cost technology available to produce a good.
D) all the wants of a given household.

19) A drop in the price of a compact disc shifts the demand curve for prerecorded tapes leftward. From 19)
that you know compact discs and prerecorded tapes are
A) normal goods. B) substitutes. C) inferior goods. D) complements.

20) A substitute is a good 20)


A) of higher quality than another good. B) that is not used in place of another good.
C) that can be used in place of another good. D) of lower quality than another good.

21) People buy more of good 1 when the price of good 2 rises. These goods are 21)
A) normal goods. B) complements. C) substitutes. D) inferior goods.

22) Which of the following pairs of goods are most likely substitutes? 22)
A) compact discs and compact disc players B) lettuce and salad dressing
C) cola and lemon lime soda D) peanut butter and gasoline

3
23) The demand for a good increases when the price of a substitute ________ and also increases when 23)
the price of a complement ________.
A) falls; falls B) rises; falls C) rises; rises D) falls; rises

24) A complement is a good 24)


A) used in conjunction with another good. B) used instead of another good.
C) of lower quality than another good. D) of higher quality than another good.

25) Suppose people buy more of good 1 when the price of good 2 falls. These goods are 25)
A) substitutes. B) inferior. C) normal. D) complements.

26) As the opportunity cost of a good decreases, people buy 26)


A) more of that good but less of its complements.
B) less of that good and also less of its complements.
C) less of that good but more of its complements.
D) more of that good and also more of its complements.

27) People come to expect that the price of a gallon of gasoline will rise next week. As a result, 27)
A) next week's supply of gasoline decreases.
B) the price of a gallon of gasoline falls today.
C) today's supply of gasoline increases.
D) today's demand for gasoline increases.

28) The demand curve for a normal good shifts leftward if income ________ or the expected future 28)
price ________.
A) decreases; falls B) increases; rises C) increases; falls D) decreases; rises

29) If income increases or the price of a complement falls, 29)


A) the supply curve of a normal good shifts leftward.
B) the supply curve of a normal good shifts rightward.
C) the demand curve for a normal good shifts rightward.
D) the demand curve for a normal good shifts leftward.

30) If income decreases or the price of a complement rises, 30)


A) there is an upward movement along the demand curve for the good.
B) there is a downward movement along the demand curve for the good.
C) the demand curve for a normal good shifts leftward.
D) the demand curve for a normal good shifts rightward.

31) Normal goods are those for which demand decreases as 31)
A) the price of a substitute falls. B) the price of a complement falls.
C) the good's own price rises. D) income decreases.

4
32) A normal good is a good for which 32)
A) there are very few complements.
B) demand decreases when income increases.
C) demand increases when income increases.
D) there are few substitutes.

33) Most goods 33)


A) have vertical demand curves. B) have vertical supply curves.
C) are normal goods. D) are complements to each other.

34) A normal good is a good for which demand 34)


A) increases when income increases. B) decreases when population increases.
C) increases when population increases. D) decreases when income increases.

35) Inferior goods are those for which demand increases as 35)
A) income decreases. B) income increases.
C) the price of a substitute rises. D) the price of a substitute falls.

36) By definition, an inferior good is a 36)


A) normal substitute good.
B) good for which demand decreases when its price rises.
C) want that is not expressed by demand.
D) good for which demand decreases when income increases.

37) If a good is an inferior good, then purchases of that good will decrease when 37)
A) the demand for it increases. B) population increases.
C) income increases. D) the price of a substitute rises.

38) An inferior good is a good for which demand 38)


A) increases when population increases. B) decreases when income increases.
C) decreases when population increases. D) increases when income increases.

39) When economists speak of preferences as influencing demand, they are referring to 39)
A) the availability of a good to all income classes.
B) directly observable changes in prices and income.
C) the excess of wants over the available supplies.
D) an individual's attitudes toward goods and services.

40) In 2000 there were 200,000 gas grills demanded at a price of $500. In 2001 there were more than 40)
200,000 gas grills demanded at the same price. This increase could be the result any of the
following EXCEPT
A) an increase in the supply of gas grills.
B) an increase in population.
C) an increase in income if gas grills are a normal good.
D) a fall in the price of natural gas, a complement for a gas grill.

5
41) A change in the price of a good 41)
A) shifts the good's demand curve but does not cause a movement along it.
B) does not shift the good's demand curve but does cause a movement along it.
C) shifts the good's demand curve and also causes a movement along it.
D) neither shifts the good's demand curve nor causes a movement along it.

42) A reduction in the price of a good 42)


A) does not shift the good's demand curve leftward but does decrease the quantity demanded.
B) shifts the good's demand curve leftward but does not decrease the quantity demanded.
C) shifts the good's demand curve leftward and also decreases the quantity demanded.
D) neither shifts the good's demand curve leftward nor decreases the quantity demanded.

43) A decrease in quantity demanded caused by an increase in price is represented by a 43)


A) movement up and to the left along the demand curve.
B) movement down and to the right along the demand curve.
C) leftward shift of the demand curve.
D) rightward shift of the demand curve.

44) A change in which of the following alters buying plans for cars but does NOT shift the demand 44)
curve for cars?
A) a 10 percent decrease in the price of car insurance
B) a 20 percent increase in the price of a car
C) a 5 percent increase in people's income
D) an increased preference for walking rather than driving

45) Which of the following would NOT shift the demand curve for turkey? 45)
A) a change in tastes for turkey B) a decrease in the price of ham
C) an increase in income D) a change in the price of a turkey

46) When we say demand increases, we mean that there is a 46)


A) movement to the right along a demand curve.
B) movement to the left along a demand curve.
C) leftward shift of the demand curve.
D) rightward shift of the demand curve.

6
47) In the figure above, which movement reflects an increase in demand? 47)
A) from point a to point e B) from point a to point c
C) from point a to point b D) from point a to point d

48) In the figure above, which movement reflects a decrease in demand? 48)
A) from point a to point d B) from point a to point e
C) from point a to point c D) from point a to point b

49) In the figure above, which movement reflects a decrease in quantity demanded but NOT a 49)
decrease in demand?
A) from point a to point c B) from point a to point e
C) from point a to point d D) from point a to point b

50) In the figure above, which movement reflects how consumers would react to an increase in the 50)
price of a non-fruit snack?
A) from point a to point b B) from point a to point d
C) from point a to point c D) from point a to point e

51) In the figure above, which movement reflects an increase in the price of a substitute for fruit 51)
snacks?
A) from point a to point d B) from point a to point e
C) from point a to point b D) from point a to point c

52) In the figure above, which movement reflects an increase in the price of a complement for fruit 52)
snacks?
A) from point a to point b B) from point a to point d
C) from point a to point e D) from point a to point c

7
53) In the figure above, which movement reflects how consumers would react to an increase in the 53)
price of a fruit snack that is expected to occur in the future?
A) from point a to point b B) from point a to point e
C) from point a to point c D) from point a to point d

54) In the figure above, which movement reflects an increase in income if fruit snacks are an inferior 54)
good?
A) from point a to point d B) from point a to point c
C) from point a to point b D) from point a to point e

55) In the figure above, which movement reflects an increase in income if fruit snacks are a normal 55)
good?
A) from point a to point d B) from point a to point e
C) from point a to point b D) from point a to point c

56) In the figure above, which movement reflects a decrease in population? 56)
A) from point a to point d B) from point a to point c
C) from point a to point e D) from point a to point b

57) The quantity supplied of a good is 57)


A) equal to the difference between the quantity available and the quantity desired by all
consumers and producers.
B) the same thing as the quantity demanded at each price.
C) the amount that the producers are planning to sell at a particular price during a given time
period.
D) the amount the firm would sell if it faced no resource constraints.

58) The quantity supplied of a good or service is the quantity that a producer 58)
A) actually sells at a particular price during a given time period.
B) should sell at a particular price during a given time period.
C) is willing to sell at a particular price during a given time period.
D) needs to sell at a particular price during a given time period.

59) A fall in the price of a good causes producers to reduce the quantity of the good they are willing to 59)
produce. This fact illustrates
A) a change in supply. B) the law of demand.
C) the nature of an inferior good. D) the law of supply.

60) Each point on a supply curve represents 60)


A) the highest price sellers can get for each unit over time.
B) the lowest price buyers will accept per unit of the good.
C) the lowest price for which a supplier can profitably sell another unit.
D) the highest price buyers will pay for the good.

8
61) Because of increasing marginal cost, most supply curves 61)
A) are horizontal. B) have a negative slope.
C) are vertical. D) have a positive slope.

62) A supply curve shows the relation between the quantity of a good supplied and 62)
A) the price of the good. Usually a supply curve has negative slope.
B) income. Usually a supply curve has positive slope.
C) income. Usually a supply curve has negative slope.
D) the price of the good. Usually a supply curve has positive slope.

63) A supply curve differs from a supply schedule because a supply curve 63)
A) is a graph and the supply schedule is a table.
B) holds the number of suppliers constant, whereas the supply schedule allows the number to
vary.
C) holds resource prices constant, whereas the supply schedule allows them to vary.
D) represents one firm, whereas the supply schedule represents all firms in the market.

64) Which of the following is NOT held constant while moving along a supply curve? 64)
A) prices of resources used in production B) expected future prices
C) the number of sellers D) the price of the good itself

65) If a producer can use resources to produce either good A or good B, then A and B are 65)
A) substitutes in consumption. B) complements in consumption.
C) complements in production. D) substitutes in production.

66) Good A and good B are substitutes in production. The demand for good A increases so that the 66)
price of good A rises. The increase in the price of good A shifts the
A) demand curve for good B rightward. B) demand curve for good B leftward.
C) supply curve of good B rightward. D) supply curve of good B leftward.

67) Blank tapes and prerecorded tapes are substitutes in production. An increase in the price of a blank 67)
tape will cause
A) a decrease in the supply of prerecorded tapes.
B) an increase in the quantity supplied of prerecorded tapes but not in the supply.
C) a decrease in the quantity supplied of prerecorded tapes but not in the supply.
D) an increase in the supply of prerecorded tapes.

68) Good A and good B are substitutes in production. The demand for good A decreases, which lowers 68)
the price of good A. The decrease in the price of good A
A) increases the demand for good B. B) decreases the demand for good B.
C) increases the supply of good B. D) decreases the supply of good B.

9
69) An increase in the number of fast-food restaurants 69)
A) increases the demand for substitutes for fast-food meals.
B) raises the price of fast-food meals.
C) increases the supply of fast-food meals.
D) increases the demand for fast-food meals.

70) Over the past decade technological improvements that have lowered the cost of producing an 70)
automobile have increased
A) the demand but not the supply of automobiles.
B) both the supply and the demand for automobiles.
C) the supply but not the demand for automobiles.
D) neither the supply nor the demand for automobiles.

71) Which of the following will shift the supply curve for good X leftward? 71)
A) a situation in which quantity demanded exceeds quantity supplied
B) an increase in the cost of the machinery used to produce X
C) a decrease in the wages of workers employed to produce X
D) a technological improvement in the production of X

72) Which of the following does NOT shift the supply curve? 72)
A) an increase in the price of the good
B) a fall in the price of a substitute in production
C) a decrease in the wages of labor used in production of the good
D) a technological advance

73) If the price of a good changes but everything else influencing suppliers' planned sales remains 73)
constant, there is a
A) rotation of the initial supply curve around the initial price.
B) new supply curve that is to the right of the initial supply curve.
C) new supply curve that is to the left of the initial supply curve.
D) movement along the supply curve.

74) A decrease in the quantity supplied is represented by a 74)


A) rightward shift in the supply curve. B) movement down the supply curve.
C) leftward shift in the supply curve. D) movement up the supply curve.

75) Which of the following causes an increase in the quantity supplied of good X but NOT in the 75)
supply of good X?
A) an increase in the price of X
B) an increase in the price of good Y, a complement in the production of X
C) an improvement in the technology for producing X
D) a reduction in the price of resources used to produce X

10
76) In the figure above, an increase in the supply of oil would result in a movement from 76)
A) point a to point d. B) point a to point e.
C) point a to point b. D) point a to point c.

77) In the figure above, an increase in the quantity of oil supplied but NOT in the supply of oil is 77)
shown by a movement from
A) point a to point c. B) point a to point b.
C) point a to point e. D) point a to point d.

78) In the figure above, a decrease in the quantity of oil supplied but NOT in the supply of oil is shown 78)
by a movement from
A) point a to point e. B) point a to point d.
C) point a to point b. D) point a to point c.

79) In the figure above, which movement could be caused by an increase in the wages of oil workers? 79)
A) point a to point d B) point a to point b
C) point a to point c D) point a to point e

80) In the figure above, which movement could be caused by the development of a new, more efficient 80)
refining technology?
A) point a to point e B) point a to point c
C) point a to point b D) point a to point d

11
81) The figure above represents the market for candy. People become more concerned that eating 81)
candy causes them to gain weight, which they do not like. As a result, the
A) demand curve will not shift, and the supply curve shifts from S1 to S2.
B) demand curve shifts from D1 to D2and the supply curve shifts from S1 to S2.
C) demand curve shifts from D2 to D1 and the supply curve shifts from S2 to S1.
D) demand curve shifts from D2 to D1 and the supply curve will not shift.

82) The above figure represents the market for oil. Because of the development of a new deep sea 82)
drilling technology the
A) demand curve shifts from D1 to D2 and the supply curve shifts from S1 to S2.
B) demand curve shifts from D1 to D2 and the supply curve will not shift.
C) demand curve will not shift, and the supply curve shifts from S1 to S2.
D) demand curve will not shift, and the supply curve shifts from S2 to S1.

83) The above figure represents the market for oil. When terrorists blow up a major refinery the 83)
A) demand curve for oil will not shift, and the supply curve for oil shifts from S2 to S1.
B) demand curve for oil shifts from D1 to D2 and the supply curve for oil will not shift.
C) demand curve for oil shifts from D1 to D2 and the supply curve for oil shifts from S2 to S1.
D) demand curve for oil will not shift, and the supply curve for oil shifts from S1 to S2.

84) The above figure represents the market for bicycles. When there is a physical fitness craze the 84)
A) demand curve for bicycles shifts from D1 to D2.
B) demand curve for bicycles shifts from D2 to D1.
C) supply curve of bicycles shifts from S1 to S2.
D) demand curve and the supply curve of bicycles do not shift.

12
85) The above figure represents the market for french fries at fast food joints. If the price of potatoes 85)
rises and simultaneously people become concerned that french fries can cause heart attacks
A) the demand curve for french fries will shift from D2 to D1 and the supply curve of french fries
will shift from S2 to S1.
B) the demand curve for french fries will shift from D2 to D1 and the supply curve of french fries
will not shift.
C) the demand curve for french fries will not shift, and the supply curve of french fries will shift
from S1 to S2.
D) the demand curve for french fries will shift from D2 to D1 and the supply curve of french fries
will shift from S1 to S2.

86) The interaction of supply and demand explains 86)


A) both the prices and the quantities of goods and services.
B) the quantities of goods and services but not their prices.
C) the prices of goods and services but not their quantities.
D) neither the prices nor the quantities of goods and services.

87) When the quantity demanded equals quantity supplied 87)


A) the government must be intervening in the market.
B) there is a shortage.
C) there is a surplus.
D) none of the above

13
88) In the above figure, if the demand curve is D2, then 88)
A) an increase in price will cause the demand curve to shift to D3.
B) the equilibrium price will be P1 and the equilibrium quantity will be Q2.
C) the equilibrium price will be P1 and the equilibrium quantity will be Q1.
D) there will be a shortage equal to Q2 - Q1.

89) When the price is below the equilibrium price, the quantity demanded 89)
A) is less than the equilibrium quantity. The quantity supplied exceeds the equilibrium quantity.
B) exceeds the equilibrium quantity. The quantity supplied is less than the equilibrium quantity.
C) exceeds the equilibrium quantity. So does the quantity supplied.
D) is less than the equilibrium quantity. So is the quantity supplied.

90) A price below the equilibrium price results in 90)


A) a further price fall. B) a shortage.
C) excess supply. D) a surplus.

91) Which of the following correctly describes how price adjustments eliminate a shortage? 91)
A) As the price falls, the quantity demanded increases while the quantity supplied decreases.
B) As the price rises, the quantity demanded decreases while the quantity supplied increases.
C) As the price falls, the quantity demanded decreases while the quantity supplied increases.
D) As the price rises, the quantity demanded increases while the quantity supplied decreases.

92) A shortage causes the 92)


A) supply curve to shift rightward. B) price to rise.
C) price to fall. D) demand curve to shift leftward.

14
93) If the quantity demanded exceeds the quantity supplied, then there is 93)
A) a shortage and the price is above the equilibrium price.
B) a surplus and the price is below the equilibrium price.
C) a shortage and the price is below the equilibrium price.
D) a surplus and the price is above the equilibrium price.

94) A surplus occurs when the price is 94)


A) equal to the equilibrium price.
B) greater than the equilibrium price.
C) less than the equilibrium price.
D) None of the above because the existence of a surplus is independent of the price of the good.

95) If the quantity supplied exceeds the quantity demanded, then there is 95)
A) a shortage and the price is below the equilibrium price.
B) a surplus and the price is below the equilibrium price.
C) a surplus and the price is above the equilibrium price.
D) a shortage and the price is above the equilibrium price.

96) The price of a good will fall if 96)


A) the price of a complement falls.
B) there is a surplus at the current price.
C) the quantity demanded exceeds the quantity supplied.
D) the current price is less than the equilibrium price.

97) The equilibrium price in the above figure is 97)


A) $2. B) $8. C) $4. D) $6.

15
98) The equilibrium quantity in the above figure is 98)
A) 400 units. B) 300 units. C) 600 units. D) 200 units.

99) At a price of $10 in the above figure, there is 99)


A) a surplus of 400 units. B) a shortage of 200 units.
C) a surplus of 200 units. D) a shortage of 400 units.

100) At a price of $4 in the above figure, 100)


A) there is a surplus of 200 units. B) the equilibrium quantity is 400 units.
C) the quantity supplied is 400 units. D) there is a shortage of 200 units.

101) If the good in the above figure is a normal good and income rises, then the new equilibrium 101)
quantity
A) is more than 300 units.
B) is less than 300 units.
C) could be less than, equal to, or more than 300 units.
D) is 300 units.

102) The initial supply and demand curves for a good are illustrated in the above figure. If there are 102)
technological advances in the production of the good, then the new price for the good
A) is $6.
B) is more than $6.
C) could be less than, equal to, or more than $6.
D) is less than $6.

103) The initial supply and demand curves for a good are illustrated in the above figure. If there is a rise 103)
in the price of the resources used to produce the good, then the new price
A) is less than $6.
B) is more than $6.
C) could be less than, equal to, or more than $6.
D) is $6.

16
The Market for Wapanzo Beans
Quantity Demanded Price Quantity Supplied
(millions of pounds (dollars per (millions of pounds
per year) pound) per year)
Case Case Case Case Case Case
1 2 3 A B C
15 10 5 1 2 3
12 8 4 2 2 4 6
9 6 3 3 3 6 9
6 3 2 4 4 8 12
3 2 1 5 5 10 15

104) Refer to the table above. Suppose that in normal years demand is represented by Case 2 and 104)
supply is represented by Case B. In a normal year the price of wapanzo beans will be
A) $3 per pound. B) $4 per pound. C) $2 per pound. D) $1 per pound.

105) Refer to the table above. Suppose that in normal years demand is represented by Case 2 and 105)
supply is represented by Case B. In a normal year the equilibrium quantity of wapanzo beans will
be
A) 8 million pounds. B) 4 million pounds.
C) 6 million pounds. D) 2 million pounds.

106) Refer to the table above. Suppose that in normal years demand is represented by Case 2 and 106)
supply is represented by Case B. If there is a drought in the wapanzo bean growing region then
supply will ________ and demand will ________.
A) stay at case B; become case 3 B) stay at case B; become case 1
C) become case A; become case 1 D) become case A; stay at case 2

107) Refer to the table above. Suppose that in normal years demand is represented by Case 2 and 107)
supply is represented by Case B. If there is exceptionally good growing weather in the wapanzo
bean growing region then supply will ________ and demand will ________.
A) stay at case B; become case 1 B) become case C; stay at case 2
C) become case C; become case 3 D) become case C; become case 1

108) Refer to the table above. Suppose that in normal years demand is represented by Case 2 and 108)
supply is represented by Case B. If it is discovered that wapanzo beans help prevent cancer then
supply will ________ and demand will ________.
A) stay at case B; become case 1 B) become case C; stay at case 2
C) become case A; become case 1 D) become case C; become case 1

109) When the demand for a good decreases, its equilibrium price ________ and equilibrium quantity 109)
________.
A) rises; decreases B) falls; decreases C) falls; increases D) rises; increases

17
110) If good A is a normal good and income increases, the equilibrium price of A 110)
A) and the equilibrium quantity will increase.
B) and the equilibrium quantity will decrease.
C) will rise and the equilibrium quantity will decrease.
D) will fall and the equilibrium quantity will increase.

111) The price of a gallon of milk falls. Which of the following is a possible cause? 111)
A) a discovery that milk cause diabetes
B) a drought that reduces supplies of feed grains fed to cows that produce milk
C) an increase in the income of the average household, with milk being a normal good
D) a decrease in the price of oatmeal, a complement to milk

112) Assume that beef and pork are substitutes for consumers. There is a drought in the cattle grazing 112)
areas. The drought will cause the
A) supply curve for pork to shift rightward. B) supply curve for pork to shift leftward.
C) demand curve for pork to shift leftward. D) demand curve for pork to shift rightward.

113) An increase in demand combined with no change in supply causes 113)


A) a decrease in demand because the supply curve does not shift.
B) the equilibrium price to fall.
C) a movement rightward along the demand curve.
D) the equilibrium price to rise.

114) Goods A and B are complementary goods (in consumption). The cost of a resource used in the 114)
production of A decreases. As a result,
A) the equilibrium price of B will fall and the equilibrium price of A will rise.
B) the equilibrium prices of both A and B will rise.
C) the equilibrium price of B will rise and the equilibrium price of A will fall.
D) the equilibrium prices of both A and B will fall.

115) When demand decreases and supply does not change, the equilibrium price 115)
A) rises and the equilibrium quantity decreases.
B) rises and the equilibrium quantity increases.
C) falls and the equilibrium quantity increases.
D) falls and the equilibrium quantity decreases.

116) When supply decreases and demand does not change, the equilibrium quantity 116)
A) decreases and the price rises. B) increases and the price falls.
C) decreases and the price falls. D) increases and the price rises.

18
117) Beef and leather belts are complements in production. If people's concern about health shifts the 117)
demand curve for beef leftward, the result in the market for leather belts will be a
A) lower equilibrium price for a leather belt because there is an increase in the supply of leather
belts.
B) higher equilibrium price for a leather belt because there is a decrease in the supply of leather
belts.
C) lower equilibrium price for a leather belt because there is a decrease in the supply of leather
belts.
D) higher equilibrium price for a leather belt because there is an increase in the supply of leather
belts.

118) You observe that the price of a good rises and the quantity decreases. These observations can be 118)
the result of
A) the supply curve shifting rightward. B) the demand curve shifting rightward.
C) the demand curve shifting leftward. D) the supply curve shifting leftward.

119) Leather belts and leather shoes are substitutes in production. If style changes increase the demand 119)
for leather belts, the supply curve of leather shoes will shift
A) rightward and the equilibrium price of leather shoes will fall.
B) leftward and the equilibrium price of leather shoes will rise.
C) leftward and the equilibrium price of leather shoes will fall.
D) rightward and the equilibrium price of leather shoes will rise.

120) If both demand and supply increase, what will be the effect on the equilibrium price and quantity? 120)
A) The price will rise but the quantity could either increase, decrease, or remain the same.
B) The quantity will increase but the price could either rise, fall, or remain the same.
C) Both the price and the quantity will increase.
D) The price will fall but the quantity will increase.

121) If both the demand and supply increase, the equilibrium quantity 121)
A) decreases and the price rises.
B) increases and the effect on price is indeterminate.
C) decreases and the effect on price is indeterminate.
D) increases and the price falls.

122) The price will rise and the equilibrium quantity might increase, decrease, or stay the same when 122)
the
A) demand and the supply of a good both increase.
B) demand and the supply of a good both decrease.
C) demand for a good decreases and the supply of it increases.
D) demand for a good increases and the supply of it decreases.

19
123) The price will fall and the equilibrium quantity might increase, decrease, or stay the same when the 123)
A) demand for a good increases and the supply of it decreases.
B) demand and the supply of a good both decrease.
C) demand for a good decreases and the supply of it increases.
D) demand and the supply of a good both increase.

124) The equilibrium quantity will decrease and the price might rise, fall, or stay the same when the 124)
A) demand and the supply of a good both decrease.
B) demand for a good increases and the supply of it decreases.
C) demand for a good decreases and the supply of it increases.
D) demand and the supply of a good both increase.

125) The equilibrium quantity of a good will increase and its equilibrium price might rise, fall, or stay 125)
the same when
A) its demand decreases and supply increases.
B) its demand increases and supply decreases.
C) its demand and supply both increase.
D) its demand and supply both decrease.

126) The price of compact disc players fell over the past decade because a combination of improving 126)
technology, rising incomes, and falling prices of compact discs caused the
A) demand curve for compact disc players to shift rightward faster than the supply curve of
compact disc players shifted rightward.
B) supply curve of compact disc players to shift rightward faster than the demand curve for
compact disc players shifted rightward.
C) demand curve for compact disc players to shift leftward and the supply curve of compact disc
players to shift leftward.
D) supply curve of compact disc players to shift rightward and the demand curve for compact
disc players to shift leftward.

127) Which of the following will always raise the equilibrium price? 127)
A) an increase in demand combined with a decrease in supply
B) a decrease in both demand and supply
C) an increase in both demand and supply
D) a decrease in demand combined with an increase in supply

20
128) In the above figure, a change in quantity demanded with unchanged demand is represented by a 128)
movement from
A) point a to point c.
B) point a to point e.
C) point a to point b.
D) None of the above represent a change in the quantity demanded with an unchanged demand.

129) In the above figure, a change in quantity supplied with unchanged supply is represented by a 129)
movement from
A) point b to point e. B) point b to point a.
C) point e to point c. D) point a to point e.

130) In the above figure, if D2 is the demand curve, then a price of P3 would result in 130)
A) a surplus of Q3 - Q1. B) a shortage of Q4 - Q3.
C) a surplus of Q4 - Q0. D) a shortage of Q3 - Q1.

131) In the above figure, if D2 is the original demand curve for a normal good and income decreases, 131)
which price and quantity may result?
A) point c, with price P3 and quantity Q3 B) point a, with price P2 and quantity Q2
C) point b, with price P1 and quantity Q1 D) point d, with price P1 and quantity Q3

132) In the above figure, if D2 is the original demand curve and the price of a substitute in consumption 132)
rises, which price and quantity may result?
A) point c, with price P3 and quantity Q3 B) point d, with price P1 and quantity Q3
C) point a, with price P2 and quantity Q2 D) point b, with price P1 and quantity Q1

21
133) In the above figure, if D2 is the original demand curve and consumers come to expect that the price 133)
of the good will rise in the future, which price and quantity may result?
A) point a, with price P2 and quantity Q2 B) point c, with price P3 and quantity Q3
C) point d, with price P1 and quantityQ3 D) point b, with price P1 and quantity Q1.

134) In the above figure, if D2 is the original demand curve and the population falls, which price and 134)
quantity may result?
A) point d, with price P1 and quantity Q3 B) point c, with price P3 and quantity Q3
C) point b, with price P1 and quantity Q1 D) point a, with price P2 and quantity Q2

135) In the figure, the equilibrium price is initially $3 per bushel of wheat. If suppliers come to expect 135)
that the price of a bushel of wheat will rise in the future, but buyers do not, the current equilibrium
price will
A) not change.
B) fall.
C) rise.
D) perhaps rise, fall, or stay the same, depending on whether there are more demanders or
suppliers in the market.

136) In the figure, the equilibrium price is initially $3 per bushel of wheat. If buyers come to expect that 136)
the price of a bushel of wheat will rise in the future, but sellers do not, the current equilibrium price
will
A) rise.
B) fall.
C) not change.
D) perhaps rise, fall, or stay the same, depending on whether there are more demanders or
suppliers in the market.

22
137) Let Qd stand for the quantity demanded, Qs stand for the quantity supplied, and P stand for price. 137)
If Qd = 20 - 2P and Qs = 5 + 3P, then the equilibrium price is
A) $2. B) $3. C) $4. D) $1.

138) LetQd stand for the quantity demanded, Qs stand for the quantity supplied, and P stand for price. 138)
If Qd = 20 - 2P andQs = 5 + 3P, then the equilibrium quantity is
A) 14. B) 5. C) 20. D) 3.

139) A consumer might consider in-line skates and elbow-pads to be 139)


A) unrelated goods.
B) substitutes.
C) products with upward sloping demand curves.
D) complements.

140) A decrease in the price of a game of bowling shifts the 140)


A) demand curve for bowling balls rightward.
B) supply curve of bowling balls leftward.
C) supply curve of bowling balls rightward.
D) demand curve for bowling balls leftward.

141) If a decrease in the price of gasoline increases the demand for large cars, then 141)
A) gasoline and large cars are complements in consumption.
B) large cars are an inferior good.
C) gasoline is an inferior good.
D) gasoline and large cars are substitutes in consumption.

142) Gruel is an inferior good. Hence, a decrease in people's incomes 142)


A) shifts the supply curve of gruel leftward.
B) shifts the demand curve for gruel rightward.
C) shifts the demand curve for gruel leftward.
D) decreases the quantity of gruel supplied.

143) An unusually warm winter 143)


A) shifts the supply curve of gloves leftward.
B) shifts the demand curve for gloves rightward.
C) shifts the demand curve for gloves leftward.
D) shifts the supply curve of gloves rightward.

144) A rise in the price of a good causes producers to supply more of the good. This statement 144)
illustrates
A) the nature of an inferior good. B) the law of demand.
C) the law of supply. D) a change in supply.

23
145) The price of jet fuel falls. This fall shifts the 145)
A) supply curve of airplane trips rightward.
B) demand curve for airplane trips leftward.
C) demand curve for airplane trips rightward.
D) supply curve of airplane trips leftward.

146) If there is surplus of a good, then the quantity demanded ________ the quantity supplied and the 146)
price will ________.
A) is less than; rise B) is less than; fall
C) is greater than; fall D) is greater than; rise

147) Pizza and hamburgers are substitutes for consumers. A fall in the price of a pizza ________ the 147)
price of a hamburger and ________ the quantity of hamburgers.
A) raises; decreases B) lowers; decreases
C) raises; increases D) lowers; increases

148) How does an unusually warm winter affect the equilibrium price and quantity of gloves? 148)
A) It lowers both the price and the quantity.
B) It raises both the price and the quantity.
C) It raises the price and decreases the quantity.
D) It lowers the price and increases the quantity.

149) You notice that the price and quantity of wheat both decrease. This observation can be the result of 149)
the
A) demand curve for wheat shifting leftward.
B) supply curve of wheat shifting rightward.
C) demand curve for wheat shifting rightward.
D) supply curve of wheat shifting leftward.

150) A technological improvement lowers the cost of producing coffee. At the same time, consumers' 150)
preferences for coffee increase. The equilibrium price of coffee will
A) rise, fall, or stay the same, depending on the relative size of the shifts in the demand and
supply curves.
B) remain the same.
C) fall.
D) rise.

151) Which of the following definitely causes a fall in the equilibrium price? 151)
A) a decrease in both demand and supply
B) an increase in demand combined with a decrease in supply
C) a decrease in demand combined with an increase in supply
D) an increase in both demand and supply

24
152) CD players rise in price while pre-recorded audio tapes fall in price. The combined effect of these 152)
two changes is to create
A) a leftward shift of the demand curve for portable audio tape players, such as a Walkman.
B) a rightward shift of the demand curve for portable audio tape players, such as a Walkman.
C) a rightward shift of the supply curve for portable audio tape players, such as a Walkman.
D) a leftward shift of the supply curve of portable audio tape players, such as a Walkman.

153) Walkman Watch expects a recession to occur. Knowing that a Walkman is a normal good, you 153)
predict that the demand for a Walkman
A) will increase. B) might increase or decrease.
C) will decrease. D) will remain unchanged.

154) Wages for workers producing Walkmans and similar products will rise next year. Walkman Watch 154)
asks you to predict the effect of this change in next year's market for Walkmans. You predict that
the major effect will be that the
A) demand curve for a Walkman will shift leftward.
B) supply curve for a Walkman will shift rightward.
C) supply curve for a Walkman will shift leftward.
D) demand curve for a Walkman will shift rightward.

155) Producers of Walkmans are able to lower the wage rate that they pay to their workers. Walkman 155)
Watch asks you to predict the effect on the Walkmans. You predict that the
A) quantity supplied will decrease. B) price will rise.
C) supply curve will shift leftward. D) supply curve will shift rightward.

156) The wage rate paid by Walkman producers falls and at the same time the price of raw materials 156)
used in the production of Walkmans rises. You predict that the supply curve of Walkmans will
A) surely shift leftward. B) surely become steeper.
C) shift either leftward or rightward. D) surely shift rightward.

157) Walkmans play cassette tapes. Producers of Walkmans expect that a new technology for producing 157)
CD players will be available next year. Walkman Watch asks you to predict the effect of the new
technology on the market for Walkmans. You predict that
A) the demand curve for Walkmans will shift leftward and the price will fall.
B) the price will rise, and so will the quantity demanded.
C) the price will fall, and the quantity demanded will increase.
D) the demand curve for Walkmans will shift rightward and the price will rise.

158) Producers of Walkmans will be able to lower the wage rate that they pay to their workers. 158)
Walkman Watch asks you to predict the effects on the supply of Walkmans, and the price of a
Walkman. You predict that the supply curve shifts
A) leftward, and the price is constant. B) rightward, and the price falls.
C) leftward, and the price rises. D) rightward, and the price is constant.

25
Answer Key
Testname: UNTITLED3.TST

1) A
2) A
3) C
4) B
5) D
6) C
7) D
8) A
9) A
10) B
11) B
12) A
13) B
14) C
15) B
16) B
17) C
18) A
19) B
20) C
21) C
22) C
23) B
24) A
25) D
26) D
27) D
28) A
29) C
30) C
31) D
32) C
33) C
34) A
35) A
36) D
37) C
38) B
39) D
40) A
41) B
42) A
43) A
44) B
45) D
46) D
47) D
48) C
49) B
50) B
1
Answer Key
Testname: UNTITLED3.TST

51) A
52) D
53) D
54) B
55) A
56) B
57) C
58) C
59) D
60) C
61) D
62) D
63) A
64) D
65) D
66) D
67) A
68) C
69) C
70) C
71) B
72) A
73) D
74) B
75) A
76) C
77) C
78) D
79) A
80) C
81) D
82) C
83) A
84) A
85) A
86) A
87) D
88) B
89) B
90) B
91) B
92) B
93) C
94) B
95) C
96) B
97) D
98) B
99) A
100) D
2
Answer Key
Testname: UNTITLED3.TST

101) A
102) D
103) B
104) A
105) C
106) D
107) B
108) A
109) B
110) A
111) A
112) D
113) D
114) C
115) D
116) A
117) B
118) D
119) B
120) B
121) B
122) D
123) C
124) A
125) C
126) B
127) A
128) B
129) B
130) A
131) C
132) A
133) B
134) C
135) C
136) A
137) B
138) A
139) D
140) A
141) A
142) B
143) C
144) C
145) A
146) B
147) B
148) A
149) A
150) A
3
Answer Key
Testname: UNTITLED3.TST

151) C
152) B
153) C
154) C
155) D
156) C
157) A
158) B

Das könnte Ihnen auch gefallen